You are on page 1of 227

Any screen.

Any time.
Anywhere.
Activate the eBook version
of this title at no additional charge.

Expert Consult eBooks give you the power to browse and find content,
view enhanced images, share notes and highlights—both online and offline.

Unlock your eBook today.


1 Visit expertconsult.inkling.com/redeem Scan this QR code to redeem your
eBook through your mobile device:
2 Scratch off your code
3 Type code into “Enter Code” box

4 Click “Redeem”
5 Log in or Sign up
6 Go to “My Library”
Place Peel Off
It’s that easy! Sticker Here

For technical assistance:


email expertconsult.help@elsevier.com
call 1-800-401-9962 (inside the US)
call +1-314-447-8200 (outside the US)
Use of the current edition of the electronic version of this book (eBook) is subject to the terms of the nontransferable, limited license granted on
expertconsult.inkling.com. Access to the eBook is limited to the first individual who redeems the PIN, located on the inside cover of this book,
at expertconsult.inkling.com and may not be transferred to another party by resale, lending, or other means.
2015v1.0
Second Edition

The
Brigham
Intensive
Review of
Internal
Medicine
QUESTION & ANSWER COMPANION

Ajay K. Singh, MBBS, FRCP, MBA


Senior Associate Dean
Global and Continuing Education
Harvard Medical School
Physician, Renal Division
Brigham and Women’s Hospital
Boston, MA

Sarah P. Hammond, MD
Assistant Professor of Medicine
Harvard Medical School
Division of Infectious Diseases
Department of Medicine
Brigham and Women’s Hospital
Boston, MA

Joseph Loscalzo, MD, PhD


Hersey Professor of the Theory and Practice of Physic
Harvard Medical School
Chairman, Department of Medicine
Physician-in-Chief
Brigham and Women’s Hospital
Boston, MA
1600 John F. Kennedy Blvd.
Ste 1800
Philadelphia, PA 19103-2899

THE BRIGHAM INTENSIVE REVIEW OF INTERNAL MEDICINE


QUESTION & ANSWER COMPANION, SECOND EDITION ISBN: 978-0-323-48043-7

Copyright © 2019 by Elsevier, Inc. All rights reserved.

No part of this publication may be reproduced or transmitted in any form or by any means, electronic or me-
chanical, including photocopying, recording, or any information storage and retrieval system, without permis-
sion in writing from the publisher. Details on how to seek permission, further information about the Publisher’s
permissions policies and our arrangements with organizations such as the Copyright Clearance Center and the
Copyright Licensing Agency, can be found at our website: www.elsevier.com/permissions.

his book and the individual contributions contained in it are protected under copyright by the Publisher (other than
as may be noted herein).

Notices

Knowledge and best practice in this ield are constantly changing. As new research and experience broaden our
understanding, changes in research methods, professional practices, or medical treatment may become necessary.
Practitioners and researchers must always rely on their own experience and knowledge in evaluating and using
any information, methods, compounds, or experiments described herein. In using such information or methods
they should be mindful of their own safety and the safety of others, including parties for whom they have a profes-
sional responsibility.
With respect to any drug or pharmaceutical products identiied, readers are advised to check the most current
information provided (i) on procedures featured or (ii) by the manufacturer of each product to be administered,
to verify the recommended dose or formula, the method and duration of administration, and contraindications.
It is the responsibility of practitioners, relying on their own experience and knowledge of their patients, to make
diagnoses, to determine dosages and the best treatment for each individual patient, and to take all appropriate
safety precautions.
To the fullest extent of the law, neither the Publisher nor the authors, contributors, or editors, assume any liabil-
ity for any injury and/or damage to persons or property as a matter of products liability, negligence or otherwise,
or from any use or operation of any methods, products, instructions, or ideas contained in the material herein.

Previous edition copyrighted 2014 by Oxford University Press.

Library of Congress Cataloging-in-Publication Data

Names: Singh, Ajay, 1960- editor. | Loscalzo, Joseph, editor.


Title: he Brigham intensive review of internal medicine question & answer
companion / [edited by] Ajay K. Singh, Joseph Loscalzo.
Description: Second edition. | Philadelphia, PA : Elsevier, [2019] | Includes
index.
Identiiers: LCCN 2017042502 | ISBN 9780323480437 (pbk. : alk. paper)
Subjects: | MESH: Internal Medicine--methods | Physical Examination--methods
| Problems and Exercises
Classiication: LCC RC46 | NLM WB 18.2 | DDC 616--dc23 LC record available at
https://lccn.loc.gov/2017042502

Executive Content Strategist: Kate Dimock


Senior Content Development Specialist: Joan Ryan
Publishing Services Manager: Catherine Jackson
Book Production Specialist: Kristine Feeherty
Design Direction: Patrick Ferguson

Printed in China
Last digit is the print number: 9 8 7 6 5 4 3 2 1
Contributors

Amy Bessnow, MD Ole-Petter R. Hamnvik, MB BCh BAO, MMSc


Instructor in Medicine Assistant Professor of Medicine
Harvard Medical School Harvard Medical School
Department of Medicine Division of Endocrinology, Diabetes, and Hypertension
Brigham and Women’s Hospital Department of Medicine
Dana-Farber Cancer Institute Brigham and Women’s Hospital
Boston, MA Boston, MA
Hematology and Oncology Endocrinology

Robert Burakof, MD, MPH Galen V. Henderson, MD


Vice Chair for Ambulatory Services Assistant Professor of Medicine
Department of Medicine Harvard Medical School
Weill Cornell Medical College Department of Neurology
New York, NY; Brigham and Women’s Hospital
Site Chief Boston, MA
Division of Gastroenterology and Endoscopy General Internal Medicine
New York–Presbyterian Lower Manhattan Hospital
New York, NY; Jennifer A. Johnson, MD
Visiting Scientist Assistant Professor of Medicine
Harvard Medical School Harvard Medical School
Boston, MA Division of Infectious Diseases
Gastroenterology Department of Medicine
Brigham and Women’s Hospital
Elizabeth Gay, MD Boston, MA
Member of the Faculty of Medicine Infectious Diseases
Harvard Medical School
Division of Pulmonary and Critical Care Medicine Ann S. LaCasce, MD
Department of Medicine Associate Professor of Medicine
Brigham and Women’s Hospital Harvard Medical School
Boston, MA Department of Medical Oncology
Pulmonary and Critical Care Medicine Dana-Farber Cancer Institute
Department of Medicine
Sarah P. Hammond, MD Brigham and Women’s Hospital
Assistant Professor of Medicine Boston, MA
Harvard Medical School Hematology and Oncology
Division of Infectious Diseases
Department of Medicine Ernest I. Mandel, MD
Brigham and Women’s Hospital Instructor in Medicine
Boston, MA Harvard Medical School
Infectious Diseases Division of Renal Medicine
General Internal Medicine Department of Medicine
Brigham and Women’s Hospital
Boston, MA
Nephrology and Hypertension

iii
iv Contributors

Muthoka L. Mutinga, MD Scott L. Schissel, MD


Assistant Professor of Medicine Instructor in Medicine
Harvard Medical School Harvard Medical School
Division of Gastroenterology, Hepatology, and Endoscopy Chief, Department of Medicine
Department of Medicine Brigham and Women’s Faulkner Hospital
Brigham and Women’s Hospital Division of Pulmonary and Critical Care Medicine
Boston, MA Brigham and Women’s Hospital
Gastroenterology Boston, MA
Pulmonary and Critical Care Medicine
Anju Nohria, MD, MSc
Assistant Professor of Medicine Lori Wiviott Tishler, MD
Harvard Medical School Assistant Professor of Medicine
Division of Cardiovascular Medicine Harvard Medical School
Department of Medicine Division of General Internal Medicine and Primary Care
Brigham and Women’s Hospital Department of Medicine
Boston, MA Brigham and Women’s Hospital
Cardiovascular Disease Boston, MA
General Internal Medicine
Molly Perencevich, MD
Instructor in Medicine Derrick J. Todd, MD, PhD
Harvard Medical School Instructor of Medicine
Division of Gastroenterology, Hepatology, and Endoscopy Harvard Medical School
Department of Medicine Division of Rheumatology, Immunology, and Allergy
Brigham and Women’s Hospital Department of Medicine
Boston, MA Brigham and Women’s Hospital
Gastroenterology Boston, MA
Rheumatology
Megan Prochaska, MD
Research Fellow in Medicine
Harvard Medical School
Department of Medicine
Brigham and Women’s Hospital
Boston, MA
Nephrology and Hypertension
Preface

Preparing for the American Board of Internal Medicine their eforts and commitment to this project. We asked
(ABIM) certifying or recertifying examination requires them to put themselves “in the head” of the ABIM to
knowledge and clinical experience that can be evaluated identify the topics that might be addressed in the board
by successfully answering questions in a test format. In examination. We believe this book will be a valuable
this Question and Answer book, our goal is to provide the study tool to gauge one’s knowledge in preparation for the
reader with 450 questions across 9 subspecialties in inter- examination.
nal medicine. hese questions test knowledge on topics rel- We wish to thank Stephanie Tran and Michelle Deraney
evant to the ABIM boards. As a companion to the Brigham for supporting us in the development of this book. Without
Intensive Review of Internal Medicine, now in its third edi- them this book would not have been possible. Our thanks
tion, this book is focused on how one applies knowledge to also go to our families who have supported all of our aca-
answer board questions successfully. he annotated answers demic activities, including this important project.
are detailed, and they review the steps in critical thinking
required to get to the correct answer. Ajay K. Singh, MBBS, FRCP, MBA
he authors who have contributed questions and anno- Joseph Loscalzo, MD, PhD
tated answers to this book are some of our most senior Sarah P. Hammond, MD
physicians in the department. We sincerely thank them for

v
Contents

1. Infectious Diseases, 1
Sarah P. Hammond and Jennifer A. Johnson

2. Hematology and Oncology, 28


Amy Bessnow and Ann S. LaCasce

3. Rheumatology, 49
Derrick J. Todd

4. Pulmonary and Critical Care Medicine, 78


Scott L. Schissel and Elizabeth Gay

5. Endocrinology, 104
Ole-Petter R. Hamnvik

6. Nephrology and Hypertension, 129


Megan Prochaska and Ernest I. Mandel

7. Gastroenterology, 145
Muthoka L. Mutinga, Molly Perencevich,
and Robert Burakof

8. Cardiovascular Disease, 165


Anju Nohria

9. General Internal Medicine, 191


Lori Wiviott Tishler, Sarah P. Hammond,
and Galen V. Henderson

Index, 212

vi
1
Infectious Diseases
SARAH P. HAMMOND AND JENNIFER A. JOHNSON

1. A 28-year-old woman who has lived her entire life in donor for polycystic kidney disease and has had no
Providence, Rhode Island, presents 3 days after return- episodes of rejection since. Her donor was cytomega-
ing from a 2-week trip to hailand complaining of lovirus (CMV) immunoglobulin G (IgG) negative,
fever to 102°F, muscle aches, and severe retroorbital and she was CMV IgG positive before transplant. Her
headache. She has no gastrointestinal symptoms. She chronic medications for transplant include tacrolimus,
traveled only to the towns of Bangkok, Chiang Mai, low-dose prednisone, and mycophenolate mofetil and
and Phuket. She attended a travel clinic before travel- have not changed in several years. She is a third-grade
ing and was told there was no malaria in these towns, teacher and has recently been taking care of the class
so she did not take prophylaxis. She denied contact pets, which include two goldish and a hamster. On
with bodies of fresh water. Examination is unremark- presentation she has a fever to 102.1°F and has difuse
able other than temperature of 101.8°F. Remarkable tenderness of the abdomen without rebound.
laboratory indings include a leukocyte count of 2200 he most likely cause of her present illness is:
cells/µL3, hematocrit of 37%, and platelets of 62,000 A. Salmonellosis
cells/µL3. Chemistries are normal. A peripheral blood B. Medication-induced diarrhea related to the cumu-
smear for parasites is sent and is negative. lative efects of mycophenolate
Which of the following is the most likely diagnosis C. Cytomegalovirus colitis
in this traveler? D. Norovirus gastroenteritis
A. Leptospirosis E. Irritable bowel syndrome
B. Malaria
C. Typhoid 4. A 24-year-old man with ulcerative colitis presents in
D. Hepatitis A January for a irst primary care clinic visit with you as
E. Dengue his new primary care physician. He was diagnosed with
ulcerative colitis involving the entire colon 10 months
2. A 55-year-old male smoker with severe chronic ago and initially was treated with corticosteroids and
obstructive pulmonary disease (COPD) is hospitalized mesalamine. In the last 3 months he has been doing
in the medical intensive care unit. He now requires well on mesalamine and azathioprine, which are his
intubation and mechanical ventilation for hypercarbic only medications. He works as a paralegal and hopes
respiratory failure after failing noninvasive ventilation. to attend law school in the next few years. He lives
To reduce this patient’s risk for developing ventila- with his girlfriend of 2 years with whom he is monoga-
tor-associated pneumonia, you recommend: mous. He uses condoms for birth control. He is feeling
A. Elevation of the head of the bed to 15 degrees to well today. Physical examination is unremarkable; he is
prevent aspiration afebrile and well appearing. You review his immuniza-
B. Suctioning of subglottic secretions tion history—he has not received any vaccines within
C. Twenty-four hours of prophylactic systemic anti- the past 6 years; his last vaccination was the conjugate
biotics, especially if the intubation was emergent meningococcal vaccine at age 18.
D. Daily changing of the ventilatory circuit In addition to vaccinating for inluenza and human
E. Nasotracheal intubation rather than orotracheal papillomavirus, he should also receive which of the
intubation following vaccines?
A. Tetanus, diphtheria (Td) vaccine
3. A 47-year-old woman with a history of renal transplan- B. Pneumococcal 13-valent conjugate (PCV13) vaccine
tation presents with 4 days of profuse nonbloody diar- C. Haemophilus inluenzae B vaccine
rhea, abdominal pain, and high fevers. She received her D. Pneumococcal 23-valent polysaccharide vaccine
kidney transplant 10 years ago from a living unrelated E. Meningococcus B vaccine

1
2 C HA P T E R 1 Infectious Diseases

5. A 36-year-old man is found to have a positive tuber- hepatitis B DNA not detected, hepatitis C IgG nega-
culosis interferon gamma release assay result as part tive. She was treated with six cycles of rituximab,
of a workplace screening program. He is originally cyclophosphamide, doxorubicin, vincristine, and
from Bangladesh and was vaccinated with the bacil- prednisone (R-CHOP), which ended several weeks
lus calmette-Guérin (BCG) vaccine during childhood. ago and achieved complete remission based on posi-
He immigrated to the United States 6 months ago. He tron emission tomography (PET) CT imaging. Basic
reports feeling well. He has no fever, cough, or weight laboratory indings when she presents now are notable
loss. Physical examination is normal. for aspartate aminotransferase (AST) 527 U/L, alanine
he next best step in his management should be: aminotransferase (ALT) 495 U/L, total bilirubin 3.5
A. Sputum for smear microscopy and mycobacterial mg/dL, with a normal international normalized ratio
culture (INR). Her past history is notable for immigrating to
B. Initiation of isoniazid prophylaxis to prevent reac- the United States from rural Vietnam 2 years ago. She
tivation of latent tuberculosis infection had a PPD skin test at the time she immigrated and
C. Chest x-ray to assess for active pulmonary disease she was treated for latent tuberculosis with a 9-month
D. Perform a tuberculin skin test (puriied protein course of isoniazid that inished before the diagnosis of
derivative [PPD] test) to conirm the skin test lymphoma.
result A likely cause of her abnormal liver function tests
and malaise is:
6. A previously well 62-year-old man presents to the hos- A. Hepatitis C infection resulting from blood trans-
pital with increasing weakness in his lower extremities. fusion
Examination reveals decreased relexes symmetrically, B. Recurrence of her lymphoma
which progresses proximally over the course of several C. Acute hepatitis A infection
hours. He is diagnosed with Guillain–Barré syndrome D. Delayed isoniazid toxicity
and admitted to the intensive care unit for treatment. E. Reactivation of hepatitis B infection
On history, he reports several days of nausea, vomit-
ing, and diarrhea approximately 2 months prior. 9. A 34-year-old teacher presents to her primary care
he most likely infectious cause of his gastrointes- physician with 1 week of severe cough. Her symptoms
tinal illness was: began 2 weeks prior with a mild fever and rhinorrhea.
A. Campylobacter She has been experiencing posttussive emesis every
B. Giardia couple of hours. She takes levothyroxine for hypo-
C. Salmonella thyroidism but otherwise has no chronic illnesses.
D. Cryptosporidium Whooping cough is suspected, and a polymerase chain
E. Escherichia coli O157:H7 reaction (PCR) of a respiratory specimen is sent to test
for Bordetella pertussis.
7. A 24-year-old woman calls your oice complaining of he best management is:
burning with urination, and increased urinary urgency A. Treat with codeine-containing cough syrup for
and frequency. She reports no fever, nausea, vomiting, symptom control and wait for conirmation of B.
or lank pain. She has been in a monogamous rela- pertussis infection.
tionship for 3 years, and she had one prior episode of B. It is too late to treat for B. pertussis with antibiot-
cystitis, more than a year ago. ics; reassure her that the cough will improve in the
Which of the following agents is the best treatment next 2–4 weeks and administer the TDaP vaccine
for acute uncomplicated cystitis? now.
A. Cephalexin 500 mg twice daily for 7 days C. Start empiric azithromycin for a 5-day course.
B. Ciproloxacin 250 mg twice daily for 3 days D. Start empiric levoloxacin for a 7-day course.
C. Nitrofurantoin macrocrystals 100 mg twice daily E. It is too late to treat for B. pertussis with antibiot-
for 5 days ics; administer immunoglobulin to provide passive
D. Amoxicillin 500 mg three times daily for 7 days immunity.

8. A 65-year-old woman with recently diagnosed difuse 10. A 57-year-old man from lower Delaware presents to
large B-cell lymphoma presents with malaise, nausea, an urgent care center after being bitten by a tick. He
and mild jaundice. She was diagnosed with difuse reports that he spent many hours in his garden over the
large B-cell lymphoma after developing massive right weekend and was bitten by many insects. He returned
cervical lymphadenopathy and daily fevers 7 months to his work as an accountant after the weekend and
ago. She was profoundly anemic when she presented has been mostly spending time inside since then. his
and required a blood transfusion at that time. Pretreat- morning, a Tuesday, he noticed an engorged tick at
ment work up revealed the following: HIV antibody/ his waist line, which he removed. At the moment he
antigen negative, hepatitis A IgG positive, hepatitis B feels well other than worrying about getting sick from
surface antigen negative, hepatitis B core IgG positive, this tick. He has had no fevers, rashes, joint pains, or
CHAPTER 1 Infectious Diseases 3

headache. He brought the removed tick with him, and team notes that this is the third patient with C. diicile
it appears to be an engorged deer tick. infection on their unit in the past 2 weeks.
he best management plan is: To help reduce further horizontal transmission, you
A. Ceftriaxone 250 mg intramuscular × 1 recommend:
B. Amoxicillin 500 mg by mouth three times a day A. Identiication and treatment of asymptomatic car-
for 14 days riers
C. Check Lyme disease serologies, and treat with B. Strict adherence to standard precautions, includ-
antibiotics if the serology is positive. ing hand hygiene with an alcohol hand rub
D. Doxycycline 100 mg by mouth twice a day for 14 C. Use of a chlorine-containing cleaning agent to
days address environmental contamination
E. Doxycycline 200 mg by mouth × 1 D. Contact precautions for any patient suspected of
having C. diicile infection until 48 hours of ther-
11. A 53-year-old woman with well-controlled non– apy have been given
insulin-dependent diabetes and obesity develops an E. Prophylactic metronidazole for all patients on the
area of swelling and pain on the right thigh. Over unit
about 3 days, the area becomes luctuant and she
develops a large area of erythema around it. She 14. A 67-year-old woman with end-stage renal disease is
presents to the emergency room, where she is found admitted from hemodialysis with hypoxia, fever, hypo-
to have a low-grade fever but is otherwise stable. tension, and cough. She is diagnosed with multifocal
he collection, which measures 2.5 cm in diameter, pneumonia. Her course is complicated by respiratory
is lanced, producing a small amount of purulent failure requiring ongoing mechanical ventilation. She is
material, which is drained and swabbed for culture. treated with levoloxacin, piperacillin-tazobactam, and
he culture of the wound later grows Staphylococcus vancomycin, and her respiratory status improves over
aureus. the irst 2 days of admission. Her medical history is
In addition to excellent wound care, which of the notable for type 2 diabetes, hypertension, and end-stage
following antibiotics would be the best choices for out- renal disease. She has been on hemodialysis for the last
patient treatment of this skin and soft tissue infection? 7 weeks through a tunneled catheter while a istula in
A. Levoloxacin the right upper extremity matures. On hospital day 4
B. Clarithromycin she redevelops daily fevers as high as 102.5°F and hemo-
C. Trimethoprim-sulfamethoxazole dynamic instability that persists for 2 days. On hospital
D. Penicillin day 6, blood cultures are reported as growing yeast.
E. Rifampin he best management in addition to repeating
blood cultures is:
12. A 26-year-old graduate student presents for evaluation A. Start intravenous luconazole 800 mg daily.
after being bitten on the right shin by her neighbor’s B. Remove the tunneled dialysis catheter, stop unnec-
playful Labrador puppy. Her past medical history is essary antibiotics, and observe.
notable for undergoing splenectomy for treatment C. Start intravenous isavuconazonium.
of idiopathic thrombocytopenic purpura at age 24, D. Start intravenous caspofungin.
which was curative. She takes no medications and has E. Start intravenous liposomal amphotericin.
no allergies. On examination she has normal vital signs
and appears well. On the right shin are two puncture 15. A 43-year-old woman calls to report that she is very
marks, which are still bleeding slightly. here is no concerned because she found a large lump under
purulence and no surrounding erythema of the skin. her left arm. She has no personal or family history
he most appropriate management is: of malignancy. She has no other complaints. On
A. Sequester the puppy and treat the patient with examination, her vital signs are normal. She has one
human rabies immunoglobulin. tender 2 × 3 cm mobile mass under her left arm,
B. Oral trimethoprim-sulfamethoxazole with overlying erythema. She works at an urban ani-
C. No therapy is necessary. mal shelter. At home she has two recently adopted
D. Intravenous ceftriaxone kittens, one of whom was ill about 2 months earlier.
E. Oral amoxicillin-clavulanate She tried to force-feed the kitten antibiotics and was
scratched repeatedly.
13. A 67-year-old female is recovering following an elective Among cat-associated zoonoses, the most likely
total hip arthroplasty. On postoperative day 5, she com- pathogen in this case is:
plains of worsening diarrhea and abdominal pain. Her A. Bartonella henselae
white blood cell count has risen from 9000 to 21,000 B. Toxocara cati
cells/µL. Testing of the stool reveals a positive glutamate C. Toxoplasma gondii
dehydrogenase antigen and a positive toxin A/B assay D. Yersinia pestis
conirming the diagnosis of Clostridium diicile. he E. Pasteurella multocida
4 C HA P T E R 1 Infectious Diseases

16. A 26-year-old woman presents for a new primary care of 527 mg/dL. Urinalysis is remarkable for ketones and
provider visit. She has no health complaints, and her glucose. CT of the sinuses reveals opaciication of the
medical history is notable only for appendectomy at left frontal, sphenoid, and ethmoid sinuses. Endoscopic
age 23. She immigrated to the United States from evaluation by an otolaryngologist reveals a black eschar
Vietnam 7 years ago. She recently inished her under- over the left middle turbinate.
graduate degree and will start a new job as a irst-grade In addition to treatment with insulin for diabetic
teacher in a few weeks. She is sexually active with her ketoacidosis, appropriate management includes:
boyfriend of 1 month and reports using condoms for A. Voriconazole
birth control. She has a history of occasionally smok- B. Lipid formulation of amphotericin B
ing marijuana but has not done so for over 3 years. C. Caspofungin
In addition to giving inluenza and tetanus/diphthe- D. Fluconazole
ria/acellular pertussis vaccines, screening for hepatitis B E. Cefepime
infection is ordered. Which detail in the patient’s history
indicates that she should be screened for hepatitis B? 19. An obese 57-year-old man with asthma who smokes
A. New sexual partner two packs per day of cigarettes is admitted with 3
B. Woman of childbearing age days of fever, nonproductive cough, and shortness of
C. History of marijuana use breath. Home medications include luticasone inhaler
D. Country of birth twice a day and albuterol inhaler as needed for wheez-
E. New profession as a teacher ing. Oxygen saturation is 94% on 2 L of oxygen by
nasal cannula. Chest x-ray shows dense multifocal
17. A 45-year-old woman was diagnosed with HIV infec- consolidations. He is admitted to the medical service
tion (224 CD4 T cells/µm3) and smear-negative, cul- and treated with levoloxacin. he following labora-
ture-positive pulmonary tuberculosis after presenting tory indings are sent:
with chronic cough. Chest CT showed left lower lobe Urine legionella antigen: positive
interstitial iniltrates. Rapid tuberculosis drug suscepti- Urine streptococcal antigen: negative
bility testing showed no evidence of drug resistance. he Expectorated sputum Gram stain: no polys, 1+ epi-
patient was started on isoniazid, rifampin, ethambutol, thelial cells
and pyrazinamide. After 2 weeks, the patient was started Expectorated sputum bacterial culture: 2+ oral lora,
on antiretroviral therapy with emtricitabine/tenofovir 1+ Candida albicans
and dolutegravir. Her cough initially improved but Based on these results the best treatment plan is:
then worsened after about 1 month of tuberculosis A. Stop levoloxacin and add micafungin.
treatment. She also developed progressive shortness of B. Stop levoloxacin; start azithromycin and mica-
breath. A chest x-ray showed new extensive right upper fungin.
lobe opacities. Sputum smear microscopy was negative. C. Continue levoloxacin and add luconazole.
he patient should now: D. Continue levoloxacin, add luconazole, and pur-
A. Switch to an empiric regimen for treatment of sue bronchoscopy.
multidrug-resistant tuberculosis. E. Continue levoloxacin and observe.
B. Initiate systemic corticosteroids to control symp-
toms of paradoxical tuberculosis immune recon- 20. A 25-year-old woman presents to the emergency
stitution inlammatory syndrome (IRIS). department with fever and back pain. he patient
C. Enroll in directly observed tuberculosis treatment has been using intravenous heroin for the past few
given the high likelihood of poor adherence to years; she had one prior episode of soft tissue abscess
therapy. after injection but no other illnesses in the past. She
D. Begin trimethoprim/sulfamethoxazole for treat- now complains of 2 weeks of fevers, sweats, muscle
ment of Pneumocystis jirovecii infection. aches, and some low back pain. On examination she
E. Proceed to open lung biopsy. is tachycardic, diaphoretic, febrile (102°F), and ill
appearing. Cardiac examination reveals a new sys-
18. A 45-year-old man presents to the emergency room with tolic murmur. Blood is drawn for basic laboratory
2 days of severe headache over the left eye and fevers. He indings and blood cultures (two sets). Given her ill
reports nearly constant tearing of the left eye and tender- appearance, the admitting physician decides to start
ness over the left eye for 24 hours. His medical history empiric antibiotics for the most likely pathogens
is notable for type 1 diabetes mellitus complicated by immediately.
peripheral neuropathy and chronic kidney disease. His he best empiric antibiotic regimen for this patient is:
examination is notable for an ill-appearing middle-aged A. Vancomycin + cefepime
man with chemosis of the left eye and periorbital and B. Vancomycin + gentamicin + rifampin
palpebral erythema extending over the left side of the C. Vancomycin + caspofungin
nose. Laboratory data are notable for a blood glucose D. Ampicillin + gentamicin
CHAPTER 1 Infectious Diseases 5

21. A 47-year-old male with quadriplegia secondary to in bilateral cervical and groin distribution. He
a motor vehicle accident as a young adult presents has a maculopapular rash over his chest and back.
with fever, fatigue, and foul-smelling urine. He has Laboratory data: WBC 2200/µL, hematocrit 30%,
a chronic indwelling Foley catheter due to urinary platelets 103,000/µL, blood urea nitrogen (BUN)
retention, and he reports multiple hospitalizations 20 mg/dL, creatinine 1.0 mg/dL, AST 66/µL, ALT
for catheter-associated urinary tract infections. On 72/µL, alkaline phosphatase 120/µL, total bilirubin
arrival to the emergency department, he is noted to 0.9 mg/dL. Blood cultures are drawn.
be confused with a temperature of 102.3°F, a respira- Which of the following is the most appropriate
tory rate of 23 breaths per minute, a heart rate of 116 panel of tests to order next?
beats per minute, and a blood pressure of 75/43 mm A. Hepatitis A, B, and C serologies
Hg. Urinalysis inds 3+ leukocyte esterase and positive B. Urine gonorrhea and chlamydia probes, syphilis
nitrite with microscopy revealing 100–200 WBC per serology
high-powered ield (HPF) with 3+ bacteria. Blood and C. Rheumatoid factor (RF), echocardiogram
urine cultures are obtained and laboratory indings are D. Blood smear, lactate dehydrogenase (LDH), bone
pending. marrow biopsy
In addition to administering broad-spectrum anti- E. Blood smear, Epstein-Barr virus (EBV) serologies,
biotics, the next best step is: CMV serologies, HIV 1/2 antigen/antibody, HIV
A. Initiation of dopamine as a vasopressor RNA
B. Administration of intravenous hydrocortisone
C. Administration of 30 mL/kg of crystalloid 24. A 48-year-old male presents to the emergency
D. Initiation of norepinephrine as a vasopressor department complaining of shortness of breath wors-
ening over the past 2 days and weakness. Admission
22. A 23-year-old woman with acute myeloid leukemia chest x-ray reveals pulmonary edema, and laboratory
(AML) who underwent standard induction chemo- indings are notable for potassium 6.8 mmol/L. On
therapy with cytarabine and daunorubicin 11 days review of his past medical history, you learn that he
ago has recurrent fever to 103.1°F associated with mal- has severe type 1 diabetes mellitus complicated by
aise and sweats. She also endorses a sore mouth and nephropathy for which he has been on peritoneal
throat as well as mild diarrhea, which she attributes to dialysis for the past year. He reports 100% compli-
her recent chemotherapy. Her laboratory indings are ance with his peritoneal dialysis regimen at home.
notable for an absolute neutrophil count of 28. She Two months ago he was admitted for methicillin-
irst developed fever and neutropenia 8 days ago and resistant Staphylococcus aureus (MRSA) bacteremia
was treated with empiric cefepime. Her fevers resolved related to an infected foot ulcer for which he was
within 24 hours, and she remained afebrile until 2 treated with a course of intravenous vancomycin. He
hours ago. When fevers redeveloped, blood cultures is admitted for acute management of hyperkalemia
were drawn. and to transition to hemodialysis. On hospital day 4,
Which is the most appropriate antimicrobial agent one day after surgery to create an arteriovenous is-
to add? tula, he reports an increasingly productive cough and
A. Fluconazole spikes a fever to 101.1°F. A repeat chest x-ray reveals
B. Vancomycin a new consolidative opacity in the right lung.
C. Caspofungin Based on clinical and radiologic evidence suggest-
D. Meropenem ing pneumonia, you recommend:
E. Daptomycin A. Levoloxacin
B. Ertapenem + vancomycin
23. A 22-year-old man presents to his primary care C. Piperacillin/tazobactam + amikacin
physician’s office with fever and sore throat. The D. Imipenem + ciproloxacin
patient was feeling well until 1 week ago when he E. Cefepime + levoloxacin + vancomycin
developed fever, malaise, fatigue, and sore throat;
he later developed diarrhea. He has lost 5 pounds 25. A 21-year-old college student with moderate to
in the past month. The patient is a student at a local severe asthma presents to the student health cen-
college, he does not smoke, he drinks alcohol a few ter with a sore mouth for several days. hough it
times per week, and he does not use any recreational is bothersome, it has not interfered with his eating
drugs. He is sexually active with men and has had or drinking. He denies odynophagia. Ten days ago
two new male partners over the past few months. he was hospitalized for 2 days with a severe asthma
He is up to date on all of his vaccinations. On lare. He was treated with a 2-week prednisone taper
examination he is thin, febrile, and mildly tachy- for his asthma and with a 5-day course of azithro-
cardic. His oropharynx is erythematous without mycin for possible respiratory tract infection. His
exudates; he has palpable small lymphadenopathy regular medications include inhaled salmeterol and
6 C HA P T E R 1 Infectious Diseases

luticasone. On examination he is a relatively well- Based on identiication of bacteria grown in blood


appearing young man with white curd-like plaques culture, the suspicion for endocarditis diminishes
adherent to the soft and hard palate. signiicantly.
In addition to education about rinsing the mouth Which of the following bacteria grew in culture?
after using a steroid inhaler and considering HIV test- A. Haemophilus parainluenzae
ing, the next best step is: B. Eikenella corrodens
A. Clotrimazole troches C. Streptococcus gallolyticus
B. Chlorhexidine mouthwash D. Bacteroides fragilis
C. Referral to oral medicine for biopsy E. Enterococcus faecalis
D. Oral posaconazole
E. Single-dose intravenous micafungin 29. A 28-year-old woman presents to the emergency
department with fever, malaise, abdominal pain, nau-
26. A 26-year-old female is hospitalized in a burn unit sea, and vomiting. Initial laboratory indings show
after sufering deep burns over 55% of her body in WBC 1800/µL, hematocrit 27%, platelets 100,000/µL,
a house ire. She remains intubated with access via AST 170/UL, ALT 195/UL, alkaline phosphatase 422
central venous catheter. On hospital day 10, the IU/L, and total bilirubin 2.1 mg/dL. A CT scan of the
microbiology lab calls with the result of a blood cul- abdomen and pelvis reveals difuse lymphadenopathy
ture that was sent in the setting of a fever to 102°F. and hepatosplenomegaly. Further diagnostics reveal
he lab reports that Acinetobacter baumannii is that HIV antibody is positive with a CD4 count of
growing in the blood, which is testing positive for 18/µL (4% of total lymphocytes). Biopsy of a lymph
extended-spectrum beta-lactamases and a carbapen- node reveals numerous organisms on acid-fast stain-
emase. ing. PCR of the sample with numerous organisms is
Based on these results, which antibiotic is most consistent with mycobacterium avium intracellulare (or
likely to be efective against this pathogen? mycobacterium avium complex [MAC]). he patient is
A. Meropenem started on treatment with clarithromycin and ethambu-
B. Colistimethate tol while awaiting the results of resistance testing from
C. Ciproloxacin mycobacterial isolator blood cultures.
D. Piperacillin/tazobactam When is the best time to start antiretroviral therapy
E. Gentamicin for this patient?
A. Start three antiretroviral drugs in a staggered fash-
27. A 37-year-old woman with HIV (CD4 count 523/µL, ion, adding one every 2 weeks over the next 6
HIV viral load <20/µL), asthma, and allergic rhinitis weeks.
presents to her primary care physician oice com- B. A few days after initiation of clarithromycin and
plaining of worsening symptoms of allergic rhinitis. ethambutol, but within 2 weeks
She has had worsening rhinorrhea, itchy and watery C. After 2 weeks on clarithromycin and ethambutol,
eyes, and dry cough over the past few weeks this if no side efects
spring. Her current medications are tenofovir, emtric- D. After completion of 6 weeks of clarithromycin and
itabine, darunavir, ritonavir, loratadine, and inject- ethambutol, to decrease risk of immune reconsti-
able medroxyprogesterone. She requests an additional tution inlammatory syndrome (IRIS)
medication for control of her allergic rhinitis symp- E. After the patient is discharged from the hospital,
toms. in the outpatient setting with documented patient
Which of the following drugs should not be pre- capacity for adherence to follow-up and medica-
scribed because of a potentially harmful drug–drug tions
interaction?
A. Oral cetirizine 30. A 45-year-old man with a long-standing history of well-
B. Inhaled albuterol controlled HIV presents to his primary care physician
C. Ophthalmic nedocromil for a routine visit. He is feeling well, with no symp-
D. Inhaled luticasone tomatic complaints. He reports 100% adherence to his
E. Oral montelukast antiretroviral regimen: tenofovir, emtricitabine, and ril-
pivirine. He works in real estate and lives with his hus-
28. A 48-year-old man presents with 4 days of fevers, band and their dog. He smokes approximately one pack
night sweats, and malaise. He has a history of bicus- of cigarettes per day, as he has for the past 20 years. He
pid aortic valve with aortic regurgitation. On exami- drinks six alcoholic beverages per week, and he does not
nation he is febrile (102.4°F) and tachycardic. Blood use recreational drugs. His family history is notable for
cultures are drawn. A transthoracic echocardiogram coronary artery disease in both parents but no cancer in
does not show any valvular vegetations or abscess. irst-degree relatives. Vital signs: heart rate 82 beats per
CHAPTER 1 Infectious Diseases 7

minute, blood pressure 137/86 mm Hg, BMI 32 kg/ and fatigue. he patient has a history of childhood
m2, and physical examination is unremarkable. Labora- asthma, but she has not used inhalers or other medi-
tory indings show CD4 count 470/µL, HIV viral load cations for asthma in several years. She is otherwise
is <20/µL (undetectable), and complete blood count healthy, and she takes no medications. She has a his-
(CBC) and chem-20 are normal. You plan to continue tory of a rash reaction to clarithromycin. On exami-
his current antiretroviral regimen, and you discuss addi- nation, her temperature is 99.0°F, heart rate is 92
tional health care maintenance eforts with him. beats per minute, blood pressure is 132/85 mm Hg,
Given his history, which of the following is the respiratory rate is 18 breaths per minute, and oxy-
most important health care maintenance item to pur- gen saturation is 96% on room air. She appears
sue during this visit? mildly uncomfortable and is coughing during the
A. Start sulfamethoxazole/trimethoprim for prophy- examination; she has no lesions in the oropharynx,
laxis against Pneumocystis pneumonia. sclerae are clear, maxillary sinuses are mildly tender
B. Counsel patient to decrease alcohol intake, and on percussion, there is no nasal discharge/drainage,
refer for alcohol dependence treatment. tympanic membranes are clear, and she has a few
C. Counsel patient to quit smoking, and discuss small (<1.5 cm) palpable cervical lymph nodes. Her
medications and supports for smoking cessation. lungs are clear except for occasional faint expiratory
D. Screen for toxoplasma serostatus to determine risk wheezes.
for toxoplasma reactivation in the future. he most appropriate management is:
E. Refer for early colon cancer screening by colonos- A. Oral moxiloxacin, inhaled albuterol, and intrana-
copy. sal oxymetazoline
B. Oral amoxicillin-clavulanate, inhaled luticasone,
31. An 84-year-old man with a history of coronary artery and saline nasal irrigation
disease, diabetes mellitus, and chronic renal insui- C. Oral dextromethorphan, inhaled albuterol, and
ciency presents to the emergency room with progres- intranasal ipratropium
sive headache, fevers, and neck pain. he patient was D. Intramuscular ceftriaxone, oral pseudoephedrine,
in his usual state of health until 4 days prior when the and inhaled salmeterol
staf and his friends at his assisted living facility noted E. Intramuscular inluenza vaccine, oral guaifenesin,
he started complaining of feeling ill with headache and and inhaled tiotropium
nausea. On the morning of presentation he was found
in his room confused. In the emergency room he was 33. A 29-year-old woman who is otherwise healthy pres-
febrile and confused, and became somnolent during ents for a routine prenatal visit at 14 weeks’ gestational
his care there. Head CT showed no acute processes. A age. She is feeling well and has no symptomatic com-
lumbar puncture was performed, cerebrospinal luid plaints, and physical examination is consistent with
(CSF) examination showed glucose 22 mg/dL (serum normal pregnancy, otherwise unremarkable. She has
glucose 112 mg/dL), protein 97 mg/dL, red blood routine prenatal laboratory indings checked, which
cell (RBC) 7, WBC 489 with 87% neutrophils, 7% show the following results: hemoglobin 11.2 g/dL,
monocytes, and 6% lymphocytes. CSF is sent to the rubella IgG positive, HIV-1/2 antigen/antibody nega-
microbiology lab for Gram stain and culture. tive, treponemal IgG (by enzyme immunoassay [EIA])
Which of the following is the most appropriate positive. Follow-up rapid plasma reagin (RPR) is also
empiric antibiotic regimen to initiate while awaiting positive, with a titer of 1:16, and FTA-ABS is also pos-
the results of the CSF Gram stain and culture? itive. he patient has never had prior syphilis testing.
A. Vancomycin, ceftriaxone, ampicillin She reports a history of severe allergy to penicillin with
B. Vancomycin, cefepime, acyclovir a “feeling of throat closing.”
C. Vancomycin, meropenem he most appropriate management for this patient is:
D. Ampicillin, gentamicin, acyclovir A. No treatment now due to risk of toxicity; follow
E. Ampicillin, trimethoprim-sulfamethoxazole, amp- clinically and repeat syphilis testing at 20 weeks’
hotericin B gestational age
B. Treat with doxycycline 100 mg orally twice daily
32. A 27-year-old woman presents to her primary care for a 21-day course.
physician complaining of fever, cough, sinus pres- C. Treat with ceftriaxone 1 g IM once daily for 10
sure, and malaise. She reports onset of symptoms 5 days.
days ago, with fevers for the irst 2 days with tem- D. Treat with azithromycin 2 g orally in a single dose.
peratures as high as 101.1°F. Highest temperature in E. Allergy consultation and admission for desensiti-
the past 3 days has been 99.8°F. She reports ongo- zation to penicillin in order to facilitate treatment
ing symptoms of frequent cough productive of scant with benzathine penicillin G 2.4 million units IM
white sputum, nasal congestion, mild sinus pressure, once weekly for 3 weeks
8 C HA P T E R 1 Infectious Diseases

34. A 24-year-old man with well-controlled HIV presents and on sinonasal examination he has septal perfo-
for routine follow-up primary care visit and notes some ration with no obvious exudates or other abnor-
dysuria for the past several days. Physical examination malities. Blood tests show white blood cell count of
is unremarkable. Urinalysis shows 10 WBC, no epi- 4200/µL, hemoglobin 9.1 g/dL, and creatinine 2.2
thelial cells, 2 RBC, no bacteria per HPF. Urine testing mg/dL. Urinalysis shows too numerous to count red
for gonorrhea is positive by nucleic acid ampliication blood cells. Chest x-ray shows some abnormal opac-
test (NAAT) probe, and urine chlamydia NAAT probe ities, so chest CT is obtained, which shows multiple
is negative. he patient has no known drug allergies. pulmonary nodules.
Which of the following is the recommended treat- he test most likely to suggest the diagnosis in this
ment regimen? case is:
A. Levoloxacin 500 mg orally once daily for 7 days + A. Sputum smear for acid-fast bacilli (AFB)
doxycycline 100 mg orally twice daily for 7 days B. AFB smear of biopsy of a pulmonary nodule
B. Ceftriaxone 250 mg IM single dose C. Serum test for antineutrophilic cytoplasmic anti-
C. Ceixime 400 mg orally single dose + doxycycline bodies (ANCA)
100 mg orally twice daily for 7 days D. Serum test for galactomannan
D. Ceftriaxone 250 mg IM single dose + azithromy- E. Serum interferon-gamma release assay (IGRA)
cin 1 g orally single dose
37. A 62-year-old woman with multiple sclerosis and a
35. A 32-year-old man presents to his primary care phy- neurogenic bladder now has a chronic indwelling uri-
sician complaining of painful perianal lesions. he nary catheter, after failing management with intermit-
patient is sexually active with multiple male partners. tent use of urinary straight catheters. She presents for
He has had negative screening for sexually transmitted a routine primary care visit and has no current symp-
diseases, including HIV, gonorrhea, and chlamydia, in tomatic complaints. She is interested in discussing
the past (his last screening tests were 6 months ago). strategies to decrease the risk of urinary tract infec-
On physical examination he is overall well appearing, tions in the future. In the past when she has developed
but in the perianal area he has multiple shallow ulcer- urinary tract infections they often precipitated a wors-
ations grouped in the right perianal area. here is no ening of her multiple sclerosis, and she often requires
rectal discharge and no palpable lymphadenopathy, hospitalization, so she hopes to prevent the need for
but the ulcerations are tender and painful even when hospitalization in the future.
not palpated. Which of the following strategies would be most
he type of diagnostic test that is most likely to successful at achieving her goals?
conirm the diagnosis of this active condition is: A. Monitoring for early signs and symptoms of uri-
A. Bacterial culture of a swab of the ulcers nary tract infection, with expedited early urinaly-
B. Viral culture of a swab of the ulcers sis, urine culture, and empiric treatment while
C. Urine NAAT probe awaiting culture results when symptoms develop
D. Blood serologic test B. Routine screening with urinalysis for pyuria at
E. Urinary antigen test regular intervals with early empiric treatment for
urinary tract infection if pyuria is detected, even in
36. A 47-year-old man presents to the emergency depart- the absence of symptoms
ment with complaint of hemoptysis. He was born C. Addition of gentamicin solution to the catheter
and raised in upstate New York, currently manages drainage bag at regular intervals
a restaurant and bar that he and his siblings own, D. Chronic prophylaxis with methenamine salts to
and is married with two children. His only travel decrease bacteria
in the last few years was a trip to Montreal. He has E. Chronic prophylaxis with ciproloxacin to decrease
been healthy until approximately 6 months prior bacteria and infections
when he developed sinusitis. He has been treated by
his primary care physician with courses of amoxi- 38. A 45-year-old man with a history of prior open reduc-
cillin, amoxicillin-clavulanate, and moxiloxacin tion and internal ixation (ORIF) of a left femur frac-
for episodes of sinusitis over the past few months, ture in the past now presents with his third episode
but his symptoms persist. He has also had several of cellulitis in the left leg. He was well until 1 day
episodes of epistaxis in the last few months. Over prior when he developed sudden onset of malaise,
the past 2 weeks he developed a cough, which was fever, nausea, and erythema and pain in the left leg.
initially nonproductive, but during the past 2 days He presented to the emergency room overnight and
he had a few episodes of hemoptysis. On physical was treated empirically with vancomycin overnight.
examination he has lost 5 lb since his last exami- He improves gradually overnight. In discussion the
nation 1 month prior, he is thin but comfortable, following day he asks whether there are any strategies
CHAPTER 1 Infectious Diseases 9

to decrease the frequency of his episodes of cellulitis in and takes no medications. She lives in New Hamp-
the future. shire and spends time walking her dog in the woods
Which of the following antibiotics, when taken regu- frequently, but she does not remember any speciic
larly as prophylaxis, has been shown to decrease the inci- tick bites. Approximately 10 days earlier she developed
dence of cellulitis among patients with recurrent cellulitis? upper respiratory infection (URI) symptoms, which
A. Sulfamethoxazole-trimethoprim improved over a few days and then resolved after 7
B. Doxycycline days. Over the past 3–4 days she developed right ear
C. Clarithromycin pain. On physical examination she is noted to have a
D. Levoloxacin right facial droop and some lesions with serous drain-
E. Penicillin age in the right external ear canal. Her mucous mem-
branes are dry.
39. A 32-year-old woman presents for routine prenatal Which of the following is the most appropriate
care at 12 weeks’ gestational age. She is taking prenatal treatment for this patient?
vitamins and is feeling well. She reports that a friend A. Doxycycline 100 mg orally twice daily for 14 days
recently gave birth to a daughter who was diagnosed B. Valacyclovir 1000 mg orally three times daily for
with congenital toxoplasmosis, and she would like to 14 days
know how to prevent this infection during her preg- C. Ciproloxacin otic solution to the right ear 4 times
nancy. She is a kindergarten teacher. She lives with her per day for 7 days
spouse, who cares for their 10-year-old indoor cat. She D. Ciproloxacin otic solution + amoxicillin-clavula-
likes to garden in her free time. nate 875/125 mg orally twice daily for 10 days
In addition to washing fruits and vegetables before E. Prednisone 60 mg orally once daily for 5 days
eating, which of the following lifestyle changes is rec-
ommended to reduce risk for acute toxoplasma infec- 42. A 26-year-old man presents with dysuria, which has
tion during pregnancy? been persistent for more than 1 week. Urinalysis shows
A. Cook meat to “well done.” 12 WBC, 1 RBC, and no bacteria per HPF, and the
B. Avoid ingestion of pork or any pork products. urine culture is negative. Urine NAAT probes for chla-
C. Give the cat up for adoption. mydia and gonorrhea are negative. Serum testing for
D. Stop gardening. HIV 1/2 antigen/antibodies is negative.
E. Take leave from work at the start of the third tri- Which of the following organism is the most likely
mester to avoid transmission from her students. cause of the patient’s symptoms?
A. Trichomonas vaginalis
40. A 74-year-old man with poorly controlled diabetes, cor- B. Herpes simplex virus (HSV)
onary artery disease, end-stage renal disease, and periph- C. Mycoplasma genitalium
eral vascular disease presents with pain at a chronic foot D. Haemophilus ducreyi
ulcer site. he patient has had a nonhealing ulcer on his E. E. coli
left great toe for several months but no other symptoms.
Over the past few days he developed purulent drain- 43. A 71-year-old woman with hypertension develops tem-
age from the ulcer bed, as well as erythema, pain, and poral headaches, fevers, and weight loss. She is diagnosed
swelling of the toe, which is now tracking up the foot. with giant cell arteritis by temporal artery biopsy and
he margins of the toe ulcer have also started to turn starts treatment with prednisone 50 mg per day. Her
black. He has a fever of 100.7°F at the time of presen- symptoms improve markedly, and the prednisone dose
tation. Laboratory indings reveal a white blood cell is slowly tapered starting 3 weeks later. Ten weeks after
count of 13,500 cells/µL. Blood cultures are sent, and starting the prednisone taper she develops fevers in the
debridement—during which cultures of the base will be 100–101°F range despite taking 20 mg of prednisone
obtained—is planned for later in the day. per day. She also notes a dry cough. She is treated with a
Which of the following antibiotic regimens would course of azithromycin but continues to have a cough and
be appropriate initial therapy for this patient while fevers. She also notes dyspnea on exertion. She presents to
awaiting debridement and culture results? the emergency department, where she is found to have a
A. Vancomycin temperature of 100.6°F, respiratory rate of 38 breaths per
B. Cefazolin and metronidazole minute, and oxygen saturation of 92% while breathing
C. Ampicillin-sulbactam ambient air. On exam she has no jugular venous disten-
D. Vancomycin and piperacillin-tazobactam tion or peripheral edema. She has faint bilateral crackles
E. Ertapenem in both lungs. A chest x-ray shows bilateral interstitial
iniltrates. Serum β-d-glucan is >500 pg/mL.
41. A 67-year-old woman presents to the emergency room In addition to treatment with antibiotics for com-
with right ear pain. She is otherwise healthy at baseline munity-acquired pneumonia, which of the following
10 C HA P T E R 1 Infectious Diseases

treatments is most appropriate additional empirical Staphylococcus aureus (MRSA). he patient is treated
therapy? with vancomycin while an inpatient, and his fevers
A. Ivermectin resolve. He is then discharged with oral linezolid for
B. Trimethoprim-sulfamethoxazole another 10 days. After 7 days he returns to the emer-
C. High-dose steroids gency room complaining of recurrent fever and mal-
D. Furosemide aise. His temperature is 103.7°F, heart rate is 116 beats
E. heophylline per minute, blood pressure is 180/92 mm Hg, and
oxygen saturation is 98% on room air. On examina-
44. A 22-year-old man presents to primary care physi- tion he is somewhat agitated and unable to sit still.
cian for routine follow-up. He is sexually active with Lungs are clear to auscultation bilaterally, there are no
men, with three partners in the past year. He had a murmurs on cardiac exam, abdomen is soft and non-
recent urgent care visit for urethritis, was diagnosed tender, and the lower left leg prior incision and drain-
with gonorrhea, and was treated with ceftriaxone and age site is healing.
azithromycin. He is now feeling well, with no cur- he most likely cause of this patient’s new symp-
rent symptomatic complaints. he physician focuses toms is:
this routine visit on sexually transmitted infections A. Drug–drug interaction
(STIs), including risk-reduction counseling, conirm- B. Recurrent MRSA abscess
ing completion of HPV-vaccine series, distribution of C. MRSA bacteremia due to endocarditis
condoms, and discussion of preexposure prophylaxis D. Hospital-acquired pneumonia
(PrEP) for HIV prevention. he patient is interested E. Clostridium diicile colitis
in PrEP and inquires about the usual treatment plan
and the risks and beneits associated with PrEP. 46. A 53-year-old man with a prior history of idiopathic
Which of the following statements about antiretro- thrombocytopenic purpura treated with a course of
viral PrEP is true? steroids, rituximab, and ultimately splenectomy 4
A. HIV screening with combination antigen/anti- months ago now presents with fever and chills. he
body test should be performed at baseline before patient spent a week on Nantucket for a summer vaca-
initiation of PrEP and annually while on PrEP. tion and was feeling well until 2 days after he returned
B. Taking tenofovir/emtricitabine once daily with from vacation, when he developed fevers as high as
excellent adherence for PrEP can decrease risk of 103.2°F, shaking chills, and headache. He presented
HIV acquisition by about 25%. to a local hospital, where initial complete blood count
C. Patients taking PrEP should have regular screening showed WBC 12,000/µL, hemoglobin 8.2 g/dL,
for other STIs at least every 6 months, including platelets 80,000/µL, normal electrolytes, serum creati-
syphilis, gonorrhea, and chlamydia. nine 2.1 mg/dL, and alkaline phosphatase 206 IU/L.
D. In the HIV uninfected population there is little A blood smear showed parasites within the red blood
risk for tenofovir nephrotoxicity while taking cells; the parasitemia burden was assessed as 12%.
PrEP, so there is no need for monitoring of renal Which of the following is the most appropriate
function. treatment regimen for this patient at this point?
E. In most populations the risks of PrEP outweigh A. Azithromycin and atovaquone/proguanil
the beneits because patients who are prescribed B. Quinine and clindamycin and consideration of
PrEP have increased sexual risk behaviors and red cell exchange transfusion
increased incidence of STIs, including HIV when C. Ceftriaxone and doxycycline
nonadherent to PrEP. D. High-dose corticosteroids, intravenous immuno-
globulin, and initiation of plasmapheresis
45. A 55-year-old man with a history of hypertension, E. Intravenous artesunate
coronary artery disease, and depression presents with
a soft tissue infection on the leg. He has no prior his- 47. A 36-year-old man with psoriasis, for which he takes
tory of soft tissue infections. His current medications methotrexate, traveled to Arizona for 2 weeks for a
are lisinopril, clopidogrel, aspirin, atorvastatin, and family reunion and developed a fever 2 days before
duloxetine. He has no known medication allergies. On returning to his home in New York. He was feeling
examination, he has a fever (temperature is 101.2°F) well during the trip and enjoyed the irst week of the
but vital signs are otherwise normal. here is an area reunion. He bunked with extended family, including
of erythema on the lower left leg, originating from a small children and two dogs. He ate food at the hotel
punctate wound where the patient states he sustained a and drank primarily bottled water. He went hiking in
spider bite. he area is warm, swollen, and tender, and the desert on three occasions. He swam in the hotel
there is a central 4-cm area of luctuance. Incision and pool but engaged in no fresh water swimming. he
drainage of the abscess is performed at the bedside. weather was dry throughout the trip and very windy at
A culture of drained pus grows methicillin-resistant times. Two days before returning home he developed
CHAPTER 1 Infectious Diseases 11

fever, fatigue, malaise, dry cough, and chest pain. He 49. A 53-year-old woman who is otherwise healthy devel-
took acetaminophen and rested, then returned home ops fever, headache, malaise, and then cough, which
to New York. After 10 days the symptoms had not is persistent and worsens over a couple of days. he
signiicantly improved, so he presented to his primary patient lives in Missouri, where she works on a dairy
care provider. Physical examination was remarkable farm and also tends sheep for wool as an additional
only for low-grade fever. Chest x-ray showed subtle source of income. She spends her spare time hunting
left hilar iniltrate and lymphadenopathy. He was deer and rabbits. She is married and is monogamous
treated with a 5-day course of azithromycin with no with her husband. At the time of presentation she was
change in his symptoms. mildly hypoxic and febrile, and her condition rapidly
he best diagnostic test to send at this point would be: worsened, ultimately requiring mechanical ventila-
A. Blood serologic test for Coccidioides tion. Chest imaging showed multifocal iniltrates and
B. Serum 1,3-β-d-glucan testing progressive pleural efusions, as well as hilar lymph-
C. Urinary legionella antigen adenopathy. At the time of admission her laboratory
D. Biopsy of the hilar lymph node for fungal culture indings were normal with the exception of WBC
E. Serum galactomannan 14,000/µL. However, she developed progressive renal
failure and abnormal liver function tests over the irst
48. A 31-year-old man with a prosthetic aortic valve 2 days after admission. Blood, urine, and sputum cul-
presents with fevers. He had his aortic valve replaced tures are all negative repeatedly. She has been treated
with a mechanical prosthesis at age 24 due to con- with vancomycin, cefepime, and metronidazole with
genital bicuspid aortic valve, and he has been doing no improvement.
well since then. His medications include warfarin he most likely etiologic organism for her current
and lisinopril. He has no known drug allergies. At condition is:
the time of admission to the hospital, blood cultures A. Tropheryma whipplei
are positive for MRSA. After extensive evaluation, no B. Babesia microti
source for the bacteremia is identiied. He is treated C. Borrelia lonestari
for presumed prosthetic valve endocarditis, even in D. Francisella tularensis
the absence of suggestive indings on transesophageal E. Anaplasma phagocytophilum
echocardiogram. His antibiotic regimen is vancomy-
cin, gentamicin, and rifampin. After 3 days the blood 50. A 22-year-old woman who is healthy at baseline sus-
cultures clear of bacteria, he clinically improves, and tains minor blunt trauma to the right thigh after she
he is eventually discharged to a rehab facility to bumps her leg on the edge of a table. Within hours
complete a course of vancomycin, gentamicin, and after the bump she develops fever and severe right leg
rifampin. After 3 weeks he returns to the emergency pain such that she is barely able to walk. She presents
room with an acute stroke, which appears embolic on to a local emergency room, where she is febrile and
MRI/MRA of the brain. Laboratory indings show hypotensive. Her right thigh appears dusky, and she
WBC 12,000/µL, hemoglobin 9.8 g/dL, platelets complains of pain tracking down to the foot and up
167,000/µL, creatinine 0.9 mg/dL, liver function to the lower abdomen. She is taken immediately to the
tests are normal, partial thromboplastin time 36.0 operating room, where operative exploration reveals
seconds, and INR 1.2. Blood cultures are negative at necrotizing myositis of the muscles of the thigh with-
48 hours of incubation. out gas formation, with necrotizing soft tissue infec-
he most likely cause of the patient’s new stroke is: tion tracking down the leg and up to the groin. Some
A. Persistent infectious vegetation on the prosthetic of the debrided tissue is sent for Gram stain and cul-
aortic valve ture to aid with choice of antibiotics.
B. Toxicity from gentamicin he most likely pathogen is:
C. Aortic valvular dysfunction due to perivalvular A. Methicillin-sensitive Staphylococcus aureus
abscess B. E. coli
D. Hypercoagulable state due to loss of gut lora while C. Group A streptococci
on antibiotics D. Methicillin-resistant S. aureus
E. Drug–drug interaction of warfarin with rifampin E. Aeromonas hydrophila

Chapter 1 Answers
1. ANSWER: E. Dengue travelers returning with fever should be evaluated for
Leptospirosis can present in many diferent ways, malaria (even in cases where they had reported tak-
including headache, muscle aches, and fever, but it ing prophylaxis), the normal hematocrit and lack of
is almost always associated with freshwater exposure, other laboratory abnormalities in this young woman
such as swimming or white-water rafting. Although all are reassuring. Typhoid can also present with only
12 C HA P T E R 1 Infectious Diseases

headache and fever, and in fact although it is caused that result directly from surgery (e.g., wound infection).
by Salmonella species. it can often cause little to no hose transplanted 1–6 months before or those recently
gastrointestinal symptoms. he marked thrombocyto- treated for rejection are at risk for opportunistic infec-
penia would be unusual for this diagnosis. Hepatitis tions (such as CMV enteritis). Opportunists like CMV
A is also a risk for travelers, but the lack of gastroin- are much less likely in those transplanted more than 6
testinal symptoms and lack of liver enzyme elevation months ago and not treated for rejection recently (as
argue against it. Dengue is the second most common in this case). Patients more than 6 months after trans-
cause of systemic febrile illness travelers returning to plant who have not been treated for recent rejection are
the United States from foreign travel (12%), after at highest risk for community-acquired pathogens, like
malaria, and dengue was the most frequently identi- Salmonella and norovirus. his patient has been in con-
ied cause of systemic febrile illness among travelers tact with a hamster, which may increase her risk for Sal-
returning from Southeast Asia. Diagnosis is generally monella infection (hamsters have been associated with
clinical, although acute and convalescent sera can be Salmonella outbreaks). Although she may also be at risk
sent. Treatment is supportive. for norovirus infection as a school teacher, Salmonella
Hagmann SH, Han PV, Staufer WM, et al. Travel-associated dis- is much more likely in this case based on her history of
ease among US residents visiting US GeoSentinel clinics after high fevers and abdominal pain along with diarrhea in
return from international travel. Fam Pract. 2014;31:678–687. the absence of nausea or vomiting. Among medications
used for posttransplant immunosuppression, mycophe-
2. ANSWER: B. Suctioning of subglottic secretions nolate mofetil commonly causes diarrhea; however, the
here are a number of strategies that can help reduce new onset after years of mycophenolate and the associ-
the risk of ventilator-associated pneumonia in an intu- ated fevers all make mycophenolate an unlikely cause of
bated patient. hese usually focus around minimizing this acute illness.
sedation, preventing aspiration, reducing colonization Fishman JA, Rubin, RH. Infection in organ-transplant recipi-
of the airway and digestive tract, and minimizing con- ents. N Engl J Med. 1998;338:1741–1751; Centers for Dis-
tamination of the ventilatory circuit. To prevent aspira- ease Control and Prevention. Outbreak of multidrug-resistant
tion, the head of the bead should be maintained in a Salmonella typhimurium associated with rodents purchased at
semirecumbent position (elevated 30–45 degrees) rather retail pet stores—United States, December 2003-October
than in a fully recumbent position. One should also use 2004. MMWR Morb Mortal Wkly Rep. 2005;54:429–433.
a cufed endotracheal tube with cuf pressure set at ≥20
cm H2O with subglottic secretion drainage. To reduce 4. ANSWER: B. Pneumococcal 13-valent conjugate
colonization of the airway and digestive tract, orotra- (PCV13) vaccine
cheal intubation is preferred to nasotracheal intubation his patient is a 24-year-old man with ulcerative
to decrease the risk of sinusitis, acid-reducing medica- colitis for which he takes immunosuppressive medi-
tions such a histamine receptor 2 (H2)–blocking agents cations. Based on his immunocompromised state he
and proton pump inhibitors should be avoided except should be vaccinated for pneumococcus with both
in patients at high risk for ulcers/gastritis, and regular the conjugated and the polysaccharide vaccines. In
oral care with an antiseptic solution (e.g., chlorhexi- an immunocompromised patient under 65 years old
dine) should be performed. Routine use of oral or intra- who has never been vaccinated for pneumococcus,
venous antibiotics for prophylaxis is not recommended. the conjugate vaccine should be given irst, followed
Finally, to minimize contamination of the ventilatory by the polysaccharide vaccine (Option D) at least 8
circuit, the tubing should only be changed when visibly weeks later. Although the patient may be due for teta-
soiled or malfunctioning. nus vaccination, unless he has a clear history that he
Klompas M, Branson R, Eichenwald EC, et al. Strategies to pre- previously received the tetanus, diphtheria, acellular
vent ventilator-associated pneumonia in acute care hospitals: pertussis (TDaP) vaccine, the TDaP vaccine should
2014 update. Infect Control Hosp Epidemiol. 2014;35:915– be given now, not the tetanus, diphtheria vaccine
936; American horacic Society; Infectious Diseases Society of (Option A). he Advisory Committee on Immuni-
America. Guidelines for the management of adults with hos- zation Practices (ACIP) recommends that all persons
pital-acquired, ventilator-associated, and healthcare-associated above age 10 should be vaccinated with TDaP once
pneumonia. Am J Respir Crit Care Med. 2005;171:388–416.
due to outbreaks of pertussis in adolescents and adults
related to waning pertussis immunity. his patient
3. ANSWER: A. Salmonellosis has no speciic indications for Haemophilus inluen-
Infection risk after solid organ transplantation zae B vaccine at this time (Option C); after standard
depends on two key factors: when the transplant occurred childhood vaccination this vaccine is typically only
and whether the patient has recently been treated for indicated in adults in speciic circumstances such as
organ rejection. Transplant recipients who underwent stem cell transplantation or splenectomy. he menin-
transplantation less than a month ago are typically at gococcal B vaccine (Option E, available in two dif-
highest risk for nosocomial infections and infections ferent formulations with diferent schedules) was irst
CHAPTER 1 Infectious Diseases 13

approved in 2014. It is currently recommended for a 7. ANSWER: C. Nitrofurantoin macrocrystals 100


select population including individuals with asplenia, mg twice daily for 5 days
those with acquired or congenital complement dei- In 2010 the Infectious Diseases Society of America
ciency, or those who were potentially exposed in an (IDSA) updated guidelines for the management of
outbreak of meningococcus type B infection. Unlike uncomplicated cystitis in premenopausal women. By
the conjugate meningococcal ACWY vaccine, it is not far the most common cause of uncomplicated cystitis
routinely recommended for young adults between the is E. coli. Recommended irst-line regimens include
age of 16 and 23, but it is a consideration. nitrofurantoin macrocrystals 100 mg orally twice daily
Kim DK, Bridges CB, Harriman KH, et al. Advisory Commit- × 5 days, trimethoprim-sulfamethoxazole double-
tee on Immunization Practices Recommended Immuniza- strength orally twice daily × 3 days, fosfomycin 3 g sin-
tion Schedule for Adults Aged 19 Years or Older—United gle dose, or pivmecillinam 400 mg twice daily × 5 days
States, 2016. MMWR Morb Mortal Wkly Rep. 2016;65:88– (not available in the United States). here is no single
90; Bennett NM, et  al. MMWR. 2012;61:816–819;
best agent in this group that is superior to all others for
Folaranmi T, Rubin L, Martin SW, et al. Use of serogroup
B meningococcal vaccines in persons aged ≥10 years at
the empiric management of uncomplicated cystitis.
increased risk for serogroup B meningococcal disease: he choice between agents should be individualized
recommendations of the Advisory Committee on Immu- and based on the patient’s history, including allergies,
nization Practices, 2015. MMWR Morb Mortal Wkly Rep. local resistance patterns, drug availability, and cost.
2015;64:608–612. Trimethoprim-sulfamethoxazole is efective and inex-
pensive, but allergic reactions are more common, and
5. ANSWER: C. Chest x-ray to assess for active pul- in some US geographic areas, resistance rate among
monary disease community-acquired E. coli isolates exceeds 20% and
he patient should undergo chest x-ray to evaluate therefore it is not recommended in this type of set-
for active pulmonary tuberculosis. Radiologic disease ting. Nitrofurantoin is more expensive but efective.
can be present in the absence of symptoms. Either a Fosfomycin is not as efective and is also expensive.
tuberculin skin test (PPD) or an interferon gamma However, they both exert little “collateral damage.”
release assay alone is acceptable to test for tuberculosis his term refers to the indirect adverse efects of anti-
exposure, and a positive test result from either is suf- biotic agents, including their propensity to select
icient to diagnose latent infection. Because interferon drug-resistant organisms and promote colonization
gamma release assays are not afected by previous BCG and infection with multidrug-resistant bacteria. A
vaccine, this is the preferred screening test for patients 3-day regimen of luoroquinolones such as ciproloxa-
who have previously received this vaccine. Sputum cin is highly eicacious in the management of uncom-
evaluation should only be pursued if the patient has plicated cystitis. However, luoroquinolones are no
concerning symptoms or signs of active disease on longer recommended as irst-line empiric agents due
chest imaging. he Centers for Disease Control and to their potential to impact gastrointestinal lora and
Prevention (CDC) web site provides updated infor- also for potential side efects. Beta-lactam antibiotics,
mation about diagnosis and management of latent including amoxicillin and cephalexin, tend to have
tuberculosis: http://www.cdc.gov/tb/default.htm. inferior eicacy and more adverse efects compared to
the recommended agents. hey are therefore consid-
6. ANSWER: A. Campylobacter ered second-line agents for uncomplicated cystitis and
Campylobacter infection can result in Guillain-Barré are recommended when other agents cannot be used.
syndrome (GBS) several weeks after diarrhea. Approx- Gupta K, Hooton TM, Naber KG, et al. International clinical
imately 1/1000 reported Campylobacter illnesses leads practice guidelines for the treatment of acute uncomplicated
cystitis and pyelonephritis in women: A 2010 update by the
to GBS, and up to 40% of GBS in the United States
Infectious Diseases Society of America and the European
may be triggered by Campylobacter. Acute Campylo- Society for Microbiology and Infectious Diseases. Clin
bacter gastrointestinal illness can present with diarrhea Infect Dis. 2011;52:e103–e120; Food and Drug Adminis-
(possibly bloody), cramping, abdominal pain, and tration Safety Announcement; July 26, 2016; http://www.
fever, sometimes with nausea and vomiting, and can fda.gov/Drugs/DrugSafety/ucm511530.htm.
last from 2 to 10 days. It is not usually spread from one
person to another, but most cases are associated with 8. ANSWER: E. Reactivation of hepatitis B infection
contact with raw or undercooked poultry. As few as Rituximab, a monoclonal antibody to CD-20 (a
100 organisms can cause illness. Illness can also result B-lymphocyte marker), is a well-described cause of hep-
from contact with stool of an ill pet dog or cat. Antibi- atitis B reactivation characterized by redevelopment of
otics are indicated in severe cases. high levels of circulating hepatitis B virus, transaminitis,
Hughes RAC, Cornblath DR. Guillain-Barré syndrome. Lan- and in some cases a symptomatic hepatitis lare. Inactive
cet. 2005;366:1653–1666; Vucic S, Kiernan MC, Corn-
carriers of hepatitis B (surface antigen positive, but with
blath DR. Guillain-Barré syndrome: an update. J Clin
Neurosci. 2009;16:733–741.
normal liver function tests [LFTs] and low amounts of
14 C HA P T E R 1 Infectious Diseases

circulating virus) and also patients with serologic evi- and the tick was attached for long enough to trans-
dence of previous hepatitis B infection (hepatitis B core mit the infection (at least 36 hours) are at substantial
IgG positive, surface antigen negative) are both at risk risk for developing Lyme disease. he IDSA recom-
for reactivation after therapy with rituximab. Reacti- mends either of two potential management strategies
vation tends to occur early (weeks to months) after in this case: a single oral dose of doxycycline 200 mg ×
rituximab therapy in “inactive carriers,” whereas it is 1 within 72 hours of tick removal or close observation
more often a late complication of rituximab therapy in for rash or other symptoms of early Lyme disease. A
patients with serologic evidence of previous infection, full treatment course for Lyme disease (Options B and
occurring up to a year after rituximab therapy ends. D) or a single dose of antibiotics that might be active
here is very low risk of developing hepatitis C against Lyme other than the doxycycline regimen
infection as a consequence of blood transfusion in above (Option A) are not recommended. Serologies
the United States (<1 in 2 million transfusions), and are unlikely to be helpful in a case of recent exposure as
hepatitis C infection does not frequently cause an seroconversion within 1–2 days or exposure is unlikely
acute illness as described here (Option A). Transami- (Option C).
nitis would be an unusual manifestation of relapse of Wormser GP, Dattwyler RJ, Shapiro ED, et  al. he clinical
lymphoma (Option B). Hepatitis A can cause an acute assessment, treatment, and prevention of Lyme disease,
symptomatic hepatitis similar to this case presenta- human granulocytic anaplasmosis, and babesiosis: clinical
tion, but this patient was immune to hepatitis A based practice guidelines by the Infectious Diseases Society of
America. Clin Infect Dis. 2006;43:1089–1134.
on prechemotherapy serologies so it is unlikely here
(Option C). Isoniazid can cause idiosyncratic drug-
induced liver injury characterized by signiicantly 11. ANSWER: C. Trimethoprim-sulfamethoxazole
elevated transaminases, but this would be an unlikely his patient has a purulent skin and soft tissue
cause of new elevation in LFTs many months after the infection that was appropriately drained and cul-
treatment course ended (Option D). tured, which is the mainstay of treatment. Uncom-
Perrillo RP, Gish R, Falck-Ytter YT. American Gastroenterological plicated purulent skin and soft tissue infections
Association Institute technical review on prevention and treat- are most often caused by Staphylococcus aureus.
ment of hepatitis B virus reactivation during immunosuppres- Although in the absence of systemic illness this
sive drug therapy. Gastroenterology. 2015;148:221–244. type of infection can be managed with drainage
alone, a recent study comparing drainage and tri-
9. ANSWER: C. Start empiric azithromycin for a 5-day methoprim-sulfamethoxazole to drainage and pla-
course. cebo found high clinical cure rates in those treated
he clinical incubation of Bordetella pertussis is with trimethoprim-sulfamethoxazole. Antibiotics
5–21 days, and patients are considered infectious active against S. aureus, MRSA in particular, are
until 3 weeks after symptom onset. B. pertussis is a recommended while cultures of purulent mate-
highly communicable disease, and patients with sus- rial are pending. Appropriate antibiotics include
pected infection should be treated pending test results. trimethoprim-sulfamethoxazole or doxycycline,
All nonpregnant patients who present with 3 weeks both of which are usually active against methicillin-
or less of symptoms (6 weeks or less if pregnant) are sensitive and resistant strains of S. aureus. Some of
potentially contagious and so should be treated. he the other antibiotic options in this question have
treatment of choice for adults is azithromycin, clar- limited activity against S. aureus (levoloxacin, clar-
ithromycin, or erythromycin. he CDC recommends ithromycin), but none of the other options are rec-
trimethoprim-sulfamethoxazole as the second-line ommended for the treatment of purulent skin and
agent to treat those with macrolide antibiotic allergies soft tissue infection. Penicillin may be active against
or intolerance. Fluoroquinolones have not been shown susceptible S. aureus infection, but this susceptibil-
to be efective as therapy. Neither vaccination nor ity pattern is rare; penicillin is more appropriate to
administration of immunoglobulin would adequately treat uncomplicated nonpurulent cellulitis, which
treat B. pertussis infection. is more often due to streptococci. Notably, rifampin
Tiwari T, Murphy TV, Moran J, et al. Recommended antimi-
is an oral antibiotic with good oral bioavailability
crobial agents for the treatment and postexposure prophy-
laxis of pertussis: 2005 CDC Guidelines. MMWR Recomm
and activity against S. aureus. However, rifampin
Rep. 2005;54(RR14):1–16. should not be used as monotherapy in the treatment
of these infections due to a low barrier to resistance.
Talan DA, Mower WR, Krishnadasan A, et al. Trimethoprim-
10. ANSWER: E. Doxycycline 200 mg by mouth × 1
sulfamethoxazole versus placebo for uncomplicated skin
In the mid-Atlantic and Northeast, including Dela- abscess. N Engl J Med. 2016;374:823–832; Stevens DL,
ware, the deer tick Ixodes scapularis is the vector for Bisno AL, Chambers HF, et al. Practice guidelines for the
Borrelia burgdorferi, which causes Lyme disease. Indi- diagnosis and management of skin and soft tissue infections:
viduals bitten by ticks in an endemic area where there 2014 update by the Infectious Diseases Society of America.
is clear evidence that the bite was due to a deer tick Clin Infect Dis. 2014;59:147–159.
CHAPTER 1 Infectious Diseases 15

12. ANSWER: E. Oral amoxicillin-clavulanate (caspofungin, micafungin, anidulafungin) is the irst-


Splenectomy results in increased vulnerability line agent for treating candidemia in an ill patient such
to overwhelming infection and sepsis due to certain as this. Fluconazole may also be a reasonable agent to
organisms, including Streptococcus pneumoniae, Hae- use initially in select patients (such as the elderly or
mophilus inluenzae, Neisseria meningitidis, and Cap- diabetic patients) who are hemodynamically stable,
nocytophaga canimorsus. C. canimorsus is an anaerobic have not previously been treated with an antifungal in
gram-negative rod that is part of canine and feline oral the azole class, and are not at risk for resistant fungi. In
lora. When inoculated into humans, C. canimorsus addition, luconazole is renally cleared and therefore
can cause severe infection in certain hosts, includ- the dose ofered in Option A would not be appropri-
ing those without a spleen. Given this risk, typically ate for a patient on hemodialysis. Isavuconazonium
asplenic patients who sustain a dog or cat bite in which (Option C) and liposomal amphotericin (Option E)
the skin is broken are treated with a short course of also likely have activity against Candida, but neither
preventative oral antibiotics. First-generation cepha- are recommended as irst-line agents for this indica-
losporins (such as cephalexin) and trimethoprim- tion. In most cases, central venous catheters need to
sulfamethoxazole are not typically active; beta-lactam/ be removed in the setting of candidemia (Option B),
beta-lactamase inhibitors and third-generation ceph- but candidemia needs to be treated as quickly as pos-
alosporins are usually active. In this case the patient sible—fast treatment can portend better outcomes, so
is well clinically so has no indication for intravenous Option B is incorrect.
antibiotics (ceftriaxone); thus amoxicillin-clavulanate Pappas PG, Kaufman CA, Andes DR, et al. Clinical practice
is the most appropriate choice. Rabies is unlikely in guideline for the management of candidiasis: 2016 update
this case because the puppy is a domestic pet. by the Infectious Diseases Society of America. Clin Infect
Butler T. Capnocytophaga canimorsus: an emerging cause of sep- Dis. 2016;62:e1–50.
sis, meningitis, and post-splenectomy infection after dog
bites. Eur J Clin Microbiol Infect Dis. 2015;34:1271–1280. 15. ANSWER: A. Bartonella henselae
All of the pathogens listed including Bartonella
13. ANSWER: C. Use of a chlorine-containing cleaning henselae, Toxocara cati, Toxoplasmosis gondii, and Yer-
agent to address environmental contamination sinia pestis can be transmitted to humans by cats. he
Standard environmental cleaning detergents are not most likely cause of this patient’s symptoms is B. hense-
sporicidal. To adequately clean the room of a patient with lae. Systemic bartonellosis can sometimes present with
Clostridium diicile infection, it is necessary to use a chlo- a primary lesion that develops 3–10 days following
rine-containing cleaning agent (1000–5000 ppm avail- inoculation from a bite or scratch from an infected cat
able chlorine) or other sporicidal agent. Even in outbreak (usually kitten). Tender lymphadenopathy may develop
settings, routine identiication of asymptomatic carriers is proximally after 1–10 weeks with overlying erythema
not recommended and treatment is not efective in reduc- that can suppurate and may last weeks. Rare complica-
ing horizontal transmission. Strict adherence to contact tions include neuroretinitis (stellate macular exudates;
precautions, including gowning and gloving on entry to “macular star”), encephalopathy/transverse myelitis, or
a patient’s room and hand hygiene with soap and water, endocarditis. In immune-suppressed individuals, dis-
are key measures in reducing horizontal transmission of seminated illness can present as bacillary angiomatosis.
C. diicile. he spore form of C. diicile is resistant to T. cati is a helminthic infection that can be passed from
killing by alcohol. Contact precautions should continue cats to humans via fecal oral transmission; it is typically
at least for the duration of the diarrhea, with many hos- asymptomatic but can cause cutaneous larva migrans.
pitals continuing contact precautions for the duration of Toxoplasmosis is a protozoan infection that similarly
the inpatient admission. here is no evidence at this time can be spread from cats to humans via fecal oral trans-
to support the use of prophylactic antibiotics to prevent mission. his infection is also often asymptomatic in
horizontal transmission. normal hosts but can cause a mono-like illness with
Cohen SH, Gerding DN, Johnson S, et  al. Clinical practice cervical adenopathy and occasionally can cause retinal
guidelines for Clostridium diicile infection in adults: 2010 disease in normal hosts. Y. pestis, the cause of plague, is
update by the Society for Healthcare Epidemiology of Amer- transmitted uncommonly by feline leas and is generally
ica (SHEA) and the Infectious Diseases Society of America a rare illness in the United States.
(IDSA). Infect Control Hosp Epidemiol. 2010;31:431–455. Goldstein EJC, Abrahamian FM. Diseases transmitted by cats.
Microbiol Spectr. 2015;3:IOL5-0013-2015.
14. ANSWER: D. Start intravenous caspofungin.
he most likely cause of yeast in blood cultures 16. ANSWER: D. Country of birth
in this patient with recent antibiotic exposure and he US Preventive Services Task Force updated rec-
an indwelling hemodialysis catheter is Candida spe- ommendations for hepatitis B screening in 2014 based
cies. Based on IDSA guidelines for the management on extensive review of available data. Individuals who
of Candida infections, an echinocandin antifungal should be screened for hepatitis B infection include
16 C HA P T E R 1 Infectious Diseases

those born in countries where the prevalence of HBV intranasal eschar. he most common infection syn-
infection is higher than 2%, those born in the United dromes in this clinical context include mucormyco-
States to parents born in countries or regions where sis and aspergillosis. Appropriate empiric antifungal
the prevalence of hepatitis B infection is higher than therapy in suspected cases of fungal sinusitis includes
8% and were not vaccinated in infancy, those who are an antifungal agent active against Mucorales and
HIV-infected, injection drug users, men who have Aspergillus species such as amphotericin B or a lipid
sex with men, and household contacts or sexual part- formulation of amphotericin (which may be less neph-
ners of individuals with chronic hepatitis B infection. rotoxic in this individual with chronic kidney disease).
Based on these recommendations, the reason that this Voriconazole and caspofungin are active against Asper-
patient needs to be screened for hepatitis B infection gillus species but do not have activity against Mucora-
is her country of birth. he estimated prevalence of les when given alone. Fluconazole has no activity in
hepatitis B infection in Vietnam and Asia in general Aspergillus species or Mucorales. Two azole antifungal
is above 2%. Other regions of high hepatitis B infec- drugs, posaconazole and isavuconazole, have in vitro
tion prevalence include Africa and parts of the Middle activity against Mucorales, though typically a lipid for-
East and Central and South America. In addition to mulation of amphotericin in combination with surgi-
the above screening recommendations, the CDC also cal debridement is the treatment of choice for initial
recommends hepatitis B screening for all individuals management of these types of infections.
in whom immunosuppressive therapy (including che- Chitasombat MN, Kontoyiannis DP. Treatment of mucormy-
motherapy) is planned, individuals on hemodialysis, cosis in transplant patients: role of surgery and of old and
blood and body luid donors, all pregnant women, new antifungal agents. Curr Opin Infect Dis. 2016;29:340–
and infants of HBV-infected mothers. 345.
LeFevre ML. Screening for hepatitis B virus infection in non-
pregnant adolescents and adults: U.S. Preventive Services 19. ANSWER: E. Continue levoloxacin and observe.
Task Force recommendation statement. Ann Intern Med. his patient has Legionella pneumonia based on a
2014;161:58–66; Weinbaum CM, Williams I, Mast EE, clinical syndrome consistent with community-acquired
et al. Recommendations for identiication and public health pneumonia and a positive urinary Legionella antigen,
management of persons with chronic hepatitis B virus infec- which has high speciicity for the diagnosis of Legionella
tion. MMWR Recomm Rep. 2008;57:RR-8. serotype 1. herefore treatment of Legionella with either
levoloxacin or azithromycin, which are considered
17. ANSWER: B. Initiate systemic corticosteroids to con- irst-line therapy for Legionella pneumonia, is indicated.
trol symptoms of paradoxical tuberculosis immune Gram stain and culture of the expectorated sputum in
reconstitution inlammatory syndrome (IRIS). this case suggest that the specimen was orally contami-
he clinical syndrome is classic for paradoxical tuber- nated and does not represent the lower respiratory tract.
culosis IRIS, which is characterized by new or worsen- he growth of Candida species from sputum typi-
ing indings of tuberculosis disease within 2–3 months cally indicates colonization and does not require anti-
of initiation of antiretroviral therapy. Manifestations can fungal treatment as described in the IDSA guidelines
be seen within lung parenchyma, the central nervous sys- for the management of Candida infections (thus
tem, and at serosal surfaces (pleural efusions, ascites, or Options A–D are incorrect). his patient may have
pericardial efusions). Mild symptoms can be controlled been orally colonized with Candida due to the use of
with nonsteroidal antiinlammatory medications, but inhaled luticasone (particularly if he is not rinsing his
moderate to severe symptoms often require systemic cor- mouth after use).
ticosteroids. Multidrug-resistant tuberculosis is unlikely Phin N, Parry-Ford F, Harrison T, et al. Epidemiology and clin-
given the negative drug susceptibility test. he rapid ical management of Legionnaires’ disease. Lancet Infect Dis.
onset of symptoms is not consistent with loss of control 2014;14:1011–1021; Pappas PG, Kaufman CA, Andes
of tuberculosis infection due to poor adherence. Pneumo- DR, et al. Clinical practice guideline for the management of
cystis jirovecii is in the diferential diagnosis but less likely candidiasis: 2016 update by the Infectious Diseases Society
given the CD4 T cell count above 200 cells/µL3. of America. Clin Infect Dis. 2016;62:e1–50.
Panel on Antiretroviral Guidelines for Adults and Adolescents.
Guidelines for the use of antiretroviral agents in HIV-1-in- 20. ANSWER: A. Vancomycin + cefepime
fected adults and adolescents. Department of Health and his patient most likely has bacterial endocardi-
Human Services. <http://aidsinfo.nih.gov/contentiles/lvgu tis. Because this patient is febrile and ill appearing,
idelines/AdultandAdolescentGL.pdf/>; accessed 08.05.16. it is appropriate to start empiric antibiotics as soon as
blood cultures have been drawn in order to prevent
18. ANSWER: B. Lipid formulation of amphotericin B further complications of infection. he most common
he case patient’s presentation is consistent with pathogens in this type of presentation in an intravenous
a severe sinus infection with an angioinvasive mold drug user include Staphylococcus aureus, coagulase-neg-
resulting in a rapidly progressive infection with an ative staphylococci, viridans streptococci (β-hemolytic
CHAPTER 1 Infectious Diseases 17

streptococci, oral lora), and occasionally aerobic gram- Likewise, intravenous hydrocortisone is not recom-
negative bacilli or fungi. Empiric therapy should target mended if hemodynamic stability is restored with
these bacteria. Enterococci are a less frequent cause of luid resuscitation and vasopressors.
endocarditis, though still important to consider— Singer M, Deutschman CS, Seymour CW, et  al. he hird
empiric treatment for staphylococci and streptococci International Consensus Deinitions for Sepsis and Septic
will also include empiric treatment for enterococci. Can- Shock (Sepsis-3). JAMA. 2016;315:801–810; Dellinger
dida endocarditis occurs in injection drug users but is RP, Levy MM, Rhodes A, et al. Surviving sepsis campaign:
international guidelines for management of severe sepsis
less common than the bacterial pathogens.
and septic shock: 2012. Crit Care Med. 2013;41:580–637.
he best empiric therapy among the choices given
is vancomycin and cefepime, which would treat S.
aureus (including MRSA), enterococci, streptococci, 22. ANSWER: C. Caspofungin
and aerobic gram-negative organisms (including Pseu- Neutropenic patients with fever that persists or
domonas). Another reasonable regimen would include recurs after 4–7 days of empiric antibiotics are at high
vancomycin and ceftriaxone. Although Option B, risk for fungal infection, particularly due to Candida
which includes vancomycin, gentamicin, and rifampin, and Aspergillus species. Neutropenic patients with this
would have activity against the organisms of concern, fever pattern should be treated with empiric antifun-
this is the regimen recommended for treatment of gal therapy. Based on IDSA guidelines, appropriate
prosthetic valve MRSA endocarditis. Rifampin has a antifungal therapy at this juncture would include an
low barrier of resistance to bacteria so is not typically echinocandin (such as caspofungin or micafungin),
used during active bacteremia and is typically only voriconazole, or a preparation of amphotericin.
Freifeld AG, Bow EJ, Sepkowitz KA, et  al. Clinical practice
indicated to treat staphylococcal prosthetic valve endo-
guideline for the use of antimicrobial agents in neutropenic
carditis or speciic rare causes of endocarditis such as patients with cancer: 2010 update by the Infectious Diseases
Brucella. he remaining antibiotic treatment options Society of America. Clin Infect Dis. 2011;52:e56–e93.
do not adequately target the most common pathogens
as listed above.
Baddour LM, Wilson WR, Bayer AS, et al. Infective endocardi-
23. ANSWER: E. Blood smear, EBV serologies, CMV
tis in adults: diagnosis, antimicrobial therapy, and manage- serologies, HIV 1/2 antigen/antibody, HIV RNA
ment of complications: a scientiic statement for healthcare his patient has a “mono-like illness,” characterized
professionals from the American Heart Association. Circu- by fever, sore throat, malaise, fatigue, rash, and lymph-
lation. 2015;132:1435–1486. adenopathy. Mild hepatitis and pancytopenia may also
be a part of the syndrome. he most common causes
21. ANSWER: C. Administration of 30 mL/kg of crys- of this syndrome in young adults are viral pathogens,
talloid including EBV and CMV for those who were not
his patient is at risk for sepsis based on the clinical already infected during childhood. Serologies for both
criteria of altered mental status, increased respiratory EBV and CMV are helpful diagnostics here. Acute
rate (>22 breaths per minute), and low systolic blood retroviral syndrome, or acute HIV, may also present
pressure (<100 mm Hg), which make up the quick as a mono-like illness, often with concurrent diarrhea
Sequential Organ Failure Assessment (qSOFA). he and weight loss just as this patient had. he incidence
qSOFA and the more detailed SOFA score, which is of new HIV infections in the United States is high-
based on Pao2, platelet count, bilirubin, mean arterial est among men who have sex with men at present,
pressure, Glasgow coma scale, serum creatinine, and so this patient has a documented risk factor as well.
urine output, allow clinicians to identify patients with Acute HIV is the most likely diagnosis for this patient.
sepsis who are at increased risk for mortality to allow In acute HIV, the HIV antibody is often still nega-
for appropriate early intervention. tive, before seroconversion, but the HIV p24 antigen
he Surviving Sepsis Campaign recommends that is positive. Fourth-generation HIV tests check serum
the following be completed within 3 hours: mea- for HIV antibodies and p24 antigen. he CDC rec-
surement of lactate, obtaining blood cultures before ommends the following testing algorithm for patients
administration of antibiotics, administration of broad- with suspected established or acute HIV infection:
spectrum antibiotics, and administration of 30 mL/kg initial HIV 1/2 antigen/antibody testing; if the test is
of crystalloid for hypotension or lactate ≥4 mmol/L. reactive, then the sample is relexively tested specii-
here is good evidence that early antibiotics (within cally for HIV1 and HIV2 antibodies. his antibody
1 hour of recognizing sepsis) improves patient out- diferentiation test would be positive for either HIV1
comes. Norepinephrine is the irst-choice vasopressor or HIV2 antibodies in established infection, whereas
to maintain a mean arterial pressure ≥65; however, it may be negative in acute infection. hus patients
vasopressors should be reserved for hypotension that is with discordant results on the HIV 1/2 antigen/anti-
nonresponsive to luid resuscitation. Dopamine is not body and antibody diferentiation test (positive/nega-
recommended except in highly select circumstances. tive) should have further testing with an HIV RNA
18 C HA P T E R 1 Infectious Diseases

(or viral load), which should be positive and is usually high risk for mortality, two drugs that treat Pseudomo-
very high in acute HIV. In cases where suspicion is nas, including beta-lactams, quinolones, or aminogly-
high for acute HIV, like this case, it is also reasonable cosides (but preferably not to include two agents in
to check HIV RNA upfront because it becomes posi- the same class), and a single agent to treat for S. aureus,
tive in acute infection a few days before the HIV anti- including MRSA (either vancomycin or linezolid), are
gen/antibody test. recommended. herefore among the choices given,
Although hepatitis serologies may be helpful at some cefepime, levoloxacin, and vancomycin would be the
point if the liver function tests continue to be abnor- best choice. Current guidelines emphasize the impor-
mal, viral hepatitis would not explain the full constel- tance of referring to local antibiograms data to choose
lation of this patient’s symptoms or lab abnormalities, the best antibiotic combination for patients at each
and the transaminases are not high enough to be con- center. Antibiotics can be adjusted based on clinical
sistent with acute viral hepatitis, so these would not improvement and culture results at 48 to 72 hours.
be the appropriate next tests to order. It is appropriate Kalil AC, Metersky ML, Klompas M, et  al. Management of
to consider the possibility of new sexually transmitted adults with hospital-acquired and ventilator-associated
infections (STIs) given his new sexual partners, and pneumonia: 2016 clinical practice guidelines by the Infec-
secondary syphilis can also present with fever and rash tious Diseases Society of America and the American ho-
racic Society. Clin Infect Dis. 2016;63:e61–e111.
and lymphadenopathy but would be unlikely to present
with diarrhea, weight loss, and pancytopenia. An echo-
cardiogram might be an appropriate test if the blood 25. ANSWER: A. Clotrimazole troches
cultures are positive, but there are no other clues spe- he patient in this case has clinical evidence of oral
ciic to endocarditis here aside from the fever. A blood candidiasis (thrush). He recently received treatment
smear and LDH may be appropriate tests at this point with systemic steroids and antibiotics and is also on a
given the patient’s pancytopenia of uncertain etiology, chronic inhaled steroid, all of which raise his potential
but a bone marrow biopsy should not be pursued until risk for thrush. In order to minimize the possibility
further noninvasive diagnostics have been performed. that the thrush resulted from the use of an inhaled ste-
Centers for Disease Control and Prevention and Association of roid, reviewing proper oral hygiene after use could be
Public Health Laboratories. Laboratory testing for the diag- helpful. Despite these risk factors, it is also reasonable
nosis of HIV infection: updated recommendations. Avail- to consider HIV testing; the CDC currently recom-
able at <http://stacks.cdc.gov/view/cdc/23447/>. Published mends testing for all patients in the health care set-
June 27, 2014. Accessed 08.05.16. ting. Appropriate treatment for mild thrush includes
clotrimazole troches or miconazole buccal tablets; oral
24. ANSWER: E. Cefepime + levoloxacin + vancomycin nystatin solution or pastilles is also an alternative. Sys-
his patient has hospital-acquired pneumonia (HAP), temic oral luconazole is typically reserved for moder-
which is deined as “pneumonia not incubating at the ate or severe oral thrush. Posaconazole and micafungin
time of hospital admission and occurring 48 hours or are broader spectrum systemic antifungal agents than
more after admission” by the IDSA. he appropriate luconazole and are not indicated for the treatment of
antibiotic choice in this setting depends on three patient mild oral thrush in a normal host, though posacon-
factors: azole is sometimes used for luconazole-refractory
1. Risk for infection with MRSA thrush in a compromised host. Chlorhexidine mouth-
• Risk for MRSA is increased if the patient has wash is approved for treatment of gingivitis but does
received IV antibiotic therapy in the last 90 not have antifungal activity and will not treat oral
days or is hospitalized in a unit where more thrush.
than 20% of S. aureus isolates are methicillin- Pappas PG, Kaufman CA, Andes DR, et al. Clinical practice
resistant or if this statistic is not known. guideline for the management of candidiasis: 2016 update
2. Risk for mortality by the Infectious Diseases Society of America. Clin Infect
• Risk for mortality is increased if the patient Dis. 2016;62:e1–50.
requires ventilatory support for or has
septic shock due to HAP. 26. ANSWER: B. Colistimethate
3. Risk for infections due to multidrug-resistant org- Acinetobacter baumannii is an aerobic gram-negative
anisms including Pseudomonas species. coccobacillus that can cause hospital outbreaks with
• Risk for resistant organisms is increased high rates of mortality. Outbreaks are typically asso-
if the patient has received IV antibiotic ciated with colonized respiratory support equipment,
therapy in the last 90 days. irrigation solutions, and intravenous solutions. he
Based on the patient’s previous course of IV antibi- most common infections include ventilator-associated
otics for bacteremia within the last 3 months, he is at pneumonia, infection of surgical wounds and burns,
increased risk for both multidrug-resistant pathogens and bacteremia. Over 50% of A. baumannii isolates
and MRSA. For patients in this setting or in those at in US hospitals are multidrug resistant. Colistimethate
CHAPTER 1 Infectious Diseases 19

(colistin) is virtually always active in vitro. he pres- eicacy, of these agents. Efavirenz is both a substrate
ence of a beta-lactamase means that both cefepime and inducer of isoforms of the cytochrome P450 sys-
and piperacillin/tazobactam, the two most active beta- tem, but interactions with efavirenz are generally less
lactam antibiotics against A. baumannii, are likely to common and less severe, though the interaction with
be inefective. he presence of a carbapenemase means methadone is unpredictable and can be problem-
that imipenem/cilastatin, meropenem, and doripenem atic. Inhaled albuterol, ophthalmic antihistamines,
are all unlikely to be efective. Finally, most multidrug- oral cetirizine, and oral montelukast would not be
resistant A. baumannii isolates are resistant to luoro- expected to cause a problematic drug interaction for
quinolones. Even if the isolate tests susceptible to a this patient.
luoroquinolone, resistance often develops during use. Panel on Antiretroviral Guidelines for Adults and Adolescents.
Fishbain J, Peleg AY. Treatment of Acinetobacter infections. Clin Guidelines for the use of antiretroviral agents in HIV-
Infect Dis. 2010;51:79–84. 1-infected adults and adolescents. Department of Health
and Human Services. Available at http://www.aidsinfo.nih.
27. ANSWER: D. Inhaled luticasone gov/ContentFiles/AdultandAdolescentGL.pdf. Accessed
08.08.16.
here are many possible problematic drug–drug
interactions with antiretroviral medications and com-
monly prescribed drugs. Physicians should review 28. ANSWER: D. Bacteroides fragilis
medication lists for any possible drug–drug interac- Bacteroides fragilis is an anaerobic gram-negative
tions before prescribing new medications to patients bacteria that colonizes the intestines and is cultured
who are currently taking antiretroviral medications. from the blood in the setting of bowel perforation.
Computer software applications that predict drug It is not a common cause of endocarditis. Common
interactions from a physician-entered medication list pathogens that cause endocarditis include Staphylo-
are particularly helpful in this setting. he most prob- coccus aureus, the viridans streptococci, Streptococcus
lematic antiretroviral agent in terms of risk of drug gallolyticus (previously known as Streptococcus bovis),
interactions is ritonavir. Ritonavir is the most potent the HACEK organisms (Haemophilus parainluenzae,
cytochrome P450 3A4 inhibitor of all currently avail- Haemophilus aphrophilus, Haemophilus paraphrophilus,
able medications. he other protease inhibitors, such Actinobacillus, Cardiobacterium hominis, Eikenella cor-
as atazanavir and darunavir, and the pharmacologic rodens, Kingella kingae), and enterococci (community
booster agent cobicistat (available in several cofor- acquired, without another obvious source). Positive
mulations and alone), also inhibit CYP 3A4. hese blood cultures from at least two separate cultures with
medications interact with many commonly prescribed any of those organisms are considered major criteria
drugs, including warfarin, the statins, combination for the diagnosis of endocarditis by the modiied Duke
estrogen/progesterone oral contraceptives, clarithro- criteria, or highly positive antibody titer to Coxiella
mycin, rifampin, amiodarone, benzodiazepines, carba- burnetii (because this organism is exceedingly diicult
mazepine, phenytoin, sildenail, tadalail, meperidine, to culture). he other major criterion for diagnosis of
methadone, and peripherally administered cortico- endocarditis is an echocardiogram with indings con-
steroids. Peripherally administered corticosteroids sistent with infective endocarditis (mobile intracardiac
(inhaled luticasone, in this case) may reach systemic mass, abscess, new dehiscence of prosthetic valve, or
levels in patients who are taking ritonavir, which new valvular regurgitation). A diagnosis of endocar-
can lead to corticosteroid excess (features of Cush- ditis by Duke criteria requires fulillment of either
ing syndrome) and then later lead to corticosteroid two major criteria (as listed previously), or one major
insuiciency when the medication is withdrawn. he and three minor criteria, or ive minor criteria. his
systemic efects of peripherally administered cortico- patient meets two minor criteria for the diagnosis of
steroids with ritonavir are unpredictable and thus are endocarditis at the time of presentation: fever and
diicult to monitor and treat. Peripherally adminis- predisposing cardiac lesion (the bicuspid valve with
tered corticosteroids, including inhaled, intranasal, regurgitation). Other minor criteria are as follows:
injected (joint injections for treatment of pain), and other predisposing risk (injection drug use), evidence
even ophthalmic, should be avoided in patients tak- of emboli (arterial emboli, pulmonary infarcts, Jane-
ing ritonavir. If these medications are necessary, then way lesions, conjunctival hemorrhage), immunologic
an HIV specialist should be consulted. In addition complications (glomerulonephritis, Osler nodes), or
to the aforementioned list of drug–drug interactions, positive blood cultures that do not meet the major
there are other common drug–drug interactions with criteria or serologic evidence of an infection with an
antiretroviral medications. Both atazanavir and rilpi- organism consistent with infective endocarditis not
virine require stomach acid for absorption to thera- satisfying a major criterion (e.g., Bartonella henselae).
peutic levels, so coadministration of acid blockers (H2 he diagnosis of infective endocarditis is “possible” if
blockers and especially proton pump inhibitors) can only one major and one minor criteria are fulilled, or
lead to decreased blood levels, and therefore decreased if three minor criteria are fulilled.
20 C HA P T E R 1 Infectious Diseases

Baddour LM, Wilson WR, Bayer AS, et al. Infective endocardi- as opportunistic infections and AIDS-related malig-
tis in adults: diagnosis, antimicrobial therapy, and manage- nancies (e.g., Kaposi sarcoma), there are other risks
ment of complications: a scientiic statement for healthcare with prolonged HIV infection. Prolonged HIV infec-
professionals from the American Heart Association. Circu- tion is associated with increased risk of cardiovascu-
lation. 2015;132:1435–1486.
lar disease and non–AIDS-related malignancies, such
as lung cancer. Health care maintenance, including
29. ANSWER: B. A few days after initiation of clar- addressing modiiable risk factors and pursuing age-
ithromycin and ethambutol, but within 2 weeks appropriate cancer screening, is extremely important
Antiretroviral therapy (ART) should be started in these patients. For this patient the most important
within 2 weeks of diagnosis of most opportunistic modiiable risk factor for cardiovascular disease and
infections, with the exception of cryptococcal men- malignancy is his smoking. Among all of the health
ingitis and tuberculosis meningitis. here is a risk care maintenance items that could be addressed with
of immune reconstitution inlammatory syndrome the patient during this visit, smoking cessation (if
(IRIS) in patients who start ART with low CD4 this can be achieved) is the most likely to lead to a
count, high viral load, and active opportunistic infec- reduction in morbidity and mortality. It is important
tion. IRIS may be less pronounced if the opportunistic to address smoking cessation with him through coun-
infection is treated irst. However, IRIS may be treated seling (use of motivational interviewing is especially
with corticosteroids if needed. Patients have increased helpful here) and discussion of nicotine replacement
mortality from AIDS-related causes if ART is not therapy (nicotine patches, etc.) and other medications
started early enough. It may be appropriate to start to aid in smoking cessation (varenicline, bupropion).
treatment for opportunistic infections irst to ensure his patient does not meet criteria to start prophylaxis
tolerance of those medications for a couple of days for Pneumocystis pneumonia because his CD4 count is
prior to ART and start control of those infections, but well above 200, and he is not at risk for reactivation of
then ART should be started soon afterwards, within latent toxoplasmosis because his CD4 count is greater
2 weeks of the diagnosis of opportunistic infection. than 100. A discussion about alcohol use to screen for
here is no reason to wait until hospital discharge to warning signs of abuse or dependence may be appro-
start ART. ART may be started during hospitalization priate at this visit, but his current reported number of
and transitioned to the outpatient setting in the same alcoholic beverages per week is below the threshold of
way that other new medications (e.g., statins, beta- concern for men, so this is less important to address
blockers, and clopidogrel after new cardiac event) are than smoking cessation. he patient should be referred
started during hospitalization and transitioned to the for colonoscopy screening at age 50, but in the absence
outpatient setting. ART should not be started in a of symptoms or a personal or family history of polyps
staggered fashion, one at a time over weeks. or colon cancer, there is no indication to refer for early
Zolopa A, Andersen J, Powderly W, et  al. Early antiretroviral
colonoscopy screening.
therapy reduces AIDS progression/death in individuals with
acute opportunistic infections: a multicenter randomized
strategy trial. PLOS One. 2009;4:e5575; Panel on Antiretro- 31. ANSWER: A. Vancomycin, ceftriaxone, ampicillin
viral Guidelines for Adults and Adolescents. Guidelines for his patient is an elderly man who is living in a
the use of antiretroviral agents in HIV-1-infected adults and close community at his assisted living facility, now
adolescents. Department of Health and Human Services. presenting with probable bacterial meningitis. he
Available at http://www.aidsinfo.nih.gov/ContentFiles/Adu most common pathogens in this case are Streptococ-
ltandAdolescentGL.pdf. Accessed 08.08.16. cus pneumoniae, Neisseria meningitidis, and Listeria
monocytogenes. he combination of vancomycin and
30. ANSWER: C. Counsel patient to quit smoking, and high-dose ceftriaxone is used empirically when S.
discuss medications and supports for smoking ces- pneumoniae meningitis is a possibility because of the
sation. small but signiicant prevalence of penicillin-resistant
his patient has well-controlled HIV with a CD4 and even cefotaxime-resistant S. pneumoniae. Ampi-
count well above the range that would be concerning cillin is used for treatment of Listeria meningitis.
for opportunistic infections. If he continues to take he other antibiotic options that are listed are not
his antiretrovirals regularly, then he should continue to appropriate for this patient. his patient has no his-
have a robust immune system and low risk of oppor- tory of neurosurgical procedures, so he does not need
tunistic infections. In fact, patients with HIV who are empiric treatment with an antipseudomonal cepha-
diagnosed early (before diagnosis of AIDS), started losporin, such as cefepime or ceftazidime, or a car-
on antiretroviral therapy quickly, and maintained on bapenem, such as imipenem or meropenem.
antiretroviral therapy without interruptions are esti- he CSF examination of this patient shows dei-
mated to have a life expectancy that approaches that nite predominance of neutrophils with minimal lym-
of the general population. Although this patient is phocytes and low glucose, which is more suggestive
unlikely to develop AIDS-related complications, such of bacterial meningitis rather than viral meningitis,
CHAPTER 1 Infectious Diseases 21

so acyclovir is not required for initial treatment. never indicated as a part of treatment of an acute ill-
he combination of ampicillin and gentamicin may ness, so Option E is incorrect. his patient should
be useful for such pathogens as ampicillin-sensitive be treated with supportive care, including symptom
enterococci or Streptococcus agalactiae (group B Strep- management—all of the medications in Option C
tococcus), but it does not include appropriate empiric may be used for symptom management, so this is the
treatment for S. pneumoniae or N. meningitidis. Fun- correct answer.
gal meningitis is rare, seen primarily in patients who Chow AW, Benninger MS, Brook I, et al. IDSA clinical practice
are immunocompromised or those who have been guideline for acute bacterial rhinosinusitis in children and
instrumented or undergone epidural injections with adults. Clin Infect Dis. 2012;54:e73–e112.
contaminated material such as in the 2012 outbreak
involving steroid injections contaminated with Exse- 33. ANSWER: E. Allergy consultation and admission
rohilum rostratum. his patient does not have a his- for desensitization to penicillin in order to facili-
tory of immunocompromise, instrumentation, or tate treatment with penicillin G 2.4 million units
epidural injections of medications, so fungal menin- IM once weekly for 3 weeks
gitis is extremely unlikely, and empiric therapy with Syphilis infection during pregnancy can have
amphotericin B is not appropriate. serious consequences for both the mother and
Kainer MA, Reagan DR, Nguyen DB, et  al. Fungal infec- fetus if left untreated. herefore it is important to
tions associated with contaminated methylprednisolone in treat immediately with the most aggressive, data-
Tennessee. N Engl J Med. 2012;367:2194–2203; Tunkel supported efective treatment for syphilis, which is
AR, Hartman BJ, Kaplan SL, et al. Practice guidelines for penicillin. Pregnant women who are diagnosed with
the management of bacterial meningitis. Clin Infect Dis. syphilis should always be treated with penicillin,
2004;39:1267–1284. even if this requires hospitalization for desensitiza-
tion to penicillin, in the case of penicillin allergy.
32. ANSWER: C. Oral dextromethorphan, inhaled alb- All other treatments for syphilis are potentially less
uterol, and intranasal ipratropium efective, and doxycycline is contraindicated during
his patient has rhinosinusitis that is improving pregnancy. his patient may be presumed to have
without antibiotic treatment already, based on ces- late latent syphilis, as she is asymptomatic with
sation of fevers 3 days ago and only mild residual infection of unknown duration, so treatment with
symptoms. Acute rhinosinusitis (“sinusitis”) is an weekly IM penicillin for 3 weeks would be appro-
extremely common condition, especially among priate. Follow-up RPR titers should be monitored
adults age 45–74 years, and the cause is viral in more to ensure that they decrease appropriately.
than 90% of cases, with bacterial infection account- Workowski KA, Bolan GA. Sexually transmitted diseases treat-
ing for less than 10% of cases. Symptoms of upper ment guidelines, 2015. MMWR Recomm Rep. 2015;64:
respiratory tract infections or sinusitis may include 1–137.
nasal congestion, headache, ear pain, cough, fever,
and purulent nasal discharge. he indications for 34. ANSWER: D. Ceftriaxone 250 mg IM single dose +
antibiotic treatment of sinusitis are when symptoms azithromycin 1 g orally single dose
suggest bacterial infection with any one or combina- his patients has uncomplicated gonococcal urethri-
tion of three patterns: (1) persistent or not improv- tis. Fluoroquinolone-resistant strains of Neisseria gon-
ing symptoms (≥10 days); (2) severe symptoms for orrhoeae are now disseminated throughout the United
≥3–4 days (including temperature >39°C or 102°C, States. So, luoroquinolones have not been recom-
or 3–4 consecutive days of sinus pain at the begin- mended for treatment of gonorrhea in the United States
ning of the illness); or (3) worsening or “double- since 2007 (Option A). In addition, Asian and Euro-
sickening” (onset of new/worsening symptoms as pean countries have reported ceixime-resistant strains
viral upper respiratory infection is starting to resolve and treatment-failures. In the United States, gonorrhea
after 5–6 days, indicating bacterial superinfection). surveillance in the last decade has also suggested that
his patient meets none of these criteria; therefore the eicacy of ceixime may be reduced, and strains not
antibiotic treatment including moxiloxacin (Option susceptible to ceixime are also typically not susceptible
A), amoxicillin-clavulanate (Option B), and ceftriax- to tetracyclines like doxycycline. In 2015 the CDC
one (Option D) are not indicated. Furthermore, for changed their recommendations for the treatment of
patients who do meet criteria for therapy and are not uncomplicated cervical, urethral, and rectal gonorrhea
penicillin-allergic, quinolones and third-generation infection to a single preferred regimen of intramuscular
cephalosporins are no longer considered irst-line ceftriaxone and single-dose oral azithromycin. Ceftri-
therapy; amoxicillin-clavulanate remains the treat- axone, also a third-generation cephalosporin like ceix-
ment of choice for patients who do need treatment. ime, remains part of the regimen of choice because the
Inluenza vaccine, although not contraindicated in intramuscular delivery leads to long-lasting high levels
this situation because the patient is now afebrile, is of antibiotic relative to a single oral dose of ceixime.
22 C HA P T E R 1 Infectious Diseases

Ceixime remains an alternative option for treatment nodules and hemoptysis may be due to fungal infec-
of uncomplicated gonorrhea when given with a single tion or tuberculosis, or nontuberculous mycobac-
dose of azithromycin, but not in combination with terial infection. However, when the entirety of the
doxycycline (Option C). Concurrent treatment with patient’s history is reviewed, the salient points are
azithromycin in either case is important because most as follows: no obvious risk factors for tuberculosis
gonococci are also susceptible to azithromycin even or immunosuppression at baseline, chronic sinusitis
with diminished susceptibility to cephalosporins. Con- that was unresponsive to repeated treatments with
current treatment may also decrease the rates of devel- antibiotics, new pulmonary nodules and hemopty-
opment of antibiotic-resistant gonorrhea and also treats sis, and blood tests showing anemia and hematuria.
for the possibility of concomitant Chlamydia coinfec- Taken together, this syndrome is most consistent
tion in cases where this infection is present. with granulomatosis with polyangiitis (previously
Workowski KA, Bolan GA. Sexually transmitted diseases treat- known as Wegner granulomatosis). he American
ment guidelines, 2015. MMWR Recomm Rep. 2015;64:1– College of Rheumatology–endorsed diagnostic cri-
137. teria for granulomatosis with polyangiitis stipulate
that patients should meet at least 2 of 4 criteria that
35. ANSWER: B. Viral culture of a swab of the ulcers include “nasal or oral inlammation,” abnormal chest
he most likely diagnosis in this case is anogeni- imaging, abnormal urinary sediment, and granu-
tal herpes simplex virus (HSV) infection. he features lomatous inlammation of biopsy. Most patients
that are consistent with this diagnosis are the grouped with active granulomatosis will have positive serum
ulcerations, which are painful, with absence of sig- ANCA testing (up to 80%), especially if the disease
niicant lymphadenopathy or rectal discharge, which is severe, but biopsy would be deinitive. AFB smears
would be expected with some other conditions such from sputum or from biopsy samples would be posi-
as gonorrhea or lymphogranuloma venereum (LGV). tive if the patient had tuberculosis or nontubercu-
Diagnostics for active HSV infection are somewhat lous mycobacterial infection. Serum galactomannan
limited, and the diagnosis is often made clinically fol- would be positive if the patient had invasive aspergil-
lowed by somewhat empiric treatment. Viral culture losis. Serum IGRA may be positive in patients with
of a swab of the lesion has a high speciicity, but the tuberculosis.
sensitivity is limited and operator dependent. In cer- Leavitt RY, Fauci AS, Bloch DA, et al. he American College of
tain institutions the direct luorescent antibody test- Rheumatology 1990 criteria for the classiication of Wegen-
ing (DFA) of a scraping of the base of the lesion may er’s granulomatosis. Arthritis Rheum. 1990;33:1101–1107.
be highly sensitive and speciic, but it is also operator
dependent and therefore not consistent across insti- 37. ANSWER: A. Monitoring for early signs and symp-
tutions. PCR for HSV is both sensitive and speciic toms of urinary tract infection, with expedited
from CSF during episodes of HSV meningitis and early urinalysis, urine culture, and empiric treat-
can also be checked from a swab of an ulcer. Serologic ment while awaiting culture results when symp-
testing for HSV type-speciic antibodies will demon- toms develop
strate infection that was acquired at some point in the Patients with chronic indwelling urinary catheters
past if the patient is having a recurrent outbreak and often have pyuria and bacteria even in the absence of
will develop within the irst several weeks of infection other signs or symptoms of active urinary tract infec-
for new exposures, but serologies are not helpful in tion. Routine screening with urinalysis and urine cul-
diagnosing an active outbreak. Urine NAAT probes ture is not recommended in these patients because this
are helpful for diagnosis of gonorrhea or chlamydia will lead to overtreatment of asymptomatic patients,
but not helpful for diagnosis of HSV. Bacterial stud- and it increases the risk for antibiotic-resistant infec-
ies would not be expected to be positive during active tions in the future. Early diagnosis and treatment,
HSV infection, which is a viral infection. based on prior available culture results while waiting
Workowski KA, Bolan GA. Sexually transmitted diseases treat- for pending acute cultures, is the safest approach to
ment guidelines, 2015. MMWR Recomm Rep. 2015;64:1– management to decrease the risk of overtreatment.
137. Antibiotic treatments into the urinary drainage bag
are not recommended, and chronic prophylaxis with
36. ANSWER: C. Serum test for antineutrophilic cyto- methenamine salts or empiric broad antibiotics (such
plasmic antibodies (ANCA) as ciproloxacin) without any review of culture data or
his question stem describes a patient with an prior infections are not recommended.
“infection mimicker,” which can be diagnostically Hooton TM, Bradley SF, Cardenas DD, et al. Diagnosis, pre-
challenging. Many of the features of this patient’s ill- vention, and treatment of catheter-associated urinary tract
ness may be consistent with an infectious etiology— infection in adults: 2009 International Clinical Practice
chronic sinusitis, especially in immunocompromised Guidelines from the Infectious Diseases Society of America.
patients, may be due to fungal infection. Pulmonary Clin Infect Dis. 2010;50:625–663.
CHAPTER 1 Infectious Diseases 23

38. ANSWER: E. Penicillin typically spread directly from human to human in a


Recurrent cellulitis with episodes as described for teacher-student relationship, and pregnant women
this patient (sudden onset of fever, systemic symp- can keep a pet cat as long as contact with feces is
toms, and pain and erythema of the leg) is commonly minimized (e.g., another person cleans the litter box)
attributed to recurrent streptococcal infection, though and hands are washed carefully after any potential
the microbiologic diagnosis is not conirmed in most exposure.
cases. Risk factors for recurrent cellulitis may include Montoya JG, Remington JS. Management of Toxoplasma
prior surgery or prior infections in the afected limb, gondii infection during pregnancy. Clin Infect Dis.
lower-extremity edema, poor hygiene status, skin 2008;47(4):554–566; American College of Obstetricians
ulcerations, and onychomycosis. A randomized, pla- and Gynecologists. Practice bulletin no. 151: cytomega-
lovirus, parvovirus B19, varicella zoster, and toxoplasmo-
cebo-controlled trial of chronic suppressive/prophy-
sis in pregnancy. Obstet Gynecol. 2015;125:1510–1525;
lactic oral penicillin showed that this was an efective Lopez A, Dietz VJ, Wilson M, et al. Preventing congenital
strategy to decrease recurrences of cellulitis; incidence toxoplasmosis. MMWR Recomm Rep. 2000;49:59–68.
of cellulitis was 37% among placebo recipients and
22% among penicillin recipients during the follow-
up period. Many of the antibiotic options listed have 40. ANSWER: D. Vancomycin and piperacillin-tazobactam
good eicacy at treating staphylococcal infections, and his patient has poorly controlled diabetes with
some have good eicacy at treating streptococcal infec- a new progressive soft tissue infection arising from
tions (e.g., levoloxacin), but only penicillin has been a nonhealing toe ulcer. he black areas of the toe
proven to reduce risk of recurrence of cellulitis in a are concerning for necrotic tissue, which in this
placebo-controlled clinical trial. case would raise the concern for “wet gangrene,” or
homas KS, Crook AM, Nunn AJ, et al. Penicillin to prevent infected gangrenous tissue. his patient requires surgi-
recurrent leg cellulitis. N Engl J Med. 2013;368:1695–1703; cal debridement and may require amputation of the
Stevens DL, Bisno AL, Chambers HF, et al. Practice guide- toe, depending on the extent of the infection. Based
lines for the diagnosis and management of skin and soft on the presence of his local symptoms (purulent drain-
tissue infections: 2014 update by the Infectious Diseases age, tenderness, pain), the size of the ulcer (>2 cm),
Society of America. Clin Infect Dis. 2014;59:147–159. and his systemic signs and symptoms of infection
(fever >100.4°F and white blood cell count >12,000
39. ANSWER: A. Cook meat to “well done.” cells/µL), this patient has a severe infection based on
Toxoplasma gondii is a parasite found in soil and IDSA classiications. Antibiotics should be started
contaminated or undercooked foods. Cats are the immediately while awaiting results of cultures and
deinitive host and shed oocytes that eventually operative management. Cultures should be obtained
become infectious to humans or other animals if con- from deep tissue typically by biopsy or debride-
sumed (typically through poorly washed produce, ment after the wound has been cleaned, as cultures
undercooked meat, or inadequate hand washing after obtained from supericial swabs of the wound may be
soil exposure). Acute infection may be associated with less accurate. For severe infections, empirical therapy
malaise, fever, and lymphadenopathy, but it is often for Staphylococcus aureus, including MRSA, as well as
asymptomatic. Acute infection during pregnancy gram-negative bacilli, including Pseudomonas aeru-
may result in congenital infection of the newborn, ginosa when risk factors such as a high prevalence of
which can cause long-term complications such as Pseudomonas infection are present, is indicated. Of the
blindness. Early diagnosis and treatment during preg- listed antibiotic regimens, only piperacillin-tazobac-
nancy is essential to preventing complications. How- tam and vancomycin treat for these possibilities.
ever, prevention of infection is ideal. he CDC and Lipsky BA, Berendt AR, Cornia PB, et al. 2012 Infectious Dis-
the American College of Obstetrics and Gynecology eases Society of America clinical practice guideline for the
recommend that pregnant women be advised about diagnosis and treatment of diabetic foot infections. Clin
several lifestyle changes to reduce the risk of exposure. Infect Dis. 2012;54:132–173.
his includes careful washing of fruits and vegetables
before consumption because they may be contami- 41. ANSWER: B. Valacyclovir 1000 mg orally three
nated with cysts from the soil. All meats should be times daily for 14 days
cooked to “well done” because uncooked meats may his patient has Ramsay Hunt syndrome type II,
also carry infection. Although pork products may also otherwise known as herpes zoster oticus. his is a rare
carry infection, they do not need to be universally syndrome that is caused by reactivation of herpes zos-
avoided as long as they are cooked properly. Lastly, ter virus in the geniculate ganglion, which leads to a
pregnant women should wear gloves and carefully triad of symptoms including (1) ear pain, tinnitus, and
wash hands afterwards when they anticipate soil expo- vertigo, (2) ipsilateral peripheral facial nerve paraly-
sure, such as with gardening (but it need not be given sis, and (3) vesicles in the external auditory canal, ear,
up as long as gloves are worn). Toxoplasmosis is not mouth, or anterior two-thirds of the tongue. Patients
24 C HA P T E R 1 Infectious Diseases

may also experience altered sensation in the ear canal Risk for PCP in patients treated with corticosteroids
and loss of moisturization of the eyes and mouth. depends on the dose and duration of the steroids. In
he concurrence of facial droop with ear canal symp- a study of 116 patients with PCP who did not have
toms, changes in mucous membranes moisture, and HIV, 91% had been on corticosteroids when PCP
skin lesions make this syndrome unique. his zoster was diagnosed, and the mortality rate was 49%. he
reactivation is treated with acyclovir or valacyclovir. median steroid dose and duration in this study was 30
he addition of corticosteroids, such as prednisone mg of prednisone per day for 12 weeks, but it was seen
to antiviral therapy, is controversial but has not been in patients on doses as low as 16 mg per day and for
shown to be of beneit in a randomized study. he durations as short as 8 weeks.
facial droop alone can be a symptom of Lyme disease he features of the presentation that are consistent
(especially with the epidemiologic risk factors listed with Pneumocystis pneumonia include prolonged cor-
for this patient) and would be appropriately treated ticosteroid exposure (especially in the context of recent
with doxycycline but would not explain the patient’s tapering), cough and shortness of breath, which are
other symptoms. Otitis externa may be treated with progressive despite antibiotics; signiicant hypoxia on
otic ciproloxacin drops, and if concurrent otitis media presentation; and bilateral iniltrates on chest imaging.
was suspected, then additional treatment with amoxi- he serum β-d-glucan assay may be positive in the set-
cillin-clavulanate would be appropriate. ting of Pneumocystis pneumonia, especially in immu-
Worme M, Chada R, Lavallee L. An unexpected case of Ramsay nocompromised patients, and also in some other
Hunt syndrome: case report and literature review. BMC Res fungal infections. Treatment is with trimethoprim-
Notes. 2013;6:337; Kansu L, Yilmaz I. Herpes zoster oticus sulfamethoxazole, and corticosteroids should be added
(Ramsay Hunt syndrome) in children: case report and lit- when there is severe disease as evidenced by a large A–a
erature review. Int J Pediatr Otorhinolaryngol. 2012;76:772–
gradient.
776.
Ivermectin would be used to treat Strongyloides
stercoralis hyperinfection, which can cause respiratory
42. ANSWER: C. Mycoplasma genitalium failure in patients who are latently infected and then
his patient has nongonococcal urethritis (NGU) receive corticosteroids, but it is less common in the
based on the clinical diagnosis of urethritis and the United States and is typically manifested with other
negative testing for gonorrhea. he most common symptoms and problems also including diarrhea,
cause of NGU is Chlamydia infection, which causes abdominal pain, gram-negative sepsis, or meningitis
an estimated 15%–40% of all NGU, but this patient related to increased worm burden and transit. hough
also had negative testing for Chlamydia. Other organ- fevers could suggest laring of her giant cell arteritis
isms that cause NGU include Mycoplasma genitalium, such that increased steroids are necessary, her pulmo-
Trichomonas vaginalis, HSV, and occasionally adenovi- nary symptoms and lack of recurrent headaches make
rus. After Chlamydia, M. genitalium is likely the next this less likely. Pulmonary edema is also a consider-
most common cause of NGU at about 15%–25% and ation, but on exam she has no evidence of congestive
is treated with azithromycin. Notably, this organism heart failure and this would not explain the fevers.
is less responsive to doxycycline, and, furthermore, Yale SH, Limper AH. Pneumocystis carinii pneumonia in
azithromycin resistance with this infection is thought patients without acquired immunodeiciency syndrome:
to be increasing in the United States. In patients with associated illness and prior corticosteroid therapy. Mayo
NGU who do not respond to azithromycin, M. geni- Clin Proc. 1996;71:5–13; Kermani TA, Ytterberg SR, War-
talium is the most common cause and retreatment rington KJ. Pneumocystis jiroveci pneumonia in giant cell
with moxiloxacin is indicated. Trichomonas is a less arteritis: a case series. Arthritis Care Res. 2011;63:761–765.
common cause of urethritis but is a consideration in
men who have sex with women and do not respond to 44. ANSWER: C. Patients taking PrEP should have reg-
empiric therapy for NGU with azithromycin or doxy- ular screening for other STIs at least every 6 months,
cycline, particularly in areas where Trichomonas infec- including syphilis, gonorrhea, and chlamydia.
tion is prevalent. Haemophilus ducreyi is the causative he CDC recommends tenofovir/emtricitabine
organism in chancroid, which is an ulcerative STI and once daily for PrEP for HIV prevention for patients
does not typically cause urethritis. with high risk of HIV acquisition, including men
Workowski KA, Bolan GA. Sexually transmitted diseases treat- who have sex with men as well as heterosexual men
ment guidelines, 2015. MMWR Recomm Rep. 2015;64:1–137. and women with increased sexual risk, and injection
drug users. he CDC has extensive online resources
43. ANSWER: B. Trimethoprim-sulfamethoxazole on PrEP for both patients and providers at http://w
his patient has Pneumocystis jirovecii pneumonia, ww.cdc.gov/hiv/risk/prep/. Providers should follow
also still commonly known as PCP. Besides HIV infec- these online guidelines when prescribing PrEP, includ-
tion or AIDS, PCP is most commonly seen in immu- ing baseline and regular HIV testing every 3 months
nocompromised patients treated with corticosteroids. (not 12), regular STI screening, and monitoring renal
CHAPTER 1 Infectious Diseases 25

function because there is risk for nephrotoxicity with common tick-borne illness (Ixodes scapularis), and
tenofovir. Taking tenofovir/emtricitabine for PrEP has no reported travel history to a malaria-endemic
with excellent adherence can reduce the risk of HIV area, so it is presumed that the infection in this case
acquisition by as much as 92% in this patient popula- is babesiosis rather than malaria. Severe babesiosis, as
tion. Since the advent of PrEP, many providers have in this case, is deined as infection resulting in renal,
raised concerns that patients taking PrEP would com- hepatic, or pulmonary end-organ dysfunction, high
pensate with increased sexual risk behaviors, resulting grade parasitemia (>10%), or signiicant hemolysis.
in increased incidence of STIs. However, the PROUD Mild babesiosis may be treated with azithromycin
study of PrEP in a real-world setting demonstrated no and atovaquone (but not with proguanil, so Option
increased incidence of STIs in the PrEP arm, and no A is wrong). Severe babesiosis is generally treated with
evidence of increased risky behaviors in the PrEP arm. quinine (oral quinine or intravenous quinidine when
US Public Health Service. Preexposure prophylaxis for the pre- necessary) and clindamycin. he major risk factors for
vention of HIV infection in the United States—2014. Avail- severe babesiosis in this patient are his history of sple-
able at: http://www.cdc.gov/hiv/pdf/prepguidelines2014. nectomy and rituximab therapy within the last 6–12
pdf. Accessed 29.09.16; McCormack S, Dunn DT, Desai months. His age (i.e., >50 years old) is also a risk fac-
M, et al. Pre-exposure prophylaxis to prevent the acquisition
tor for severe disease. Patients with severe babesiosis
of HIV-1 infection (PROUD): efectiveness results from the
pilot phase of a pragmatic open-label randomised trial. Lan-
should be evaluated for red cell exchange transfusion,
cet. 2016;387:53–60. especially if the patient also has a history of splenec-
tomy and/or recent rituximab therapy. his patient
meets many criteria for red cell exchange transfusion,
45. ANSWER: A. Drug–drug interaction and it should be pursued immediately to prevent fur-
his patient has serotonin syndrome due to interac- ther complications from this disease. Plasmapheresis,
tion of linezolid with duloxetine. Linezolid is a mild steroids, and IVIG (Option D) are not used to treat
monoamine oxidase inhibitor, and when combined babesiosis. Ceftriaxone and doxycycline (Option C)
with certain medications including selective sero- may be used to treat other tick-borne infections (Lyme
tonin reuptake inhibitors, drug-induced serotonin disease and anaplasmosis) but would not be efective
excess can result. he syndrome is characterized by treatment for babesiosis. Notably, babesiosis, Lyme
autonomic instability, cognitive/behavioral changes, disease, and anaplasmosis are transmitted by the same
and neuromuscular excitability; thus the signs and species of tick, I. scapularis, and so coinfection should
symptoms of serotonin syndrome can include tachy- be considered in ill patients. Intravenous artesunate
cardia, hyperthermia, hypertension, agitation, restless- is appropriate therapy for severe malaria but not for
ness, and confusion. his condition can be fatal if not babesiosis.
addressed quickly, which is why there is a black-box Wormser GP, Dattwyler RJ, Shapiro ED, et  al. he clinical
warning on linezolid to avoid concurrent administra- assessment, treatment, and prevention of Lyme disease,
tion with selective serotonin reuptake inhibitors, sero- human granulocytic anaplasmosis, and babesiosis: clinical
tonin-norepinephrine reuptake inhibitors, and some practice guidelines by the Infectious Diseases Society of
other serotonin-active medications. Based on the his- America. Clin Infect Dis. 2006;43:1089–1134; Krause PJ,
tory given, there is no evidence of recurrent soft tissue Gewurz BE, Hill D, et  al. Persistent and relapsing babe-
infection on physical exam and no reason to suspect siosis in immunocompromised patients. Clin Infect Dis.
2008;46:370–376.
MRSA bacteremia without another source, without
murmurs on cardiac exam, and without evidence of
sepsis (blood pressure is high, rather than low). he 47. ANSWER: A. Blood serologic test for Coccidioides
abdominal examination is normal, and there are no his patient most likely has primary pulmonary
reported abdominal symptoms, so C. diicile infection coccidioidomycosis, or “valley fever.” he features of
is also unlikely. his presentation that are consistent with this diagno-
Lawrence KR, Adra M, Gillman PK. Serotonin toxicity associ- sis are as follows: exposure in an endemic area (hik-
ated with the use of linezolid: a review of postmarketing ing in dry windy climate in the American Southwest),
data. Clin Infect Dis. 2006; 42:1578–1583. somewhat immunosuppressed (due to his psoriasis
and methotrexate treatment), and acute-onset illness
46. ANSWER: B. Quinine and clindamycin and con- with fever, fatigue, chest pain, and cough, which all
sideration of red cell exchange transfusion persisted despite antibiotic therapy. he chest x-ray
his patient has severe babesiosis. Intraerythro- may show an iniltrate, or it may be normal. Labo-
cytic parasites (protozoa) are diagnostic of either ratory indings often also show new eosinophilia, but
malaria (caused by multiple diferent Plasmodium spe- this may also be normal. his patient has ongoing
cies) or babesiosis (in the United States most often symptoms, but they are not worsening, so his disease is
caused by Babesia microti). his patient has trav- relatively mild and uncomplicated, despite his under-
eled to Nantucket, where babesiosis is a relatively lying treatment with methotrexate. Serologic testing
26 C HA P T E R 1 Infectious Diseases

for Coccidioides (IgM and IgG) is often diagnostic in K production and therefore hypocoagulable state,
these cases—if the serology is positive, then the patient rather than hypercoagulable state.
most likely has active Coccidioides infection, because Lee CA, hrasher KA. Diiculties in anticoagulation manage-
serologic reactivity tends to wane within months of ment during coadministration of warfarin and rifampin.
an infection. However, it sometimes takes months for Pharmacotherapy. 2001;21:1240–1246.
antibody tests to be positive, so sputum culture may
be the only way to diagnose early infection. Many 49. ANSWER: D. Francisella tularensis
patients with this infection will have self-limited dis- Tularemia is the infection caused by Francisella
ease, often even subclinical. Directed treatment, with tularensis, which is a gram-negative bacterium. People
antifungals such as itraconazole or liposomal ampho- may develop tularemia infection after contact with
tericin, is generally only required in severe, dissemi- infected animals or via insect vectors such as ticks.
nated, or prolonged infection. Most of the cases in the United States are reported
Although the fungal antigen 1,3-β-d-glucan (Option in Arkansas, Missouri, Kansas, South Dakota, Okla-
B) can be positive in active coccidioidomycosis, it is homa, and California. After exposure, patients may
not a speciic test because it can also be positive in the be asymptomatic, or they may develop any one of the
setting of other invasive fungal infections such as dis- six major clinical forms of tularemia: ulceroglandular
seminated candidiasis and histoplasmosis. CT imag- tularemia, glandular tularemia, oculoglandular tulare-
ing would similarly be abnormal but not speciic for mia, pharyngeal (oropharyngeal) tularemia, typhoidal
coccidioidomycosis. Hilar lymph node biopsy (Option tularemia, or pneumonic tularemia. Ulceroglandular
D) may also reveal the diagnosis but would not be an and pneumonic tularemia are the two most commonly
initial diagnostic test of choice. Serum galactomannan diagnosed forms of the disease. his patient has pneu-
(Option E) is only helpful for making the diagnosis monic tularemia. Primary pneumonic disease occurs
of aspergillosis typically in an immunocompromised after inhalation of the organism from a source, such
host. as an infected animal—this is most common among
Galgiani JN, Ampel NM, Blair JE, et al. 2016 Infectious Dis- farmers, sheep workers, landscapers, and hunters.
eases Society of America (IDSA) clinical practice guideline Pneumonic tularemia may present similarly to pneu-
for the treatment of coccidioidomycosis. Clin Infect Dis. monic plague (caused by Yersinia pestis), except that
2016;63:e112–e146. pulmonary disease often consists of peribronchial inil-
trates or lobar consolidations in pneumonic tularemia,
48. ANSWER: E. Drug–drug interaction of warfarin whereas it is often rounded and cavitated pulmonary
with rifampin iniltrates or nodules in setting of pneumonic plague.
Rifampin is not a commonly prescribed antibi- Patients may have some laboratory abnormalities,
otic, but it is used in several speciic situations and including elevated or depressed WBC and abnormal
has numerous drug–drug interactions because it is liver function tests, but laboratory indings may also
a potent inducer of cytochrome P450 3A. Patient be normal. Patients may develop respiratory failure
medication lists should be reviewed carefully for and empyema with pneumonic tularemia, and infec-
potential interactions whenever rifampin is started, tion may even spread beyond the lungs, with compli-
and a plan for monitoring or adjustment in treat- cations such as meningitis and endocarditis. Diagnosis
ment should be made whenever possible. In this case, is primarily by serologies and clinical suspicion, but
rifampin induces the metabolism of warfarin so that the organism may be cultured (with diiculty) if cul-
the patient has approximately half the previously tures are speciically requested on cysteine-containing
available warfarin dose after rifampin is started. If supportive media. When tularemia is suspected, the
the INR is not monitored carefully in order to facili- laboratory must be notiied in order to take special
tate warfarin dose adjustments, then patients will precautions because it is highly transmissible in the
quickly become subtherapeutic in this setting. Even laboratory setting. Treatment is with doxycycline or
with proper monitoring and dose adjustments, some ciproloxacin for mild disease, and with streptomycin
patients are unable to achieve therapeutic warfarin or gentamicin for severe disease.
levels while on rifampin and may require concurrent Tropheryma whipplei is the etiologic agent in Whip-
treatment with enoxaparin or other medications. ple disease, which commonly presents with abdominal
his patient had a recent transesophageal echocar- pain, diarrhea, weight loss, and joint pains. Babesia
diogram, which did not show any valvular vegeta- microti is a parasite that is transmitted by ticks primar-
tions or perivalvular abscess, and the blood cultures ily in New England and to some extent in the upper
on this presentation are negative, so there is no evi- Midwest; babesiosis is characterized by fever, anemia,
dence to suggest treatment failure at this point. Gen- thrombocytopenia, and often respiratory distress, but
tamicin has many associated potential toxicities, but this patient had no anemia or parasites visualized on
it does not cause stroke. Loss of gut lora while on initial laboratory indings. Borrelia lonestari is thought
antibiotic therapy often leads to decreased vitamin to be the causative agent in Southern tick–associated
CHAPTER 1 Infectious Diseases 27

rash illness (STARI), which is a Lyme disease–like which is more often polymicrobial and purulent as
infection described in patients in the southeastern and compared with this presentation of “streptococcal
south-central United States. Anaplasma phagocytophi- gangrene.” Necrotizing soft tissue infections can also
lum causes anaplasmosis, formerly known as human be caused by polymicrobial lora, which can include
granulocytic ehrlichiosis, which is another tick-borne E. coli, other gram-negative pathogens, and anaerobes;
illness that often presents with fever, headache, and lab this type of necrotizing skin and soft tissue infection
abnormalities but rarely causes pulmonary disease. most commonly occurs in the groin and genital area
Centers for Disease Control and Prevention. Tularemia— (“Fournier gangrene”). Clostridium species including
United States, 2001-2010. MMWR Morb Mortal Wkly Rep. C. perfringens and C. septicum also cause necrotiz-
2013;62:963–966. ing skin and soft tissue infection with gas formation
(which was not present in this nonclostridial case) and
50. ANSWER: C. Group A streptococci myonecrosis. Aeromonas hydrophila is a rare cause of
his patient has a necrotizing soft tissue infec- necrotizing skin and soft tissue infection associated
tion with myonecrosis after blunt trauma. his is with trauma in or exposed to fresh water. he history
most often caused by group A beta-hemolytic strep- for this patient was not consistent with this pathogen.
tococci. Treatment is emergent surgical debridement Stevens DL, Bisno AL, Chambers HF, et al. Practice guidelines
and aggressive antibiotics, preferably with high-dose for the diagnosis and management of skin and soft tissue
IV penicillin and clindamycin to reduce toxin forma- infections: 2014 update by the Infectious Diseases Society
tion once the pathogen is identiied. If antibiotics are of America. Clin Infect Dis. 2014;59:147–159.
started in the absence of Gram stain results, then a
more broad-spectrum regimen would be appropriate Acknowledgment
until culture data are available, including vancomycin
and piperacillin-tazobactam or a carbapenem. Staphy- he authors and editors gratefully acknowledge the con-
lococci, including methicillin-resistant or methicillin- tributions of the previous authors—Michael Calderwood,
sensitive, can cause necrotizing soft tissue infections, Rebeca Plank, Dylan Tierney, and Sigal Yawetz.
2
Hematology and Oncology
AMY BESSNOW AND ANN S. LACASCE

1. An 80-year-old man presents with several months of symptoms. Her CA 19-9 is drawn and found to be
epigastric pain, weight loss, fatigue, and dyspnea. he normal. You refer the patient for magnetic resonance
patient was diagnosed with pernicious anemia 5 years cholangiopancreatography (MRCP), which conirms a
ago and has been receiving regular vitamin B12 injec- 1.9-cm mucinous lesion involving the main pancreatic
tions. At that time, he had a colonoscopy, which was duct, with classic features of an intraductal papillary
notable for extensive sigmoid diverticulosis. His com- mucinous neoplasm of the pancreas (IPMN).
plete blood count (CBC) today reveals a hematocrit of What is the proper next step in management?
24% and a mean cell volume (MCV) of 75 fL. A. Follow-up CT scan in 6 months
What malignancy is most likely responsible for his B. Follow-up MRCP in 6 months
symptoms? C. Referral to a pancreatic surgeon for resection
A. Myelodysplastic syndrome D. See the patient in 6–12 months to assess symp-
B. Gastric cancer toms, but obtain no further imaging studies.
C. Colon cancer E. his is a benign inding; no speciic follow-up is
D. Mucosa-associated lymphoid tissue (MALT) lym- required.
phoma
E. Pancreatic cancer 4. A 35-year-old woman originally from the Dominican
Republic comes to the clinic for her irst visit. She has
2. A 45-year-old man presents with back pain, jaundice, no signiicant medical problems and has had two chil-
and weight loss of 20 pounds. A CT scan of the abdo- dren. After the birth of her second child 3 years ago, she
men reveals a large mass in the pancreatic head. His was told to take iron tablets twice daily. Aside from an
cancer antigen (CA) 19-9 is elevated at 496 U/mL. he oral contraceptive, this is her only medication. Results
patient’s family history is notable for the following. His of her laboratory studies are shown in Table 2.1.
father was diagnosed with colon cancer at age 72. His What is the most appropriate management?
mother is an only child and was diagnosed with breast A. Phlebotomy for hemochromatosis
cancer at age 51. His maternal grandfather died of pan- B. Continue current iron therapy and initiate workup
creatic cancer at age 55. His sister was diagnosed with for chronic inlammatory process.
ovarian cancer at 52 and is currently in remission. C. Switch therapy from oral iron sulfate to iron dex-
he patient is most likely to have which of the fol- tran.
lowing hereditary cancer syndromes? D. Discontinue iron therapy; send for ferritin and
A. Lynch syndrome hemoglobin electrophoresis.
B. BRCA2 mutation
C. CDKN2A mutation 5. A 74-year-old man with diabetes mellitus controlled
D. Li-Fraumeni syndrome with an oral agent presents for routine follow-up.
E. CDH1 mutation His other medical problems include hypertension,
for which he takes an angiotensin-converting enzyme
3. A 60-year-old healthy woman undergoes a CT scan (ACE) inhibitor, and benign prostatic hypertrophy.
of the abdomen for signs and symptoms of nephroli- On review of his records, you note that his hema-
thiasis. he scan conirms a small right kidney stone tocrit has been gradually declining over the past 3
but also reveals a 2-cm cystic lesion involving the main years. Results of his laboratory studies are shown in
pancreatic duct. After spontaneously passing the kid- Table 2.2.
ney stone, the patient follows up in your clinic stat- What is the most likely etiology of his anemia?
ing that she feels entirely well. She denies jaundice, A. Combined iron and vitamin B12 deiciency
abdominal or back pain, weight loss, or any other B. Medication efect from the ACE inhibitor

28
CHAPTER 2 Hematology and Oncology 29

TABLE TABLE
2.1 Laboratory Results for Question 4 2.3 Complete Blood Count Results for Question 6

White blood cell 4600/mm3 (4000–10,000) Hemoglobin 5.6 g/dL


count
Hematocrit 17%
Hematocrit 35% (36–48)
Mean cell volume 123 fL
Mean cell volume 66 fL (80–95)
White blood cell count 3500/mm3
Red blood cell 6.0 (4.2–5.6)
count Platelets 70,000/µL

Platelets 256,000/mm3 (150,000–450,000) Bilirubin 2.3 mg/dL

Iron 150 µg/dL (40–159)


Total iron-binding 275 µg/dL (250–400) TABLE
capacity 2.4 Laboratory Results for Question 7

Hemoglobin 11.6 g/dL


TABLE Hematocrit 31%
2.2 Laboratory Results for Question 5
Mean cell volume 83 fL
White blood cell 7000/mm3 (4000–10,000) Reticulocytes 7.0%
count
Platelets 220,000/mm3
Hematocrit 28% (36–48)
White blood cell count 6500/mm3
Mean cell volume 84 fL (80–95)
Blood smear Spherocytes, increased
Red blood cell dis- 15 (10–14.5) reticulocytes
tribution width
Coombs test Negative
Platelets 340,000/mm3 (150,000–450,000)
Blood urea nitrogen 35 mg/dL (9–25)
Creatinine 1.9 mg/dL (0.7–1.3) years. She thinks that she has always tired more easily than
her friends, and she has been told several times that she is
Lactate 230 U/L (107–231)
dehydrogenase anemic. She has been treated on several occasions with
iron pills, but not in the past 2 years. Her physical exami-
nation reveals that she has scleral icterus and a spleen tip
palpable below the left costal margin (Table 2.4).
C. Erythropoietin deiciency he patient is most likely to respond to which of
D. Anemia due to marrow replacement by metastatic the following?
prostate cancer A. Corticosteroids
B. Intravenous iron
6. A 50-year-old man presents complaining of dyspnea on C. Splenectomy
exertion. He was in a car accident 5 years ago with mul- D. Eculizumab
tiple abdominal injuries, resulting in a splenectomy and
a resection of several feet of his terminal ileum. After a 8. A 39-year-old man with sickle cell anemia is admitted
prolonged recovery, he returned to work and his normal for management of pneumonia. He presented with a
activities. For the last 4–6 months, he has had trouble 2-day history of a dry, nonproductive cough and fever
climbing the stairs to his bedroom without stopping to to 101°F, and he was found to have a right lower lobe
catch his breath. He is on no medications, has not lost iniltrate. On admission, his oxygen saturation was
weight, and has a well-balanced diet (see Table 2.3 for 94% on room air, and he was not short of breath.
CBC). Peripheral smear showed macrocytic erythrocytes, He is placed on cefuroxime and given intravenous
hypersegmented neutrophils, and decreased platelets. hydration (Table 2.5). One day later, he complains of
What is the most likely diagnosis? increasing shortness of breath and is found to have an
A. Myelodysplastic syndrome oxygen saturation of 86% on room air. Chest radiog-
B. Vitamin B12 deiciency raphy reveals bilateral lower lobe opacities.
C. Sideroblastic anemia What are the best next steps?
D. Hemolytic anemia A. Continue current antibiotic coverage and admin-
ister supplemental O2.
7. An 18-year-old woman is referred for the evaluation of B. Continue current antibiotic coverage, administer
mild jaundice. She thinks she has had it intermittently for supplemental O2, and obtain a V/Q scan.
30 C HA P T E R 2 Hematology and Oncology

TABLE 10. In which of the following patients should yearly


2.5 Laboratory Results for Question 8 screening with low-dose computed tomography for
lung cancer not be considered?
CBC on Admission A. A 47-year-old woman with a 2-pack-per-day active
White blood 18,000/mm3 (4000–10,000) tobacco use habit and a history of lung cancer in
cell count both her mother and father
Hematocrit 21% (36–48) B. A 57-year-old woman with a history of a 1-pack-
per-day tobacco use habit from ages 15 to 50
Platelets 247,000/mm3 (150,000–
450,000) C. A 70-year-old man with a history of asbestos expo-
sure and 50-pack-year history of cigarette smoking
who quit smoking 12 years ago
D. A 75-year-old woman who has smoked a half-pack
TABLE Laboratory Studies in the Emergency Room of cigarettes per day since age 30
2.6 for Question 9 E. A 60-year-old man with a 50-pack-year history of
cigarette smoking whose last cigarette was after a
White blood cell 14,000/mm3 (4000–10,000)
count heart attack at age 50
Hematocrit 20% (36–48)
11. A 47-year-old woman required emergency craniotomy
Platelets 317,000/mm3 (150,000–450,000) and repair of an aneurysm. On postoperative day 2, she
Reticulocyte count 0.3% (0.6–2.8) develops left calf pain and is found to have thrombus
in the popliteal vessel. Intravenous (IV) unfractionated
Total bilirubin 1.8 mg/dL (0.2–1.2)
heparin is started given her recent neurosurgery. Two
Direct bilirubin 0.3 mg/dL (0.0–0.3) days later, warfarin is started. On postoperative day 7,
Lactate dehydro- 240 U/L (135–225) she is found to have right leg swelling and dyspnea.
genase Evaluation reveals pulmonary emboli and thrombus in
the right common femoral vein. Her platelet count is
noted to be 87,000/µL, down from 320,000/µL at the
time of surgery.
C. Continue current antibiotic coverage, administer Next steps in her management include:
supplemental O2, and transfuse packed red blood A. Immediately stop all heparin exposure, including
cells (PRBCs). IV line lush.
D. Add coverage for atypical organisms and adminis- B. Start IV direct thrombin inhibitor such as argatro-
ter supplemental O2. ban or bivalirudin.
E. Add coverage for atypical organisms, administer C. Give 10 mg IV vitamin K to reverse warfarin.
supplemental O2, and exchange transfuse. D. Test for heparin-platelet factor 4 antibodies.
E. All of the above
9. A 54-year-old African American man with sickle cell
anemia presents to the emergency room with a 2-day 12. A 32-year-old woman 8 weeks postpartum is seen
history of fatigue and pain in his back and lower by her obstetrician for fevers, fatigue, and bruis-
extremities. He scores his pain at 5 out of 10. He has ing. In addition to elevated temperature as well as
had episodes of pain in the past and is well known some bruises and petechiae on examination, she is
to the emergency department staf. His temperature is found to have a hematocrit of 21%, platelet count
99°F, heart rate is 108 beats per minute, blood pressure of 23,000/µL, and creatinine of 2.4 mg/dL. Values
is 120/70 mm Hg, and oxygen saturation on room air at time of delivery were normal. While waiting to get
is 96% (see Table 2.6). He is started on intravenous more laboratory serum tests, she develops right arm
(IV) normal saline and morphine. weakness and confusion. She is sent directly to the
What is the best next step? emergency room and admitted to the intensive care
A. Admit to the hospital and start broad-spectrum unit.
antibiotics. What should your immediate next step be?
B. Admit to the hospital and transfuse packed red A. Request neurology consult for electroencephalo-
blood cells (PRBCs). gram (EEG).
C. Admit to the hospital and arrange for emergent B. Transfuse with platelets.
RBC exchange transfusion. C. Give IV luids for dehydration.
D. Manage pain aggressively and discharge home D. Check peripheral smear and ind schistocytes; ini-
with close hematology follow-up. tiate plasmapheresis.
E. Manage pain aggressively and discharge home on E. Send for ADAMTS13 level and wait for results
oral antibiotics. before treating.
CHAPTER 2 Hematology and Oncology 31

13. A 39-year-old man is seen by a new primary care physi- C. Mixing study
cian. “von Willebrand disease (vWD)” is listed in his past D. Platelet aggregation studies
medical history, but the patient does not know many E. d-Dimer
details of this. He had an episode of gastrointestinal (GI)
bleeding in college requiring hospitalization and RBC 17. A 63-year-old man with type 2 diabetes mellitus and
transfusion when taking nonsteroidal antiinlammatory hypertension presents with recurrent painless hema-
drugs (NSAIDs). He was told he has vWD and to avoid turia. He undergoes a CT scan of the abdomen and
aspirin and NSAIDs. Review of systems is negative for pelvis, which shows an enhancing 6.3-cm left kid-
recent episodes of bleeding, bruising, or epistaxis. His ney mass highly suspicious for a renal cell carcinoma.
father has had some bleeding episodes in the past. here were multiple retroperitoneal enlarged lymph
What tests do you send to evaluate for vWD? nodes, and the lung bases showed four or ive pul-
A. Factor VIII activity monary nodules, with the largest measuring 1.1 cm.
B. vWF antigen level he patient has a good performance status. His CBC
C. Ristocetin cofactor and comprehensive metabolic panel (CMP) results
D. vWF multimer gel electrophoretic analysis were within normal limits, except for a hemoglobin of
E. All of the above 11.5 g/dL.
he next steps would include all except:
14. A 43-year-old woman undergoes elective cholecystectomy A. Biopsy of the kidney mass
for gallstones. Her past medical history is unremarkable, B. Biopsy of the lung lesions
and she is on no medications. On postoperative day 4, C. Chest CT, brain magnetic resonance imaging
she develops left lower-extremity calf pain and is found to (MRI), bone scan
have popliteal vein thrombus by compression ultrasound D. Initiating sunitinib or pazopanib
(US). She has no past history of thrombosis. She weighs E. Proceed with cytoreductive nephrectomy.
68 kg and has normal renal function.
Which is the best treatment option? 18. A 72-year-old man who is an active smoker presents
A. Compression stockings with dysuria and vague abdominal/pelvic pain. His
B. Aspirin 325 mg once daily urinalysis reveals 10–20 RBCs/high-power ield. He
C. Low-molecular-weight heparin overlapping with undergoes an abdominal ultrasound that is unremark-
warfarin targeting international normalized ratio able except for potential thickening of the bladder
1.5–2.0 wall on the left side. Cystoscopy shows a 5-cm bladder
D. Rivaroxaban 15 mg twice daily × 3 weeks, then 20 mass. he patient undergoes transurethral resection of
mg once daily bladder tumor (TURBT), which reveals high-grade
E. Intravenous unfractionated heparin (IV UFH) urothelial cancer with focal invasion into the lamina
propria. No muscle is available in the specimen.
15. he patient in Question 14 has no past history of he next step would be:
thrombosis, despite two pregnancies. Her father had a A. Radical cystectomy
deep vein thrombosis (DVT) at age 43, and his father B. Partial cystectomy (left bladder wall)
had a pulmonary embolism in his 60s. She has two C. Proceed with bacille Calmette-Guérin (BCG)
daughters, ages 12 and 15. immunotherapy.
She should be tested for thrombophilia: D. Repeat TURBT
A. True E. Chemoradiation to the bladder mass
B. False
19. A 51-year-old woman presents with recurrent urinary
16. A 32-year-old woman undergoes laparoscopic resection tract infections (UTIs) for the past 6 months and pel-
of a benign complex ovarian cyst by her gynecologist. vic pain for 6 weeks. A CT scan of her pelvis shows a
One week later she presents complaining of increasing large bladder mass. Cystoscopy followed by TURBT
lower back and abdominal pain. On examination, she shows a poorly diferentiated urothelial carcinoma.
has a 7-cm × 4-cm subfascial hematoma from the umbi- Tumor is invading into the muscularis propria. Chest
licus to the right lower quadrant and a 6-cm × 3-cm and abdominal CT do not show distant metastases or
suprapubic ecchymosis. Her prothrombin time (PT) enlarged lymph nodes. Her serum creatinine is 0.75
is 12.3 seconds (normal 11.2–13.4 seconds), and her mg/dL. Her bone scan is normal. he patient has an
activated partial thromboplastin time (aPTT) is 72.4 excellent performance status.
seconds (normal 23.8–36.6 seconds). Repeat testing Which is the correct statement?
reveals PT 12.8 seconds and aPTT 81.5 seconds. A. Neoadjuvant cisplatin-based chemotherapy is
What is the next test to evaluate an isolated elevated indicated based on a 15% absolute reduction in
aPTT? death.
A. Factor XIII level B. Adjuvant cisplatin-based chemotherapy is indi-
B. Fibrinogen cated based on a 15% absolute reduction in death.
32 C HA P T E R 2 Hematology and Oncology

C. Neoadjuvant cisplatin-based chemotherapy is Which of the following is not a risk associated with
indicated based on a 5% absolute reduction in androgen deprivation therapy?
death. A. Osteoporosis and bone fracture
D. Adjuvant cisplatin-based chemotherapy is indi- B. Prolactinoma
cated based on a 5% absolute reduction in death. C. Hot lashes
E. Proceed with radical cystectomy. D. Anemia
E. Increase in subcutaneous adipose tissue
20. A 59-year-old man, previously healthy and taking only
one 81-mg aspirin per day, was referred for prostate 23. Immune therapy with programmed death receptor 1
biopsy after a screening showed a serum prostate-spe- (PD-1) inhibition can be efective and is often well tol-
ciic antigen (PSA) of 4.8 ng/mL and unremarkable erated in patients with a variety of advanced malignan-
digital rectal examination (DRE). A 12-core needle cies. In which of the following patients should immune
biopsy was performed, revealing two cores on the left therapy with PD-1 inhibition not be considered?
with Gleason 3 + 3 adenocarcinoma involving up to A. A patient with metastatic squamous cell carci-
20% of each core. All other cores showed no evidence noma of the epiglottis that recurs after platinum-
of disease. he patient asks you about imaging to com- based therapy and unknown tumor programmed
plete his diagnostic workup. death receptor ligand 1 (PD-L1) expression
What would you advise? B. A patient with newly diagnosed metastatic non-
A. Endorectal coil MRI only small cell lung cancer, poor performance status,
B. CT scan of the abdomen/pelvis only and 10% tumor PD-L1 expression
C. Bone scan only C. A patient with metastatic nonsmall cell lung can-
D. CT scan of the abdomen/pelvis and bone scan cer refractory to platinum-based chemotherapy
E. No further workup necessary and 0% tumor PD-L1 expression
D. A 30-year-old patient with recurrent classic Hodg-
21. A 61-year-old man presents with a history of hyper- kin lymphoma after autologous transplant and
tension and hyperlipidemia, and he is taking aspirin, brentuximab vedotin
hydrochlorothiazide, and simvastatin daily. He has been E. An elderly patient with metastatic melanoma and
undergoing PSA screening. His digital rectal examina- heart disease
tion has been normal, but a recent PSA of 5.4 ng/mL
prompted a prostate biopsy. Biopsy demonstrated 1 24. A 76-year-old woman with stage IV nonsmall cell lung
of 12 cores positive for Gleason 4 + 3 prostate cancer cancer has been receiving nivolumab for the past 3
involving 15% of the core. His urologist ordered an months after her disease recurred following chemother-
endorectal coil MRI that showed a 1.6-cm left-sided apy with carboplatin and pemetrexed. She was feeling
lesion suspicious for tumor. here was no evidence quite well, but a few days after her last infusion she calls
of extracapsular extension, seminal vesicle involve- to report rapidly progressive shortness of breath and dry
ment, or enlarged lymph nodes. A bone scan was also cough over the past 24 hours. On evaluation, she has
ordered, but it showed no evidence of metastases. he no fever, she is tachycardic to the 110s, her blood pres-
patient is anxious about the side efects associated with sure is mildly elevated at 155/90 mm Hg, and she her
prostate cancer treatment and asks for your guidance. respiratory rate is 22 breaths per minute. Her lungs are
Which of the following is not a recommended clear to auscultation bilaterally, and she is able to speak
option for primary management of this patient’s pros- in full sentences easily, although she becomes dyspneic
tate cancer? walking briskly across the room. She is referred to the
A. Active surveillance ER, where her chest radiograph is unremarkable and
B. Radical prostatectomy electrocardiogram shows only tachycardia. Pulmonary
C. External beam radiation therapy embolism (PE)-protocol CT is without visualized clot
D. Interstitial brachytherapy but is notable for difuse ground-glass opacities and
E. All of the options above are acceptable. areas of mildly increased interstitial markings.
Which of the following interventions is indicated?
22. A 66-year-old man presented with hip pain and was A. A course of antibiotics for suspected pneumonia
ultimately diagnosed with prostate cancer that had B. Enoxaparin for suspected small-vessel PE
metastasized to the ribs and pelvic bones. His PSA at C. Prednisone at least 1 mg/kg or equivalent
the time of diagnosis was 180 ng/mL. He was started D. A and C
on androgen deprivation therapy with leuprolide, a E. A course of antibiotics and inhaled corticosteroids
gonadotropin-releasing hormone (GnRH) agonist.
After 6 months on treatment, his pain resolved and his 25. A 40-year-old woman with a strong family history
PSA has decreased to 0.8 ng/mL. He presents to the of breast cancer is considering taking tamoxifen for
clinic to discuss side efects of his therapy. chemoprevention. She calls to discuss potential side
CHAPTER 2 Hematology and Oncology 33

efects of the medication. She reports she does not did not demonstrate other nodules or mediastinal or
want to experience premature menopause, weight hilar adenopathy.
gain, or depression. What is the most appropriate next step for follow-
What do you advise her? up of the lung nodule?
A. Tamoxifen causes none of these side efects. A. Repeat chest CT in 3 months
B. She should take raloxifene instead. B. Repeat chest CT in 6 months
C. She should take exemestane instead. C. Repeat chest CT in 2 years
D. Tamoxifen may cause weight gain and depression, D. PET/CT now
but not premature menopause. E. Referral for CT-guided biopsy
E. She should wait until after menopause to start
tamoxifen. 29. A 58-year-old woman with minimal prior medical
history and with good performance status is found to
26. A 68-year-old man with a 40-pack-year smoking have a new 2.2-cm peripheral right lower lobe mass.
history and chronic obstructive pulmonary disease Complete staging evaluation demonstrates only the
(COPD) presented to his local emergency room after solitary lung mass, with no other sites of disease in the
falling on his bathroom rug and hitting his head and mediastinum or distantly. CT-guided biopsy conirms
right ribs. A noncontrast head CT scan did not show a diagnosis of nonsmall cell lung cancer, adenocarci-
evidence of bleeding. A chest radiograph did not show noma histology.
any broken ribs, but it did incidentally show a spicu- What is the most appropriate next step in manage-
lated right upper lobe lung mass measuring 3.6 × 2.4 ment?
cm. A subsequent biopsy conirms nonsmall cell lung A. Chemotherapy
cancer. B. Cryoablation
Which of the following radiographic studies is not C. Repeat CT scan in 6 months
recommended as part of a subsequent staging evalua- D. Stereotactic radiation
tion? E. Surgical resection
A. Brain MRI with gadolinium
B. Chest CT with IV contrast 30. A 25-year-old woman presents with a dry cough, inter-
C. Positron emission tomography (PET)-CT mittent drenching night sweats, and difuse pruritus
D. Skeletal survey with plain ilms without identiiable rash. On physical examination,
she appears tired. Her vital signs are notable for a tem-
27. A 63-year-old woman who had smoked two packs of perature of 99.9°F, heart rate of 110 beats per minute,
cigarettes daily for 30 years presents with several weeks blood pressure of 100/80 mm Hg, respiratory rate of
of increasing cough and dyspnea with exertion, fol- 18 breaths per minute, and oxygen saturation of 98%
lowed over the last few days with lethargy and swelling on room air. Her physical examination is notable for
of her face and arms. Initial evaluation in the emer- the absence of peripheral lymphadenopathy. Her chest
gency room reveals a serum sodium of 120 mEq/L, is clear to auscultation bilaterally. Cardiac examination
blood urea nitrogen (BUN) 17 mg/dL, and creatinine reveals tachycardia with a soft systolic ejection mur-
0.8 mg/dL. Chest CT with IV contrast is negative for mur. She had no splenomegaly. Her lower extremities
pulmonary embolus but demonstrates a 5.5-cm right are notable for linear excoriations. Laboratory stud-
hilar mass with associated mediastinal adenopathy and ies reveal a white blood cell count of 14,300/µL with
compression of the superior vena cava; furthermore, 80% neutrophils and 5% lymphocytes, 10% eosino-
there appear to be multiple osseous and hepatic metas- phils, and 5% monocytes. Her basic metabolic panel
tases. is normal.
What is the most likely diagnosis? What is the next most appropriate step in her man-
A. Breast cancer agement?
B. Bronchial carcinoid tumor A. Obtain a lactate dehydrogenase (LDH) level.
C. Non-Hodgkin lymphoma B. Order a PET/CT scan.
D. Small cell lung cancer C. Refer to dermatology for evaluation.
E. hymoma D. Obtain a chest x-ray.
E. Send for Epstein-Barr virus (EBV) serologies.
28. A 70-year-old man with a history of kidney stones
and a 20-pack-year smoking history underwent CT of 31. A 70-year-old man presents to the emergency depart-
the abdomen as part of an evaluation for lank pain. ment for evaluation of upper respiratory symptoms
CT demonstrated recurrence of nephrolithiasis but with low-grade fevers. He appears clinically well. His
also demonstrated an incidental, peripheral, 5-mm physical examination is notable for a 1-cm palpable
left lower lobe (LLL) lung nodule. Chest CT was per- lymph nodes in the bilateral neck and axilla. His
formed and again showed the small LLL nodule but spleen tip is palpable just below the left costal margin.
34 C HA P T E R 2 Hematology and Oncology

Laboratory studies reveal a white blood cell count of thyroid-stimulating hormone (TSH) of 2.2 mIU/L,
18,000/µL with 75% lymphocytes, with smudge cells, and prolactin of 360 ng/mL. You suspect a prolac-
15% neutrophils, and 5% monocytes. His chemistries tinoma and order a pituitary MRI, which reveals a
are normal, including LDH. 10-mm × 12-mm pituitary lesion. here is no cavern-
What is the next most appropriate step? ous sinus invasion or compression of the optic nerves
A. Obtain peripheral blood low cytometry. by the pituitary tumor.
B. Refer for excisional lymph node biopsy. You advise the patient that the initial treatment of
C. Order a PET/CT to evaluate the best node for choice is:
biopsy. A. Medical therapy with a somatostatin analogue
D. Perform a bone marrow aspiration and biopsy. B. Observation only
E. Treat with antibiotics and recheck CBC in 1 C. Transsphenoidal pituitary tumor resection
month. D. Medical therapy with a dopamine agonist
E. Radiation therapy to the pituitary
32. A 65-year-old woman presents complaining of a mass
in the left neck without fevers, night sweats, weight 35. Which of the following antidepressants is preferred in
loss, or localizing symptoms. On examination, she has women taking tamoxifen for hormone receptor–positive
normal vital signs and is found to have a 2.5-cm left breast cancer?
anterior cervical node, a 2-cm left axillary node, and A. Paroxetine
a 3-cm right inguinal node. Her examination is oth- B. Fluoxetine
erwise normal. Laboratory studies, including a CBC C. Venlafaxine
with diferential, basic metabolic panel, and LDH, are D. Bupropion
normal. E. Duloxetine
What is the next most appropriate step in her man-
agement? 36. Which of the following patients would be a good can-
A. Refer to interventional radiology for a needle didate for novel oral anticoagulant (NOAC) therapy?
biopsy of the most easily accessible lymph node. A. A 65-year-old woman with rheumatic heart dis-
B. Refer to otolaryngology for ine-needle aspiration ease and a metal prosthetic mitral valve
of the neck mass. B. A 35-year-old woman with a newly diagnosed
C. Refer to a surgeon for an excisional lymph node PE, history of multiple second-trimester preg-
biopsy. nancy losses, and positive antiphospholipid anti-
D. Obtain a PET/CT scan. bodies
E. Send for peripheral blood low cytometry. C. A 60-year-old man with new-onset nonvalvular
atrial ibrillation and a history of a diverticular
33. A 66-year-old postmenopausal woman with hyper- bleed 5 years ago
tension and hypothyroid is diagnosed with stage D. A 60-year-old man with a newly diagnosed PE and
IA ER/PR-positive, HER2/neu-negative invasive metastatic nonsmall cell lung cancer
ductal carcinoma. She undergoes lumpectomy fol- E. A 60-year-old man with newly diagnosed DVT
lowed by radiation, and chemotherapy is deferred and renal insuiciency
based on a low risk of recurrence and low likelihood
of beneit. She is started on an aromatase inhibitor 37. A 64-year-old woman presents with abdominal bloat-
(AI). ing, early satiety, and constipation. On bimanual
For which of the following is she now at signii- exam, an enlarged right ovary is noted. Ultrasound
cantly increased risk due to her aromatase inhibitor? demonstrates a complex 10-cm × 8-cm mass highly
A. DVT/PE concerning for malignancy as well as a moderate
B. Hypothyroid amount of free luid in the pelvis.
C. Osteoporosis Which of the following is not potentially indicated?
D. Endometrial cancer A. Abdominal/pelvic CT
E. Glucose intolerance B. Laboratory testing for cancer antigen 125 (CA-125)
C. Colonoscopy
34. A 35-year-old woman with a history of regular men- D. Referral to gynecologic oncology for consideration
ses presents with complaints of bilateral galactorrhea. of surgery
She notes that her menses have been more irregular E. Referral to interventional radiology for needle-
over the last year and that her last menstrual period guided biopsy of the adnexal mass
was approximately 6 weeks ago. She reports occa-
sional headaches but otherwise feels well. She does 38. A 22-year-old man presents with headache and
not take any medication or herbal supplements. peripheral vision loss. Brain imaging reveals a large sel-
Laboratory testing reveals a negative pregnancy test, lar mass with both solid and cystic components that is
CHAPTER 2 Hematology and Oncology 35

compressing the optic chiasm. Humphrey visual ield presents to Student Health for evaluation. Her physi-
testing demonstrates bitemporal hemianopsia. He cal examination is notable for scattered, quarter-sized
has no previous medical history, his body mass index bruises on her upper arms and thighs. Laboratory stud-
(BMI) is 20 kg/m2, and he is taking no medications. ies are notable for a white blood cell count of 500/µL,
His initial pituitary functional evaluation revealed hemoglobin of 7 g/dL, and platelets of 50,000/µL.
central hypogonadism and central hypothyroidism. A peripheral smear is notable for 80% large atypical
His adrenal function was normal. MRI characteris- cells with folded, bilobed, kidney-shaped nucleoli and
tics suggested the mass was a craniopharyngioma. He elongated, bluish-red rods within the cytoplasm.
was taken to surgery to remove the pituitary mass in What translocation is the deining feature of this
attempts to decompress the optic chiasm and restore disease?
his vision. He did well in the immediate postopera- A. t(11;14)
tive period, but within 24 hours, he began to develop B. t(15;17)
polyuria. His urine output increased to 400 mL/h and C. t(9;22)
was very dilute with a urine speciic gravity of less than D. t(8;14)
1.001. He complained of extreme thirst. His serum E. t(14;18)
sodium increased to 148 mEq/L, and his fasting glu-
cose was elevated at 106 mg/dL. 41. A 65-year-old man with past medical history of hyper-
What diagnosis are you suspecting in this patient? tension presents to his primary care physician (PCP)
A. Syndrome of inappropriate antidiuretic hormone with several weeks of fatigue and dyspnea on exer-
secretion (SIADH) tion (DOE). He notes feeling more winded during
B. Diabetes mellitus, type 2 his 3-mile walks. His vital signs are stable. His blood
C. Central diabetes insipidus pressure is 120/60 mm Hg, and his O2 saturation is
D. Nephrogenic diabetes insipidus 97% on room air. He had no hepatosplenomegaly. His
E. None of the above complete blood count is notable for white blood cell
count of 4000/µL with 60% neutrophils, hemoglobin
39. A 55-year-old woman was involved in a motor vehi- of 9 g/dL, and platelets of 110,000/µL. he laboratory
cle accident as an unrestrained driver. Although she indings are notable for a vitamin B12 of 700 pg/mol,
did not lose consciousness, she sustained a signiicant folate of 20 ng/mL, TSH of 1.2 mIU/mL, testosterone
head trauma and was brought by ambulance to the of 400 ng/dL, and erythropoietin of 90 U/L. A bone
emergency department. She was evaluated and found marrow biopsy is done and is remarkable for dysplas-
to have a normal neurologic examination, but due to tic erythroid precursors and megakaryocytes with 1%
the mechanism of her accident, brain imaging by non- blasts. Cytogenetics are normal. He has 2 points—1
contrast CT scan was performed. She was incidentally for Hgb of 9 g/dL and 1 for normal cytogenetics—
found to have a large sellar lesion, estimated at approx- based on his Revised International Prognostic Scoring
imately 2 cm in greatest diameter. No acute intracra- System (IPSS-R) score.
nial hemorrhage was identiied. She had a laceration What would be the most appropriate recommenda-
on her forehead that was repaired, and because she was tion for therapy for his myelodysplastic syndrome?
otherwise clinically stable, she was discharged with a A. Allogeneic stem cell transplant
plan to follow up with her primary care provider for B. Induction chemotherapy with an anthracycline +
further evaluation of this pituitary mass. cytarabine
In evaluating a patient with a newly discovered C. Lenalidomide
pituitary mass, what are the important initial clinical D. Erythropoiesis-stimulating agent (ESA)
considerations? E. No treatment necessary
A. Evaluation for mass efects (headaches, visual loss,
cranial nerve abnormalities) 42. A 35-year-old woman presents to her PCP for her
B. Evaluation for pituitary hormonal hypersecretion annual visit. She has been feeling well. She denies
C. Evaluation for pituitary hormonal hypofunction fevers, night sweats, and weight loss. She denies GI
D. All of the above symptoms, including nausea, vomiting, abdominal
E. None of the above pain, and early satiety. She denies easy bleeding, bruis-
ing, or history of clots. On examination, her spleen
40. A 22-year-old college student notes increasing fatigue is palpable two ingerbreadths below the costal mar-
over the past few weeks that she attributes to staying gin. A complete blood count is notable for a white
up late studying for inal examinations. Over the past blood cell count of 60,000/µL, hemoglobin of 10 g/
2–3 days she has noted several new bruises on her dL, and platelets of 600,000/µL. Review of the periph-
upper and lower extremities. She does not recall ante- eral smear is notable for a large percentage of basophils
cedent trauma leading to the bruising. his morning and eosinophils, along with myelocytes and metamy-
she developed a bloody nose lasting 20 minutes. She elocytes. JAK2 V617F mutation is negative.
36 C HA P T E R 2 Hematology and Oncology

What is the most appropriate next step in manage- E. Despite normal vWF levels, pregnant women with
ment? vWD have increased risk of bleeding during deliv-
A. Start hydroxyurea. ery and should be treated prophylactically with
B. Check blood cultures and start empiric antibiotics Humate-P.
for possible infection.
C. Start high-dose aspirin (325 mg twice daily). 45. A 68-year-old lifelong-nonsmoking woman in excel-
D. Send peripheral blood for cytogenetics to evaluate lent general health is brought to the ER after seizure-like
for the t(9;22) chromosome translocation. activity was observed by her family during dinner. She is
E. Determine a leukocyte alkaline phosphatase (LAP) somnolent on arrival but within 30 minutes is alert and
score. oriented and without focal neurologic indings. Noncon-
trast head CT shows no bleed, and brain MRI with and
43. A 60-year-old woman with past medical history of without contrast demonstrates three foci of enhancement
hypertension and COPD is brought to the emergency measuring up to 2 cm and located in the right parietal,
room by her daughter. She is lethargic and is having occipital, and left frontal lobes. here is surrounding vaso-
diiculty walking. She has noted increasing dii- genic edema, particularly of the largest lesion in the left
culty breathing over the past few hours. Her physi- frontal cortex. However, there is no mass efect or shift.
cal examination is notable for temperature of 100.7°F, CT of the chest, abdomen, and pelvis demonstrate a spic-
blood pressure of 150/70 mm Hg, respiratory rate of ulated left upper lobe lung mass and multiple bony lesions.
24 breaths per minute, and O2 saturation of 88% on Biopsy of the left upper lobe mass demonstrates poorly dif-
room air. She has diiculty with inger-to-nose and ferentiated adenocarcinoma consistent with lung primary.
rapid alternating movements. She has ine crackles Which step would not be included in the manage-
bilaterally. A complete blood count is notable for a ment of this patient?
white blood cell count of 110 × 106, hemoglobin of 8, A. Radiation oncology consultation for consideration
and platelets of 75,000/µL. A chest x-ray shows bilat- of urgent whole-brain radiotherapy (WBRT)
eral iniltrates. B. Intravenous dexamethasone
What would be the appropriate next step in man- C. Initiation of levetiracetam
agement of this patient? D. Radiation oncology consultation for consideration
A. Give a unit of PRBCs. of stereotactic radiosurgery (SRS)
B. Start antibiotics for possible pneumonia and fol- E. Molecular testing of tumor specimen for targe-
low for symptom improvement before initiating table mutations
more invasive procedures.
C. Start steroids and antibiotics for COPD lare. 46. In which of the following patients would you be most
D. Start leukapheresis while establishing diagnosis. concerned about initiating therapy with ibrutinib for
E. Establish diagnosis and start appropriate therapy. symptomatic chronic lymphocytic leukemia (CLL)?
A. A patient with bulky nodal disease and anemia
44. A 32-year-old woman presents in active labor on a Fri- B. A patient with a p53 mutation and rapid disease
day evening. She tells the admitting obstetrician who recurrence after ludarabine, cyclophosphamide,
is covering for a group practice that she has a history of and rituximab
heavy periods and mucosal bleeding and was told she C. A patient with a history of pneumocystis pneumo-
has von Willebrand disease. She uses a nasal inhaler nia and recurrent varicella zoster virus infections
at the start of her periods, and it reduces bleeding. D. A patient with poorly controlled atrial ibrillation
Her mother and sister have similar bleeding histories. and platelets of 20,000/µL
You ind laboratory values done prior to her preg- E. A patient with a history of immune colitis
nancy showing vWF antigen of 42%, vWF activity of
38%, and factor VIII of 49%. All of these are below 47. Your patient is a 56-year-old schoolteacher who pre-
the lower limits of normal. here is a note stating that sented with about 2 months’ history of intermittent
the values are compatible with type I von Willebrand blood in the toilet bowl with bowel movements. A
disease. colonoscopy is performed that demonstrates a mass in
Which of the following statements is/are true? the midsigmoid, and biopsy conirms adenocarcinoma.
A. von Willebrand factor levels go down during preg- He undergoes laparoscopic hemicolectomy, and inal
nancy. pathology reveals a 3-cm, moderately diferentiated
B. Placement of an epidural catheter is contraindi- adenocarcinoma invading through the muscle layer into
cated in a patient with von Willebrand disease. the serosa with two of nine lymph nodes positive.
C. he correct treatment would be infusion of recom- What would be the next step?
binant coagulation factor VIIa (NovoSeven). A. Reoperate for more complete lymph node dissection.
D. von Willebrand factor levels become normal dur- B. Referral to medical oncologist for consideration of
ing pregnancy in patients with type I disease. chemotherapy
CHAPTER 2 Hematology and Oncology 37

C. Follow-up colonoscopy in 1 year lumbosacral back pain for 2 weeks. His physical exam-
D. Referral to radiation oncologist for postoperative ination is normal except for localized tenderness over
radiation L4, with no neurologic deicits.
What is the best next step?
48. A 42-year-old man newly diagnosed with Burkitt lym- A. Increase his scheduled pain medications and see
phoma presents to the emergency room 3 days after him for follow-up in 1 month.
beginning chemotherapy. He complains of decreased B. Arrange for an urgent MRI of the spine to evaluate
urine output. Laboratory data show serum potassium for possible cord compression.
of 6.5 mg/dL, BUN of 48 mg/dL, serum creatinine of C. Arrange for radiation therapy within 24 hours for
2.4 mg/dL, uric acid of 10.9 mg/dL, and phosphorus probable cord compression.
of 6.4 mg/dL, with uric acid crystals on his urinalysis. D. Emergency neurosurgical evaluation for suspected
He is already on allopurinol. spinal cord compression
What treatment is least likely to be efective at this
point? 50. Which one of the following statements is true regard-
A. Hydration and furosemide, monitoring urine out- ing superior vena cava (SVC) syndrome?
put and weight to maintain euvolemia A. Establishing a tissue diagnosis is the most impor-
B. Acute management of hyperkalemia with calcium tant step in management of most patients with
gluconate and insulin and dextrose SVC syndrome.
C. Initiation of hemodialysis if diuresis does not occur B. SVC syndrome should be treated with IV throm-
D. A recombinant uricase, such as rasburicase bolytics and stenting of the vena cava.
E. Increase in dose of allopurinol C. Diagnosis is usually established by chest x-ray,
then venography if needed.
49. A 69-year-old man with prostate cancer and wide- D. Breast cancer and germ cell tumors are the most
spread bone metastases complains of worsening common malignant causes of SVC syndrome.

Chapter 2 Answers
1. ANSWER: B. Gastric cancer and pancreatic cancer, is highly suggestive of BRCA
Pernicious anemia is an autoimmune disorder that mutation. Both BRCA1 and BRCA2 mutations are
is characterized by autoantibodies against intrinsic associated with increased risk of pancreatic cancer,
factor, destruction of gastric parietal cells, vitamin but BRCA2 is much more strongly associated. BRCA2
B12 deiciency, and gastric achlorhydria. Over time, mutation is thought to be responsible for approxi-
the condition progresses to chronic atrophic gastritis, mately 15% of familial pancreatic cancer. Although
intestinal metaplasia, dysplasia, and ultimately gastric Lynch syndrome (hereditary nonpolyposis colorec-
adenocarcinoma. he patient described here has devel- tal cancer [HNPCC]) is associated with a modestly
oped occult upper gastrointestinal (GI) bleeding from increased risk of pancreatic cancer, his father’s diagno-
gastric cancer, stemming from long-standing perni- sis of colon cancer at age 72 is not highly suggestive of
cious anemia and atrophic gastritis. Myelodysplastic Lynch syndrome. Li-Fraumeni syndrome results from
syndrome has not been associated with pernicious ane- a mutation in the p53 tumor suppressor gene, and it
mia, nor is it associated with iron deiciency. Although is most notably associated with early diagnosis of sar-
the patient likely has iron-deiciency anemia, perni- coma, brain tumors, breast cancer, and leukemia, but
cious anemia is also not associated with colon can- not pancreatic cancer. Mutations in CDKN2A result
cer. Additionally, an advanced colon cancer would in the familial atypical multiple-mole melanoma syn-
be unlikely, given the patient’s colonoscopy indings drome, which is associated with markedly increased
5 years previously. MALT lymphoma is highly linked risk of melanoma and pancreatic cancer. Germline
to Helicobacter pylori infection, not pernicious anemia. mutations in CDH1 (E-cadherin) are associated with
Although some reports indicate an increased risk of a very high risk of difuse gastric cancer and breast can-
pancreatic cancer associated with pernicious anemia, cer, but not pancreatic cancer.
pancreatic cancer would not cause iron-deiciency
anemia. 3. ANSWER: C. Referral to a pancreatic surgeon for
resection
2. ANSWER: B. BRCA2 mutation he patient was incidentally found on an imaging
his patient presents with a likely diagnosis of pan- study to have a main duct IPMN. IPMNs are prema-
creatic adenocarcinoma. In addition to his young age, lignant lesions of the pancreatic ductal epithelium,
his family history raises concern for a possible heredi- which have the potential to progress to pancreatic
tary cancer syndrome. he patient’s maternal family adenocarcinoma. IPMNs involving the side branches
history, including early breast cancer, ovarian cancer, of the pancreatic ductal system (so-called branch duct
38 C HA P T E R 2 Hematology and Oncology

IPMNs) carry an approximate 10%–20% risk of pro- blood cell distribution width (RDW) to be abnor-
gressing to pancreatic cancer. For patients with branch mally high. Furthermore, vitamin B12 deiciency
duct IPMNs who do not wish to have surgery, active (which can be a complication of metformin) should
surveillance with serial MRI or pancreas protocol CT also depress the white blood cell count and the plate-
is considered an acceptable option. his patient has let count. ACE inhibitors are not typically associ-
an IPMN of the main pancreatic duct, which carries ated with anemia. In a patient with benign prostatic
approximately a 70% risk of progression to pancreatic hypertrophy, the sudden development of prostate
adenocarcinoma. For this reason, all patients diag- cancer leading to marrow iniltration and anemia
nosed with a main duct IPMN, without signiicant with a normal white count and platelet count and
contraindications to surgery, should be referred to an a normal lactate dehydrogenase (LDH) would seem
experienced pancreatic surgeon for consideration of highly unlikely.
resection.
Tanaka M, Fernández-del Castillo C, Adsay V, et  al. Inter- 6. ANSWER: B. Vitamin B12 deiciency
national consensus guidelines 2012 for the manage- his patient has megaloblastic anemia secondary to
ment of IPMN and MCN of the pancreas. Pancreatology. malabsorption of vitamin B12, which is absorbed in
2012;12(3):183–197. the terminal ileum. Because people with a good diet
usually have substantial body stores of vitamin B12,
4. ANSWER: D. Discontinue iron therapy; send for patients typically do not develop vitamin B12 dei-
ferritin and hemoglobin electrophoresis. ciency until several years following failure of vitamin
his young woman has a microcytic anemia B12 absorption. Megaloblastic anemia causes pancyto-
despite 3 years of oral iron therapy. Because patients penia because the failure of DNA synthesis afects all
are frequently noncompliant with oral iron, one rapidly dividing cells. Patients may also have mouth
must always consider that the patient has not taken ulcers, symptoms of malabsorption, smooth tongue,
the medication. Although the best measure of iron and neurologic abnormalities.
stores would be to check serum ferritin, her iron and Sideroblastic anemia typically presents with a
total iron-binding capacity (TIBC) show an iron microcytic anemia. Hemolytic anemia should not
saturation of over 50%, so it seems unlikely that cause leukopenia or thrombocytopenia. Although
she remains iron deicient. A ferritin level will allow myelodysplasia can be associated with macrocytosis, it
you to determine if she is iron replete or iron over- rarely causes an MCV over 100–110 fL. Furthermore,
loaded. he profound microcytosis in the face of a although you may see pseudo Pelger-Huët anomaly
near-normal hematocrit, mildly elevated RBC count, (bilobed neutrophils) in the neutrophils in patients
and presumed adequate iron stores should lead one with myelodysplastic syndrome, hypersegmented
to consider a diagnosis of thalassemia minor. Indeed, polys are seen only in megaloblastic anemia.
an MCV this low is extremely rare in iron deiciency Stabler SP. Vitamin B12 deiciency. N Engl J Med. 2013;
and would never occur in the absence of profound 368(21):149–160.
anemia. Beta-thalassemia minor can be diagnosed
by hemoglobin electrophoresis, and it is established 7. ANSWER: C. Splenectomy
by the demonstration of an elevated hemoglobin A2 he patient has anemia with increased reticulocytes,
(>4%). Given the minimal anemia, it is even more and spherocytes are seen on the peripheral smear. he
likely that this represents alpha-thalassemia, which diagnostic distinction to be made is between a con-
can be diagnosed only by DNA studies, because it genital hemolytic anemia and an immune hemolytic
does not result in an abnormal hemoglobin electro- anemia. he patient’s long history of jaundice and the
phoresis. relatively well-compensated hemolysis is most sugges-
Peters M, Heijboer H, Smiers F, Giordano PC. Diagnosis and tive of hereditary spherocytosis. Hereditary sphero-
management of thalassaemia. BMJ. 2012;344:e228. cytosis, if severe enough to require therapy, responds
very well to splenectomy.
5. ANSWER: C. Erythropoietin deiciency he negative Coombs test makes autoimmune
he patient has a normochromic, normocytic hemolytic anemia (AIHA) unlikely, although about
anemia. he most likely reason for the gradual dec- 10% of patients with AIHA may have a negative
rement in his hematocrit is that he has had a gradual Coombs test. Steroids are the preferred irst-line ther-
decline in his erythropoietin level coincident with apy for AIHA. he reticulocytosis makes iron dei-
his moderate renal insuiciency. his can be readily ciency extremely unlikely. Although hemolysis is the
assessed by checking an erythropoietin level, which hallmark of paroxysmal nocturnal hemoglobinuria
should be several hundred with a hemoglobin/ (PNH), patients with PNH have intravascular hemo-
hematocrit in this range. lysis secondary to surface activation of complement
Although this could be a result of combined iron and therefore do not present with spherocytes in the
and vitamin B12 deiciency, one would expect the red peripheral blood.
CHAPTER 2 Hematology and Oncology 39

8. ANSWER: E. Add coverage for atypical organisms populations, which demonstrated a statistically signii-
and administer supplemental O2, and exchange cant decrease in lung cancer deaths in the CT-screened
transfusion. group (lung cancer mortality reduction of 20% [CI
he patient presented with a clearly deined right 3.8–26.7], all-cause mortality reduction of 6.7% [CI
lower lobar pneumonia that progressed to acute chest 1.2–13.6]). Several other screening studies are under-
syndrome (ACS). ACS is an acute illness characterized way or under analysis. Additionally, there are multiple
by a constellation of fever, chest pain, shortness of breath, guidelines as to appropriate populations in which to
hypoxia, and a new iniltrate(s) on chest radiograph. ACS consider screening. he broadest guidelines include
is a life-threatening complication of sickle cell disease individuals with all the following characteristics:
often caused by atypical organisms such as Mycoplasma • Aged 55–79
pneumoniae. It is not uncommon for adults with sickle • At least a 30-pack-year smoking history
cell disease to present with a pain crisis or pneumonia • Active tobacco use within the past 15 years
and deteriorate in the hospital to ACS. he treatment for he duration of screening is unclear, but all guide-
ACS is antibiotic coverage for typical and atypical organ- lines recommend yearly screening for a minimum of
isms, red blood cell exchange transfusion to a goal hemo- 3 years and a maximum recommendation of yearly
globin S (sickle) <30% and other supportive care. While screening until patients no longer meet high-risk crite-
simple transfusion can be very helpful to increase the oxy- ria. here are no data at this time to suggest a mortality
gen-carrying capacity of blood and blood volume itself, beneit in screening patients under age 50.
it does not rapidly decrease the percentage of circulating National Lung Screening Trial Research Team. Reduced lung-
sickle cells. hus in situations such as ACS where sickling cancer mortality with low-dose computed tomographic
of RBCs is the chief problem, exchange transfusion is the screening. N Engl J Med. 2011;365(5):395–409.
preferred treatment. A pulmonary embolus would not
explain this patient’s new pulmonary iniltrates. 11. ANSWER: E. All of the above
Vichinsky EP, Neumayr LD, Earles AN, et  al. Causes and his patient has a clinical diagnosis of heparin-
outcomes of the acute chest syndrome in sickle cell disease. induced thrombocytopenia (HIT) with develop-
N Engl J Med. 2000;342(25):1855–1865. ment of new clots while on heparin for 5 days, and
with a platelet count decrease greater than 50% of
9. ANSWER: B. Admit to the hospital and transfuse initial count. Despite recent neurosurgery, lifesaving
packed red blood cells. treatment requires doing A, B, and C immediately.
Vasoocclusive pain crises are the hallmark of sickle Although testing should be sent to conirm diagnosis,
cell disease, and patients may be admitted to the hospital waiting for results before starting treatment is contrain-
very frequently for pain control. However, not all presen- dicated and can lead to poor outcome, including death.
tations of individuals with sickle cell disease are for a pain Greinacher A. Heparin-induced thrombocytopenia. N Engl J
crisis. A signiicant inding in this presentation is ane- Med. 2015; 373(3):252–261.
mia accompanied by reticulocytopenia. his patient has
transient red cell aplasia (TRCA) due to parvovirus B19. 12. ANSWER: D. Check peripheral smear and ind
His anemia is likely to worsen because of the shortened schistocytes; initiate plasmapheresis.
lifespan of sickled red blood cells (about 20 days) and the his patient has the classic pentad of indings asso-
inability of his bone marrow to produce reticulocytes. ciated with thrombotic thrombocytopenic purpura
Reticulocytopenia begins about 5 days postexposure and (TTP). Although disseminated intravascular coagu-
continues for 7 to 10 days. Recovery is spontaneous, but lation (DIC) from retained products of conception
support with RBC transfusions is necessary. he two could result in microangiopathic changes and throm-
indications for RBC exchange transfusion are stroke/ bocytopenia, DIC is not usually associated with renal
TIA and acute chest syndrome. his patient has no indi- failure or focal neurologic indings. Risk factors for
cation for broad-spectrum antibiotics. TTP include pregnancy, HIV infection, autoimmune
Serjeant BE, Hambleton IR, Kerr S, et  al. Haematological disorders, and others. Rapid initiation of plasmapher-
response to parvovirus B19 infection in homozygous sickle- esis is critical, even if conirmatory laboratory test
cell disease. Lancet. 2001;358(9295):1779–1780. results are not available.
George JN, Nester CM. Syndromes of thrombotic microangi-
10. ANSWER: A. A 47-year-old woman with a 2-pack- opathy. N Engl J Med 2014;371(7):654–666.
per-day active tobacco use habit and a history of
lung cancer in both her mother and father 13. ANSWER: E. All of the above
Low-dose CT screening in high-risk patient popula- his patient gives a good history for vWD, with
tions has been associated with the potential for a signii- mucosal bleeding in the setting of impaired plate-
cant reduction in lung cancer mortality. he National let function due to NSAIDs that was severe enough
Lung Screening Trial (NLST) compared annual low- to require red cell transfusion. All of the tests listed
dose chest CT to chest x-ray for 3 years in high-risk above are required to make a diagnosis of vWD and
40 C HA P T E R 2 Hematology and Oncology

determine the type (I, II, or extremely unlikely III). be normal. Platelets have no impact on the aPTT.
vWF antigen level measures the actual vWF protein d-Dimer results would not be of help in determining
level, but vWF also functions to carry FVIII and pro- the etiology of prolonged aPTT in this postoperative
long its plasma half-life as well as bind to platelets— patient with normal PT.
hence the need to measure factor VIII activity and
ristocetin cofactor activity, a functional surrogate assay 17. ANSWER: D. Initiating sunitinib or pazopanib
for patient vWF platelet binding. vWF multimer gel It is mandatory to have tissue diagnosis prior to
electrophoresis determines vWF multimer size, which initiating systemic therapies. Although renal cell carci-
is needed in order to classify vWD type II subtypes. noma (RCC) is statistically by far the most likely diag-
Leebeek FWG, Eikenboom JCJ. Von Willebrand’s disease. nosis, transitional cell cancer (TCC) of the renal pelvis
N Engl J Med. 2016;375(21):2067–2080. is a consideration. In that case, removal of the primary
would not be indicated, and the patient would pro-
14. ANSWER: D. Rivaroxaban ceed with a diferent systemic therapy.
his patient is an ideal candidate for treatment of acute
deep vein thrombosis (DVT) with rivaroxaban—she is 18. ANSWER: D. Repeat TURBT
young, has no signiicant health problems or medications, It is very important to have muscle present on the
and has limited duration of therapy for provoked venous biopsy report to document whether muscle invasion is
thromboembolism (VTE). his dose (15 mg twice daily present for appropriate staging and to guide therapy.
for 3 weeks followed by 20 mg once daily) is the treat- Supericial disease: BCG for high-risk supericial
ment dose for acute DVT or pulmonary embolism. disease.
Aspirin or compression stockings are inadequate Muscle-invasive disease: neoadjuvant chemother-
treatment for acute VTE. A target INR of 1.5–2.0 is too apy + cystectomy.
low for acute VTE treatment. Although IV UFH could
be considered, at 96 hours postoperatively, she should 19. ANSWER: C. Neoadjuvant cisplatin-based chemo-
have adequate hemostasis to allow oral anticoagulant therapy is indicated based on a 5% absolute reduc-
therapy with a short-acting new oral anticoagulant. tion in death.
In patients with good performance status and good
15. ANSWER: A. True kidney function, neoadjuvant cisplatin-based chemo-
Although she has had a provoked DVT, her father had therapy is standard. It is associated with a 5% abso-
a DVT at a young age and she has two daughters who lute reduction in death and 15% relative reduction in
might consider oral contraceptive pill (OCP) use. Find- death. Data for adjuvant therapy are limited, although
ing an inherited thrombophilia in this patient would not in practice it is frequently considered in patients who
change current management or duration of anticoagula- did not receive neoadjuvant therapy. he preferred
tion treatment for provoked DVT (3 months), but if approach in this healthy patient with normal renal
present, it would prompt testing of her daughters prior function is neoadjuvant cisplatin-based chemotherapy
to OCP use. Additionally, results could afect manage- followed by radical cystectomy.
ment of potential future pregnancies of her daughters. Bellmunt J, Orsola A, Leow JJ, et al. Bladder cancer: ESMO
Practice Guidelines for diagnosis, treatment and follow-up.
16. ANSWER: C. Mixing study Ann Oncol. 2014;25(suppl 3):iii40–iii48.
A mixing study should be done next to determine if
the aPTT is prolonged due to the absence of a coagula- 20. ANSWER: E. No further workup necessary
tion factor in the intrinsic coagulation pathway or to the he most common sites of prostate cancer metas-
presence of a circulating inhibitor. A mixing study is per- tases are bone and lymph nodes. For patients with
formed by mixing patient plasma with normal pooled low-risk disease (T1c [normal DRE] or T2a [tumor
plasma in a 1:1 ratio. he normal plasma supplies involving one-half or less of one lobe] and PSA <10 ng/
enough of the coagulation factors to correct a congeni- mL and Gleason score <7), the likelihood of spread of
tal deiciency and normalize the aPTT. If an inhibitor is disease visible by imaging is exceedingly low. Further
present, such as an acquired FVIII inhibitor, lupus anti- workup is not recommended for these patients. CT of
coagulant, or even drugs such as unfractionated heparin the abdomen/pelvis and bone scan are recommended
or bivalirudin, the addition of plasma will not correct for men with serum PSA >20 ng/mL, or serum PSA
the aPTT, because the factors in the normal plasma will >10 ng/mL with a positive DRE, or a Gleason ≥8
be afected. If the mixing study corrects the aPTT, then tumor, or evidence of tumor extension through the
individual factor levels can be performed to determine prostate capsule. Several institutions use endorectal
which is deicient (XII, XI, IX, VIII). If the aPTT does coil MRI as part of the initial workup, but the clinical
not correct, tests for inhibitors need to be performed. utility of this modality is not yet clear.
Factor XIII activity is not measured by the aPTT. Mohler JL, Armstrong AJ, Bahnson RR, et al. Prostate cancer,
Because the PT is normal, ibrinogen levels should version 1.2016. J Natl Compr Canc Netw. 2016;14(1):19–30.
CHAPTER 2 Hematology and Oncology 41

21. ANSWER: A. Active surveillance can bind to PD-1 on T cells, one strategy for tumor eva-
Given the long natural history of prostate cancer, sion of the immune response. Blockade of PD-1 with
the frequency of indolent disease in a high percent- pharmaceutical inhibitors (currently approved agents:
age of patients, and signiicant morbidity associated nivolumab, pembrolizumab) can result in increased
with local treatment, active surveillance is increasingly immune recognition and killing of tumor cells. PD-1
recommended by urologists and medical oncologists. inhibition is currently approved in all of the settings in
Prospective series clearly demonstrate that active sur- this question with the exception of up-front therapy for
veillance (serial PSAs and physical examinations every stage IV lung cancer where tumor PD-L1 expression
3–4 months along with repeat biopsies every 1–2 years) is less than 50%. In most instances, quantiication of
can safely be used to manage certain patients, avoiding tumor PD-L1 expression is not required for initiation
(or at the very least forestalling) life-altering therapies. of therapy, although response rates appear to be higher
However, not all patients are good candidates for active in many cases in tumors with increased PD-L1 expres-
surveillance. For patients <70 years old, criteria for sion. he exception is frontline therapy for metastatic
entry into most active surveillance series include early- lung cancer, where >50% PD-L1 expression is required.
stage disease, Gleason score ≤6, and serum PSA ≤10 ng/ Studies of frontline platinum doublet chemotherapy
mL. here are data suggesting that low-volume Glea- versus PD-1 inhibition failed to demonstrate improved
son 3 + 4 disease can be managed successfully by active outcomes for immune therapy in patients with tumors
surveillance, particularly for patients older than age 70. with lower or no PD-L1 expression and thus are not
However, a diagnosis of localized Gleason 4 + 3 disease recommended in this setting.
in a patient with life expectancy >10 years, as in the case Giroux Leprieur E, Dumenil C, Julie C, et al. Immunotherapy
above, warrants deinitive treatment. revolutionises non-small-cell lung cancer therapy: results,
Mohler JL, Armstrong AJ, Bahnson RR, et al. Prostate cancer, perspectives and new challenges. Eur J Cancer. 2017 Apr
version 1.2016. J Natl Compr Canc Netw. 2016;14(1):19–30. 10;78:16–23.

22. ANSWER: B. Prolactinoma 24. ANSWER: D. A and C


Androgen deprivation therapy (ADT) is the irst- PD-1 and PD-L1 inhibition, though often well
line treatment for advanced prostate cancer and is used tolerated, are associated with a wide variety of autoim-
as neoadjuvant treatment along with radiation therapy mune toxicities. Among the most common are endo-
for intermediate- and high-risk localized disease. After crine dysfunction (hyper- or hypothyroid, pituitary
about 1 week on ADT, GnRH receptors in the pitu- dysfunction), colitis, pneumonitis, and rash. Less com-
itary are downregulated, and LH and FSH production mon toxicities include pancreatitis, nephritis, hepatitis,
are diminished. his results in a profound reduction diabetes, and myocarditis. Pneumonitis and colitis are
in testosterone production by the testicles and induces the most frequently cited causes of life-threatening or
tumor responses in 80%–90% of patients. Castrate even fatal toxicities. he mainstay of therapy for severe
testosterone levels are associated with several common reactions is permanent cessation of drug and immediate
side efects: loss of libido, osteopenia/osteoporosis, hot initiation of immune suppression with corticosteroids.
lashes, decrease in lean body mass, increase in subcu- Milder toxicities can often be managed with temporary
taneous adipose tissue, thinning of body hair, decrease cessation of drug plus or minus courses of oral cortico-
in penile and testicular size, gynecomastia, anemia, and steroids. he optimal dose and corticosteroid type for
mild fatigue. More rarely, treatment may be associated severe reactions is unknown, but the equivalent of at
with cognitive decline or depression. Some studies have least 1 mg/kg of prednisone is a widely accepted mini-
observed an increased incidence of cardiovascular events mum. In this patient with cough, dyspnea, absence of
in those receiving ADT; others have found no convinc- fever, and the listed CT indings, nivolumab-induced
ing association. Associations between ADT and devel- pneumonitis is highly likely. Pneumonitis is a diagno-
opment of colorectal cancer have also been reported, but sis of exclusion, however, and other etiologies must be
a true causal relationship has not been irmly established. considered and evaluated. Because atypical pneumonia
here is no known association with other cancers. could have a similar presentation, it is prudent to treat
this patient for both infection and pneumonitis.
23. ANSWER: B. A patient with newly diagnosed met- Maughan BL, Bailey E, Gill DM, Agarwal N. Incidence of
astatic lung cancer and 10% tumor programmed immune-related adverse events with program death recep-
death receptor ligand 1 (PD-L1) expression tor-1- and program death receptor-1 ligand-directed thera-
pies in genitourinary cancers. Front Oncol. 2017;7:56.
Immune therapy with PD-1 inhibition is currently
approved in several malignancy types and settings,
with many more indications under study. PD-1 is a 25. ANSWER: A. Tamoxifen causes none of these side
checkpoint protein expressed on T cells, and activa- efects.
tion of PD-1 leads to downregulation of the immune In placebo-controlled, randomized trials of tamoxi-
response. PD-L1 is expressed on some tumor cells and fen for breast cancer prevention among patients
42 C HA P T E R 2 Hematology and Oncology

without a personal history of breast cancer, there was 27. ANSWER: D. Small cell lung cancer
no increased risk of premature menopause, weight he indings of a large hilar mass with superior vena
gain, or depression. Both raloxifene and exemestane cava (SVC) syndrome, hepatic and bony metastases,
have data only for postmenopausal women. and hyponatremia in a 63-year-old woman with a
signiicant smoking history are most consistent with
26. ANSWER: D. Skeletal survey with plain ilms a small cell lung cancer (SCLC). SCLC is the tumor
In the case of lung cancer, the most common sites type most closely associated with smoking: About
of involvement are the mediastinum and contralat- 95% of patients with SCLC have a smoking history.
eral lung, liver, bone, brain, and adrenal glands. he SCLC tends to occur in the central chest, so SVC
appropriate radiographic staging workup of lung can- syndrome or tracheal compression can be presenting
cer should include imaging of all of those sites. symptoms. In fact, SCLC is the most common cause
Brain imaging is required for all patients with small of SVC syndrome. SCLC tends to be aggressive in its
cell lung cancer, patients with nonsmall cell lung can- course, with metastatic disease found at the time of
cer of stage IB or greater, or any patient with lung initial diagnosis in 60%–70% of patients with SCLC.
cancer with any neurologic symptom. In this case, the SCLC can also be associated with a number of para-

t
ne
unexplained fall in the setting of new lung cancer war- neoplastic syndromes. he most common of these is
rants dedicated brain imaging. MRI is preferred over hyponatremia due to the syndrome of inappropriate
CT for brain imaging, though CT with contrast is a antidiuretic hormone secretion (SIADH).

e.
reasonable alternative. Regardless of modality, IV con-
trast (gadolinium for MRI, iodinated contrast for CT) 28. ANSWER: B. Repeat chest CT in 6 months

in
is required for suicient detection of brain metastases. he Fleischner Society has published guidelines
he prior noncontrast head CT is insuicient. for the follow-up and management of nodules <8

ic
A chest CT with IV contrast that is performed spe- mm detected incidentally at nonscreening CT. hese
ciically for lung cancer staging should be expanded guidelines make a distinction between low- and high-
to include the liver and adrenal glands. he admin-
istration of IV contrast enhances the ability to detect
ed risk patients, and nodule sizes ≤4 mm, >4–6 mm,
>6–8 mm, and >8 mm. High-risk patients are deined
hepatic metastases. Furthermore, by delineating the as those with a history of smoking or other known risk
sm
mediastinal vasculature with IV contrast, the clini- factors. In this case, a 5-mm nodule in a patient with a
cian can better assess the size and extent of mediastinal 20-pack-year smoking history merits a follow-up CT
lymph nodes. in 6–12 months. For a nodule that small, it would be
PET/CT or bone scan is the best modality to look diicult to reliably perform a CT-guided biopsy, and
ok

for bone metastases in lung cancer. PET/CT is the it may be below the size threshold for luorodeoxyglu-
preferred modality. A plain ilm skeletal survey is not cose avidity on PET/CT (Table 2.7).
sensitive enough to detect occult bone metastases in
bo

lung cancer. 29. ANSWER: E. Surgical resection


Ettinger DS, Wood DE, Aisner DL, et  al. Non-small cell lung he gold standard therapy for newly diagnosed
cancer, version 5.2017, NCCN Clinical Practice Guidelines early-stage lung cancer is anatomic resection. In
e

in Oncology. J Natl Compr Canc Netw. 2017;15(4):504–535. patients with good performance status and adequate
://

TABLE Guidelines for Management of Small Pulmonary Nodules Detected on CT Scans: A Statement From the
tp

2.7 Fleischner Society


Nodule Size (mm)a Low-Risk Patient High-Risk Patient
ht

≤4 No follow-up needed Follow-up CT at 12 months; if unchanged, no


further follow-up
>4–6 Follow-up CT at 12 months; if unchanged, no further Initial follow-up CT at 6–12 months, then at
follow-up 18–24 months if no change
>6–8 Initial follow-up CT at 6–12 months, then at 18–24 Initial follow-up CT at 3–6 months, then at 9–12
months if no change and 24 months if no change
>8 Follow-up CT at around 3, 9, and 24 months; dynamic Same as for low-risk patient
contrast-enhanced CT; PET; and/or biopsy

NOTE: Newly detected indeterminate nodule in persons 35 years of age or older.


Nonsolid (ground-glass) or partly solid nodules may require longer follow-up to exclude indolent adenocarcinoma.
aAverage of length and width.

From MacMahon H, Austin JH, Gamsu G, et al. Guidelines for management of small pulmonary nodules detected on CT scans: a statement from the Fleischner
Society. Radiology. 2005;237(2):395–400.
CHAPTER 2 Hematology and Oncology 43

lung function, the goal is anatomic resection (i.e., Excisional lymph node biopsy and bone marrow biopsy
resection of the mass and its associated lymphatic are not necessary, given that the diagnosis may be estab-
drainage). his is classically achieved via lobectomy, lished using low cytometry. PET/CT is not necessary
though segmentectomy (resection of an anatomic seg- in the evaluation of CLL. CT scans may be obtained in
ment of a lung lobe) may be adequate for some tumors patients with adverse cytogenetics, such as 11q and 17p
that are small enough and in the right location. deletion, but they are not necessary at this time. he
For patients who are poor operative candidates, ste- patient’s symptoms are suggestive of a viral infection,
reotactic body radiation therapy (SBRT) can be con- and antibiotics are not indicated.
sidered. he potential use of stereotactic radiation for
early-stage lung cancer may be limited by tumor size 32. ANSWER: C. Refer to a surgeon for an excisional
and location. In general, tumors >4 cm are not gen- lymph node biopsy.
erally amenable to deinitive control with stereotac- he patient presents with difuse, asymptomatic
tic radiation. Furthermore, there have been concerns lymphadenopathy suggestive of involvement by an
about stereotactic radiation to central tumors because indolent lymphoma. To establish the diagnosis, an ade-
early trials of this modality raised concerns about col- quate tissue biopsy is critical. Given that she has palpa-

t
ne
lapse of adjacent airways. ble peripheral lymphadenopathy, referral to a surgeon
Chemotherapy is not considered a curative modal- for an excisional lymph node biopsy is the most appro-
ity in lung cancer, and it has not been shown to be an priate way to obtain adequate diagnostic tissue. Needle

e.
appropriate up-front therapy for potentially resectable biopsies lack adequate tissue to evaluate for architec-
early-stage nonsmall cell lung cancer. ture. Peripheral blood low cytometry is unlikely to

in
Modalities such as cryoablation and radiofrequency yield a diagnosis in the setting of a normal white blood
ablation continue to be investigated in lung cancers cell count and diferential. PET/CT scan is indicated
but are not currently considered a standard curative

ic
for staging in aggressive lymphomas and Hodgkin lym-
treatment option in early-stage lung cancer. phoma after a diagnosis has been established.
More recent data suggest that this may be achieved
by segmentectomy, depending on a tumor’s size and
ed
33. ANSWER: C. Osteoporosis
location. For patients who are poor operative candi- Aromatase inhibitors are associated with estrogen
sm
dates, consider stereotactic radiation. deiciency–induced bone loss. Patients starting an AI
Ettinger DS, Wood DE, Aisner DL, et  al. Non-small cell lung should undergo baseline bone density testing, and test-
cancer, version 5.2017, NCCN Clinical Practice Guidelines ing should continue periodically for the duration of the
in Oncology. J Natl Compr Canc Netw. 2017;15(4):504–535. course of therapy. Although formal guidelines do not
ok

exist, many experts recommend testing every 2 years


30. ANSWER: D. Obtain a chest x-ray. while a patient is on AI therapy. If identiied, osteopenia
his young woman presents with signs and symp- and osteoporosis should be managed aggressively and
bo

toms worrisome for lymphoma, particularly Hodg- according to expert guidelines. Additionally, all women
kin lymphoma (HL). Although pruritus is a frequent should be counseled regarding lifestyle optimization for
symptom in patients with HL, intense pruritus with- bone health, including physical activity, smoking cessa-
e

out a rash is quite rare. On the other hand, B symp- tion, and adequate calcium and vitamin D intake.
://

toms (fevers, drenching night sweats, and unexplained Increased risk for DVT/PE and endometrial can-
weight loss) are common in this disease. In addition, cer (Answers A and D) are associated with tamoxifen.
tp

she is a young adult at the median age of presentation Hypothyroid and glucose intolerance are not known
of HL. Finally, the leukocytosis with lymphopenia and adverse efects of hormonal therapy for breast cancer.
elevated eosinophils may be seen with HL. Given that Patel S. Disruption of aromatase homeostasis as the cause
ht

she does not have peripheral lymphadenopathy, the of a multiplicity of ailments: a comprehensive review. J Steroid
next most logical step in her management is to obtain Biochem Mol Biol. 2017;168:19–25.
a chest x-ray to look for a mediastinal mass. LDH is
rarely elevated in HL. A PET/CT scan is done as part 34. ANSWER: D. Medical therapy with a dopamine
of staging once a diagnosis has been established. She agonist
does not have symptoms typical for EBV infection. Medical therapy with a dopamine agonist is con-
sidered the irst-line initial therapy for symptomatic
31. ANSWER: A. Obtain peripheral blood low cytom- prolactinomas. Dopamine agonists have both antipro-
etry. liferative and antisecretory efects on prolactin-produc-
his patient presents with the classic indings of a ing pituitary cells and typically result in both decreased
patient with CLL with difuse lymphadenopathy and prolactin secretion and tumor shrinkage. Cabergoline
splenomegaly with a lymphocytosis and smudge cells is preferred over bromocriptine for most patients due
on a peripheral smear. he diagnosis of CLL may be to its increased eicacy and tolerability. Somatostatin
established using peripheral blood low cytometry. analogues are used in the treatment of acromegaly.
44 C HA P T E R 2 Hematology and Oncology

Transsphenoidal surgery is reserved for patients who hormone (ADH), or an inadequate renal response to
do not tolerate or are not responsive to the dopamine ADH. Patients with diabetes insipidus often complain
agonists. Radiation would be considered only in very of extreme thirst and will excrete large amounts of very
rare cases of advanced or progressive prolactinomas that dilute urine. Without this hormone, individuals cannot
have failed medical or surgical interventions. Observa- adequately concentrate their urine and, if not allowed
tion alone would not be the preferred approach in a access to liquids, may develop severe, life-threatening
patient with a macroadenoma (size >10 mm), because hypernatremia. Central diabetes insipidus is due to
control of tumor growth would be desired. deiciency, either partial or complete, of ADH. Central
diabetes insipidus can occur after any pituitary surgery.
35. ANSWER: C. Venlafaxine It can be transient or permanent and is seen more fre-
here are multiple drug interactions between tamox- quently in patients with larger tumors or tumors that
ifen and antidepressants based on the potential for tend to invade the pituitary stalk, such as craniopha-
CYP2D6 inhibition by many antidepressants. CYP2D6 ryngiomas. he diagnosis is not likely SIADH, because
converts tamoxifen to endoxifen, its active form. Venla- this condition is typically associated with hyponatre-
faxine has the least efect on CYP2D6 of the commonly mia. Type 2 diabetes mellitus would also not be a likely

t
ne
used antidepressants. Paroxetine, luoxetine, and bupro- cause of this patient’s acute severe polyuria and polydip-
pion are strong inhibitors and should be avoided if pos- sia, because his BMI is normal and his fasting glucose is
sible. Duloxetine and sertraline are moderate inhibitors. only mildly elevated. Given the clinical presentation of

e.
Citalopram and escitalopram are mild inhibitors. Des- a pituitary lesion, his diabetes insipidus would be much
venlafaxine and mirtazapine likely have minimal efect more likely be central than nephrogenic.

in
on tamoxifen/endoxifen, but direct studies are lacking. Lobatto DJ, de Vries F, Zamanipoor Najafabadi AH, et al.
Preoperative risk factors for postoperative complications in
endoscopic pituitary surgery: a systematic review. Pituitary.

ic
36. ANSWER: C. A 60-year-old man with new-onset
nonvalvular atrial ibrillation and a history of a 2017 Sep 15. [Epub ahead of print]
ed Prete A, Corsello SM, Salvatori R. Current best practice in
diverticular bleed 5 years ago
the management of patients after pituitary surgery. her Adv
NOACs (rivaroxaban, edoxaban, apixaban, dabiga-
Endocrinol Metab. 2017;8(3):33–48.
tran) are approved in nonvalvular atrial ibrillation and
sm
DVT/PE. Current contraindications include patients
with active malignancies (studies underway), metallic 39. ANSWER: D. All of the above
heart valves, the antiphospholipid antibody syndrome his patient was found to have an incidental
(studies underway), and renal insuiciency (glomeru- pituitary lesion. All three of the answer choices are
ok

lar iltration rate [GFR] <15 mL/min for edoxaban important for the initial evaluation of the patient. A
and apixaban, and GFR <30 mL/min for rivaroxaban thorough history must be obtained to elicit symp-
and dabigatran). Data are insuicient regarding safety toms of mass efects and pituitary dysfunction (both
bo

in pregnant or lactating women. Prior GI bleeding is hypersecretion and hypofunction). Having a complete
not an absolute contraindication to NOAC use. understanding of pituitary physiology will help guide
Steinberg BA. How I use anticoagulation in atrial ibrillation. the questions related to determining the function of
e

Blood. 2016;128(25):2891–2898. each hormonal axis. he clinician should look for both
://

signs and symptoms to suggest hormonal hypersecre-


37. ANSWER: E. Referral to IR for needle-guided tion (i.e., hyperprolactinemia, growth hormone excess,
or hypercortisolism) or hormonal hypofunction (i.e.,
tp

biopsy of the adnexal mass


If clinical suspicion of ovarian malignancy is high, central adrenal insuiciency, central hypothyroidism,
any of the above evaluations is reasonable (although central hypogonadism, GH deiciency, diabetes insipi-
ht

only referral to gynecologic oncology is required), with dus). Laboratory evaluation should be performed to
the exception of needle biopsy due to the potential risk assess pituitary hormonal function. MRI should be
for malignant seeding of the needle tract. Colonos- performed to better characterize the structure of the
copy should be considered in women who are found mass. Taken together, both laboratory and imaging
to have mucinous histology at surgical resection, or studies will help guide further evaluation and treat-
for women in whom colorectal cancer is clinically sus- ment of the patient presenting with a pituitary mass.
pected at presentation.
40. ANSWER: B. t(15;17)
38. ANSWER: C. Central diabetes insipidus he age distribution of patients with acute promy-
he symptoms and presentation in this case are most elocytic leukemia (APL) difers from other forms of
concerning for the development of postoperative cen- acute myeloid leukemia (AML). APL is uncommon
tral diabetes insipidus. Diabetes insipidus is a syndrome in the irst decade of life; its incidence increases dur-
of hypotonic polyuria and is due to either a deiciency ing the second decade, reaches a plateau during early
of arginine vasopressin, also known as antidiuretic adulthood, and then remains constant until it decreases
CHAPTER 2 Hematology and Oncology 45

after age 60 years. he median age is 40 years old, thus he Revised IPSS (IPSS-R) incorporates a larger
considerably lower than for other myeloid leukemias. number of cytogenetic abnormalities, divided into
APL is a medical emergency deined by the pres- ive prognostic categories, a lower cutof for absolute
ence of a reciprocal translocation between the long neutrophil count (<800/µL vs. <1800/µL), and difer-
arms of chromosomes 15 and 17, with the creation of ent weights for the clinical parameters to better predict
a fusion gene, PML-RARA, which links the retinoic outcomes in newly diagnosed patients. Patients with a
acid receptor alpha (RARA) gene on chromosome low IPSS-R (1.5 to 3) have a median survival of 5.3
17 with the promyelocytic leukemia (PML) gene on years and risk to 25% AML transformation of 10.8
chromosome 15. years. he main goals of therapy are to control symp-
A key component of therapy is the use of all-trans toms due to cytopenias, improve quality of life, and
retinoic acid (ATRA), which promotes the terminal minimize the toxicity of therapy. here is no evidence
diferentiation of malignant promyelocytes to mature that the treatment of asymptomatic patients improves
neutrophils. Coagulopathy is a common presenting long-term survival.
feature and should be corrected with blood products Immediate treatment is indicated for patients with
as needed (platelets, fresh frozen plasma, cryopre- symptomatic anemia or thrombocytopenia or recur-

t
ne
cipitate) in addition to the administration of ATRA rent infections in the setting of neutropenia. his
as soon as it is available. Correction of coagulopathy patient is developing worsening dyspnea on exertion as
and administration of ATRA should be performed a result of his anemia, so therapy could be considered.

e.
immediately with suspicion of APL and should not be Approximately 15%–25% of unselected patients with
deferred until a irm diagnosis is available. MDS have an improvement in hemoglobin when treated

in
t(11;14): Overexpression of cyclin D1 in mantle with recombinant human erythropoietin. he Nordic
cell lymphomas (MCLs) is strongly associated with the MDS Group developed a predictive model for epoetin

ic
t(11;14)(q13;q32), a translocation between the cyclin use in MDS: patients requiring 2 U of PRBCs or more
D1 locus and the immunoglobulin heavy chain (IgH) per month and those with serum epoetin level >100 U/L
locus. Approximately 50% to 65% of MCLs will show
the t(11;14) by cytogenetics, but by luorescence in
ed (especially >500 U/L) are less likely to respond.
Allogeneic transplant is the only curative treat-
situ hybridization (FISH), a much higher fraction of ment for MDS. Transplant at the time of diagnosis
sm
cases with cyclin D1 overexpression contain BCL-1/ is shown to have the greatest beneit in patients in
IgH fusion genes. more advanced IPSS risk groups: intermediate-2 and
t(9;22): he vast majority of patients (90%–95%) high risk. Because this patient has low-risk disease,
demonstrate the t(9;22)(q34;q11.2) reciprocal translo- strategies to reduce the need for transfusions is rec-
ok

cation that results in the Ph chromosome-BCR-ABL1 ommended with allogeneic transplant at the time of
fusion gene or its product, the BCR-ABL1 fusion disease progression.
mRNA. he BCR-ABL1 fusion gene is the target of As is the case with allogeneic hematopoietic stem
bo

therapies such as dasatanib, nilotinib, and imatinib. cell transplant, high-intensity chemotherapy regimens
t(8;14): Burkitt lymphoma is associated with a are generally reserved for patients with intermediate-2
translocation between the long arm of chromosome or high-risk IPSS scores. his allows for the avoidance
e

8, the site of the c-MYC oncogene (8q24), and one of treatment-related morbidity and mortality in most
://

of three locations on immunoglobulin (Ig) genes. he patients with a relatively good prognosis.
most common translocation is with the Ig heavy chain Lenalidomide improves anemia in some patients
tp

gene on chromosome 14 (approximately 80%). with MDS and a normal marrow karyotype, but this
t(14;18): Approximately 85% of patients with agent is particularly efective in MDS with an intersti-
follicular lymphoma have t(14;18), which results in tial deletion of chromosome 5q. In studies, it has been
ht

the overexpression of B-cell leukemia/lymphoma 2 associated with transfusion independence in 67% of


(BCL2), an oncogene that blocks programmed cell transfusion-dependent patients.
death (apoptosis), leading to prolonged cell survival. Greenberg PL, Tuechler H, Schanz J, et  al. Revised interna-
DeAngelo DJ. Tailored approaches to induction therapy for tional prognostic scoring system for myelodysplastic syn-
acute promyelocytic leukemia. J Clin Oncol. 2017;35(6):583–586. dromes. Blood. 2012;120(12):454–465.
Adams J, Nassiri M. Acute promyelocytic leukemia: a review
and discussion of variant translocations. Arch Pathol Lab Med. 42. ANSWER: D. Send peripheral blood for cytogenet-
2015;139(10):1308–1313. ics to evaluate for the t(9;22) chromosome translo-
cation.
41. ANSWER: D. Erythropoiesis-stimulating agent Chronic myelogenous leukemia (CML) accounts
(ESA) for approximately 15%–20% of leukemias in adults.
he International Prognostic Scoring System It has an annual incidence of 1–2 cases per 100,000,
(IPSS) is the most widely used prognostic classiica- with a slight male predominance. he clinical presen-
tion system for myelodysplastic syndrome (MDS). tation with splenomegaly, immature myeloid forms,
46 C HA P T E R 2 Hematology and Oncology

and basophilia and eosinophilia is suggestive of CML In the setting of her WBC count and chest x-ray
or another myeloproliferative disorder. he lack of indings, it is likely that she has pulmonary evidence of
JAK2 and normal Hgb level makes polycythemia vera leukostasis, presenting with dyspnea, tachypnea, and
less likely. Approximately 15%–30% of patients with hypoxemia. Respiratory acidosis and cor pulmonale
CML have platelet counts >600,000/µL. may also occur. Chest x-ray can show difuse bilat-
he diagnosis of CML is conirmed by the dem- eral iniltrates. Approximately 80% of patients with
onstration of the Philadelphia chromosome t(9;22) leukostasis are febrile, usually as a result of associated
(q34;q11.2), the BCR-ABL1 fusion gene, or the BCR- inlammation. Although starting empiric antibiotics
ABL1 fusion mRNA by conventional cytogenetics, is reasonable, waiting to see if the patient improves
luorescence in situ hybridization (FISH) analysis, or would waste valuable time during which the patient
reverse transcription–polymerase chain reaction (RT- can undergo leukapheresis.
PCR). Transfusions would increase blood viscosity and
Hydroxyurea can be used to reduce white blood increase the risk of worsening the respiratory compro-
cell counts while awaiting conirmation of a suspected mise and intracranial hemorrhage.
diagnosis of CML in a patient with signiicant leuko- Leukapheresis should be initiated while establish-

t
ne
cytosis (e.g., >80 × 109 white cells/L). his patient’s ing a diagnosis in patients who have symptomatic
WBC count is only 60× 109 white cells/L, and she is leukostasis. It is a temporizing measure, thus a diag-
asymptomatic from her disease, so waiting for a diag- nosis needs to be made promptly and deinitive ther-

e.
nosis is appropriate. apy should be started. herapy such as hydroxyurea
his patient is afebrile and has no other symptoms reduces the WBC count by 50% to 80% within 24 to

in
of infection. he relative basophilia is also less con- 48 hours and should be used as the sole therapy only
sistent with an infectious cause, thus starting empiric in patients with hyperleukocytosis without symptoms

ic
antibiotics for a presumed reactive leukocytosis is not of leukostasis. While induction chemotherapy serves
needed. to both rapidly decrease the circulating WBC count
While essential thrombocythemia (ET) is still in
the diferential diagnosis because 50% of patients with
ed and target the leukemia cells in the bone marrow,
reducing the WBC count within 24 hours of initia-
ET do not have the JAK2 mutation, starting high-dose tion, the patient is rapidly worsening, and it will take
sm
aspirin in this patient is not recommended. Higher time to make the diagnosis.
doses of aspirin (900 mg/d) have been associated with
increased gastrointestinal hemorrhage as compared 44. ANSWER: D. von Willebrand factor levels become
with low-dose aspirin (75–325 mg/d). normal during pregnancy in patients with type I
ok

Although morphologically normal, the neutrophils disease.


in CML are cytochemically abnormal. Leukocyte alka- Approximately 85% of patients with von Wil-
line phosphatase (LAP) is typically low in CML. he lebrand disease (vWD) have type I disease where
bo

low LAP score is useful in excluding a reactive leuko- there is a decrease in the vWF level with no abnor-
cytosis, typically due to infection, in which the score mality in vWF function. For reasons that are not well
is typically elevated or normal, or polycythemia vera, understood, during pregnancy, the vWF levels in type
e

which is also associated with an increased LAP activity. I vWD patients normalize. hus, it is safe to use an
://

Although the LAP score may provide useful support- epidural catheter for anesthesia. After delivery of the
ive data, it is neither sensitive nor speciic for CML. baby and the placenta, the vWF level returns to the
tp

prepregnancy level. hus, about 25% of women with


43. ANSWER: D. Start leukapheresis while establish- type I vWD require treatment with DDAVP 2–3 days
ing diagnosis. after delivery but are not at risk for excess bleeding
ht

his patient has a WBC count >100,000/µL with during labor and delivery.
anemia and thrombocytopenia. She has a likely diag- Leebeek FWG, Eikenboom JCJ. Von Willebrand’s disease. N
nosis of leukemia with an elevated WBC count lead- Engl J Med. 2016;375(21):2067–2080.
ing to hyperleukocytosis and leukostasis.
Hyperleukocytosis occurs in 10%–20% of patients 45. ANSWER: A. Radiation oncology consultation for
with AML and more commonly in the monocytic consideration of urgent whole-brain radiotherapy
and myelomonocytic variants. Leukostasis results (WBRT)
from increased blood viscosity as a direct complica- WBRT is a reasonable option for patients with
tion of the presence of large leukemic blasts and the multiple brain lesions not amenable to SRS and/
interaction of the blasts with the endothelium to form or poor performance status. In patients with good
aggregates, leading to thrombi in the microcircula- performance status and limited metastatic lesions,
tion. Patients are usually symptomatic when the WBC SRS is preferred. Although precise guidelines as
count reaches levels >100,000. If left untreated, the to size and number of lesions are not agreed upon,
1-week mortality rate is approximately 20% to 40%. most studies consider SRS for one to three lesions
CHAPTER 2 Hematology and Oncology 47

(and occasionally more) measuring 3 cm or less per the two positive nodes make the patient stage III and
lesion. Although there is a decreased risk of intracra- guidelines recommend adjuvant chemotherapy. A fol-
nial progression at 1 year with WBRT following SRS, low-up colonoscopy in 1 year would be recommended
there is no survival beneit. WBRT is associated with as part of surveillance for this patient, though it is not
potential long-term cognitive impairment, especially the next step. Radiation is rarely used in colon cancer,
in memory and learning. In this patient with good because local recurrence rates are very low; radiation is
performance status and a limited number of lesions, considered for stages II and III rectal cancer.
SRS is the preferred initial approach for management Benson AB 3rd, Venook AP, Cederquist L, et  al. Colon can-
of brain metastases. cer, version 1.2017, NCCN Clinical Practice Guidelines in
Because she has had a seizure, both corticosteroids Oncology. J Natl Compr Canc Netw. 2017;15(3):370–398.
and anticonvulsant therapy are indicated. In asymp-
tomatic patients with edema, corticosteroids should 48. ANSWER: E. Increase in dose of allopurinol
be considered cautiously. If primary CNS lymphoma Tumor lysis syndrome (TLS) is acute cell lysis
is suspected, corticosteroids should be avoided unless caused by chemotherapy and radiation therapy. he
clinically necessary. here is no proven role for prophy- release of intracellular products (e.g., uric acid, phos-

t
ne
lactic anticonvulsant therapy, but because this patient phates, calcium, potassium) overwhelms the body’s
has had a seizure, treatment should be initiated. homeostasis mechanism. TLS is more common with
All patients with lung cancer should have their hematologic malignancies or cancers with readily

e.
tumors tested for targetable mutations, particularly epi- growing tumors, particularly acute leukemias and
dermal growth factor receptor (EGFR) and ALK. More high-grade lymphomas such as Burkitt lymphoma

in
comprehensive testing may identify additional thera- (BL). Patients with BL are at high risk of TLS and
peutic targets as well. However, treatment of symptom- uric acid nephropathy, especially during chemother-
atic brain lesions should not be deferred while awaiting

ic
apy. Prophylactic allopurinol and aggressive hydra-
molecular testing results. tion should be administered. TLS usually presents

46. ANSWER: D. A patient with poorly controlled


ed with acute kidney injury and metabolic derange-
ments such as hyperphosphatemia, hyperkalemia,
atrial ibrillation and platelets of 20,000/µL and hypocalcemia. Treatment includes inpatient
sm
Ibrutinib is a highly efective and generally well- monitoring, vigorous luid resuscitation, allopuri-
tolerated agent in CLL. It is a potent and irreversible nol or urate oxidase (uricase) therapy to lower uric
inhibitor of Bruton tyrosine kinase (BTK), which is acid levels, and hemodialysis if renal failure develops.
important in B-cell signaling pathways. BTK inhibi- Because the patient is already on allopurinol, it is
ok

tion with ibrutinib can lead to decreased proliferation unlikely that further adjustment of allopurinol dos-
and survival of malignant B cells. Key potential adverse ing will be efective, at least acutely.
efects include increased risk of atrial ibrillation and Howard SC, Jones DP, Pui CH. he tumor lysis syndrome.
bo

bleeding, likely due to ibrutinib-induced platelet dys- N Engl J Med. 2011; 364(19):1844–1854.
function. In the patient with thrombocytopenia and
poorly controlled AF, ibrutinib would be more likely to 49. ANSWER: B. Arrange for an urgent MRI of the
e

cause serious adverse events than in the other patients spine to evaluate for possible cord compression.
://

described in the vignette. Ibrutinib is efective in bulky his patient likely has epidural spinal cord com-
disease, and anemia is not a contraindication. Unlike pression caused by a tumor compressing the dural
tp

FCR, ibrutinib is nearly as efective in patients who sac. his can cause permanent neurologic impairment
harbor p53 mutations as compared with those who even if treatment is delayed for only a few hours, and
do not. Ibrutinib is not associated with a signiicantly therefore it requires attention and treatment with
ht

increased risk of opportunistic infections or colitis. some urgency. Epidural spinal cord compression is
Idelalisib, an oral kinase inhibitor also used in CLL, is associated with renal, prostate, and most commonly
associated with these potential adverse events. breast and lung cancers. he thoracic spine is most
Leong DP, Caron F, Hillis C, et al. he risk of atrial ibrillation often frequently afected, accounting for 70% of
with ibrutinib use: a systematic review and meta-analysis. patients with the condition. One should suspect an
Blood 2016;128(1):138–140. epidural metastasis if the patient complains of new
pain that worsens when the patient is lying down or
47. ANSWER: B. Referral to medical oncologist for with palpation of vertebral bodies, which is charac-
consideration of chemotherapy teristic of this condition. Late neurologic signs such
his patient has stage III colon cancer. Although as incontinence and loss of sensory function are
proper pathology staging recommends sampling and associated with permanent paraplegia. MRI has sur-
analysis of at least 12 lymph nodes for ideal staging, passed myelography as the imaging study of choice. If
it is not going to impact the patient’s current man- neurologic symptoms are present, the patient should
agement to have more lymph nodes sampled, because be treated with steroids. his treatment should not
48 C HA P T E R 2 Hematology and Oncology

be delayed while awaiting diagnostic study results. dyspnea; dysphagia; and swelling or discoloration of
Use of high-dose dexamethasone (up to 100 mg) is the neck, face, or upper extremities. Often, collateral
controversial; clinical trials have shown that it has venous circulation causes distention of the supericial
unclear beneits and signiicantly more serious side veins in the chest wall. Tissue diagnosis (i.e., sputum
efects at higher doses. Most patients with epidural cytology, thoracentesis, bronchoscopy, or needle aspi-
and spinal cord compression need radiation treat- ration) often is necessary to direct treatment decisions.
ment (up to 3000 Gy) or surgery.
Acknowledgment
50. ANSWER: A. Establishing a tissue diagnosis is
the most important step in management of most he authors and editors gratefully acknowledge the con-
patients with SVC syndrome. tributions of the previous authors—Lawrence Shulman,
he SVC syndrome is caused by the gradual com- Wendy Y. Chen, Yuksel Urun, Toni K. Choueiri, Jean M.
pression of the superior vena cava, leading to edema Connors, Peter Enzinger, Nancy Berliner, Maureen M.
and retrograde low. Lung cancer is the most common Okam, Mark M. Pomerantz, David M. Jackman, Brett E.
malignant cause, although lymphoma, metastatic

t
Glotzbecker, Edwin P. Alyea, Daniel J. DeAngelo, Rob-

ne
mediastinal tumors, and indwelling catheters also can ert I. Handin, Jefrey A. Meyerhardt, and Whitney W.
cause SVC syndrome. Symptoms may include cough; Woodmansee

e.
in
ic
ed
sm
ok
e bo
://
tp
ht
3
Rheumatology
DERRICK J. TODD

1. A 76-year-old woman has been admitted to the hospi- C. Seronegative rheumatoid arthritis of the elderly

t
ne
tal for fever, cough, dyspnea, and hypoxemia. She has a D. Wrist osteoarthritis
history of primary hyperparathyroidism and has refused E. Wrist fracture
surgical intervention. Chest radiograph shows a lobar

e.
iniltrate, and sputum culture grew Streptococcus pneu- 2. A 92-year-old woman has been in the hospital for 3
moniae. She is diagnosed with pneumococcal pneumo- weeks with unexplained fever, malaise, and weight

in
nia and treated with levoloxacin based on antibiotic loss. She reports no other localizing symptoms. Com-
sensitivities. Her clinical status improves markedly. prehensive physical examination (including pelvic and

ic
On hospital day 4, just prior to discharge, she develops breast evaluations) has failed to reveal any abnormali-
rapid-onset painful swelling in the right wrist associated ties other than generalized muscle wasting and men-
with redness and warmth. She is afebrile, normotensive,
and does not appear to be in a toxic condition. he
ed tal lassitude. She has had a battery of laboratory and
imaging studies without a speciic diagnosis. Com-
right wrist is red, warm, swollen, and very painful to plete blood count shows hemoglobin 9.1 g/dL (with
sm
active or passive range of motion. Several right metacar- normal red blood cell indices), platelets 525 × 103
pophalangeal (MCP) joints are similarly involved, but cells/mm3, and normal white blood count with dif-
the left hand and wrist are completely normal. Blood ferential. Erythrocyte sedimentation rate (ESR) is 110
cultures drawn on the day of admission (prior to anti- mm/h. Albumin is 2.8 g/dL, and alkaline phosphatase
ok

biotics) have shown no growth. Plain radiograph of the is 250 U/L (normal 40–130). Transaminases, biliru-
right hand/wrist is as shown in Fig. 3.1. bins, electrolytes, renal parameters, and serum protein
What is the most likely diagnosis? electrophoresis (PEP) are all normal. Urinalysis and
bo

A. Streptococcus pneumoniae septic arthritis urine PEP are likewise normal. Cultures of urine and
B. Acute pseudogout arthritis blood have revealed no growth of microorganisms.
he patient has no risk factors for tuberculosis infec-
e

tion, and PPD test was negative. Serologic testing for


://

syphilis, Lyme disease, rheumatoid factor, antinuclear


antibodies, and antineutrophil cytoplasmic antibod-
tp

ies are all negative. Chest radiographs and computed


tomography (CT) scan of the abdomen/pelvis do not
demonstrate lymphadenopathy, culprit masses, hem-
ht

orrhages, or abscesses. Endoscopy and colonoscopy are


normal, as are abdominal and transvaginal ultrasound
evaluations. Vascular ultrasonographic imaging of the
extremities show no evidence of thrombosis.
Which of the following is the most appropriate next
step in trying to identify a source of the patient’s fever?
A. Repeat blood cultures and treat with empiric cef-
triaxone.
B. Interferon-gamma release assay to assess for prior
exposure to Mycobacterium tuberculosis.
R
C. Question family members out of concern for elder
abuse.
• Fig. 3.1 Plain radiograph of the right wrist and hand of the patient D. Liver biopsy
described in Question 1. E. Temporal artery biopsy

49
50 C HA P T E R 3 Rheumatology

3. A 37-year-old previously healthy woman presents to the C. Sickle cell disease


outpatient clinic with 6 months of progressively achy D. Alcoholism
hands, wrists, elbows, knees, ankles, and feet. Symptoms E. Gaucher disease
are worse in the morning and are alleviated by physical
activities. Functionally, she is inding it diicult to per- 5. A 27-year-old woman is diagnosed with systemic lupus
form her duties as an orthopedic hand surgeon because erythematosus (SLE) based on a history that includes
of reduced manual strength and dexterity. Naproxen the following: 5 months of swollen joints in the hands,
has provided minimal relief. She reports fatigue but no photosensitive malar rash, documented pericarditis,
other associated symptoms. Prior history is notable only lymphopenia, autoimmune hemolytic anemia, and
for uterine ibroids, treated by total hysterectomy at age elevated antinuclear antibody (ANA) 1:1280 difuse
35. he patient denies any history of tobacco use, and pattern. Further serology demonstrates an elevated
her alcohol intake is minimal. Physical examination anti-Smith antibody, but other serologies and comple-
reveals redness, warmth, and swelling of multiple proxi- ment levels are normal.
mal interphalangeal (PIP) joints, metacarpophalangeal Which of the following therapies for SLE demands
(MCP) joints, both wrists, both ankles, and both knees. regular screening for potential ocular toxicity?

t
ne
Her examination is otherwise normal. Laboratory A. Mycophenolate mofetil
analysis reveals normal comprehensive metabolic pro- B. Methotrexate
ile and normal complete blood count with diferential. C. Cyclophosphamide

e.
Rheumatoid factor (RF), anticyclic citrullinated pep- D. Hydroxychloroquine
tide (anti-CCP) antibodies, and antinuclear antibodies E. Belimumab

in
(ANA) are not detectable. CRP is 12.5 mg/L (normal
<5). Plain ilm imaging of the hands reveals swelling of 6. A 46-year-old man presents to the emergency depart-

ic
the soft tissues around the afected joints but no other ment for his ifth episode of hepatic encephalopathy as
abnormalities. a complication of cirrhotic end-stage liver disease. He
Which of the following is the most appropriate irst
step in pharmacologic management of this condition?
ed also has volume overload and episodes of congestive
heart failure, with an ejection fraction of 30%. His
A. Intravenous rituximab 1000 mg for two infusions, family is adamant that he has never consumed alcohol
sm
given 2 weeks apart excessively. Comorbidities include poorly controlled
B. Duloxetine titrated up to 60 mg once daily diabetes mellitus on insulin therapy and osteoporosis.
C. Methotrexate titrated up to 25 mg once weekly Aspiration of ascites does not suggest spontaneous bac-
D. Doxycycline 100 mg twice daily terial peritonitis. He is treated with lactulose and loop
ok

E. Repeat the RF, anti-CCP antibody, and ANA diuretics, and his clinical status improves greatly. On
tests, and consider pharmacologic treatment only hospital day 4, the patient stumbles and reports some
if one of the tests is positive. pain in the right hand and wrist. It is not red, warm,
bo

or swollen, but chronic bony deformities are present


4. A 39-year-old woman presents to her primary care pro- in the distal interphalangeal (DIP), proximal interpha-
vider with severe right hip pain. It has come on steadily langeal (PIP), and metacarpophalangeal (MCP) joints.
e

over the past 6 months in an unrelenting fashion. She Plain ilm imaging is as shown in Fig. 3.2.
://

is now barely able to walk and uses a wheelchair when What is the most likely diagnosis?
in public. Physical examination reveals a woman of A. Rheumatoid arthritis
tp

average stature in distress when trying to lay supine on B. Fifth metacarpal fracture
the examining table, preferring to keep her right hip C. Hemochromatosis
slightly lexed. She has good range of motion in the D. Primary osteoarthritis of the hands
ht

left hip. However, the right hip has severe pain with E. Gaucher disease
any degree of extension or internal rotation. She can
passively externally rotate only 30 degrees. Hip lexion 7. A 49-year-old obese male laborer with a history of
is preserved. Abdominal and knee exams are normal. prescription drug abuse presents to his primary care
Plain ilm radiography shows complete collapse of the provider with low back pain of 4 months’ duration.
right femoral head with severe “bone-on-bone” osteo- He recalls no speciic antecedent injury. Since onset,
arthritis (OA). Magnetic resonance imaging (MRI) of he has had some degree of constant low back pain,
the right hip shows avascular necrosis (AVN) of the punctuated by exacerbations of pain when he twists
femoral head, with trace efusion. here is no pathol- or turns awkwardly. Two weeks ago he was instructed
ogy of the sacroiliac joints or psoas muscle. to “take some time of” from work, and he has been
Which of the following conditions is not a risk fac- mostly lying around the house hoping that his back
tor for developing avascular necrosis of the hip? will improve. Over the past 4 months, he has been
A. Antiphospholipid antibody syndrome taking escalating doses of oxycodone (now 15 mg
B. Osteoarthritis of the hip three times daily), obtained from friends or prescribed
CHAPTER 3 Rheumatology 51

R R

t
ne
e.
• Fig. 3.2 Plain radiograph of the right wrist and hand of the patient described in Question 6.

in
ic
by physicians at nine visits to six diferent emergency notable for a swollen, boggy left knee with a massive
rooms over the past 3 months. Workup available from efusion such that she cannot fully straighten her knee
those evaluations includes normal blood work and
normal lumbar spine ilms. He reports no fever, occult
ed and walks with a limp. here are no other joint abnor-
malities. Neurologic examination is normal. Laboratory
weight loss, bladder or bowel disturbances, or intrave- analysis reveals normal comprehensive metabolic proile
sm
nous drug use. Physical examination demonstrates no and complete blood count with diferential. Rheuma-
abnormalities except that he has reduced spine lex- toid factor (RF) and anticyclic citrullinated peptide
ion and extension because of pain. He reports that the (CCP) antibody testing is negative. ESR is 66 mm/h,
oxycodone is “not cutting it.” and CRP is 44.8 mg/L (normal <5). Joint aspiration
ok

What is the most appropriate next course of action? provides 80 mL of nonbloody, slightly cloudy luid with
A. Reassurance that this will improve white blood cell count 8400 cells/mm3 (80% lympho-
B. Provide a prescription for physical therapy and cytes). Fluid is negative for crystal analysis, Gram stain,
bo

discuss an “opiate medication contract.” and culture of infectious organisms.


C. Provide a prescription for hydromorphone 2 mg What is the most appropriate next step diagnostic
three times daily, with reassurance that this will workup or management?
e

improve. A. Serologic testing for Lyme disease


://

D. MRI of the lumbar spine with referral to inter- B. MRI of the left knee
ventional radiology for any bulging or herniated C. Antinuclear antibody (ANA) testing
tp

disc D. Corticosteroid injection of the left knee


E. Discharge the patient from your practice for abuse E. Referral to orthopedic surgery for arthroscopic
of opiate-based pain medications. evaluation of the left knee
ht

8. A 28-year-old Harvard University geology graduate stu- 9. A 39-year-old male kindergarten teacher presents to the
dent presents to student health services with an 8-week emergency department (ED) with severe joint pain “all
history of swelling in her left knee. Her knee is not par- over.” He was in his usual state of good health until 3
ticularly painful, but the swelling has made it progres- days prior, when he started to feel “stif” in the hands
sively diicult for her to get around. At irst, she cut and wrists. Subsequently, he has had progressive pain,
back on mountain climbing and long-distance running. redness, and swelling in the hands, wrists, and elbows.
She then had to stop riding her bicycle to class. Now she He has diiculty moving his shoulders because of pain,
has diiculty walking more than three city blocks, and and his iancé sent him to the ED because he could not
she avoids stairs whenever possible. She has no previous get out of bed easily in the morning. He reports no other
medical history and no other joint complaints. Com- symptoms. He carries sickle cell trait but has an oth-
prehensive review of systems is otherwise unrevealing. erwise unremarkable past medical and family history.
She is up to date with immunizations and routine health Physical examination is notable for a generally well-
prevention and maintenance. Physical examination is appearing, athletic-appearing man in signiicant distress
52 C HA P T E R 3 Rheumatology

during the physical examination because of severe pain hands as sclerodactyly, and it is associated with severe
in the arms and legs. He has such redness and puiness gastroesophageal relux disease (GERD) and symp-
in the hands that the assessment of his joints is dii- tomatic Raynaud phenomenon. Vital signs are nor-
cult. His elbows do not extend fully because of joint mal. Physical examination reveals sclerodactyly with
pain, and he cannot lift his arms or legs of the exami- acrocyanosis and tightness of the skin on the face.
nation table because of pain in the shoulders and hips, He has multiple matte telangiectasias on the skin
respectively. His knees are tender and slightly swollen. and mucosal surfaces. Lungs are clear to auscultation.
Ankles, feet, and toes have pufy tenderness and are dif- Cardiac evaluation demonstrates an accentuated para-
icult to examine further because of pain. He is afebrile doxically split P2. Six-minute walk test reveals 98%
and normotensive. Heart rate is 110 beats per minute oxygen saturation at rest, which drops to 92% with
but comes down to the 90s with opiate medications ambulation. Chest radiograph is normal.
for pain control. Remainder of examination is normal. Which of the following most likely explains the
Laboratory analysis reveals hemoglobin 9.4 g/dL, with patient’s dyspnea?
mean corpuscular volume 71 fL (normal 81–97 fL), A. Physical deconditioning
and reticulocyte count 0.1%. Platelet count and white B. Scleroderma cardiomyopathy with congestive heart

t
ne
blood cell count with diferential are normal, as is the failure
comprehensive metabolic proile. ESR is 49 mm/h, and C. Pulmonary hypertension
CRP is 251 mg/L (normal <5). Radiographic imaging D. Occult aspiration from undertreated GERD

e.
of the hands and feet demonstrates soft tissue swelling E. Pulmonary embolus
but no other abnormalities.

in
What is the most likely diagnosis? 12. A 47-year-old man with a history of rheumatoid
A. Septic polyarthritis arthritis (RA) is admitted to the hospital with fever,

ic
B. Viral arthritis dyspnea, and chest iniltrate. His RA treatment of
C. Fibromyalgia syndrome methotrexate and etanercept is held. he diagnosis
D. Lyme disease
E. Rheumatoid arthritis
ed upon admission is presumed community-acquired
bacterial pneumonia, and he is treated empirically
with a combination of ceftriaxone and azithromy-
sm
10. A 68-year-old woman presents to her primary care cin. However, after 3 days his status has deteriorated
provider because of fatigue, malaise, weight loss, and such that he is placed on a mechanical ventilator for
difuse achiness. Symptoms started insidiously about hypoxemia. Chest CT scan reveals necrotic-appear-
4 months ago. She has no other localizing symp- ing pulmonary nodules.
ok

toms. Her weight has dropped by 12 pounds since Which of the following is the most appropriate
her last visit for annual examination, 9 months previ- next course of action?
ously. Physical examination reveals a frail, fatigued- A. Intravenous methylprednisolone 100 mg three
bo

appearing woman with bitemporal wasting. Her gait times daily for treatment of rheumatoid lung
is slow and deliberate. She has reduced muscle bulk B. Bronchoscopy with lavage for microscopic organ-
and strength testing in the proximal arms and legs, isms
e

mostly due to achiness. Examination is otherwise C. Open lung biopsy


://

unrevealing. Laboratory analysis shows normal com- D. PET-CT scan to identify source of primary tumor
prehensive metabolic proile, white blood cell count, in the setting of pulmonary metastases
tp

and platelet count. Hemoglobin (Hgb) is 10.0 g/dL E. Switch antibiotic therapy to levoloxacin 750 mg
with normal mean corpuscular volume. ESR is 103 daily
mm/h, and C-reactive protein (CRP) is 2.4 mg/L
ht

(normal <5). 13. A 44-year-old male landscaper presents to his primary


Which of the following is the most appropriate care provider because of knee complaints. He has had
next course of action? pain, swelling, and limited motion in the right knee for
A. Antinuclear antibody testing 5 months. His only other musculoskeletal complaint
B. Serum and urine protein electrophoresis is nontraumatic swelling of the left fourth inger such
C. Temporal artery biopsy that his wedding ring had to be cut of. He is otherwise
D. CT scan of the chest, abdomen, and pelvis healthy, except for excessive beer consumption (12 or
E. Prednisone 15 mg daily more per day). Physical examination reveals a massively
swollen right knee with near-full extension but only 60
11. A 53-year-old man with long-standing scleroderma degrees of lexion. His left fourth inger is also swol-
presents with worsening dyspnea on exertion. Over len and tender, with minimal lexion. Remainder of the
the past 2 years, he has noticed increasing diiculty joint examination is unrevealing. Laboratory analysis
climbing stairs and keeping up with his spouse when is essentially normal or negative: comprehensive meta-
walking. His scleroderma predominantly afects the bolic proile, complete blood count and diferential,
CHAPTER 3 Rheumatology 53

CRP, ESR, Lyme disease serology, antinuclear antibody,


rheumatoid factor, and anti-CCP antibody. Arthrocen-
tesis yielded 90 mL of nonbloody, cloudy, yellow luid
with 65,000 white blood cells (60% neutrophils). Fluid
analysis is negative for polarizable crystals and micro-
biologic organisms.
Which of the following is most likely to be diagnos-
tic in this case?
A. Detectable anti-Smith antibody
B. A serum uric acid of 11.5 mg/dL
C. Elevated serum angiotensin-converting enzyme
(ACE) level
D. Scaly erythematous plaques on the extensor sur-
faces of the arms and legs
E. Presence of chondrocalcinosis on plain ilm radi-

t
ne
ography of the right knee • Fig. 3.3 Hand of the patient described in Question 15.

14. A 21-year-old woman visits her primary care pro-

e.
vider because of difuse joint pains for the past 18 that include coronary artery disease, hypertension,
months. hey involve her ingers, wrists, elbows, atrial ibrillation, type 2 diabetes mellitus, and chronic

in
shoulders, hips, knees, ankles, and spine. She has tobacco use complicated by emphysema. Her only
noticed no redness, warmth, or swelling of the joints, other complaint is a new-onset rash on her face and

ic
nor any muscle pain. She reports that her shoulders hands (Fig. 3.3). Physical examination is notable for
and hips feel like they are “popping out of joint” with reduced strength. She is able to get her thighs of the
physical itness, including during her activities as a
semiprofessional ballerina. She reports palpitations
ed examining table against gravity, but not against mild
resistance testing. She has similar weakness in her
and dizziness when sitting upright quickly and states shoulders. Grip strength is preserved. Lungs have some
sm
that she has “almost passed out” on several occa- mild wheezes at the apex but also bibasilar Velcro-type
sions. Comprehensive review of systems is otherwise rales. Laboratory testing reveals elevated creatine phos-
unremarkable. Physical examination is notable for phokinase (CPK) 10,200 U/L, CK-MB 340 ng/mL,
a thin woman in no acute distress. She has normal aspartate aminotransferase (AST) 348 U/L, and alanine
ok

seated heart rate and blood pressure, and no chest aminotransferase (ALT) 419 U/L. Albumin is 3.1 g/dL.
wall abnormalities. Arms and legs are well propor- She has otherwise normal comprehensive metabolic
tioned without deformity, and she has no red, warm, proile, complete blood count with diferential, and
bo

swollen joints. She has more than 90 degrees of pas- coagulation studies. Electrocardiogram (EKG) does not
sive extension at the wrists and metacarpophalangeal demonstrate any abnormalities. Chest radiograph dem-
(MCP) joints. Elbow and knee joints do not have onstrates ibrotic changes at the bibasilar lung ields.
e

efusions, and she can extend them 190 degrees volun- Which of the following tests is most likely to be
://

tarily. Spine and hip motion is excellent; she is able to diagnostic for this condition?
place her palms lat on the loor without bending her A. Liver biopsy
tp

knees. Ocular, skin, cardiopulmonary, and neurologic B. Temporal artery biopsy


examinations are normal. Echocardiography for the C. Cardiac MRI
palpitations reveals only mild mitral valve prolapse. D. Cardiac catheterization
ht

Which of the following studies is most likely to be E. Skeletal muscle biopsy


abnormal in this patient?
A. Tilt-table test 16. A 25-year-old man presents to the emergency depart-
B. MRI of the left shoulder ment for an episode of acute right lower quadrant
C. CT scan of the right hip abdominal pain, which started abruptly 3 days prior. He
D. Serum RF and anti-CCP antibodies has had multiple episodes similar to this in the past, dating
E. Lyme disease serology back to a bout of appendicitis at age 4. Associated symp-
toms include fever, nausea, and anorexia. Medical history
15. A 68-year-old woman presents to her primary care pro- also includes presumed gout (lacking a crystal-proven
vider because of weakness of the shoulders and hips. diagnosis). Past surgical history includes four separate
For 5 weeks, she has noticed progressive diiculty abdominal surgeries: appendectomy, cholecystectomy,
climbing stairs and trouble getting out of a chair. She and two exploratory laparoscopies. here is no family
denies any arthralgias or myalgias, however. She has history of any similar condition. Physical examination
preexisting chronic dyspnea from various comorbidities reveals a thin man appearing to be in a toxic condition
54 C HA P T E R 3 Rheumatology

who is unwilling to maneuver in bed because of abdom- C. Tear of the medial meniscus
inal pain. Vital signs include temperature 39.2°C, D. Tear of the anterior cruciate ligament (ACL)
blood pressure 92/58 mm Hg, heart rate 125 beats per E. Popliteal (Baker) cyst
minute, respiratory rate 24 breaths per minute, and 99%
oxygen saturation on room air. he abdomen is slightly 18. A 21-year-old female college student presents to
distended and exquisitely tender to gentle palpation. the emergency department with new-onset fever,
Bowel sounds are absent. Rectal examination reveals headache, dyspnea, and weight gain. She was in
normal tone but no blood. Remainder of examination is her usual state of excellent health until 6 weeks
unrevealing. Laboratory analysis reveals normal compre- prior, when her current symptoms started. She has
hensive metabolic proile except for albumin 2.3 g/dL. noticed puiness around her eyes and swelling in
Creatinine is 1.0 mg/dL. Uric acid is 4.2 mg/dL. Com- her ankles. Associated symptoms include cold in-
plete blood count is notable for white blood cell count gers and toes with blue discoloration. She takes no
21.2 × 103 cells/mm3 (72% neutrophils and 13% medications regularly. Vital signs were notable for
bands), hemoglobin 10.4 g/dL (MCV 88 fL), and plate- temperature 37.9°C, blood pressure 192/110 mm
lets 630 × 103 cells/mm3. CRP is 310 mg/L and ESR Hg, heart rate 108 beats per minute, respiratory

t
ne
is 107 mm/h. Urinalysis is notable only for 4+ protein, rate 22 breaths per minute, and oxygen saturation
with a spot protein/creatinine ratio of 2.1. CT scan of 92% on room air. She is ill appearing, with peri-
the abdomen and pelvis demonstrates some free peri- orbital and lower-extremity edema. Other notable

e.
toneal luid, but no free air or other evidence of bowel features of the physical examination include lungs
rupture. with bibasilar wet rales and acrocyanosis with digi-

in
Which of the following studies is most likely to tal ulcerations. here is no sclerodactyly. Labora-
provide the correct diagnosis? tory analysis includes blood urea nitrogen (BUN)

ic
A. MEFV genetic testing 82 mg/dL, creatinine 5.2 mg/dL, albumin 1.9 g/
B. Endoscopy with biopsy of the duodenum to look dL, Hgb 8.4 g/dL with MCV 82 fL, platelet count
for lattened villi and crypts
C. Traditional angiography of the mesenteric vascula-
ed 74 × 103 cells/mm3, and white blood cell count 2.5
× 103 cells/mm3 with 80% neutrophils, 10% lym-
ture phocytes, 6% monocytes, and 3% eosinophils. Uri-
sm
D. Colonoscopy with biopsy of the terminal ileum nalysis demonstrates 50–100 RBCs with casts, +++
E. Serum antineutrophil cytoplasmic antibody (ANCA) protein, and no leukocytes. ESR is 110 mm/h, CRP
testing is 12.9 mg/L (normal <5), and antinuclear anti-
body is 1:5120 speckled pattern. Blood cultures are
ok

17. A 22-year-old female college student presents to uni- normal. EKG is normal. Chest radiograph demon-
versity health services because of bilateral knee pain. strates lufy vascular congestion and trace bibasilar
She is an avid runner, but she has had to curtail her efusions.
bo

normal regimen of 30 miles per week because of knee Which of the following additional test results
pain. It has come on rather severely over the past 3 would be most consistent with her presentation?
weeks, and localizes mostly at the anterior knee. It is A. p-ANCA–positive, antimyeloperoxidase (MPO)
e

exacerbated by rapidly standing from a chair or by antibody–negative, and antiproteinase-3 (PR3)


://

descending stairs. She denies knee swelling or lateral antibody–negative


thigh discomfort. She has not had morning stifness B. Uric acid 13.5 mg/dL and bone marrow biopsy
tp

or other joint pains. She is tearful because of the sever- with 75% clonal promyelocytes
ity of the knee pain and the efect it has had on her C. Anti–double-stranded DNA antibody 522 U/mL
exercise. She is otherwise healthy. Physical examina- (normal <5), C3 complement 52 mg/dL (nor-
ht

tion reveals a thin, athletic-appearing young woman. mal 75–175), C4 complement 8 mg/dL (normal
She has excellent range of motion in all joints except 14–40)
for the knees, where she has pain with lexion of the D. CPK 8600 U/mL, anti-Jo1 antibody 82 U/mL
knee beyond 110 degrees. She can extend fully. here (normal <5)
are no knee efusions. She is tender along the patellar E. HLA-B27–positive, sacroiliac joint fusion on plain
margins, especially if the patella is compressed down- radiography, and renal biopsy showing changes
ward during active knee extension. here is no ten- most consistent with IgA nephropathy
derness along the lateral knees, nor any posterior knee
tenderness. She has negative McMurray sign and no 19. A 58-year-old man presents to the outpatient clinic
instability on varus/valgus stress maneuvers or ante- with 3 days of an excruciatingly painful, red, warm,
rior/posterior drawer sign testing. swollen left great toe. He has had to use crutches and
Which of the following is the most likely diagnosis? been unable to sleep because of the severity of pain.
A. Patellofemoral syndrome He has had episodes similar to this in the past, and on
B. Iliotibial band syndrome one occasion he had an arthrocentesis that provided
CHAPTER 3 Rheumatology 55

a “crystal-proven” diagnosis of gout. Relevant comor- What is the most appropriate next step in manage-
bidities include stage IV chronic kidney disease (glo- ment?
merular iltration rate 19 mL/min) as a complication A. Reassure the patient that his symptoms are a com-
of hypertension. He has never had nephrolithiasis. mon complication of lumbar puncture. He should
Physical examination reveals a red, warm, very tender remain supine as much as possible for 48 hours
left irst metatarsal phalangeal joint. here are no other and touch base on Monday morning if his symp-
active joints. he bilateral olecranon bursae have large toms persist.
tophi. B. Arrange for an expedited temporal artery biopsy
What is the most appropriate next course of action? irst thing Monday morning. In the meantime,
A. Check a serum uric acid (sUA) level and initiate inform the patient to call back in the event of any
allopurinol if the sUA is >6 mg/dL. vision loss.
B. Check a 24-hour urine uric acid and initiate pro- C. Prescribe prednisone 1 mg/kg. Instruct the
benecid 500 mg twice daily if the patient is an patient to start it immediately and to call with
“under-excretor.” any new visual disturbances. Arrange for an expe-
C. Treat with colchicine 0.6-mg tablets: 2 tablets to dited temporal artery biopsy irst thing Monday

t
ne
start and 1–2 tablets per day for the next 5 days. morning.
D. Treat with naproxen 500 mg twice daily with food. D. Inform the patient that he likely has viral men-
E. Treat with intramuscular methylprednisolone 80 ingitis and should return to the ED for treat-

e.
mg ×1. ment.
E. Inform the patient that he may have a cerebral

in
20. A 50-year-old man presents to his primary care pro- aneurysm and refer him back the ED for magnetic
vider because of rapid-onset right shoulder pain of resonance angiography of the brain.

ic
3 days’ duration. He was previously healthy, recalls
no antecedent injury or trauma, and has never experi- 22. A 66-year-old obese woman with a history of chronic
enced shoulder pain of this severity before. On physical
examination, the shoulder is not visibly red, warm,
ed back pain presents for an initial evaluation of left lat-
eral hip discomfort of 5 weeks’ duration. She reports
or swollen. He experiences signiicant discomfort on no antecedent injury, but it hurts to sleep on that
sm
active abduction and forward lexion (against side at night. Symptoms do not radiate to the groin.
resistance) until he reaches 90 degrees horizontal, at On physical examination, she has an antalgic wad-
which point pain limits further motion. Passive range dling gait but demonstrates good active and passive
of motion testing achieves an additional 30 degrees in range of motion in the left hip. Passive adduction
ok

these planes. He does not tolerate resisted internal reproduces her lateral hip pain, as does palpation
or external rotation, nor resisted abduction with his over the lateral hip.
arm held slightly in front of his body in internal Which of the following is an appropriate next step
bo

rotation. in diagnosis or management of this condition?


What is the most likely diagnosis? A. Reassurance and physical therapy, focusing on pos-
A. Biceps tendonitis ture, core muscle tone, and hip range of motion
e

B. Septic arthritis B. MRI of the left hip


://

C. Acute calciic tendonitis C. Methylprednisolone taper from 24 mg down to 0


D. Glenohumeral osteoarthritis over 6 days (dropping the dose by 4 mg each day)
tp

E. Complete tear of the supraspinatus D. Whole-body bone scan


E. Bone mineral densitometry
21. An 84-year-old generally healthy man calls his pri-
ht

mary care provider (PCP) on a Saturday afternoon to 23. A 59-year-old woman with long-standing destruc-
report 3 weeks of unilateral headache, scalp tender- tive rheumatoid arthritis (RA) has recently moved to
ness, and achiness in the jaw when chewing a bagel. the area and is scheduled to undergo revision total
He has had no visual disturbances and denies any hip arthroplasty because of refractory pain and func-
head trauma. Two nights prior, he went to a local tional limitations. She is seeing you for preoperative
emergency department (ED) for these same symp- clearance evaluation. Her primary complaint is gait
toms, where he was diagnosed with a new migraine instability, urinary incontinence, and paresthesias in
headache, given acetaminophen 1000 mg three the bilateral arms. Prior records are not immediately
times daily, and instructed to call his PCP in the available, but you know that she has already had
event that his symptoms failed to improve. Labora- multiple previous orthopedic procedures, including
tory testing from the ED revealed ESR 110 mm/h, bilateral hip and knee arthroplasties, bilateral wrist
normal cerebrospinal luid analysis, and a head CT fusion surgeries, and a left elbow arthroplasty. For
scan that did not show any evidence of intracerebral her RA, she is taking etanercept and methotrexate,
hemorrhage. which will be stopped by her rheumatologist 2 weeks
56 C HA P T E R 3 Rheumatology

prior to surgery. She has no history of heart disease Which of the following serologic tests, if positive,
or identiiable risk factors other than the RA. Physi- would best explain the above clinical scenario?
cal examination reveals general muscle atrophy and A. Antinuclear antibody
multiple joint deformities. here are no red, warm, B. Hepatitis C antibody
or swollen joints. Neurologic testing reveals hyper- C. Anti-CCP antibody
relexivity of the bilateral upper extremities and posi- D. Heterophile antibody testing (monospot)
tive Hofman sign. Gait cannot be tested because of E. Lyme disease serology
instability.
What is the most important preoperative testing to 25. A 68-year-old man is seen by his primary care provider
be done in this patient? for osteoarthritic right knee pain of 12 years’ dura-
A. Rheumatoid factor (RF) and anticyclic citrulli- tion. He initially had pain only when running, but
nated peptide (CCP) antibody testing this progressed to pain on ambulation. Currently, he
B. ESR and CRP testing reports pain when walking more than two city blocks,
C. Cardiac exercise stress testing with sestamibi- standing for more than 20 minutes, and nocturnal
enhanced nuclear imaging knee pain. He reports no knee swelling, locking, or

t
ne
D. Lateral cervical spine x-ray imaging, with lexion instability. When out with friends, he cannot keep
and extension views up with them and frequently has to sit because of his
E. Lumbar spine MRI knee pain. Physical therapy and naproxen 500 mg

e.
twice daily have not adequately relieved his pain. He
24. A 48-year-old man with a history of alcoholism and has derived only 2 weeks of relief from each of three

in
cirrhosis presents to urgent care because he has been prior corticosteroid injections into the right knee
tripping over his own feet. He describes an inability joint. Comorbidities include type 2 diabetes melli-

ic
to get his toes of the ground when walking. He has tus, cerebrovascular disease with mild carotid steno-
not had any change in his baseline cirrhosis, and his sis, hypertension, hyperlipidemia, and obesity (body
mental status is lucid. He describes general arthral-
gias with some morning stifness. Physical examina-
ed mass index 34.2 kg/m2). On physical examination,
he has good range of motion in the bilateral hips, left
tion reveals stigmata of cirrhosis with scleral icterus, knee, and bilateral ankles. Both knees have hypertro-
sm
jaundice, and abdominal ascites. Gait is altered by phic osteoarthritic changes, crepitus, mild efusions,
left-leg toe drag that occurs throughout stride, and he full extension, and only 90 degrees of lexion. He has
lacks strength in the tibialis anterior muscle. Lower- genu valgus deformity of both knees and no instability
extremity relexes are otherwise intact. He has a pain- on varus/valgus stress maneuvers or anterior/posterior
ok

less rash on the legs as shown in Fig. 3.4. He has drawer sign testing. here is minimal tenderness on
difuse tenderness of muscles and joints but no red, the left but difuse joint-line tenderness on the right.
warm, or swollen joints. Laboratory testing reveals Previous right knee aspiration has yielded 25 mL of
bo

the following: serum sodium 129 mEq/L, serum cre- luid with 288 white blood cells per mm3. Plain ilm
atinine 1.4 mg/dL, normal complete blood count radiography shows tricompartmental osteoarthritic
with diferential, CRP 88.1 mg/L, ESR 99 mm/h, change of both knees with signiicant joint space nar-
e

rheumatoid factor 110 U/mL (normal <15), C3 rowing medially.


://

complement 112 mg/dL (normal 75–175), and C4 What is the most appropriate next course of action?
complement 5 mg/dL (normal 14–40). Urinalysis A. Inject the right knee with triamcinolone 40 mg,
tp

has 100–200 RBC per high-power ield, with some prescribe physical therapy, and advise weight loss.
dysmorphia. here is +++ protein. B. Switch naproxen to oxycodone 5 mg three times
daily. Reassess in 1 year.
ht

C. MRI of the right knee


D. Refer to orthopedics for arthroscopic debridement
of loose cartilage.
E. Refer to orthopedics for consideration of right
total knee arthroplasty.

26. A 28-year-old woman presents to her primary care


provider with numbness and paresthesias in the irst
three ingers of both hands. Symptoms are worst when
she irst awakens in the morning, and they impact her
profession as a hairstylist. Physical examination reveals
no redness, warmth, or swelling of the ingers, thumbs,
or wrists. She has preserved thenar muscle strength
• Fig. 3.4 Leg of the patient described in Question 24. and no atrophy. Passive forced lexion of the wrist
CHAPTER 3 Rheumatology 57

reproduces her symptoms after 30 seconds. here is comprehensive metabolic proile, complete blood
no swelling of the wrist, and range of motion is intact. count and diferential, thyroid-stimulating hormone
She is diagnosed with carpal tunnel syndrome (CTS). (TSH), creatine kinase, and C-reactive protein.
Which of the following conditions is not associated What is the next most appropriate step in diagnos-
with CTS? tic testing or management?
A. Acromegaly A. Electromyogram (EMG) with nerve conduction
B. Scleroderma studies (NCS) of all four extremities
C. Chondrocalcinosis of the wrist B. Test antinuclear antibody (ANA) and rheumatoid
D. Rheumatoid arthritis factor (RF).
E. Hypothyroidism C. Serology testing for Lyme disease
D. Recommend combination of physical therapy, sleep
27. A 51-year-old morbidly obese man presents to his routine, weight loss, and psychiatric counseling.
primary care provider with paresthesias and painless E. Initiate oxycodone 5 mg twice daily, with dose
numbness of the anterior left thigh. Symptoms are escalation as needed.
best when he irst awakens in the morning and pro-

t
ne
gressively worsen over the course of a day. He has 29. A 24-year-old woman presents to her primary care
noticed this coming on over the previous 3 weeks. provider with discomfort in her ingers and toes.
Comorbidities include diabetes mellitus. Physical Symptoms have been present for 4 years and are dis-

e.
examination reveals clearly demarcated absence of sen- tinctly worse in the winter. Cold exposure causes white
sation overlying the anterior thigh. here are no motor and blue color changes, leading to numbness and stif-

in
deicits. Lower-extremity relexes are intact. Hip range ness in the involved digits. Symptoms are alleviated
of motion is normal. by rewarming the ingers, but they are accompanied

ic
Which of the following explains the most likely by iery red erythema and discomfort. Medical history
diagnosis? is notable only for attention-deicit/hyperactivity dis-
A. Diabetic amyotrophy
B. Avascular necrosis of the left femoral head
ed order, treated with dextroamphetamine. On physical
examination, there is no erythema, warmth, or swell-
C. Entrapment neuropathy of the left lateral femoral ing in the ingers or wrists. here is no sclerodactyly or
sm
cutaneous nerve periungual erythema. Digital nailfold capillaroscopy
D. Irritation of the gluteus medius and gluteus mini- reveals mild tortuosity of capillaries without microan-
mus muscles at their tendon insertion on the left eurysms, hemorrhage, or dropout. Radial and ulnar
greater trochanter pulses are brisk, and Allen test is normal bilaterally.
ok

E. Degenerative arthritis of the left hip Which of the following is the most appropriate
next step in diagnostic workup?
28. A 49-year-old woman presents to a new primary care A. Test for antinuclear antibodies, anti-dsDNA, anti-
bo

provider with complaints of difuse joint and muscle Ro, anti-La, anti-Smith, and anti-RNP antibodies.
pains that have been present for over 20 years. he B. Test for antinuclear antibodies, anti-SCL70, and
pain has escalated over the past 18 months to the anticentromere antibodies.
e

point where she has stopped working as a cashier and C. Test for antinuclear antibodies, antiphospholipid
://

is applying for disability. She is no longer able to derive antibodies, and lupus anticoagulant.
enjoyment from her young grandchildren, because she D. Magnetic resonance angiography of the bilateral
tp

is always in pain. When asked about red, warm, or upper extremities


swollen joints, she reports a severe burning pain in her E. No additional workup necessary
arms, legs, and back. She has no signiicant comorbid-
ht

ities other than obesity (body mass index 41.1 kg/m2) 30. A 46-year-old woman with systemic sclerosis (sclero-
and chronic back pain. Review of systems is notable derma) presents to the emergency department because
for fatigue, weight gain, and poor sleep. She does not of headache, dyspnea, and lower-extremity edema.
smoke or drink excessive alcohol, but she describes a She has had recent complications of her scleroderma,
lot of stress related to the loss of her job and prolonged including Raynaud crisis with gangrene of the left
divorce proceedings over the past 2 years. Physical fourth inger, interstitial pulmonary ibrosis, and
examination is notable for an obese woman who is pericardial effusion. Vital signs include tempera-
tearful throughout the musculoskeletal exam. She has ture 36.8°C, blood pressure 140/95 mm Hg in both
difuse tenderness in the arms, legs, and thorax. here arms, heart rate 72 beats per minute, respiratory
are no obviously warm, swollen, or tender joints, but rate 24 breaths per minute, and oxygen saturation
the assessment is partly limited by obesity and her 94% on room air. Physical examination reveals an
distress even with light touch. Neurologic examina- ill-appearing individual with scleroderma involv-
tion reveals reduced efort at strength testing but nor- ing the hands, arms, face, trunk, legs, and feet. She
mal when distracted. Laboratory analysis is normal: has scattered cutaneous telangiectasia. She has dry
58 C HA P T E R 3 Rheumatology

gangrenous changes of the left fourth inger distal to 33. A 28-year-old woman presents to the emergency
the distal interphalangeal joint. Lungs have bibasi- department because of severe pain and swelling
lar Velcro rales. Cardiac examination is unrevealing. in multiple joints. Two weeks ago she was admit-
Laboratory analysis reveals BUN 40 mg/dL, creati- ted with severe gastrointestinal dysentery, from
nine 3.2 mg/dL, white blood cell count 7.2 × 103 which she has recovered. Ibuprofen has ofered little
cells/mm3, hemoglobin 8.9 g/dL, and platelet count relief. Physical examination reveals overt erythema,
85 × 103/mm3. Blood smear demonstrates schisto- warmth, and swelling of the ingers, hands, wrists,
cytes. Prothrombin time and partial thromboplastin and elbows. She has a swollen left knee. Laboratory
time are normal. Liver function studies are normal. testing reveals an ESR 88 mm/h and CRP 182 mg/L
Urinalysis shows ++++ protein and >100 RBC per (normal <5).
high-power ield. Chest radiograph is consistent with Which of the following microorganisms is not asso-
pulmonary edema. here is no cardiomegaly. EKG is ciated with reactive arthritis?
unrevealing. A. Staphylococcus aureus
Which of the following must be included in the B. Streptococcus pyogenes
next steps of managing this patient? C. Chlamydia trachomatis

t
ne
A. Plasma exchange therapy D. Yersinia pestis
B. Captopril 25 mg orally three times daily, with dose E. Salmonella enteritidis
escalation to control blood pressure

e.
C. Labetalol 100 mg orally twice daily, with dose 34. A 52-year-old obese woman presents with bilateral
escalation to control blood pressure foot pain of 3 weeks’ duration. Symptoms localize

in
D. Amlodipine 5 mg orally once daily, with dose esca- to the plantar aspect of the foot and are worse irst
lation to control blood pressure thing in the morning when she steps out of bed, last-

ic
E. Methylprednisolone 250 mg intravenously every ing for at least 2 hours. She has not noticed any red-
12 hours ness, warmth, or swelling of the feet, nor in any other

31. A 68-year-old diabetic man presents to the emergency


ed joints. Other medical conditions include osteoporo-
sis secondary to surgical menopause at age 31. Physi-
department because of right ankle pain and altered cal examination demonstrates no redness, warmth,
sm
gait. On physical examination, the ankle is swol- or swelling of the feet and no tenderness on squeeze
len posteriorly but has full passive range of motion. of the metatarsophalangeal joints. She has focal ten-
Squeezing of the calf with the patient seated does not derness to palpation at the plantar aspect of the cal-
result in dorsilexion of the ankle. Two weeks ago he caneus bones. Plain ilm radiography of the feet is
ok

had been admitted to the hospital with pneumococcal normal.


pneumonia. Which of the following is the most likely diagnosis?
What was the most likely agent used to treat his A. Occult stress fracture of the ifth metatarsal bone
bo

pneumonia? B. Plantar fasciitis


A. Imipenem C. Acute gouty arthritis of the talonavicular joint
B. Trimethoprim/sulfamethoxazole D. Charcot arthropathy
e

C. Cefpodoxime E. Occult injury of the anterior taloibular ligament


://

D. Levoloxacin
E. Azathioprine 35. A 28-year-old woman has rheumatoid arthritis (RA)
tp

that is well controlled on a combination of methotrex-


32. A 77-year-old man is recovering in the surgical inten- ate and adalimumab (anti-TNF-α monoclonal anti-
sive care unit from coronary artery bypass graft surgery. body). She will be traveling to the Brazilian Amazon
ht

On postoperative day 3, he develops severe pain of the rain forest as part of a graduate school degree program
anterior aspect of the right knee with focal erythema, in anthropology. She has never been agreeable to
warmth, and swelling. On physical examination, he immunizations in the past, but she is requesting what-
keeps his right leg fully extended and yells in pain if the ever the travel clinic advises.
knee is lexed actively or passively. he knee is exqui- In addition to malaria prophylaxis and education
sitely tender over the patella. Examination does not about appropriate use of insect repellant to avoid den-
demonstrate any perceivable knee luid for aspiration. gue and Zika virus infections, which of the following
Which of the following is the most likely diagnosis? vaccine regimens should she receive?
A. Gouty bursitis of the right prepatellar bursa A. Hepatitis A vaccine, oral typhoid vaccine, yellow
B. Gouty arthritis of the right knee fever vaccine, and MMR booster
C. Septic arthritis of the right knee B. Hepatitis A vaccine, injectable typhoid vaccine,
D. Right knee meniscal injury from unintended and yellow fever vaccine
intraoperative injury C. Hepatitis A vaccine, injectable typhoid vaccine, and
E. Quadriceps tendon rupture a letter of exemption from yellow fever vaccine
CHAPTER 3 Rheumatology 59

D. Hepatitis A vaccine, injectable typhoid vaccine, prick and proprioception in the posterior right thigh
and a letter of exemption from yellow fever and and lateral toes of the right foot.
MMR vaccines Which of the following is the most appropriate next
step in evaluation and management of this patient?
36. A 25-year-old previously healthy woman presents to A. Physical therapy and reassurance. Reassess in 4
her primary care provider with bilateral ankle pain and weeks.
a rash. Symptoms started 2 weeks ago as pain, swell- B. Plain ilm radiography of the back. If normal, then
ing, and tenderness in the bilateral ankles. Her feet are ofer physical therapy and reassurance. Reassess in
swollen to the point that she cannot wear her regular 4 weeks.
shoes and it hurts to walk. No other joints are involved. C. Schedule nonurgent MRI of the lumbar spine. If
Ten days ago, she began to notice tender erythematous there is a herniated disc, then ofer physical ther-
“lumps” on her anterior shins. Physical examination apy and reassurance. Reassess in 4 weeks.
conirms the history. She has swelling around both D. Schedule nonurgent MRI of the lumbar spine. If
ankles and reduced active range of motion because of there is a herniated disc, then refer to interven-
pain. Passive range of motion is preserved, however, tional radiology for corticosteroid injection.

t
ne
including lexion/extension and inversion/eversion at E. Referral to emergency department for urgent MRI
the ankle. he Achilles tendon is unafected. he shins of the lumbar spine
have fewer than a dozen scattered tender erythematous

e.
nodules of varying size, none larger than 1 cm. here is 38. A 50-year-old woman has had chronic systemic sar-
no overlying skin breakdown. coidosis for 4 years. She has had involvement of the

in
Which of the following tests would be expected to heart, liver, lungs, skeletal muscle, and joints. Despite
show an abnormality that would aid in the diagnosis aggressive immunosuppressive measures with metho-

ic
of her condition? trexate and inliximab, she has continued to require
A. Ankle ilm chronic corticosteroid therapy with daily prednisone.
B. Chest radiograph
C. Rheumatoid factor and anti-CCP antibody
ed he prednisone dose is currently 40 mg daily. It has
luctuated between 20 and 60 mg daily with disease
D. Serum uric acid level activity, and has been no less than 20 mg daily for
sm
E. Serum Lyme disease serologies 3 years. On physical examination, she is obese with
cushingoid features. Blood pressure is 150/95 mm Hg,
37. A 32-year-old man presents with low back pain and and body mass index is 39.5 kg/m2. Comorbidities
leg numbness. hree weeks ago, he was lifting window include steroid-induced diabetes mellitus.
ok

air-conditioning units when he felt a “pop” in his low Which of the following conditions is not a direct
back associated with severe pain. He rested his back for side efect of chronic corticosteroid therapy?
1 week and has been using a friend’s prescription for A. Increased risk of infectious illnesses
bo

hydrocodone. With this, the pain has improved slightly. B. Accelerated cataract formation
Two weeks ago, however, he began to notice constipa- C. Hyperuricemia
tion such that he has not had a bowel movement in 8 D. Avascular necrosis of bone
e

days. He has also had new-onset impotence and reports E. Proximal muscle weakness
://

increased urinary urgency with frequent dribbling of


only small volumes of urine. his is associated with par- 39. A 74-year-old woman presents to the outpatient clinic
tp

esthesias down the right posterior leg and numbness in reporting 8 weeks of sudden-onset fatigue and achiness
the perineum. Physical examination reveals a man who in the shoulders and hips. Two weeks ago she saw an
does not appear to be in a toxic condition and who has orthopedic surgeon, who diagnosed rotator cuf ten-
ht

trouble inding a comfortable position on the examina- donitis and injected 40 mg of triamcinolone into the left
tion table. Back range of motion is limited to 30 degrees subacromial bursa. She reports that, following the injec-
of forward lexion because of lumbar spine pain, and tion, “everything went away and I felt great.” However,
there is associated spasm of paraspinous muscles. Neu- this beneit only lasted 3 days, and all of her symptoms
rologic examination is entirely normal in the upper have returned. She has no other medical illnesses. She
extremities. Assessment of lower extremities reveals has not had any red, warm, or swollen joints. She denies
weakness of ankle plantar lexion on the right with intact any headaches, visual disturbances, scalp tenderness, or
ankle dorsilexion and hip lexion. His gait is altered in jaw achiness after chewing. Laboratory analysis reveals
that he shortens the length of stride during the “push- ESR 64 mm/h and CRP 76.2 mg/L (normal <5.0).
of” phase on the right leg and cannot stand on his toes Which of the following features also typically
on the right because of weakness. Relexes are normal at characterizes the condition most likely afecting this
the bilateral patellae, diminished at the left Achilles, and patient?
absent at the right Achilles. Cremasteric relex is intact. A. Morning stifness
Sensory examination reveals reduced detection of pin B. Blindness
60 C HA P T E R 3 Rheumatology

C. Elevated rheumatoid factor phototherapy. On day 9 of life, the infant develops a


D. Chondrocalcinosis patchy, well-circumscribed maculopapular rash.
E. Joint space narrowing on plain ilms of shoulders Which of the following conditions in the mother
and hips might explain the newborn’s condition?
A. Sjögren syndrome
40. A 35-year-old woman presents to urgent care for B. Lyme disease
fever, rash, and joint pain. She was in her usual state C. Marfan syndrome
of health until 3 weeks ago, when she experienced a D. Acute rheumatic fever
severe sore throat. She was seen by her primary care E. Maternal methotrexate exposure during preg-
provider. Testing for strep throat by rapid testing nancy
and culture was negative, but she was treated with
empiric amoxicillin, without improvement (com- 42. A 75-year-old man presents to his primary care pro-
pleted 10 days ago). She has subsequently developed vider for an annual physical examination. He has
pain and swelling in the wrists and high daily fevers relatively few comorbidities, but he reports ongoing
that are accompanied by a generalized rash. She chronic daily low back pain that radiates into the but-

t
ne
feels generally well between attacks but has not had tocks. He reports morning stifness and progressive
any relief of episodes with naproxen 500 mg twice pain over the course of the day. He describes diiculty
daily. Physical examination is notable for a woman in standing for more than 10 minutes, and he has

e.
of average stature who is uncomfortable but does to lean on a walker or shopping cart when trying to
not appear to be in a toxic condition. Temperature get out and about during the day. For his back pain,

in
is 38.9°C, blood pressure is 110/72 mm Hg, heart he has (1) taken tramadol 50 mg three times daily
rate is 110 beats per minute, respiratory rate is 18 and acetaminophen 1000 mg three times daily, (2)

ic
breaths per minute, and oxygen saturation is 98% engaged in regular physical therapy, and (3) received
on room air. Neck is supple, but she has cervical and multiple epidural corticosteroid injections and facet
axillary adenopathy. She has redness, warmth, and
swelling of the bilateral wrists and metacarpophalan-
ed joint injections over the past 12 months. None of
these approaches has provided adequate relief. Physi-
geal joints. Skin shows a salmon-pink maculopapular cal examination is largely unrevealing except for
sm
rash involving mostly her trunk. Laboratory analy- weakness of hip lexor strength and reduced lexion
sis is notable for a normal comprehensive metabolic and extension of the lumbar spine. His gait is slow
proile. Complete blood count reveals white blood and deliberate, but steady. Plain ilm radiography
cell count 18.5 × 103 cells per mm3 (70% neutro- and MRI from 1 year ago have revealed multilevel
ok

phils and 15% bands), Hgb 11.1 g/dL, and platelet degenerative changes with disc desiccation, nerve
count 772 × 103/mm3. CRP is 199 mg/L (normal impingement, facet joint arthropathy, and central
<5), and ESR is 92 mm/h. Blood cultures obtained 5 spinal stenosis.
bo

days prior have demonstrated no growth. ANA, RF, Which of the following is the next most appropri-
and anti-CCP antibodies are all negative. CT scan ate next course of action?
of the abdomen, chest, and pelvis reveals some mild A. Reassurance and change tramadol to oxycodone 5
e

hepatosplenomegaly and difuse adenopathy. Periph- mg three times daily


://

eral blood low cytometry does not reveal any hema- B. Reassurance and another prescription for physical
tologic malignancy. Excisional biopsy of an axillary therapy
tp

lymph node excludes a malignancy and demonstrates C. Repeat MRI of the lumbar spine
“reactive lymphadenitis.” D. Referral to interventional radiology for corticoste-
Which of the following is the most likely diagnosis? roid injection into an involved facet joint
ht

A. Lyme disease E. Referral to spine surgery for decompressive lami-


B. Adult-onset Still disease nectomy
C. Systemic lupus erythematosus
D. Acute bacterial endocarditis 43. A 72-year-old woman presents to her primary care
E. Behçet disease physician with 3 weeks of declining health because
of fatigue, cough, and dyspnea. She has a history
41. A 32-year-old woman gives birth to a baby girl at 34 of atrial ibrillation, for which she takes warfarin
weeks of gestation because of fetal distress. he new- and atenolol. She has no other signiicant medi-
born becomes cyanotic shortly after delivery. Rapid cal comorbidities. During the appointment, she
assessment identiies a heart rate of 50 beats per min- coughs up scant amounts of fresh blood. Vital signs
ute and complete heart block. With pacemaker sup- include temperature 37.2°C, heart rate 62 beats per
port, the infant’s status improves quickly, and the minute (irregular), blood pressure 162/90 mm Hg,
cyanosis resolves. he newborn develops infantile respiratory rate 25 breaths per minute, and oxygen
jaundice starting on day 5 of life, and is treated with saturation 91% on room air. Physical examination
CHAPTER 3 Rheumatology 61

shows a fatigued-appearing woman in some moder- the left knee, and arthrocentesis showed negatively
ate amount of respiratory distress, using accessory birefringent needle-shaped crystals with no growth
muscles. Lungs have difuse rales. She has mild peri- in luid culture. Current medications include pred-
orbital edema and pitting leg edema to the upper nisone 5.0 mg daily and azathioprine 150 mg daily.
shins. Urgent chest radiograph is obtained that Serum uric acid is 11.4 mg/dL.
shows patchy iniltrates in bilateral lung ields. Initial What is the most appropriate next course of action?
laboratory assessment includes the following: BUN A. Initiate allopurinol 300 mg daily and treat any
82 mg/dL, creatinine 4.1 mg/dL (baseline normal), lares of gouty arthritis with higher doses of pred-
white blood cell count 13.2 × 103 cells/mm3 with nisone “as needed.”
normal diferential, hemoglobin 9.2 g/dL, and plate- B. Check a 24-hour urine uric acid, and initiate pro-
let count 622 × 103 cells/mm3, ESR 104 mm/h, and benecid 500 mg twice daily if the patient is an
CRP 301 mg/L (normal <5). Urinalysis reveals +++ “under-excretor.”
protein and 75–100 red blood cells per high-power C. Discuss with patient’s nephrologist about alternative
ield. immunosuppressive therapies for the renal graft.
Which of the following indings would not ade- D. Increase prednisone to 15 mg daily.

t
ne
quately explain this patient’s current presentation? E. Initiate febuxostat 80 mg daily, and treat any lares
A. Antinuclear antibody 1:2560 speckled pattern, of gouty arthritis with higher doses of prednisone
anti–double-stranded DNA antibody 721 U/mL “as needed.”

e.
(normal <5), C3 complement 57 mg/dL (normal
75–175), and C4 complement 9 mg/dL (normal 45. A 48-year-old man with rheumatoid arthritis (RA) for

in
14–40) 4 years presents to his primary care physician with 5
B. Detectable antiglomerular basement membrane days of severe right knee pain and swelling. His RA has

ic
antibody been well controlled for 2 years on a combination of
C. Detectable antineutrophil cytoplasmic antibody disease-modifying antirheumatic drugs (DMARDs):
with cytoplasmic pattern and antiproteinase-3
antibody positivity
ed methotrexate and adalimumab (an injectable TNF-
α antagonist). He has no prosthetic joints and no
D. Temporal artery biopsy showing granulomatous other comorbidities. His joint examination reveals no
sm
arteritis abnormalities of the upper extremities or left leg. His
E. Urine toxicology positive for cocaine right hip and ankle move well, but the right knee is
red, warm, and swollen with a modest side efusion.
44. A 39-year-old man presents to the outpatient clinic He has a 20-degree lexion contracture of the knee and
ok

having noticed the slow accumulation of chalky pain when trying to extend it fully or lex it more than
white material in many ingers and toes (Fig. 3.5). He 45 degrees. He has no fevers and does not appear to
reports that they have occasionally become inlamed, be in a toxic condition. Laboratory analysis is notable
bo

but are not painful. He has a history of end-stage for ESR 48 mm/h and CRP 119.1 mg/L (normal <5).
renal disease from Alport syndrome, and he received Plain ilm radiography of the right knee reveals an
a cadaveric renal transplant 2 years ago (functioning efusion but no other abnormalities.
e

well with serum creatinine 1.2 mg/dL). Four years What is the most appropriate next step?
://

ago, he had an episode of acute monoarthritis of A. MRI of the right knee


B. Aspiration of the right knee for luid analysis
tp

C. Aspiration of the right knee for symptomatic


relief, followed by corticosteroid injection
D. Touch base with his rheumatologist to discuss a
ht

change in the DMARD regimen.


E. Prescribe a course of oral corticosteroids tapered
over 1 week for treatment of an RA lare.

46. A 33-year-old woman is brought to the emergency


department (ED) by ambulance. She reports fever and
progressively severe dyspnea but no cough, hemopty-
sis, or chest pain. She has a 9-month history of severe
systemic lupus erythematosus (SLE) complicated by
nonhemolytic anemia, thrombocytopenia, and class
IV lupus nephritis. Baseline laboratory tests from
3 weeks ago showed creatinine 2.8 mg/dL, hemo-
globin 10.2 g/dL, and platelet count 95 × 103 cells/
• Fig. 3.5 Hands of the patient described in Question 44. mm3. For her SLE, she is being treated aggressively
62 C HA P T E R 3 Rheumatology

with monthly intravenous cyclophosphamide 750 mg/ a generally healthy individual with limited motion
m2, prednisone 60 mg daily, and hydroxychloroquine of the spine and reduced chest excursion. Laboratory
400 mg daily. In the ED, initial vital signs show tem- analysis reveals CRP 22.8 mg/L (normal <5). Radio-
perature 37.8°C, blood pressure 90/48 mm Hg, heart graphic imaging demonstrates ankylosis of the sacro-
rate 122 beats per minute, respiratory rate 28 breaths iliac joints. A diagnosis of ankylosing spondylitis is
per minute, and oxygen saturation 88% on room air. made.
Within minutes of arrival, she exhibits increased work Which of the following will most likely to lead to
of breathing and has respiratory arrest, prompting symptomatic improvement by treating the underlying
intubation. Mechanical ventilation improves oxygen- condition?
ation saturation (94%) on 60% inhaled oxygen. Initial A. HLA-B27 testing
admission blood work shows BUN 42 mg/dL, creati- B. Prednisone 20 mg daily
nine 3.1 mg/dL, hemoglobin 7.7 g/dL, white blood C. Methotrexate 25 mg once weekly
cell count 9.2 × 103 cells/mm3 (normal diferential), D. Oxycodone 5 mg three times daily
and platelet count 88 × 103 cells/mm3. Toxicology E. Adalimumab 40 mg every other week
tests are negative for illicit drugs. Blood cultures are

t
ne
drawn and are pending. EKG demonstrates no abnor- 48. A 67-year-old woman with long-standing Sjögren syn-
malities. Chest radiograph is as shown in Fig. 3.6. drome (SjS) presents with painless irm swelling of the
Besides stabilizing the patient for her acute critical ill- left cheek of 4 months’ duration. She has had years of

e.
ness, which of the following is also indicated as the next dry eyes and dry mouth due to SjS, leading to corneal
most appropriate step in diagnosing this patient’s illness? abrasions and multiple dental caries. She also has SjS-

in
A. Chest CT angiogram with iodinated contrast associated interstitial lung disease, for which she has
B. Pulmonary function testing with corrected carbon been on chronic prednisone 5–10 mg daily and azathi-

ic
monoxide difusion capacity in the lungs (DLCO) oprine 2 mg/kg for 14 years. She has osteoporosis, for
measurement which she has been on alendronate 70 mg weekly for 10
C. Lumbar puncture
D. Cardiac MRI with gadolinium contrast
ed years. Historically, her serologic workup has included an
antinuclear antibody (ANA) 1:1280 speckled pattern,
E. Flexible bronchoscopy high-titer anti-Ro and anti-La antibodies, rheumatoid
sm
factor (RF) at 145 U/mL (normal <15), and polyclonal
47. A 42-year-old man presents to his outpatient primary hypergammaglobulinemia. She has had normal com-
care provider with back stifness. He has had these plement levels and negative anti-dsDNA, anti-Smith,
symptoms for as long as he can remember. Initially, and anti-RNP antibodies. Physical examination reveals
ok

his stifness was limited to the lumbar spine, but he asymmetric swelling of the left cheek overlying the
now reports stifness in the thoracic and cervical spine, masseter muscle. It is irm, nontender, and has poorly
accompanied by pleuritic chest discomfort. He has deined borders. She has dry mucous membranes and
bo

diiculty turning his head to look in the side mirrors lacks salivary pooling on oral inspection. She has had
when driving. His symptoms are worse in the morn- extensive dental work. here is no exposed bone, but
ing and abate over the course of the day. His medi- she still has multiple caries. She has palpable purpuric
e

cal history is notable only for episodic redness in the lesions on the legs. Laboratory analysis reveals normal
://

left eye accompanied by pain and photophobia (last comprehensive metabolic proile and complete blood
episode 12 months ago). Physical examination reveals count. ESR is 68 mm/h, CRP is 119 mg/L (normal
tp

<5), C3 complement is 66 mg/dL (normal 75–175),


and C4 complement is 3 mg/dL (normal 14–40). RF is
now normal for the irst time on record.
ht

Which of the following is the most likely diagnosis?


A. Parotid lymphoma
B. Sarcoidosis
C. Sialolithiasis
D. Suppurative bacterial sialadenitis
E. Osteonecrosis of the jaw

49. A 48-year-old woman from Guatemala presents to the


emergency department of a tertiary care center in the
United States in search of an opinion regarding her
poor health. She has been receiving hemodialysis for
12 years because of end-stage renal disease as a com-
plication of type 1 diabetes mellitus. Nine months
• Fig. 3.6 Chest radiograph of the patient described in Question 46. ago, while living in Guatemala, she experienced a
CHAPTER 3 Rheumatology 63

generalized seizure. Part of the workup included mag- in construction, and he had been managed primar-
netic resonance angiography, which revealed no intra- ily with escalating doses of oxycodone for symptoms.
cranial structural or vascular abnormalities other than Comorbidities include chronic obstructive pulmo-
moderate cerebrovascular disease. It was concluded nary disease from years of cigarette smoking, type
that her seizure was the result of an otherwise asymp- 2 diabetes mellitus, hypertension, hypercholesterol-
tomatic episode of hypoglycemia. She has not had any emia, and Hashimoto thyroiditis. Physical examina-
further seizures. However, in the past 6 months, she tion reveals “beefy hands” with boggy changes in the
has noticed thickening and hardening of the skin asso- metacarpophalangeal (MCP) and wrist joints. His
ciated with progressive immobility. Symptoms started wrists are somewhat fused, with very limited range
in her legs but have progressed to involve her arms. of motion. He is unable to extend his elbows fully
She does not describe Raynaud phenomenon. On because of 5-degree lexion contractures. Both knees
physical examination, she has tightness and hardening have modest-sized knee efusions, but he can extend
of the skin of her calves, thighs, and forearms. here is them fully. Laboratory analysis reveals a normal com-
a brawny discoloration of the skin, which is tethered prehensive metabolic proile and normal complete
down on deeper tissues such that she has lexion con- blood count with diferential. Serum uric acid is 4.0

t
ne
tractures of the ankles, knees, ingers, and elbows. Her mg/dL. Rheumatoid factor (RF), anticyclic citrulli-
face and torso are not afected, but there is a yellowish nated (CCP) antibodies, and antinuclear antibodies
discoloration of the sclera of both eyes. (ANA) are not detectable. CRP is 42.0 mg/L (normal

e.
Which of the following is the most likely diagnosis? <5). Aspiration of the right knee obtains 45 mL of
A. Scleroderma nonbloody, cloudy, yellow luid with a white blood
cell count 35,000 cells/mm3 (60% neutrophils). No

in
B. Lipodermatosclerosis
C. Scleredema diabeticorum organisms are identiied upon luid Gram stain or

ic
D. Eosinophilic fasciitis culture, and compensated polarized light microscopy
E. Nephrogenic systemic ibrosis does not reveal any birefringent crystals. Plain ilm

50. A 62-year-old male laborer presents to a new primary


ed imaging of the hands shows erosions and joint space
narrowing in multiple MCP joints and wrists.
care provider (PCP) with 3 years of persistent stifness What is the most likely diagnosis?
sm
and achiness in many joints, including the hands, A. Rheumatoid arthritis
wrists, elbows, shoulders, hips, and knees. His shoul- B. Chronic gouty arthropathy
ders and hips bother him more than anything else. At C. Primary osteoarthritis of the involved joints
the end of a long shift, his joints feel somewhat better D. Rheumatic fever
ok

than at the beginning of the day. He was told by his E. Polymyalgia rheumatic
previous PCP that he has “arthritis” related to his job
bo

Chapter 3 Answers
1. ANSWER: B. Acute pseudogout arthritis septic arthritis would indicate a high-grade bactere-
e

his patient has experienced an acute inlammatory mia. Although larger joints such as the knees, hips,
://

arthritis of the right wrist and several MCP joints, as and shoulders are more likely to be afected by sep-
evidenced by the sudden onset of redness, warmth, tic arthritis, any joint can become infected. he most
tp

and swelling, as described. Osteoarthritis (Answer likely answer is pseudogout arthritis (Answer B) based
D) rarely afects the wrist and is considered mostly on the clinical presentation of acute arthritis of the
noninlammatory. Rheumatoid arthritis (Answer C) is MCP and wrist joints. Supporting evidence includes
ht

unlikely to come on in such an abrupt manner with the chondrocalcinosis on plain ilm, more commonly
unilateral indings. Although occult injury should found in patients with hyperparathyroidism. Arthro-
always be considered, there is no history of fracture, centesis of joint luid would likely reveal elevated luid
and the plain radiograph does not support the diagno- white blood cells (>2000/mm3), negative cultures,
sis of fracture (Answer E). Septic arthritis (Answer A) and positively birefringent rhomboid-shaped crystals
is not the most likely answer for several reasons. Most consistent with calcium pyrophosphate dihydrate
important, this patient would have to have polyarticu- (Fig. 3.1).
lar septic arthritis of the wrist and several MCP joints,
a very unlikely scenario when considering her nega- 2. ANSWER: E. Temporal artery biopsy
tive blood cultures, appropriate antibiotic therapy, Giant cell arteritis (GCA) is not an uncommon
and general improvement in all other clinical param- cause for “fever of unknown origin” (FUO) in elderly
eters. Septic arthritis is often the result of hematog- patients, accounting for about 17% of FUOs in this
enous seeding of a joint from bacteremia rather than demographic group. Many of the features described in
transcutaneous inoculation of a joint. Polyarticular this case can be found in patients with unrecognized
64 C HA P T E R 3 Rheumatology

appropriate and most commonly prescribed irst-line


pharmacologic agent for the majority of patients with
RA. Methotrexate is one of many disease-modifying
antirheumatic drugs (DMARDs) available for the
treatment of RA. DMARDs reduce the signs and
symptoms of RA, and most DMARDs reduce progres-
sion of structural joint damage and preserve physical
function in patients with RA. Methotrexate is highly
teratogenic and contraindicated in patients desir-
ing pregnancy (not an issue in this patient). Risk of
hepatotoxicity and cirrhosis make methotrexate rela-
tively contraindicated in patients with excessive alco-
hol consumption. Rituximab (Answer A) is a biologic
DMARD that is approved to treat RA. However, it is
rarely prescribed as a irst-line DMARD, and eicacy

t
ne
tends to be limited to patients with “seropositive” dis-
R
ease (i.e., in those patients with detectable RF and/
or anti-CCP antibodies). he other answer options

e.
• Fig. 3.1Calcium pyrophosphate dihydrate and chondrocalcinosis of are not appropriate for this patient and unnecessar-
the hands/wrists for patient in Question 1. ily delay therapy. Duloxetine (Answer B) is sometimes

in
helpful in the management of chronic noninlamma-
GCA: anemia of chronic disease, reactive thrombo- tory musculoskeletal pain in patients with ibromyal-

ic
cytosis, elevated inlammatory indices, hypoalbu- gia syndrome. his patient’s inlammatory features are
minemia, and elevated alkaline phosphatase. Most incongruent with a diagnosis of ibromyalgia. Doxycy-
important, infectious and malignant etiologies have
been largely ruled out. here is no microbiologic evi-
ed cline (Answer D) might be used to treat Lyme arthritis
or other manifestation of Lyme disease. However, this
dence to warrant empiric levoloxacin (Answer A) or patient’s chronic polyarticular presentation is inconsis-
sm
localizing features to suggest tuberculosis infection tent with late Lyme arthritis, which usually afects only
(Answer B). Elder abuse (Answer C) should always one joint. here is no reason to repeat the RF, anti-
be considered in elderly patients with “failure to CCP antibody, or ANA tests (Answer E), and delay-
thrive” but would not explain the fevers or elevated ing treatment risks continued symptoms and onset of
ok

inlammatory indices. he elevated alkaline phospha- irreversible joint damage.


tase and hypoalbuminemia may be enough to war- Singh JA, Saag KG, Bridges Jr SL, et al. 2015 American College
rant liver biopsy, but one might expect some sort of of Rheumatology guideline for the treatment of rheumatoid
bo

abnormalities in abdominal/pelvic CT imaging or arthritis. Arthritis Rheumatol. 2016;68(1):1–26.


ultrasound studies if liver disease was the source of
fever. In this case, the abnormalities are secondary 4. ANSWER: B. Osteoarthritis of the hip
e

to GCA and will likely correct with corticosteroid Avascular necrosis (AVN) can afect almost any
://

treatment. bone, but the head of the femur is by far the most
Mourad O, Palda V, Detsky AS. A comprehensive evidence- commonly involved site. AVN can lead to cortical col-
tp

based approach to fever of unknown origin. Arch Intern lapse of bone, with resultant accelerated osteoarthritis
Med. 2003;163(5):545–551. (OA) of the adjoining joint. hus OA is the result (not
a cause) of AVN (Answer B). Pain is almost always
ht

3. ANSWER: C. Methotrexate titrated up to 25 mg the presenting complaint, but many cases can be
once weekly entirely asymptomatic and discovered incidentally. For
his patient likely has a new diagnosis of rheuma- patients with unrelenting pain, total joint arthroplasty
toid arthritis (RA). RA is the most common cause of a should be considered. he other conditions listed
chronic inlammatory polyarthritis, deined as having all increase the risk of AVN. Additional risk factors
symptoms of >6 weeks’ duration. Her history, physi- include high-dose corticosteroid therapy, radiation,
cal examination, and inlammatory indices suggest an trauma, systemic lupus erythematosus (SLE), HIV
inlammatory process. When detectable, anti-CCP infection, decompression (caisson) disease, transplant,
antibodies have high speciicity for RA (>90%), which Legg-Calvé-Perthes disease, and slipped capital femo-
is even better than RF. Even though her anti-CCP ral epiphysis (SCFE) syndrome.
antibody test is negative, RA remains the most likely
diagnosis. Up to 25% of patients with RA remain 5. ANSWER: D. Hydroxychloroquine
seronegative for the duration of their condition. Hydroxychloroquine is a generally safe and well-toler-
Weekly low-dose methotrexate (Answer C) is the most ated agent for the treatment of SLE. Rarely, however, it
CHAPTER 3 Rheumatology 65

can cause pigment to deposit in the retina, which can lead are present in this case. Hemochromatosis-related
to permanently impaired vision. hus regular ophthal- arthropathy is one of several causes of secondary osteo-
mologic screening for potential toxicity is very important. arthritis (OA), and it classically afects the MCP joints.
Chloroquine, a related agent, carries a higher risk for reti- his patient’s hand radiographs show some characteris-
nal toxicity and therefore is used less commonly. None tic changes of hemochromatosis arthropathy, including
of the other agents (Answers A–C and E) causes direct aggressive osteoarthritis changes and “hooked osteo-
visual toxicity, although each of these is immunosuppres- phytes” of the MCP joints. Although hemochromato-
sive and could increase the risk of ocular infections (e.g., sis arthropathy is largely a noninlammatory process,
zoster, toxoplasmosis). It is important to bear in mind the it is associated with chondrocalcinosis (as seen in the
late-onset complications of cyclophosphamide: steril- radiograph in Fig. 3.2) and episodes of pseudogout.
ity, lymphoma, and malignancies of the urinary tract. Involvement of MCP and wrist joints argues against
hese might not manifest until decades after a drug primary OA (Answer D), which very rarely afects these
exposure. Belimumab is a recently approved bio- joints. Primary OA tends to be limited to DIP and PIP
logic treatment for SLE. It is a monoclonal antibody joints and the base of the thumb. here are no clinical
directed against B lymphocyte stimulator. Belimumab features of inlammation to suggest rheumatoid arthri-

t
ne
has shown evidence of beneit in mostly cutaneous, tis (Answer A). Metacarpal fracture (Answer B) is not
hematologic, and articular manifestations of SLE. present in this radiograph. Gaucher disease (Answer
E) is associated with cirrhosis, osteoporosis, and joint

e.
6. ANSWER: C. Hemochromatosis pain. However, this condition mostly involves large
Hemochromatosis (Fig. 3.2) has many clinical joints (hips and knees) and is associated with avascular

in
manifestations related to iron accumulation in target necrosis, not accelerated osteoarthritis of the hands.
organs: cirrhotic liver disease, congestive heart failure, Sahinbegovic E, Dallos T, Aigner E, et  al. Musculoskeletal
disease burden of hereditary hemochromatosis. Arthritis

ic
hypogonadism, diabetes mellitus, bronzing of the skin,
and accelerated degenerative arthritis. Many of these Rheum. 2010;62(12):3792–3798.
ed
R R
sm
ok
e bo
://
tp
ht

• Fig. 3.2 Hemochromatosis arthropathy (right hand/wrist) for patient in Question 6.

7. ANSWER: B. Provide a prescription for physical infectious, or neoplastic origin, such as epidural or
therapy and discuss an “opiate medication contract.” paraspinous abscess, compressive mass, or cauda
his patient describes lumbago—musculoskeletal equina syndrome. his patient should irst engage
low back pain without evidence of any “red lags”: in physical therapy to improve core muscle strength,
fever, occult weight loss, numbness in the perineum, conditioning, and range of motion. Weight loss is
new-onset impotence, bowel or bladder retention, his- advisable. his patient also demonstrates misuse of
tory of malignancy, and recent intravenous drug abuse. oxycodone. hus discussions of an opiate medication
he presence of these indings would prompt consid- contract are indicated. It is unlikely that reassurance
eration of further imaging to assess for inlammatory, alone (Answer A) will satisfy this patient, and it will
66 C HA P T E R 3 Rheumatology

be unlikely to change his current pattern of opiate use. diseases such as systemic lupus erythematosus do not
Switching oxycodone to the more potent hydromor- experience chronic inlammatory monoarthritis of this
phone will likely only compound what appears to be nature. Although rheumatoid arthritis can occasion-
opiate misuse. MRI of the lumbar spine (Answer D) ally present as a chronic inlammatory monoarthritis,
might be appropriate if he reported any “red lags” or this presentation would be more typical of a spondy-
failed a concerted efort at physical therapy. It is inap- loarthritis condition: psoriatic arthritis, ankylosing
propriate to discharge this patient from your practice spondylitis, reactive arthritis, or the arthropathy of
(Answer E) until you can demonstrate that the patient inlammatory bowel disease (IBD arthropathy).
intentionally misconstrues facts or misrepresents him- Berende A, ter Hofstede HJ, Vos FJ, et al. Randomized trial of
self to obtain opiate-based medications, or if there is longer-term therapy for symptoms attributed to Lyme dis-
redirection of his prescription medications to other ease. N Engl J Med. 2016;374(13):1209–1220.
individuals in exchange for money or favors. An opi- Melia MT, Auwaerter PG. Time for a diferent approach to
ate medication contract would clearly indicate appro- Lyme disease and long-term symptoms. N Engl J Med.
2016;374(13):1277–1278.
priate use and misuse of opiates, such that violation
Wormser GP, Dattwyler RJ, Shapiro ED, et  al. he clinical
of the contract could provide supporting evidence to

t
assessment, treatment, and prevention of Lyme disease,

ne
discharge the patient from your practice. Many states human granulocytic anaplasmosis, and babesiosis: clinical
mandate that prescribers review an opiate prescrip- practice guidelines by the Infectious Diseases Society of
tion database prior to dispensing an opiate prescrip- America. Clin Infect Dis. 2006;43(9):1089–1134.

e.
tion to a patient. he purpose of these databases is to
track patients’ opiate prescription, prescribing physi- 9. ANSWER: B. Viral arthritis

in
cians, and dispensing pharmacies in an efort to detect his patient likely has acute viral polyarthri-
patients who “doctor hop” to receive multiple opiate tis (Answer B) due to parvovirus B19. In children,

ic
prescriptions from multiple providers. this virus is the cause of Fifth disease, characterized
Dowell D, Haegerich TM, Chou R. CDC guideline for pre- by fever, rash, and occasional joint pains. In adults,
scribing opioids for chronic pain—United States, 2016.
MMWR Recomm Rep. 2016;65(1):1–49.
ed however, inlammatory joint disease may be the only
manifestation. he condition is too recent in onset
to be diagnosed as rheumatoid arthritis (Answer E).
sm
8. ANSWER: A. Serologic testing for Lyme disease he arthritis of Lyme disease is most often a chronic
he patient presents with a chronic inlammatory inlammatory monoarthritis, and therefore Answer D
monoarthritis. While unlikely to be suppurative septic is not correct. Everything about his presentation sug-
arthritis of 8 weeks’ duration (e.g., from Staphylococcus gests an inlammatory process, and thus ibromyalgia
ok

aureus), chronic infectious processes must be consid- syndrome (Answer C) is incorrect. Septic polyarthritis
ered. In patients living in certain parts of the United (Answer A) is unlikely because patients are often much
States, such as New England and parts of the upper sicker than this patient. Septic joints are seeded hema-
bo

Midwest and Paciic Northwest, infection by the tick- togenously, and thus septic polyarthritis indicates a
borne spirochete Borrelia burgdorferi causes Lyme dis- very high level of bacteremia. Another notable feature
ease. One late-onset manifestation of Lyme disease is of parvoviral arthritis is the impairment of red blood
e

joint infection, occurring weeks or typically months cell (RBC) production (hence the inappropriately low
://

after exposure to the culprit tick. Lyme arthritis is reticulocyte count). In patients with sickle cell disease
almost invariably a chronic inlammatory monoarthri- or other states of high RBC turnover, parvoviral infec-
tp

tis, most often of the knee. Patients do not necessar- tion can cause an aplastic crisis. Diagnosis of parvovi-
ily recall the initial tick bite or any classic “bull’s-eye ral infection is conirmed by serologic or PCR-based
rash” of early Lyme disease. Lyme arthritis is not to testing; an elevated parvovirus IgM or parvovirus PCR
ht

be confused with “chronic Lyme disease,” a nebulous indicates recent infection. Treatment of parvoviral
condition that is likely not related to active Lyme dis- arthritis is supportive: pain relief with nonsteroidal
ease and encompasses a constellation of somatic com- antiinlammatory drugs (NSAIDs) or low-dose sys-
plaints that linger after standard treatment for Lyme temic corticosteroids until the viral process abates.
disease: chronic pain, fatigue, impaired cognition, and Young NS, Brown KE. Parvovirus B19. N Engl J Med.
other manifestations. Long-term antibiotic therapy for 2004;350(6):586–597.
“chronic Lyme disease” is highly controversial and has
not been proven to result in improved symptoms in 10. ANSWER: B. Serum and urine protein electropho-
multiple clinical trials. Still, some patients and advo- resis
cacy groups insist that chronic antibiotics result in a his patient presents with nonspeciic symptoms,
positive response. he other answer options could be anemia, and an elevated ESR. Although these symp-
appropriate next steps in the event that Lyme arthritis toms might represent polymyalgia rheumatica (PMR),
was ruled out by negative serologic testing. Note that the discrepantly low CRP warrants consideration
most patients with ANA-associated connective tissue of other processes. CRP is a nonspeciic indicator of
CHAPTER 3 Rheumatology 67

inlammation that can be elevated because of inlam- morbidity. Switching antibiotics (Answer E) might
mation, infection, and some malignancies. In this lead to clinical improvement if this was simply bacte-
case, factors other than inlammation have led to an rial pneumonia resistant to the initial choice of antibi-
increased ESR; an immunoglobulin paraproteinemia otics. If incorrect, however, this would delay diagnosis
increases the ESR without afecting the CRP because and treatment. PET-CT scan (Answer D) is not yet
positively charged immunoglobulin promotes electro- indicated, because malignancy has not been shown
static adhesion of negatively charged red blood cells (and indeed infection is more likely and of greater
such that they settle out of suspension faster. In this immediate concern). Finally, RA can cause necrotic
case, the patient has multiple myeloma, which would pulmonary nodules, but empiric treatment for rheu-
be detected by serum or urine protein electrophoresis matoid lung (Answer A) is inappropriate until infec-
(SPEP/UPEP). here is nothing to suggest systemic tious processes have been ruled out.
lupus erythematosus or other connective tissue disorder,
so an antinuclear antibody is unnecessary (Answer A). 13. ANSWER: D. Scaly erythematous plaques on the
Temporal artery biopsy (Answer C) is used to diagnose extensor surfaces of the arms and legs
giant cell arteritis (GCA). Sometimes GCA can present his answer describes a patient with psoriasis,

t
ne
as fever, malaise, and weight loss of unknown origin. which can also involve the scalp, external auditory
CT scanning (Answer D) can reveal occult abscess or canal, supragluteal cleft, and umbilicus. Nail pitting
solid organ malignancy. Prior to temporal artery biopsy and other nail abnormalities can also be present. In

e.
or CT scanning, however, the discrepancy in ESR and patients with psoriasis, unexplained inlammatory
CRP irst should be assessed with the much less inva- joint disease often indicates the presence of psoriatic

in
sive and more cost-efective SPEP/UPEP. Finally, it is arthritis. his can take on several forms: inlamma-
inappropriate to treat the patient empirically with low- tory monoarthritis or oligoarthritis typically of the

ic
dose corticosteroids (Answer E) for something such lower extremities (as in this case), dactylitis of ingers
as polymyalgia rheumatic (PMR) until one has ruled or toes (as in this case), a rheumatoid-pattern polyar-
out other explanations for the patient’s complaints and
laboratory abnormalities. PMR should always be con-
ed thritis, inlammatory spondyloarthritis of the spine,
accelerated osteoarthritis of the distal interphalan-
sidered a diagnosis of exclusion. geal (DIP) joints, and inlammatory enthesitis of the
sm
tendon–bone interface. In this patient, septic arthritis
11. ANSWER: C. Pulmonary hypertension and crystalline arthritides are much less likely based
he patient has pulmonary hypertension (pHTN) on arthrocentesis results, which show inlammatory
as a complication of scleroderma. Features associated luid that lacks crystals and microorganisms. Serum
ok

with pHTN include CREST-variant scleroderma and uric acid (Answer B), serum ACE level (Answer
anticentromere antibodies. his condition is suggested C), and radiographic evidence of chondrocalcinosis
by abnormalities in echocardiography and pulmonary (Answer E) are associated with gout, sarcoid arthritis,
bo

function testing (isolated low carbon monoxide difu- and pseudogout, respectively, but none of these are
sion capacity in the lungs [DLCO]). It is conirmed diagnostic for their respective conditions. Anti-Smith
by direct measurement of pulmonary artery pressures antibodies are a speciic autoantibody sometimes
e

by right heart catheterization. Treatment options found in patients with systemic lupus erythematosus
://

include endothelin antagonist agents (e.g., ambrisen- (SLE). However, SLE is unlikely in this patient clini-
tan), phosphodiesterase inhibitors (e.g., tadalail), and cal based on the negative ANA and monoarticular
tp

prostacyclin analogues. Scleroderma cardiomyopa- joint involvement.


thy is a ibrotic complication of scleroderma, associ-
ated with anti-SCL70 antibodies and difuse ibrotic 14. ANSWER: A. Tilt-table test
ht

disease. he patient demonstrates evidence of a joint


hypermobility syndrome (HMS). Several rare genetic
12. ANSWER: B. Bronchoscopy with lavage for micro- disorders can lead to HMS (e.g., Marfan syndrome
scopic organisms and genetically deinable Ehlers-Danlos syndrome
his patient is immune suppressed secondary to his [EDS]). However, this patient demonstrates none of
RA therapy. Based on his deteriorating clinical status, the characteristic nonarticular features of these rare
further investigation is warranted to assess for atypical syndromes, and thus she most likely has benign joint
infectious source. his patient could have pulmonary HMS. Formerly termed type 3 Ehlers-Danlos syndrome,
abscess, fungal pneumonia, or tuberculoma. Bron- a genetic basis for benign joint HMS is not as well
choscopy with lavage (Answer B) is the least invasive established as in the other forms of EDS. hus benign
test that will still yield clinically important informa- joint HMS remains primarily a clinical diagnosis,
tion. Lung biopsy (Answer C) might be necessary if and no further testing for genetics or other causes of
bronchoscopy fails to yield a diagnosis, but it is more joint pain is necessary (Answers B–E). Benign joint
invasive than bronchoscopy and carries greater risk of HMS is associated with dysautonomia features, such
68 C HA P T E R 3 Rheumatology

as postural orthostatic tachycardia syndrome (POTS).


he presence of this condition would be supported
by a positive tilt-table test (Answer E). Treatment of
joint complaints in patients with benign joint HMS
is challenging, but it should include primarily physi-
cal therapy to improve muscle tone and itness while
limiting extreme range-of-motion exercises. Immuno-
suppressive and antiinlammatory therapies are of no
value. Narcotics are counterproductive and can lead
to tolerance, dependency, and addiction. Psychologi-
cal support is often indicated for patients with refrac-
tory symptoms.

15. ANSWER: E. Skeletal muscle biopsy


he patient has dermatomyositis (DM; Fig. 3.3).

t
ne
his patient’s painless proximal muscle weakness • Fig. 3.3 Dermatomyositis rash.
and characteristic rash are classic for DM. he CPK,
CK-MB, AST, and ALT are all of skeletal muscle ori-

e.
gin, so testing other targets is unlikely to yield a diag- 16. ANSWER: A. MEFV genetic testing
nosis (Answers A–D). She also likely has interstitial he patient has familial Mediterranean fever (FMF),

in
lung disease (ILD), which is commonly associated one of several hereditary periodic fever syndromes.
with DM. DM is one of several idiopathic inlam- FMF most often results from a homozygous mutation

ic
matory myopathies (IIMs). Others include polymyo- of the MEFV gene. his gene encodes the pyrin pro-
sitis (PM), malignancy-associated myositis, inclusion tein, which is involved with leukocyte function. FMF
body myositis (IBM), and juvenile dermatomyositis
(J-DM). In patients with some forms of IIM, autoan-
ed is an autosomal recessive disorder, so a family history
may or may not be present. Clinical features of FMF
tibody testing can reveal an elevated antinuclear anti- include recurrent attacks characterized by fever and
sm
body (ANA) titer. A subset of patients with DM will abdominal pain. Accompanying features can include
have more speciic myositis-associated autoantibod- leukocytosis, thrombocytosis, rash, and inlammatory
ies such as anti-Jo-1, and these patients have higher joint pain (hence this patient’s questionable history
rates of DM-associated ILD. Patients with J-DM can of “gout”). Inlammatory markers are elevated dur-
ok

experience bowel vasculitis and subcutaneous calcii- ing attacks but relatively normal when asymptomatic.
cations (calcinosis cutis). he diagnosis of the various An important complication of FMF is secondary
IIMs is conirmed by skeletal muscle biopsy (Answer amyloidosis, which this patient may have, based on a
bo

E). Electron microscopy is advisable if IBM is being history of chronic untreated inlammation and unex-
considered to visualize classic vacuolar changes in plained proteinuria. Secondary renal amyloidosis can
diseased muscle. Unlike the other IIMs, however, be prevented (and to some degree reversed) by treat-
e

IBM usually includes chronic distal muscle weakness ment with daily colchicine. he patient does not have
://

and atrophy. Because muscle involvement in any of features of celiac disease or Crohn disease; hence,
the IIMs can be patchy, EMG or MRI is often used endoscopy and colonoscopy are not relevant (Answers
tp

to identify a muscle suitable for biopsy. here is an B and D). He is not experiencing recurrent mesen-
association between IIMs and malignancies, typically teric ischemia from vasculitis, and there is nothing
of the lung, GI tract, and female reproductive tract. speciically suggesting systemic vasculitis, so workup
ht

Although all patients should have age-appropriate for these conditions (Answers C and E) delays proper
cancer screening, it is not be unreasonable to perform diagnosis.
additional imaging to assess for occult cancer (espe- Ozen S, Bilginer Y. A clinical guide to autoinlammatory dis-
cially in this patient): CT scan of the chest, abdomen, eases: familial Mediterranean fever and next-of-kin. Nat Rev
and pelvis, as well as consideration of transvaginal Rheumatol. 2014;10(3):135–147.
pelvic ultrasound. Treatment of most forms of IIM
initially involves corticosteroids, often in high doses 17. ANSWER: A. Patellofemoral syndrome
(1 mg/kg) with a long and slow taper. Methotrex- his case describes a classical presentation for patel-
ate is the most commonly used nonsteroid-based lofemoral syndrome (PFS), a common mechanical
immunosuppressive agent. Unfortunately, there are disorder of young, active adults. One would expect
no efective therapies for IBM, which carries a dismal the pain of iliotibial (IT) band syndrome to be lat-
prognosis. eral. Physical examination testing does not suggest the
Dalakas MC. Inlammatory muscle diseases. N Engl J Med other options (Answers C–E). McMurray test assesses
2015; 372(18):1734–1747. for tear of the medial meniscus. Anterior drawer sign
CHAPTER 3 Rheumatology 69

testing assesses for tear of the ACL. Baker cyst should be an indication for antihyperuricemic therapy in the
be suspected in patients with posterior fullness and future, it is inappropriate to initiate it at the time
pain with knee lexion. of an acute attack (at least as monotherapy) because
sudden drops in uric acid often exacerbate gouty
18. ANSWER: C. Anti–double-stranded DNA anti- inlammation. hus Answers A and B are not the next
body 522 U/mL (normal <5), C3 complement 52 most appropriate course of action at this time. hree
mg/dL (normal 75–175), C4 complement 8 mg/dL classes of medications are traditionally used to treat
(normal 14–40) the inlammation of acute gouty arthritis: colchicine,
he patient has systemic lupus erythematosus (SLE) nonsteroidal antiinlammatory drugs (NSAIDs), and
complicated by lupus nephritis (LN), with associated corticosteroids. he indicated course of colchicine in
symptoms and indings of renal failure (peripheral edema Answer C is inappropriate for a patient with marginal
and congestive heart failure). his is supported by the renal function. Very low doses of colchicine (0.3 mg
presence of aggressive Raynaud phenomenon, leukope- every other day) might be tolerated as a prophylactic
nia (and lymphopenia), thrombocytopenia, hematuria, dose against attacks, but this is unlikely to be very
proteinuria, and elevated ANA. SLE has myriad clini- efective in an attack of acute gout such as this one.

t
ne
cal features, and LN is an important one to recognize. Answer D is inappropriate because NSAIDS are
he serologic workup of SLE includes more than just the highly nephrotoxic in patients with marginal renal
ANA, which is itself a nonspeciic test. In addition to function. Answer E is the correct answer because

e.
hematuria and proteinuria, patients with LN often (but corticosteroids provide rapid relief of inlammatory
not always) will have elevated anti-dsDNA antibodies pain in patients with acute gouty arthritis. Appropri-

in
and hypocomplementemia. Anti-Smith antibody is a ate routes of administration include intramuscular,
test speciic for SLE. Anti-Ro and anti-La antibodies are intraarticular, intravenous, and oral. Recent studies

ic
associated with SLE and Sjögren syndrome. Anti-RNP have shown that a short course of the interleukin-1
antibody is associated with an overlap condition some- receptor antagonist (IL-Ra) anakinra is highly efec-
times termed mixed connective tissue disease (MCTD), in
which there are features of SLE, scleroderma, dermato-
ed tive in treating acute gouty arthritis. Factors that
limit its use include a lack of familiarity, insurance
myositis, Sjögren syndrome, and/or rheumatoid arthri- barriers, and concerns of immunosuppression.
sm
tis. he patient does not have other features to suggest an
ANCA-associated vasculitis. Further, Answer A indicates 20. ANSWER: C. Acute calciic tendonitis
only a nonspeciic p-ANCA pattern and lacks the auto- he patient describes rapid onset of rotator cuf
antibodies associated with ANCA vasculitis (anti-MPO pain in the absence of antecedent symptoms or injury.
ok

and anti-PR3). Answer B would be seen in a patient with Acute calciic tendonitis is a common condition in
acute promyelocytic leukemia, which would not be con- which basic calcium phosphate crystals deposited along
sistent with this patient’s presentation or complete blood a tendon trigger an immune response, with resultant
bo

count. he patient does not have features of dermatomy- acute inlammation and pain. It is often unheralded by
ositis (Answer D), which largely spares the kidney. Long- any speciic antecedent injury of overuse. he physical
standing ankylosing spondylitis (Answer E) is associated examination does not suggest biceps tendonitis (Answer
e

with IgA nephropathy, but the remainder of her clinical A), because more than just forward lexion of the shoul-
://

presentation does not suggest this diagnosis. Diagnosis der is afected. Septic arthritis (Answer B) would likely
of LN is conirmed by kidney biopsy. Treatment of lupus cause a patient to appear in a toxic condition, with
tp

nephritis includes high-dose corticosteroids (1 mg/kg severe pain. Further, the examination would be lim-
slowly tapered over months) plus an immunosuppres- ited on active and passive range of motion. Likewise,
sive or cytotoxic agent such as mycophenolate mofetil glenohumeral arthritis (Answer D) would limit shoul-
ht

(MMF) or intravenous cyclophosphamide. Recent stud- der motion actively and passively. Finally, although the
ies indicate that MMF is as efective as cyclophosphamide examination suggests involvement of the supraspinatus
for the treatment of LN. MMF has a more favorable side tendon, complete tear is unlikely to explain acute-onset
efect proile than cyclophosphamide, which carries a risk symptoms in the absence of antecedent injury. If there
of sterility (especially in women aged >30). is clinical uncertainty about the presence of a complete
Hanh BH, McMahon MA, Wilkinson A, et al. American Col- rotator cuf tear, the clinician can inject 3 mL of 2%
lege of Rheumatology guidelines for screening, treatment, lidocaine into the subacromial space. If this completely
and management of lupus nephritis. Arthritis Care Res. corrects the shoulder pain and functional deicit, then
2012;64(6):797–808. complete rotator cuf tear is unlikely, as is septic arthritis
or signiicant glenohumeral arthritis.
19. ANSWER: E. Treat with intramuscular methyl-
prednisolone 80 mg ×1. 21. ANSWER: C. Prescribe prednisone 1 mg/kg.
he patient is experiencing an attack of acute Instruct the patient to start it immediately and
gouty arthritis. Although the presence of tophi would to call with any new visual disturbances. Arrange
70 C HA P T E R 3 Rheumatology

for an expedited temporal artery biopsy irst thing spine x-ray imaging, with lexion and extension views
Monday morning. (Answer D). MRI and/or CT scans of the cervical
he patient describes symptoms consistent with spine might also be indicated. Testing for RF and anti-
giant cell arteritis (GCA). A dreaded complication CCP antibodies (Answer A) is part of the diagnostic
of GCA is blindness, which results from ischemic workup of inlammatory arthritis and unnecessary in
optic neuropathy from vasculitis involving branches this case. ESR and CRP can correlate with RA disease
of the ophthalmic and retinal arteries. If blindness activity, but these do little to clarify the patient’s clini-
develops, it is often irreversible. hus empiric treat- cal presentation. It is unlikely that the patient would
ment with high-dose corticosteroids is indicated in tolerate exercise stress testing (Answer C). RA gener-
patients with possible GCA while arrangements are ally spares the lumbar spine, and lumbar spine disease
made for expedited temporal artery (TA) biopsy to would not explain the patient’s upper-extremity symp-
conirm the diagnosis. Only Answer C addresses toms. Hence, lumbar spine MRI (Answer E) is not
these issues fully. All other answers risk irreversible indicated.
blindness. Of note, once empiric corticosteroids
are started for suspected GCA, a TA biopsy should 24. ANSWER: B. Hepatitis C antibody

t
ne
be performed within 7–10 days if possible to avoid In addition to alcoholic cirrhosis, the patient has
a “false-negative” biopsy. Regardless, 10%–15% chronic hepatitis C infection complicated by cryo-
of patients with GCA will still have a negative TA globulinemic vasculitis. As in this case, cryoglobulin-

e.
biopsy. In these cases, one should consider alterna- emic vasculitis can cause palpable purpura (Fig. 3.4),
tive diagnoses or even contralateral TA biopsy, which glomerulonephritis, and mononeuritis multiplex (the

in
will pick up another 5%–10% of cases of GCA. foot drop). Bowel vasculitis, pulmonary hemorrhage,
Finally, it is important to remember that GCA rarely and cerebral vasculitis are rare complications. Immuno-

ic
if ever afects individuals younger than age 50, and logic disturbances in patients with hepatitis C include
other explanations for headache and scalp tenderness detectable rheumatoid factor (RF) and disproportion-
should be sought in these patients.
Weyand CM, Goronzy JJ. Giant-cell arteritis and polymyalgia
ed ately low C4 complement (compared with normal C3
complement). he patient does not describe or have
rheumatica. N Engl J Med. 2014;371(17):50–57. symptoms of rheumatoid arthritis (RA); hence, testing
sm
for anti-CCP antibodies (Answer E) is unnecessary at
22. ANSWER: A. Reassurance and physical therapy, this time. he remaining tests (Answers A, D, and E)
focusing on posture, core muscle tone, and hip do not address the clinical scenario and are notoriously
range of motion nonspeciic, with many “false-positive” tests.
ok

he patient has a greater trochanteric pain syn-


drome, based on clinical evaluation. Formerly, the
term trochanteric bursitis was used to describe this
bo

condition, but there is greater evidence that the term


is inaccurate. In patients with greater trochanteric
pain syndrome, there is often evidence of tendi-
e

nopathy of the gluteus medius and gluteus minimus


://

tendons rather than overt trochanteric bursitis. Fur-


ther diagnostic imaging studies (Answers B, D, and
tp

E) are not necessary upon initial evaluation of this


condition. Physical therapy is appropriate (Answer
A), and corticosteroid injection can be considered.
ht

here is little role for systemic corticosteroid treat-


ment (Answer C).
Blankenbaker DG, Ullrick SR, Davis KW, et  al. Correlation
of MRI indings with clinical indings of trochanteric pain • Fig. 3.4 Lower-extremity petechiae.
syndrome. Skeletal Radiol. 2008;37(10):903–909.

23. ANSWER: D. Lateral cervical spine x-ray imaging, 25. ANSWER: E. Referral to orthopedics for consider-
with lexion and extension views ation of right total knee arthroplasty
his patient is demonstrating signs and symptoms his patient has end-stage osteoarthritis (OA) of
of cervical myelopathy, likely as a complication of RA the right knee. He would beneit from consideration
afecting the cervical spine. Atlantoaxial instability is a of right total knee arthroplasty (TKA). He has already
rare but serious complication of RA, most often occur- attempted conservative measures, including physical
ring in patients with long-standing deforming disease. therapy and corticosteroid injections, without much
he best initial assessment for this is lateral cervical beneit. hus advising him to do this again (Answer
CHAPTER 3 Rheumatology 71

A) will likely be of little beneit. Narcotic-based ther- hip range of motion, but it is not associated with par-
apies (Answer B) might be useful for a short-term esthesias or numbness.
bridge to get him to orthopedics, but not as a stand-
alone treatment. here is little role for MRI of the 28. ANSWER: D. Recommend combination of physi-
knee or arthroscopic debridement in patients with cal therapy, sleep routine, weight loss, and psychi-
advanced knee OA (Answers C and D); these choices atric counseling.
do not change the ultimate need for TKA. Decision he patient has ibromyalgia syndrome (FMS), a
making for timing of TKA is done on a case-by-case noninlammatory, nonstructural chronic dysfunc-
basis. Pain and functional limitation are the driving tional pain syndrome of muscles, bones, and joints.
forces behind optimal timing of referral to ortho- Her history, examination, and laboratory results are
pedics. Physical therapy and weight loss remain an not consistent with an inlammatory joint disorder.
important part of preoperative management. Patients With more than 20 years of symptoms, one would
undergoing TKA should experience severe enough expect signiicant abnormalities if she had an inlam-
pain that they will derive beneit from joint replace- matory, infectious, or neuromuscular process present.
ment, but they should not be so debilitated that they hus EMG/NCS and serologic testing for connec-

t
ne
might miss a window within which optimal recov- tive tissue diseases (Answers A and B) are inappro-
ery is possible. Indeed, patients who are wheelchair priate. Likewise, tick-borne illnesses such as Lyme
bound or similarly disabled will not typically derive disease (Answer C) do not explain her constellation

e.
the same beneit as patients able to maintain muscle of symptoms. he history and examination describe
mass through active participation in perioperative hyperesthesia with allodynia (tenderness to normally

in
physical therapy. nonpainful stimulus), common features in patients
with FMS. For the most part, narcotic-based therapies

ic
26. ANSWER: C. Chondrocalcinosis of the wrist are inappropriate with patients with FMS. hey can
Carpal tunnel syndrome (CTS) is most commonly lead to tolerance, dependency, dose escalation, and
the result of an overuse syndrome of the ingers.
Flexor tendons swell slightly in response to overuse.
ed sometimes frank addiction. Rather, patients should
be encouraged to address lifestyle issues that are often
his compresses the median nerve in the carpal tun- associated with FMS: physical inactivity, obesity, sleep
sm
nel, resulting in a median nerve neuropathy. Severe disturbances, and underlying depression and anxiety
cases can lead to thenar muscle atrophy. Electromyo- (Answer D). It would be appropriate to consider phar-
gram with nerve conduction studies can be used to macotherapy for FMS if a patient is unable to address
determine the severity of median nerve deicit. Other these issues.
ok

conditions that cause swelling of wrist lexor tendons Clauw DJ. Fibromyalgia: a clinical review. JAMA. 2014;
(or the median nerve itself ) can increase the risk of 311(15):1547–1555.
carpal tunnel syndrome: acromegaly, hypothyroidism,
bo

scleroderma, rheumatoid arthritis, other inlammatory 29. ANSWER: E. No additional workup necessary
arthritides (e.g., psoriatic arthritis), and pregnancy. his patient describes Raynaud phenomenon: cold-
Chondrocalcinosis commonly afects the wrists in induced vasospasm of the digits with triphasic color
e

older individuals, but it is not associated with CTS. It changes. Most patients, including this one, have pri-
://

can lead to (1) acute lares of pseudogout arthritis, (2) mary Raynaud phenomenon, indicating an absence
chronic inlammatory “pseudo-rheumatoid” arthritis, of any underlying connective tissue disorder (CTD).
tp

or (3) accelerated degenerative “pseudo-osteoarthritis.” Patients have benign symptoms and indings, and no
further workup is indicated (Answer E). Secondary
27. ANSWER: C. Entrapment neuropathy of the left Raynaud phenomenon is the term used to describe
ht

lateral femoral cutaneous nerve Raynaud phenomenon occurring secondary to another


his patient is experiencing meralgia paresthetica— condition. hese include vascular trauma or malfor-
an entrapment neuropathy of the lateral femoral cuta- mation (Answer D); microangiopathic injury from
neous nerve. Risk factors include abdominal obesity antiphospholipid syndrome (Answer B), chemother-
and tight-itting belts/corsets. Diabetic amyotrophy apy, or other cause; and the various CTDs: systemic
(Answer A) is a lumbosacral plexopathy that occurs lupus erythematosus (Answer A), scleroderma (Answer
in patients with diabetes even with good blood sugar B), Sjögren syndrome, rheumatoid arthritis, dermato-
control. It causes pain, weakness, and muscle atrophy myositis, and mixed connective tissue disorders. In this
of the anterior thigh. Avascular necrosis (AVN) and case, the patient describes no symptoms of these other
degenerative arthritis (osteoarthritis) of the hip joint disorders, and the examination does not suggest them.
usually localizes as pain in the groin or deep buttock. hus an exhaustive serologic and imaging workup for
Hip range of motion is often impaired. Trochanteric benign primary Raynaud syndrome is unnecessary
bursitis (Answer D) causes pain overlying the trochan- and not cost efective. Factors that can exacerbate pri-
teric bursa at the lateral hip. It does not impair passive mary or secondary Raynaud attacks include cafeine
72 C HA P T E R 3 Rheumatology

ingestion, amphetamine use, anxiety, over-the-counter because of increased pressure within the inlamed
cold medications containing pseudoephedrine, and bursa. here is nothing to suggest active inlamma-
cocaine abuse. tion of the knee from gout or sepsis (Answers B or
Wigley FM, Flavahan NA. Raynaud’s phenomenon. N Engl J C). Typically, patients with these conditions will have
Med. 2016; 375(6):556–565. buildup of inlammatory luid, and the joint will be
partly lexed to alleviate buildup of intraarticular pres-
30. ANSWER: B. Captopril 25 mg orally three times sure. he examination also does not suggest meniscal
daily, with dose escalation to control blood pressure injury (reproduced with a positive McMurray test) or
his patient is experiencing a scleroderma renal quadriceps tendon rupture (which prevents extension
crisis (SRC). Evidence for this is the presence of of the knee against resistance).
scleroderma, hypertension, renal dysfunction, and
evidence of microangiopathic hemolytic anemia 33. ANSWER: A. Staphylococcus aureus
(MAHA). Other causes of MAHA include antiphos- Reactive arthritis typically presents as abrupt-
pholipid antibody syndrome, disseminated intravas- onset pain and swelling in multiple small joints. It is
cular coagulation (DIC), hemolytic-uremic syndrome associated with various infectious agents, including

t
ne
(HUS), thrombotic thrombocytopenic purpura Streptococcus pyogenes, Clostridium diicile, Chlamydia
(TTP), hypertensive emergency, hemolytic anemia/ trachomatis, Yersinia pestis, Salmonella species, Cam-
elevated liver studies/low platelets (HELLP) syndrome pylobacter species, and Shigella species. Staphylococcus

e.
of pregnancy, certain malignancies, and graft-versus- aureus is a common infectious agent, but it is not asso-
host disease. SCR afects only a subset of patients ciated with reactive arthritis.

in
with scleroderma, and highest-risk patients include
those with rapidly progressive skin disease and anti- 34. ANSWER: B. Plantar fasciitis

ic
RNA polymerase III antibody (a laboratory test that is his patient has a clinical presentation consistent
commercially available). SRC represents a hyper-renin with plantar fasciitis. his condition causes focal pain
state, leading to ischemic injury to the nephron. Even
modest degrees of hypertension (as in this case) can
ed at the origin of the plantar fascia as it comes of of
the plantar calcaneus. Obesity is a risk factor. Stress
lead to rapid renal demise. hus despite rising cre- fracture (Answer A) could still be present even with
sm
atinine levels, treatment with angiotensin-converting normal radiographs, but this patient’s examination
enzyme inhibitors (ACEi) must be rapidly initiated, does not suggest involvement of the ifth metatarsal
with the goal of normalizing blood pressure. Prior to bone, and bilateral simultaneous stress fracture would
the recognition of ACEi therapy for SRC, this con- be unusual. Acute gouty arthritis (Answer C) would
ok

dition carried a grim prognosis, accounting for a be associated with inlammatory indings of erythema,
sizable fraction of mortality in patients with sclero- warmth, and swelling. Charcot arthropathy (Answer
derma. Other antihypertensive agents (Answers C D) is a complication of sensory deprivation of the
bo

and D) might actually mask the renal injury and are extremity, such as occurs in diabetic neuropathy or
inappropriate. Plasma exchange (Answer A) would be tabes dorsalis (syphilis). he anterior taloibular liga-
indicated for patients with TTP, but it is inefective in ment (ATFL) is commonly injured in forced inversion
e

SRC. Corticosteroids (Answer E) are used for treat- injury of the ankle. Pain localizes laterally and not to
://

ment of glomerulonephritis. hey are counterproduc- the plantar foot. Treatment of plantar fasciitis is irst
tive and potentially catastrophic in patients with SRC. conservative, with stretching, weight loss, and nonste-
tp

roidal antiinlammatory drugs. If these fail, one could


31. ANSWER: D. Levoloxacin consider corticosteroid injection.
he patient has experienced quinolone-associated
ht

rupture of the Achilles tendon. his is a rare but seri- 35. ANSWER: C. Hepatitis A vaccine, injectable typhoid
ous adverse reaction to luoroquinolone-class antibi- vaccine, and a letter of exemption from yellow fever
otics. Classic features include altered gait because of vaccine
impaired plantar lexion from Achilles tendon rupture. his patient is immune suppressed because of her
he Achilles tendon is intact with a normal hompson disease-modifying antirheumatic drug (DMARD)
test; squeezing the calf should normally cause dorsi- therapy. he yellow fever, MMR, and oral typhoid
lexion at the ankle. he other antibiotics are not asso- vaccines are all live vaccines and are therefore contra-
ciated with this complication. indicated in patients on biologic DMARDs. Admin-
istering the yellow fever vaccine in particular can be
32. ANSWER: A. Gouty bursitis of the right prepatellar dangerous because life-threatening visceral infection
bursa due to the vaccine has been reported in this popula-
he physical examination suggests an acute inlam- tion. Proof of yellow fever vaccination is required
matory bursitis of the prepatellar bursa. With this for entry into many countries where yellow fever is
condition, knee lexion will cause excruciating pain endemic. herefore it is prudent in advance to provide
CHAPTER 3 Rheumatology 73

patients who cannot be vaccinated with a letter that syndrome: sensory loss in the perineum, impotence,
excuses them from vaccination. In addition, patients urinary retention, and bowel retention. If not identiied
should be carefully educated about the risks of yellow and corrected urgently, sacral nerve deicits can become
fever and how to avoid mosquito bites that could lead long standing. he remaining options (Answers A–D)
to yellow fever. MMR booster would be reasonable to delay the diagnosis of this neurosurgical emergency.
give to an immunocompetent traveler to the Amazon, Other “red lags” for low back pain not present in this
but it is not indicated here, because it is a live vaccine. case include fever, occult weight loss, history of malig-
his vaccine is not a requirement for entry into coun- nancy, and recent intravenous drug abuse. hese ind-
tries, so a letter of excuse is not needed for this vaccine ings should prompt consideration of inlammatory,
for international travel. Hepatitis A and the injectable infectious, or malignant etiologies of back pain.
typhoid vaccines are both inactivated vaccines and are Gardner A, Gardner E, Morley T, et  al. Cauda equina syn-
indicated for travelers to Brazil based on Centers for drome: a review of the current clinical and medico-legal
Disease Control and Prevention guidance. position. Eur Spine J. 2011; 20(5):690–697.
Mota LM, Oliveira AC, Lima RA, et al. Vaccination against yel-
low fever among patients on immunosuppressors with diag- 38. ANSWER: C. Hyperuricemia

t
ne
noses of rheumatic diseases [in Portuguese]. Rev Soc Bras Hyperuricemia is not a direct consequence of cor-
Med Trop. 2009;42(1):23–27. ticosteroid use. Obesity, diabetes mellitus, and hyper-
Rubin LG, Levin MJ, Ljungman P, et al. 2013 IDSA clinical tension are all side efects of corticosteroids that are
practice guideline for vaccination of the immunocompro-

e.
themselves risk factors for hyperuricemia. Corticoste-
mised host. Clin Infect Dis. 2014;58(3):309–318.
roids impair the immune system (Answer A), are a cause

in
of cataracts even in young patients (Answer B), can pre-
36. ANSWER: B. Chest radiograph cipitate avascular necrosis of bone (Answer D), and can

ic
his patient describes classic features of Lofgren cause a wasting steroid myopathy that typically presents
syndrome: ankle periarthritis and erythema nodo- as leg weakness in the hip lexors (Answer E).
sum. Chest radiograph will often demonstrate hilar
adenopathy, and biopsy of one of these lymph nodes
ed
39. ANSWER: A. Morning stifness
will show noncaseating granulomatous inlammation he most likely diagnosis is polymyalgia rheumat-
sm
consistent with sarcoidosis. his variant of sarcoidosis ica (PMR), without features to suggest giant cell arteri-
has an excellent prognosis. Patients will almost uni- tis (GCA). Morning stifness (Answer A) is commonly
versally recover fully and not experience other organ- reported in patients with PMR. Elderly patients can
speciic manifestations of sarcoid disease. Associated have incidental indings of rheumatoid factor, chon-
ok

features include oral contraceptive pill use, antecedent drocalcinosis, and joint space narrowing (Answers
streptococcal pharyngitis, inlammatory bowel disease, C–E), but these are not features of PMR. Blindness is
and gastrointestinal dysentery (speciically Yersinia a feared complication of GCA, but the patient denies
bo

species). Ankle ilm (Answer A) will not add to the other symptoms of GCA: headaches, visual distur-
clinical impression that there is ankle swelling. Sero- bances, scalp tenderness, and jaw achiness after chew-
logic testing for RA or Lyme disease (Answers C or E) ing (jaw claudication).
e

is not necessary, because the clinical assessment does Weyand CM, Goronzy JJ. Giant-cell arteritis and polymyalgia
://

not suggest inlammatory arthritis of the ankles; nor- rheumatica. N Engl J Med. 2014;371(1):50–57.
mal passive range of motion of the joint argues for a
tp

periarticular process and not articular disease. Further, 40. ANSWER: B. Adult-onset Still disease
Lyme arthritis is normally monoarticular. Finally, the his patient demonstrates many criteria that sup-
clinical assessment is not consistent with gouty arthri- port a diagnosis of adult-onset Still disease (AOSD):
ht

tis (Answer D), a condition unlikely to afect a pre- pharyngitis, fever, rash, inlammatory arthritis, leuko-
menopausal woman without any speciic risk factors. cytosis, thrombocytosis, hepatosplenomegaly, lymph-
adenopathy, and elevated inlammatory indices. Most
37. ANSWER: E. Referral to the emergency depart- important, an extensive workup for other potential
ment for urgent MRI of the lumbar spine etiologies for her condition has been negative, includ-
his patient is experiencing cauda equina syndrome: ing serologic studies. One additional useful test is the
compression of the cauda equina from a paraspinal serum ferritin level, which can sometimes be strikingly
source. In this case, an intervertebral disc has herni- elevated (>10,000 µg/mL). he clinical scenario is not
ated into the spinal canal, impairing function of sacral that of Lyme disease infection (Answer A). Behçet
nerves. he patient reports symptoms that could be disease (Answer E) is a syndrome that includes oral
mistaken for an S1 radiculopathy alone: loss of plantar ulcers, genital ulcers, and uveitis. Associated features
lexion, absent Achilles relex, and reduced posterior include fever, pyoderma gangrenosum, vasculitis, and
thigh sensation. However, he also has “red lag” features pathergy (neutrophilic pustule formation at the site of
of back pain to prompt evaluation for cauda equina innocuous skin perforation such as a sterile needle).
74 C HA P T E R 3 Rheumatology

Acute bacterial endocarditis (Answer D) is excluded by analgesics, and corticosteroid injections. His physical
the negative blood cultures, nontoxic appearance, and function is compromised, and he is experiencing weak-
time course of 3 weeks’ duration. Systemic lupus ery- ness of hip lexor muscles. Repeating these interventions
thematosus (SLE; Answer C) could explain her clini- (Answers A, B, and D) is unlikely to provide much relief.
cal presentation. However, it is essentially ruled out by Repeat MRI of the lumbar spine (Answer C) is unlikely
the negative ANA test, which has excellent negative to reveal any new pathology, and he does not describe any
predictive value. Unlike this case, SLE is more often “red lags” of back pain that would prompt workup for
associated with leukopenia and thrombocytopenia. inlammatory, infectious, or neoplastic processes. Refer-
Efthimiou P, Paik PK, Bielory L. Diagnosis and management ral to spine surgery is appropriate because decompressive
of adult onset Still’s disease. Ann Rheum Dis. 2006;65(5): laminectomy can alleviate symptoms and improve leg
564–572. weakness.

41. ANSWER: A. Sjögren syndrome 43. ANSWER: D. Temporal artery biopsy showing
he mother likely has circulating anti-Ro (anti-SSA) granulomatous arteritis
antibodies, which can cross the placenta and cause a he patient is experiencing a “pulmonary-renal syn-

t
ne
neonatal “anti-Ro syndrome” characterized by fetal drome”: pulmonary hemorrhage from difuse aveolitis
heart block and photosensitive rash (similar to subacute and crescentic glomerulonephritis. hese syndromes
cutaneous lupus of adults). Besides Sjögren syndrome, result from small-vessel vasculitis in the lung and kid-

e.
this antibody can be found in patients with systemic ney. Inlammatory indices are elevated ESR and CRP,
lupus erythematosus (SLE) or very rarely as an inciden- thrombocytosis, and anemia of chronic disease. Of

in
tal inding in healthy mothers. he rash is self-limited all answers, only giant cell arteritis/temporal arteritis
and only of cosmetic concern. he congenital heart (Answer D) is not associated with small-vessel vasculitis

ic
block, however, is often irreversible. It represents failure of this nature. he serologic studies indicate systemic
of formation of the conductive system of the heart dur- lupus erythematosus (SLE) (Answer A), Goodpasture
ing the early second trimester of pregnancy. hus careful
monitoring should be performed starting around week
ed syndrome (Answer B), and ANCA-associated vas-
culitis (Answer C). Levamisole is an antihelminthic
15 for any pregnant woman with known anti-Ro anti- agent used to “cut” cocaine. In some individuals, it
sm
body positivity, including careful monitoring of fetal
heart rate. Afected patients would beneit from having
high-risk obstetrics follow the pregnancy. Primigravida
women with anti-Ro antibodies carry an approximately
ok

5% chance of the irst pregnancy being afected by con-


genital heart block. In subsequent pregnancies, this risk
is near zero for women who had unafected irst pregnan-
bo

cies. However, the risk is much higher for those women


in whom their irst pregnancy was afected by fetal heart
block. Hydroxychloroquine appears to be efective in
e

reducing the rate of fetal heart block in at-risk moth-


://

ers. Once detected, however, fetal heart block is not


always reversible; no treatments have been shown to be
tp

highly efective for reversing conductive abnormalities.


Treatment options include high-dose betamethasone
or intravenous immunoglobulin. None of the other
ht

answers are associated with fetal heart block. Both Lyme


disease (Answer A) and acute rheumatic fever (Answer • Fig. 3.5 Tophaceous gout picture.
D) can cause heart block in the afected individual.
Marfan syndrome (Answer C) can complicate pregnan- can induce an ANCA-associated vasculitic condition
cies because of maternal aortic dissection. Methotrexate clinically indistinguishable from granulomatosis with
(Answer E) is highly teratogenic to the growing fetus, polyangiitis (GPA), formerly Wegener granulomatosis.
which often does not survive in utero exposure. Many
severe birth defects are described (including neural tube 44. ANSWER: C. Discuss with patient’s nephrologist
defects), but not congenital heart block. about alternative immunosuppressive therapies for
the renal graft.
42. ANSWER: E. Referral to spine surgery for decom- he patient is currently taking azathioprine (AZA),
pressive laminectomy an inhibitor of purine synthesis. AZA metabolites are
he patient has lumbar spinal stenosis that is refrac- further metabolized by xanthine oxidase (XO), which
tory to conservative interventions of physical therapy, normally converts purine metabolites into uric acid.
CHAPTER 3 Rheumatology 75

Although XO inhibitors such as allopurinol and for bronchial lavage to test for infectious microorgan-
febuxostat normally reduce uric acid formation, they isms (an alternative explanation for her presentation).
also lead to accumulation of AZA (and toxicity), unless he most common features of DAH are dyspnea,
the AZA dose is very much reduced. hus Answers A hypoxemia, and unexplained rapid drop in hemoglo-
and E are incorrect in this patient because of concur- bin (>2 g/dL). Fever is common, but hemoptysis is
rent azathioprine use. Answer B is incorrect because reported in only 50% of cases. hus as in this case, the
probenecid increases the risk of nephrolithiasis, a absence of hemoptysis does not exclude DAH. Missing
potentially devastating complication in patients with DAH can have dire consequences; the mortality rate
a single functioning kidney or transplanted kidney. of SLE-related DAH approaches 50%. CT angiogram
Answer D is not correct, because the patient does not with iodinated contrast (Answer A) risks contrast-
describe symptoms of acute gout, and corticosteroids induced nephropathy, and MRI with gadolinium con-
do not solve the underlying problem of hyperuricemia trast (Answer D) risks nephrogenic systemic ibrosis.
with tophaceous gouty deposits (Fig. 3.5). his patient Neither imaging study accurately permits a diagnosis
is young and likely to experience complications of the of DAH. Although pulmonary function tests (PFTs)
numerous tophi, such as a destructive arthropathy or might show elevated DLCO in the presence of alveolar

t
ne
skin breakdown complicated by infections like osteo- blood (Answer B), the patient will be unable to com-
myelitis. hus the best answer is C, with a discussion ply with PFTs while intubated. Lumbar puncture to
about alternatives to AZA so that the patient might exclude various causes of meningitis and meningoen-

e.
beneit from the addition of allopurinol or febuxostat cephalitis (Answer C) would be indicated in a patient
to prevent long-term complications of gouty tophi. such as this whose primary complaint is delirium with-

in
Alternatively, although not typically advised, the out identiiable cause, because immunosuppression
azathioprine dose could be very much reduced, and raises the risk for infection, and SLE can cause vascu-

ic
a very low dose XO inhibitor could be added, with litis and meningoencephalitis of the central nervous
careful dose titration and monitoring of blood counts. system.
Note that calcineurin inhibitors such as cyclosporine
increase serum uric acid levels but are not contraindi-
ed
cated in conjunction with XO inhibitors.
sm

45. ANSWER: B. Aspiration of the right knee for luid


analysis
The patient may have septic arthritis of the
ok

right knee. Aspiration with fluid analysis (Answer


B) is the only option that can address that con-
cern properly. In patients who present with an
bo

acute inflammatory monoarthritis, septic arthritis


should always be high in the differential diagnosis.
This is particularly true when a patient with RA
e

under good control presents with a single inflamed


://

joint. Patients with RA are at increased risk for sep-


tic arthritis for many reasons: increased vascularity
tp

of inflamed joints, immunosuppressive qualities


of DMARD therapies, underlying structural joint • Fig. 3.6 Systemic lupus erythematosus diffuse alveolar hemorrhage
chest radiograph.
disease, and frequent presence of prosthetic joints.
ht

Septic arthritis occurs most commonly from hema-


togenous seeding of the joint from bacteremia, thus
a source of infection must be sought in any patient 47. ANSWER: E. Adalimumab 40 mg every other week
with joint infection. Joint aspiration with steroid Ankylosing spondylitis (AS) is one of several
injection (Answer C) misses that this is a potential spondyloarthritides, a category of disorders that
joint infection and delays diagnosis. Likewise, the also includes psoriatic arthritis, reactive arthritis,
other answers delay diagnosis of a potentially seri- and the arthropathy of inlammatory bowel disease
ous septic arthritis. (IBD arthropathy). his patient describes years of
inlammatory back pain, worse in the morning and
46. ANSWER: E. Flexible bronchoscopy improved with activity. Symptoms of AS usually start
his patient is presenting with features of difuse alve- in the area of the sacroiliac joints (deep buttocks and
olar hemorrhage (DAH), a potentially life-threatening low back) and slowly progress up the spine. horacic
complication of SLE (Fig. 3.6). Diagnosis is made by spine involvement can cause thoracic wall pain and
lexible bronchoscopy (Answer E), which also allows afect chest expansion. AS is associated with episodes
76 C HA P T E R 3 Rheumatology

of unilateral anterior uveitis. Inlammatory mark- 49. ANSWER: E. Nephrogenic systemic ibrosis
ers are often (but not always) elevated. he diagno- Nephrogenic systemic ibrosis (NSF) is a compli-
sis is made based on symptoms of inlammatory back cation of gadolinium exposure that occurs in patients
pain and supportive imaging. Plain ilm radiographic with severely compromised renal function: severe
changes are described in this case. MRI is more sensi- acute kidney injury, stage 4 or 5 chronic kidney dis-
tive and speciic for sacroiliitis, but it is unnecessary ease, or dialysis therapy. Retained gadolinium induces
in this case. Testing for HLA-B27 haplotype (Answer a ibrotic reaction in deep dermal tissues and some-
A) is sometimes used to stratify the likelihood of AS, times in internal organs. his case describes a classic
but it is not diagnostic. Upward of 8%–10% of white presentation of NSF, including the yellowish scleral
individuals carry HLA-B27, and >95% of these peo- plaques and joint contractures from ibrosis of periar-
ple do not have AS. For years, treatment of patients ticular tissues. Scleroderma (Answer A) is highly asso-
with AS with inlammatory spine pain was limited to ciated with Raynaud phenomenon and often afects
cyclooxygenase (COX) inhibitors: nonsteroidal antiin- the face, which is spared in NSF. Lipodermatosclero-
lammatory drugs (NSAIDs). Corticosteroids, meth- sis (Answer B) is a gravity-dependent complication of
otrexate, and even opiate-based therapies (Answers chronic venous stasis that afects the lower extremi-

t
ne
B–D) are relatively inefective or not sustainable for ties only and usually does not cause joint contractures.
the long term. Sulfasalazine has a role in the treatment Scleredema diabeticorum (Answer C) is a ibrotic
of peripheral arthritis, but it is of no value in relieving reaction of dermal tissue that occurs in patients with

e.
spine pain. Antagonists of tumor necrosis factor alpha diabetes. Unlike NSF, it predominantly afects parts of
(TNF-α) have been available since the late 1990s and the upper torso, such as the shoulder and upper back.

in
are extremely efective in relieving the symptoms of Eosinophilic fasciitis is an inlammatory disorder of
inlammatory back pain in patients with AS and other fascia (not dermis) caused by aberrant eosinophilic

ic
types of spondyloarthritis. Five TNF-α antagonists are iniltration, leading to edema of the afected extremity.
currently available: adalimumab, certolizumab, etan- It is not associated with renal dysfunction, but patients
ercept, golimumab, and inliximab.
Taurog JD, Chhabra A, Colbert RA, et  al. Ankylosing
ed may have an underlying myelodysplastic syndrome.
Todd DJ, Kay J. Gadolinium-induced ibrosis. Annu Rev Med.
spondylitis and axial spondyloarthritis. N Engl J Med. 2016;67:273–291.
2016;374(26):2563–2574.
sm

50. ANSWER: A. Rheumatoid arthritis


48. ANSWER: A. Parotid lymphoma Rheumatoid arthritis (RA) is the most common
Patients with Sjögren syndrome (SjS) have a very chronic inlammatory polyarthritis afecting either
ok

high risk of developing lymphoma in the mucosa- gender or any age group. It is characterized by red-
associated lymphoid tissue (MALT) of the parotid ness, warmth, and swelling of the joints, often accom-
and submandibular glands, upward of 50–100 times panied by morning stifness. Almost any joint can be
bo

the normal population. One potential etiology is involved, with the general exception of the thoraco-
that chronic inlammation in these tissues may lead lumbar spine and distal interphalangeal (DIP) joints.
to malignant degeneration of abnormal autoim- Approximately 25% of patients with RA do not have
e

mune lymphocytes. Knowing this risk is important detectable RF or anti-CCP antibodies at any point in
://

when evaluating asymmetric glandular swelling in a their disease course (as in this case). Although these
patient with SjS. Laboratory features associated with patients (termed seronegative RA) often have a less
tp

the presence of these lymphomas include vasculitic severe disease course, they can still develop joint dam-
rash, falling complement levels, and normalization age and erosions (as noted in the ilms in this case).
of a previously abnormal RF. Imaging and surgical Inlammatory indices are often elevated, but these are
ht

referral for biopsy are indicated. Sarcoidosis (Answer nonspeciic. Several features of this case argue against
B) can afect the lacrimal and salivary glands, often chronic gouty arthritis (Answer B). He reports no
in association with fever (uveoparotid fever or Heer- history of episodic acute gouty arthritis, and crystals
fordt syndrome). Sialolithiasis and suppurative bac- were not observed in the joint luid aspirate. he rela-
terial sialadenitis (Answers C and D) can both occur tively normal uric acid does not completely exclude
in patients with SjS, but these conditions are almost gout, but the value of 4.0 mg/dL makes it unlikely
always acute and associated with pain. Ten years that the patient has a chronic, erosive gouty arthropa-
of bisphosphonate therapy increases this patient’s thy. Primary osteoarthritis (Answer C) rarely afects
risk for osteonecrosis of the jaw (Answer E). How- the MCP, wrist, or elbow joints. Rheumatic fever
ever, that condition is not suggested by the history (Answer D) can cause arthralgia and lead to joint
or physical examination, which describes extraoral deformities. Patients can experience a noninlamma-
swelling of the parotid gland and associated changes tory joint laxity (Jaccoud arthropathy) similar to that
in serologic tests. found in patients with systemic lupus erythematosus.
CHAPTER 3 Rheumatology 77

However, the patient also lacks the other clinical fea- in this case. hus although in some ways a diagnosis
tures of rheumatic fever, including the fever, rash, of exclusion, seronegative RA is the most appropriate
pharyngitis, and cardiac complications. Finally, poly- diagnosis in this case.
myalgia rheumatic (PMR) is a seronegative inlamma-
tory disorder of individuals >50 years old (Answer E). Acknowledgment
It primarily afects the shoulders and hips, as in this
case. However, PMR is not considered a cause of ero- he author and editors gratefully acknowledge the contri-
sive inlammatory polyarthritis of the extremities, as butions of the previous author, Dr. Jonathan S. Coblyn.

t
ne
e.
in
ic
ed
sm
ok
e bo
://
tp
ht
4
Pulmonary and Critical Care Medicine
SCOTT L. SC HISSEL AND ELIZABETH GAY

1. A 67-year-old woman presents to clinic to establish C. FEV1 3.0 L (100% of predicted), FVC 4.0 L (100%

t
ne
care for asthma. She has been followed for many of predicted), FEV1/FVC 0.75 (100% of predicted)
years at another facility. She has moderate persistent D. FEV1 2.0 L (66% of predicted), FVC 2.67 L (67%
disease, with several exacerbations per year. Her cur- of predicted), FEV1/FVC 0.75 (100% of predicted)

e.
rent medications include theophylline, salmeterol
inhaler twice daily, and albuterol inhaler as needed. 3. A 43-year-old man presents with several years of dys-

in
Which of the following serious adverse outcomes pnea on exertion. He also has exertional chest tightness
led to a black-box warning regarding the use of long- and has been told he has exercise-induced asthma. He
acting inhaled beta-agonist bronchodilators in the uses albuterol before exercise, but he has not noticed

ic
treatment of asthma? any improvement in his symptoms. He undergoes
A. Atrial ibrillation and lutter
B. Ventricular tachycardia and ibrillation
ed spirometry, which reveals moderate ixed obstruction,
with an FEV1 of 65% of predicted. Chest CT scan is
C. Myocardial infarction and stroke obtained and shows widespread emphysema.
sm
D. Respiratory failure and death Which of the following is the most accurate state-
E. Angioedema and anaphylaxis ment about the most likely diagnosis?
A. he diagnosis is established by a blood test.
2. A 72-year-old man presents to the clinic for evalua- B. All patients with this disease develop obstructive
ok

tion of dyspnea. He notes a 5-year history of slowly lung disease.


progressive dyspnea on exertion. He has a chronic C. he majority of persons develop clinically signiicant
cough with white to yellow phlegm, most bother- liver disease at some point during their lifetime.
bo

some in the morning. He also notes some wheezing D. A characteristic feature is a preserved difusion
with exertion. He smokes one pack of cigarettes per capacity of the lung for carbon monoxide (DLCO)
day, down from two packs per day a few years ago. on pulmonary function testing.
e

He started smoking at age 16. He worked in a beer


://

factory, with some exposure to dust. Physical exami- 4. A 45-year-old woman with asthma reports that she
nation shows the following vital signs: temperature has experienced chest tightness and wheezing approxi-
tp

37°C, heart rate 94 beats per minute, blood pressure mately 30 minutes after ingestion of aspirin 81 mg.
145/80 mm Hg, respiratory rate 20 breaths per min- Which of the following is the most appropriate
ute, and Sao2 94% on room air. Generally, he was response?
ht

alert and had no respiratory distress. he orophar- A. “Your symptoms are not likely to have been caused
ynx was clear with poor dentition. No jugular venous by aspirin at this low dose.”
distention was present. His chest had diminished B. “You are allergic to aspirin and should instead use
breath sounds with scattered end-expiratory wheezes. ibuprofen or naproxen for pain relief with antiin-
His cardiovascular presentation was regular rhythm lammatory activity.”
and rate, distant S1 and S2, with no murmurs. His C. “Because cross-sensitization is common, you
extremities were warm with no clubbing, cyanosis, or should avoid peanuts and tree nuts such as cashews
edema. and almonds.”
Which of the following pulmonary function test D. “Given your history, a leukotriene modiier such
results would you expect for this patient? as montelukast, zairlukast, or zileuton might be
A. FEV1 2.0 L (66% of predicted), FVC 4.0 L (100% particularly helpful as treatment for your asthma.”
of predicted), FEV1/FVC 0.5 (66% of predicted) E. “Given your history, a long-acting beta-agonist bron-
B. FEV1 1.0 L (33% of predicted), FVC 1.25 L (31% chodilator such as formoterol or salmeterol might be
of predicted), FEV1/FVC 0.8 (107% of predicted) particularly harmful as treatment for your asthma.”

78
CHAPTER 4 Pulmonary and Critical Care Medicine 79

5. A 68-year-old man with an 80-pack-year history of D. Pulmonary infarction


tobacco use has mild shortness of breath only while E. Sarcoidosis
climbing “steep hills,” but he has required treatment
two times in 1 year for acute exacerbations of chronic 8. An 82-year-old man presents with a 6-month history
obstructive pulmonary disease (COPD). Spirometry of nonproductive cough and gradually increasing exer-
reveals an FEV1 of 2.2 L, 80% of predicted, and an tional dyspnea, but no fever, chest pain, or hemoptysis.
FEV1/FVC ratio of 60%. He has no history of collagen vascular disease. He con-
Which of the following best describes this patient’s tinues to work as an accountant and has had no unusual
Global Initiative for Chronic Obstructive Lung Dis- exposures in his home or work environments. He takes
ease (GOLD) stage of COPD? hydrochlorothiazide for hypertension, aspirin 81 mg,
A. Stage 1A and a multivitamin. On examination, he has early club-
B. Stage 1C bing and high-pitched inspiratory crackles throughout
C. Stage 2A the lower lung zones posteriorly. Chest x-ray reveals
D. Stage 2C linear and small nodular opacities (“reticulonodular
E. Stage 3A pattern”), predominantly in the lower lung zones. Spi-

t
ne
rometry suggests a moderate restrictive pattern. Chest
6. A 45-year-old man presents with a productive cough, CT with high-resolution images reveals linear opacities,
fever, and left-sided chest pain of approximately 1 week’s most prominent in the lung periphery, with traction

e.
duration. Physical examination reveals dullness to percus- bronchiectasis and areas of “honeycombing” at the lung
sion and decreased breath sounds throughout the lower bases; there are no areas with “ground-glass” opacities

in
half of the left chest posteriorly. His white blood cell (Fig. 4.1). Fiberoptic bronchoscopy is performed with
count is 17,500/µL with 85% polymorphonuclear leu- normal visualized tracheobronchial mucosa. Analysis of

ic
kocytes and 6% band forms. A chest x-ray shows a mod- bronchoalveolar lavage luid shows no malignant cells
erate to large left pleural efusion. Sputum Gram stain and no organisms. Transbronchial lung biopsies reveal
shows many neutrophils with mixed lora, including
gram-positive cocci, gram-positive rods, and gram-nega-
ed minimal and nonspeciic inlammatory changes in the
interstitium, with collagen deposition (ibrosis) and
tive rods. Sputum culture grows only “normal oral lora” increased numbers of alveolar macrophages.
sm
after 48 hours. horacentesis succeeds in withdrawing Based on this clinical information, what is the most
only 60 mL of serous luid. Pleural luid analysis reveals likely diagnosis?
a high total protein concentration (4 g/dL), white blood A. Sarcoidosis
cells 6,500/µL with 80% polymorphonuclear cells, and B. Nonspeciic interstitial pneumonia
ok

no organisms on Gram stain or bacterial culture. Pleural C. Idiopathic pulmonary ibrosis


luid pH is 7.10; pleural luid amylase is low. he patient D. Cryptogenic organizing pneumonia (COP)
is started on azithromycin and cefotaxime in the emer- E. Langerhans cell histiocytosis
bo

gency department but fails to improve.


Which of the following is the next best step in man-
agement?
e

A. Discontinuation of azithromycin and cefotaxime


://

and initiation of levoloxacin


B. Chest tube drainage of pleural space, with thora-
tp

coscopic lysis of pleural adhesions if necessary


C. Pleural biopsy
D. Barium swallow to evaluate for esophageal rupture
ht

(Boerhaave syndrome)
E. Systemic corticosteroids for probable inlamma-
tory pleuritis related to collagen vascular disease

7. A 51-year-old woman presents with the gradual onset of


dyspnea. She denies any infectious symptoms. Her past
medical history is notable for a “kidney tumor.” Chest
x-ray shows a moderate-sized right pleural efusion. ho-
racentesis reveals 600 mL of milky-appearing luid with a
triglyceride concentration of 145 mg/dL.
Which of the following is the most likely cause of the
pleural efusion?
A. Lymphangioleiomyomatosis (LAM)
B. Pneumonia • Fig. 4.1 Chest CT scan showing linear opacities with traction bron-
C. Malignancy chiectasis and areas of “honeycombing” for patient in Question 8.
80 C HA P T E R 4 Pulmonary and Critical Care Medicine

9. A 63-year-old man presents to the pulmonary clinic D. Motor vehicle accidents


with a chronic cough and dyspnea on exertion. His E. Hypoxemic respiratory failure
exam is notable for basilar inspiratory crackles. His
social history is notable for 30 years of employment 13. A 52-year-old man with no history of smoking presents
in a shipyard. Chest CT shows basilar linear opacities with a history of frequent episodes of bronchitis. He has
and areas of “honeycombing.” a chronic barking cough that has not improved with
Which of the following test results would be steroid inhalers or bronchodilators. He has a history of
expected in this patient? arthritis but no history of gastroesophageal relux disease
A. Normal oxygen saturation at rest that falls to 82% (GERD), rashes, or exertional dyspnea. Pulmonary func-
with exertion tion tests show mild ixed obstruction and no restriction.
B. Normal oxygen saturation at rest without change Which of the following is the most likely diagnosis?
with exertion A. Scleroderma
C. Normal DLCO (difusion capacity of the lung for B. Polymyositis
carbon monoxide) C. Relapsing polychondritis
D. Increased lung static compliance D. Systemic lupus erythematosus

t
ne
E. Increased total lung capacity (TLC), functional E. Mixed connective tissue disease
residual capacity (FRC), and residual volume (RV)
on full measurement of lung volumes 14. A 30-year-old man presents for evaluation of poly-

e.
cythemia. He is found to have resting hypoxemia
10. A 67-year-old man with a diagnosis of idiopathic pul- (Sao2 = 87%) and signs of right heart failure, with

in
monary ibrosis presents to discuss treatment options. jugular venous distention and peripheral edema. He
Over the past 2 years, his FVC has declined from 75% reports excessive daytime sleepiness. His examination

ic
of predicted to 60% of predicted. He has noticed is remarkable for morbid obesity (body mass index
increased dry cough and dyspnea on exertion. [BMI] = 42 kg/m2), narrowed posterior pharyngeal
Which of the following therapies would be most likely
to prevent further lung function decline in this patient?
ed opening, and clear chest on auscultation. His hema-
tocrit is 52%. Chest x-ray is normal. Spirometry iden-
A. High-dose oral steroids tiies a pattern suggesting mild restriction. Arterial
sm
B. High-dose oral steroids plus azathioprine blood gases reveal Po2 55 mm Hg, Pco2 72 mm Hg,
C. Rituximab and pH 7.32. Continuous positive airway pressure
D. Hydroxychloroquine (CPAP) fails to correct his nocturnal hypoxemia and
E. Pirfenidone daytime hypersomnolence.
ok

Which of the following would be an expected ind-


11. A 47-year-old man with a history of asthma and ing in his evaluation?
nasal polyps presents for evaluation of daytime A. Normal ventilatory response to carbon dioxide
bo

sleepiness. He also has history of hypertension and B. Orthodeoxia


retrognathia as well as chronic insomnia and poor C. Central apneas
sleep quality. On examination, you note a height of D. Restless legs syndrome
e

62 inches but a normal BMI. His neck circumfer- E. Prolonged sleep latency period
://

ence is 15 inches.
Which of the following attributes is a risk factor for 15. A 52-year-old woman presents for evaluation of dys-
tp

obstructive sleep apnea? pnea and fatigue. Her medical history is notable for
A. Retrognathia obesity, hypertension, and type 2 diabetes. She has no
B. Dress shirt neck circumference >15 inches history of smoking, but she was placed on oxygen a
ht

C. Nasal polyps with complete nasal airlow obstruction few years ago for hypoxemia. On exam, she has a room
D. Short stature (<60 inches tall in women, <65 air oxygen saturation of 88%. Her lungs are clear, and
inches tall in men) cardiac exam shows no murmurs, with a normal S1
E. Insomnia and S2. She has trace lower extremity edema. Her
pulmonary function tests show mild restriction but
12. A 52-year-old woman with known obstructive sleep no evidence of obstruction. Polysomnography shows
apnea and an apnea-hypopnea index of 62 presents to severe obstructive sleep apnea, with an apnea-hypop-
the clinic for follow-up. She reports inability to tolerate nea index of 43.
prescribed continuous positive airway pressure (CPAP) Which of the following tests represents the next
because she feels claustrophobic. best diagnostic step?
She is at increased risk of death resulting from which A. Chest x-ray
of the following? B. Blood gas on room air
A. Aspiration C. CBC with diferential
B. Laryngospasm D. Chest CT
C. Diabetic ketoacidosis E. Six-minute walk test
CHAPTER 4 Pulmonary and Critical Care Medicine 81

16. A 62-year-old man presents with a history of wheez- Which of the following tests is most likely to iden-
ing and dyspnea. he wheezing has been refractory tify an etiology of bronchiectasis for which a speciic
to all inhalers and to a course of oral prednisone. therapy is available?
Chest CT shows a tracheal tumor. He undergoes A. Antinuclear cytoplasmic antibody (ANCA)
pulmonary function testing. B. Serum immunoglobulin G
Which of the following images represents the most C. Gluten autoantibodies (serum endomysial and
likely low–volume loop? antitissue transglutaminase antibodies)
A. Fig. 4.2 Flow–volume loop D. Bronchial biopsy examined by electron microscopy
B. Fig. 4.3 Flow–volume loop E. Full pulmonary function tests, including measure-
C. Fig. 4.4 Flow–volume loop ment of lung volumes and difusion capacity of
D. Fig. 4.5 Flow–volume loop the lung for carbon monoxide
E. Fig. 4.6 Flow–volume loop
18. A 68-year-old woman presents for evaluation of chronic
17. A 43-year-old woman with a history of frequent sinus- cough and fatigue. She describes 1 year of cough pro-
itis and pneumonia presents for evaluation. Imaging is ductive of yellow sputum. he cough failed to improve

t
ne
obtained, which shows bilateral lower lobe bronchiectasis. with empiric antibiotic courses, inhalers, and therapy for

8 8

e.
7 7
6 6

in
5 5
4 FEV1 = 2.58 FEV1 = 1.80V
4

ic
3
3
2 FEV3 = 3.14 FEV3 = 2.33
1
0
ed
2
1
1 2 3 4 5 6 7 8 0
sm
1
1 2 3 4 5 6 7 8
2 −1
3 −2
4 −3
ok

5 −4
6
−5
bo

7
−6
8
• Fig. 4.2
−7
A. Flow–volume loop for patient in Question 16.
−8
e

• Fig. 4.4 C. Flow–volume loop for patient in Question 16.


://

10
9 6
tp

8
5
7
6 4
ht

FEV1 = 1.52
5
3 FEV1 = 0.80
4
3 FEV3 = 1.74 2 FEV3 = 1.34
2
1 1
0
1 2 3 4 5 6 7 8 9 10 0
1
1 2 3 4 5 6
2 1
3
4 2
5 3
6
7 4
8
5
9
10 6
• Fig. 4.3 B. Flow–volume loop for patient in Question 16. • Fig. 4.5 D. Flow–volume loop for patient in Question 16.
82 C HA P T E R 4 Pulmonary and Critical Care Medicine

relux. Physical exam is notable for a thin, elderly woman mild airlow obstruction; a cardiac stress test shows no
in no distress, with clear lung ields. Pulmonary function evidence of myocardial ischemia.
testing reveals normal spirometry and difusion. Which of the following would be the next best step
Which of the following indings would you expect in management?
on chest CT (Fig. 4.7)? A. Measurement of prostate-speciic antigen (PSA),
A. Ground-glass opacity upper gastrointestinal series, and colonoscopy
B. Consolidation with air bronchograms B. Fiberoptic bronchoscopy and transbronchial lung
C. Honeycombing biopsy
D. Tree-in-bud nodules C. Transthoracic needle aspirate/biopsy
E. Atelectasis D. Surgical resection of the lung nodule
E. Repeat chest imaging in 3 months to assess for
19. he accompanying chest x-ray (Fig. 4.8) is most likely growth of the nodule
from a patient with which of the following diagnoses?
A. Alpha-1 antitrypsin deiciency 21. A 53-year-old woman with a history of chronic
B. Ankylosing spondylitis obstructive pulmonary disease presents for routine

t
ne
C. Bronchiectasis due to cystic ibrosis follow-up. She is doing well from a respiratory stand-
D. Granulomatosis with polyangiitis (Wegener gran- point but notes hoarseness. he hoarse voice comes
ulomatosis) and goes, but she is concerned about it.

e.
E. Lymphangioleiomyomatosis (LAM) Which one of the following medications is the
most likely cause of her hoarseness?

in
20. A 55-year-old cigarette-smoking man born and reared A. Salmeterol
in New York City is found to have a round, noncalci- B. Fluticasone

ic
ied right lower lobe nodule 1.8 cm in diameter, new C. Tiotropium
since a prior chest x-ray obtained 1 year ago. Other D. Albuterol
than his usual “smoker’s cough,” he has been free of
respiratory symptoms. His physical examination is nor-
ed E. Ipratropium

mal. Positron emission tomography (PET) scan reveals 22. A 43-year-old woman presents with exertional dys-
sm
increased uptake of radiolabeled glucose within the pnea, progressive over the past few years. She is now
lung nodule but at no other sites. Spirometry indicates breathless climbing one light of stairs. She is a lifelong
nonsmoker without a history of asthma. Her chest
8
x-ray shows mild hyperinlation. Spirometry and lung
ok

7
volumes indicate severe airlow obstruction without
6
signiicant improvement following bronchodilator.
5 Alpha-1 antitrypsin level is normal.
bo

4 FEV1 = 0.73
Which of the following is the most likely explana-
3
FEV3 = 1.70
tion for her lung disease?
2 A. She works in an old oice building with central ven-
e

1 tilation and without windows that can be opened.


://

0 B. She owns four large parrots.


−1 1 2 3 4 5 6 7 8 C. She has ulcerative colitis.
tp

−2 D. She has mixed connective tissue disease.


−3 E. She has a history of breast cancer treated with lumpec-
tomy and postsurgical radiation therapy to the breast.
ht

−4
−5
−6
−7
−8
• Fig. 4.6 E. Flow–volume loop for patient in Question 16.

• Fig. 4.8 Posteroanterior and lateral chest x-ray for patient in


• Fig. 4.7 CT for patient in Question 18. Question 19.
CHAPTER 4 Pulmonary and Critical Care Medicine 83

23. A 72-year-old woman with an 80-pack-year history of C. Hydroxychloroquine (Plaquenil) 200 mg PO twice
tobacco use and severe, GOLD stage D COPD with daily
FEV1 0.7 L, 38% of predicted is admitted to the hospi- D. Methotrexate 15 mg PO once per week
tal with 4 days of shortness of breath at rest, increased E. No therapy
cough, and cough productive of yellow sputum. A
chest x-ray reveals no pulmonary iniltrates. Her arterial 25. A 72-year-old man with COPD and severe chronic air-
blood gas on room air shows the following values: pH low obstruction presents to the emergency department
7.30, Paco2 55 mm Hg, and Pao2 58 mm Hg. with shortness of breath, productive cough, and wheez-
Which of the following is the best recommendation ing. His medications at home include tiotropium, lutica-
about treatment for this patient? sone-salmeterol combination, albuterol by metered-dose
A. Start an 8-week course of oral steroids because inhaler, aspirin (for primary prevention of cardiovascular
it is associated with fewer relapses than a 2-week events), and lisinopril for hypertension. His chest x-ray is
course of treatment. normal; oxygen saturation on 6 L/min oxygen by nasal
B. Use intravenous corticosteroids because they are prongs is 90%. Despite frequent nebulized bronchodila-
more efective and have fewer gastrointestinal side tors and systemic steroids, he has worsening respiratory

t
ne
efects than oral corticosteroids. distress and hypoxemia and is intubated and begun on
C. Avoid bilevel positive airway pressure ventilation mechanical ventilation. Ventilator settings are as fol-
because it may cause hyperinlation and auto–pos- lows: volume-cycled ventilation in assist-control mode

e.
itive end-expiratory pressure (auto-PEEP). at 18 breaths per minute, 600 mL/breath, with positive
D. Start antibiotics because the patient has a change end-expiratory pressure at 5 cm H2O and 40% inspired

in
in sputum quantity and character. oxygen. After approximately 30 minutes, he develops
E. Send for a sputum Gram stain and culture because new-onset ventricular arrhythmias. He remains afebrile

ic
these can often guide the choice of antibiotics. and normotensive. Arterial blood gases reveal the follow-
ing: Po2 195 mm Hg, Pco2 36 mm Hg, pH 7.56, and
24. A 24-year-old man is found to have bilateral hilar and
mediastinal lymphadenopathy on a routine preemploy-
ed measured bicarbonate 30 mEq/L.
Which of the following is a potential cause of his
ment chest x-ray (Fig. 4.9). He has a mild, dry cough but arrhythmia?
sm
no other respiratory or systemic symptoms. He was born A. Sudden reversal of chronic hypoxemia (acute
and raised in New England, with no history of tuberculo- hyperoxia)
sis or exposure to anyone with known tuberculosis. Phys- B. Acute metabolic alkalemia due to “unmasking” of
ical examination is normal, as are routine blood studies compensatory chronic metabolic alkalosis
ok

(complete blood count and comprehensive metabolic C. Auto-PEEP with high intrathoracic pressures and
proile). Mediastinoscopy with right paratracheal lymph inadequate minute ventilation
node biopsy reveals noncaseating granulomas. Stains for D. Acute-on-chronic respiratory acidosis due to
bo

organisms are negative. refractory exacerbation of COPD


Which of the following is the most appropriate treat- E. Hypochloremic, hypokalemic metabolic alkalosis
ment at this time? induced by combination of beta-agonists and cor-
e

A. Low-dose oral steroids (prednisone 10 mg/day, ticosteroids


://

with dose adjusted according to his response to


treatment) 26. A 67-year-old man presents to the emergency depart-
tp

B. Initial high-dose steroids (prednisone 40–60 mg/ ment with fever and cough. Chest x-ray demonstrates
day), tapered and then discontinued over approxi- a dense right lower lobe consolidation, and he is started
mately 6 months on antibiotics. While awaiting admission to the medi-
ht

cal loor, he becomes hypotensive and is unresponsive


to several liters of intravenous luid. A central line is
placed.
Which of the following statements is true regarding
the choice of norepinephrine versus dopamine as treat-
ment for his hypotension?
A. Dopamine is more likely to protect renal function.
B. Dopamine is more likely to cause cardiac
arrhythmias.
C. Norepinephrine results in more overall adverse
events.
D. Norepinephrine results in higher mortality in car-
diogenic shock.
E. Evidence for comparison of these vasopressors is
• Fig. 4.9 Posteroanterior and lateral chest x-ray for patient in limited by lack of randomized control trials.
Question 24.
84 C HA P T E R 4 Pulmonary and Critical Care Medicine

27. A 45-year-old man is admitted with severe bilateral 31. A 39-year-old woman with depression is admitted
pneumonia. He is intubated and placed on assist-control after being found intoxicated next to an empty bottle
ventilation with low tidal volumes, but the fraction of of acetaminophen. Her laboratory results are notable
inspired oxygen (Fio2) cannot be decreased below 0.9 for an alanine aminotransferase (ALT) of 55 U/L and
despite optimal PEEP. an aspartate aminotransferase (AST) of 34 U/L. Serum
Paralysis for 48 hours in this setting is most likely to and urine drug screens are pending.
result in which of the following? Which of the following is the best next step in her
A. An increase in neuromuscular weakness management?
B. A reduction in mortality A. Toxicology screen before providing further therapy
C. An increase in lung injury B. Sodium bicarbonate
D. A decrease in sedation requirements C. Activated charcoal
E. An increase in organ failure D. N-acetylcysteine
E. Gastric lavage
28. A 65-year-old man with no signiicant past medical
history is admitted with septic shock due to a left 32. A 76-year-old woman with severe COPD is admit-

t
ne
lower lobe pneumonia. He rapidly decompensates ted with 2 days of dyspnea and cough. She is alert, in
and requires vasopressors and intubation. A chest moderate respiratory distress, and has an oxygen satu-
x-ray following intubation demonstrates difuse ration of 93% on 3 L/min supplemental oxygen deliv-

e.
bilateral airspace opacities. His oxygen saturation is ered by nasal cannulae. A chest x-ray demonstrates no
95% with a Pao2 of 75 mm Hg on an Fio2 of 1.0 abnormal pulmonary opacities. She receives intensive

in
and PEEP of 7.5 cm H2O. He is 5 feet, 8 inches tall bronchodilator therapy and systemic steroids. Arterial
and weighs 100 kg, with a predicted body weight of blood gases demonstrate pH 7.27, Pco2 66 mm Hg,

ic
68.4 kg. and Pao2 74 mm Hg.
Which of the following mandatory (assist-control) Which of the following is the best next step in her care?
ventilator modes would be best for managing this
patient’s condition?
ed A. Increase supplemental oxygen
B. Intubation and mechanical ventilation
A. Volume targeted, tidal volume 800 mL C. Noninvasive ventilation
sm
B. Volume targeted, tidal volume 700 mL D. Decrease supplemental oxygen
C. Volume targeted, tidal volume 650 mL E. Initiate antibiotics
D. Volume targeted, tidal volume 410 mL
E. Pressure targeted, peak pressure 25 cm H2O 33. A 78-year-old man with severe Parkinson disease was
ok

admitted 3 days ago with an aspiration pneumonia and


29. An 87-year-old woman with severe COPD remains required intubation for respiratory failure. His fevers
in respiratory distress despite noninvasive ventilation. have resolved and his oxygenation has improved. His
bo

Preparations are made to proceed with rapid-sequence current ventilator settings are volume-cycled ventila-
intubation using propofol and succinylcholine. tion, assist-control mode, tidal volume 400 mL, PEEP
Which of the following conditions is a potential 5 cm H2O, and Fio2 0.4. His oxygen saturation is 95%.
e

contraindication to these medications? Which of the following is the best next step in care?
://

A. Neuroleptic malignant syndrome A. Extubation


B. Chronic corticosteroid use B. Spontaneous breathing trial
tp

C. Hypernatremia C. Pressure support weaning


D. Hypertension D. Decrease PEEP and Fio2
E. Hyperkalemia E. Synchronized intermittent mandatory ventilation
ht

(SIMV)
30. A 45-year-old man is admitted to the ICU with severe
pancreatitis. Over the irst 24 hours he develops hypo- 34. A 55-year-old woman is admitted with severe sepsis
tension requiring vasopressors, respiratory failure and renal failure. Her hemodynamics improve, and
requiring intubation, and renal failure. Over the last she is weaned of vasopressors. Her fever and leukocy-
several hours, his urine output has decreased, his vaso- tosis also resolve. However, she remains oliguric, and
pressor requirement has increased, and his abdomen her creatinine continues to rise.
has become progressively distended. Which of the following statements about renal
Which of the following is the best next step in his replacement therapy in this setting is most accurate?
management? A. Continuous renal replacement therapy (CRRT)
A. Abdominal CT does not have a mortality beneit compared with
B. Empiric antibiotics intermittent hemodialysis (IHD).
C. Abdominal ultrasound B. High-intensity renal replacement therapy (either
D. Measurement of bladder pressure CRRT or IHD) has consistently been demon-
E. Abdominal plain ilm strated to improve mortality.
CHAPTER 4 Pulmonary and Critical Care Medicine 85

C. CRRT reduces mortality, but only in patients on no further bleeding. His ICU course is complicated
vasopressors. by delirium and pneumonia presumed to be due to
D. CRRT reduces mortality, but only in patients on aspiration. His hemoglobin stabilizes at 7.5 g/dL.
mechanical ventilation. Which of the following is the most accurate state-
E. IHD is contraindicated in patients on vasopressors. ment about the role of red blood cell transfusion?
A. Transfusion is indicated due to concomitant coro-
35. A 45-year-old obese woman with Crohn disease compli- nary artery disease.
cated by multiple lares is admitted with fever, abdomi- B. Transfusion is indicated due to his recent gastroin-
nal pain, diarrhea, and hypotension. In the emergency testinal bleed.
department, she receives 2.5 L of intravenous luid, but C. Transfusion is associated with fewer ventilator-
her systolic blood pressure remains in the range of 70 dependent days.
mm Hg, and a central venous catheter is placed. Her D. Transfusion may be associated with an increase in
central venous pressure (CVP) is 6 mm Hg with sig- mortality.
niicant respiratory variation. Vasopressors are initiated, E. Transfusion may result in hypokalemia.
and her mean arterial pressure is currently 55 mm Hg.

t
ne
Laboratory values are notable for a leukocytosis and a 39. A 75-year-old man is admitted for pneumonia and
hematocrit of 27%; her albumin is 2.8 g/dL. sepsis. He requires vasopressors and is intubated for
Which of the following is the best next step in man- respiratory failure. He is placed on midazolam and

e.
agement? fentanyl. He is treated with ceftriaxone and azithro-
A. Increase vasopressor dose mycin for severe community-acquired pneumonia

in
B. Intravenous crystalloid drip, at a rate of 200 mL/h and continued on his outpatient aspirin, atorvastatin,
C. Intravenous administration of colloid and sertraline. Over the course of the next 18 hours,

ic
D. Red blood cell transfusion he is weaned of of vasopressors, and his white blood
E. Intravenous crystalloid bolus cell count improves; however, he is noted to become

36. An 85-year-old woman is admitted to the ICU with


ed increasingly agitated. Additional midazolam and fen-
tanyl are given, and he also receives haloperidol for
chest pain, new onset of atrial ibrillation, and an exac- presumed delirium. His vital signs are notable for a
sm
erbation of her chronic congestive heart failure (CHF). new fever to 103°F, heart rate 120 beats per minute,
After several days, she starts to improve. Heparin was and blood pressure 100/60 mm Hg. His examina-
initiated at admission, and she has recently started tion is notable for dry mucous membranes, increased
transitioning to warfarin. Starting on the ifth hos- muscle tone, and tremor and hyperrelexia with clo-
ok

pital day, her platelets decrease to 100,000/µL from nus, particularly in his lower extremities. His pupils
220,000/µL on admission. She is now noted to have are dilated with oscillatory eye movements.
new right lower extremity pain and edema. Which of the following is the next best step?
bo

In addition to discontinuing heparin, which of the A. Discontinue sertraline


following is the next best step? B. Discontinue sertraline and fentanyl
A. Await results of lower extremity ultrasound C. Initiate propofol
e

B. Initiate argatroban D. Discontinue haloperidol


://

C. Initiate low-molecular-weight heparin E. Add vancomycin


D. Continue transition to warfarin
tp

E. Await results of platelet factor 4 (PF4) antibody 40. A 45-year-old man has a witnessed cardiac arrest in a
shopping mall. Bystander cardiopulmonary resuscita-
37. An 85-year-old woman is admitted to the ICU after tion is initiated. An automated external deibrillator
ht

having a stroke. She is intubated for altered mental is applied. It advises a shock, which is delivered, but
status and inability to protect her airway. there is no return of spontaneous circulation. Para-
Which of the following interventions could increase medics arrive, and after two doses of epinephrine and
her risk of ventilator-associated pneumonia? additional shocks are given according to advanced car-
A. Elevation of the head of the bed to 30 degrees diac life support (ACLS) protocol, the patient regains a
B. Oropharyngeal decontamination pulse. When he arrives in the emergency department,
C. Gastrointestinal decontamination he is tachycardic but normotensive. He withdraws to
D. Continuous drainage of subglottic secretions painful stimuli but is otherwise unresponsive. Labora-
E. Stress ulcer prophylaxis tory data are notable for a white blood cell count of
18,000/µL, lactate 8 mEq/L, potassium 5.8 mEq/L,
38. A 52-year-old man with coronary artery disease is bicarbonate 18 mEq/L, and creatinine 1.5 mg/dL.
admitted with hematemesis. He is intubated, and an Which of the following is the most accurate statement
esophagogastroduodenoscopy (EGD) demonstrates about therapeutic hypothermia in this setting?
a bleeding gastric ulcer. Over the next 48 hours his A. herapeutic hypothermia is contraindicated due
condition stabilizes, and a repeat EGD demonstrates to risk of infection.
86 C HA P T E R 4 Pulmonary and Critical Care Medicine

B. herapeutic hypothermia is contraindicated due 44. A 65-year-old man is admitted with several days of pro-
to hyperkalemia. gressive weakness, dyspnea, and cough. His temperature
C. herapeutic hypothermia results in increased fre- is 99°F, blood pressure is 82/58 mm Hg, heart rate is 105
quency of malignant arrhythmias. beats per minute, and respiratory rate is 22 breaths per
D. herapeutic hypothermia should be initiated after minute. His oxygen saturation is 93% on supplemental
reassessment of neurologic status at 12 hours. oxygen at 2 L/min delivered by nasal cannulae. Labora-
E. herapeutic hypothermia should be continued for tory values demonstrate an elevated white blood cell
24 hours. count; asymmetric bilateral lower lobe opacities are dem-
onstrated on his chest x-ray. He is given intravenous luids
41. A 45-year-old man is intubated for a severe asthma exac- and antibiotics for pneumonia. Over the course of the
erbation. His chest x-ray demonstrates only hyperinla- next several hours, his hypotension worsens. Blood lactate
tion. His initial ventilator settings are volume-targeted, is elevated, and his extremities are cool. A central venous
assist-control mode ventilation; tidal volume 450 mL (7 catheter is placed for initiation of vasopressors. His central
mL/kg of ideal body weight); set rate 28 breaths per min- venous pressure is 14 cm H2O, and a blood gas drawn
ute; PEEP 5 cm H2O; and Fio2 0.5. he patient is not from this catheter reveals an oxygen saturation of 40%.

t
ne
triggering additional breaths and appears synchronous Which of the following is the best interpretation of
with the ventilator, and he has a measured end-expiratory his hemodynamics?
pressure of 15 cm H2O with a peak inspiratory pressure A. Distributive shock due to sepsis

e.
of 45 cm H2O. His blood pressure is 89/55 mm Hg. His B. Distributive shock due to underlying liver disease
arterial blood gases reveal the following: pH 7.38, Pco2 C. Hypovolemic shock due to sepsis

in
40 mm Hg, and Po2 120 mm Hg. D. Hemorrhagic shock
Which of the following ventilator changes would E. Cardiogenic shock

ic
you recommend?
A. No change 45. A 55-year-old man is scheduled to undergo an esopha-
B. Increase PEEP
C. Decrease respiratory rate
ed gogastroduodenoscopy for Barrett esophagus. His vital
signs, including oxygen saturation, are normal. He
D. Increase respiratory rate receives topical anesthesia with benzocaine, and con-
sm
E. Increase inspiratory-to-expiratory time (I:E) ratio scious sedation is achieved using midazolam and fen-
tanyl. Shortly after the procedure begins, his oxygen
42. An 84-year-old man is intubated for aspiration pneu- saturation drops to the mid-80% range. Other vital
monia. signs have not changed. He is placed on supplemental
ok

Which of the following interventions is most likely oxygen, but his oxygen saturation does not improve.
to be associated with an increased duration of mechan- Chest examination demonstrates clear lung ields.
ical ventilation? Which of the following is the next best diagnostic
bo

A. Daily interruption of sedation step?


B. Daily spontaneous breathing trials A. Chest x-ray
C. Use of midazolam instead of propofol B. Chest CT angiography
e

D. Use of dexmedetomidine instead of midazolam C. Arterial blood gas with cooximetry


://

E. Avoidance of routine sedative medication D. Arterial blood gas


E. Electrocardiogram
tp

43. A 55-year-old man is admitted with altered mental


status. He has a history of heavy alcohol abuse. In the 46. A 75-year-old man with severe chronic obstructive
emergency department, his vital signs are notable for pulmonary disease is intubated for an exacerbation
ht

tachycardia to 110 beats per minute but are otherwise of his lung disease. His initial ventilator settings are
normal. Laboratory values include the following: serum volume-cycled ventilation in assist-control mode, tidal
sodium 145 mEq/L, potassium 4 mEq/L, chloride 105 volume 500 mL (8 mL/kg), PEEP 5 cm H2O, and
mEq/L, bicarbonate 15 mEq/L, BUN 25 mg/dL, glu- Fio2 0.5. His initial peak inspiratory pressures are 18
cose 150 mg/dL, and creatinine 1.5 mg/dL. Urinalysis cm H2O. Several hours after intubation, his ventila-
demonstrates no ketones but is noted to have needle- tor alarms indicate increased peak inlation pressures.
shaped crystals. His serum alcohol level is 10 mg/dL, His new peak inspiratory pressure is 35 cm H2O. His
and his serum osmolarity is 325 mOsm/kg. plateau pressure is 15 cm H2O.
Which of the following is the most appropriate Which of the following is the next best step in man-
next step in his treatment? agement?
A. Ethanol A. Obtain a chest x-ray
B. Benzodiazepines B. Needle decompression
C. Fomepizole C. Endotracheal suctioning
D. Hemodialysis D. Chest tube placement
E. Bicarbonate E. Diuresis
CHAPTER 4 Pulmonary and Critical Care Medicine 87

47. A 27-year-old woman with epilepsy and a history 49. A 75-year-old man is admitted with right lower lobe
of poor adherence to therapy is admitted to the opacities and septic shock. He is treated with broad-
hospital after a tonic-clonic seizure. Shortly after spectrum antibiotics. His condition worsens over
admission, she has two more seizures. She is given the irst 24 hours. His initial iniltrate progresses to
lorazepam and intubated. She is afebrile and nor- become bilateral, and he requires intubation. His
motensive; head CT scan and serum chemistries, shock worsens to the point of requiring two vasopres-
including blood glucose, are all within normal sors despite adequate luid resuscitation.
limits. Which of the following is the most accurate statement
Which of the following statements about her man- about administration of corticosteroids in this patient?
agement is most accurate? A. Corticosteroids improve mortality in patients who
A. Infusion with phenytoin should be the initial have a low response to adrenocorticotropic hor-
treatment. mone (ACTH) stimulation.
B. Fosphenytoin is preferred over phenytoin because B. Corticosteroids improve mortality in patients who
of a decreased risk of hypotension. have inappropriately normal serum cortisol.
C. Diazepam has a slower onset of action than C. Corticosteroids will likely improve his response to

t
ne
lorazepam. vasopressors.
D. Lorazepam infusion followed by fosphenytoin is a D. Corticosteroids, if initiated for his ARDS, will be
reasonable treatment. of greater beneit if initiated after 14 days.

e.
E. Propofol is inefective for this condition. E. High-dose corticosteroids demonstrate mortality
beneit in sepsis but have an unacceptable rate of

in
48. A 67-year-old patient with COPD and coronary dis- complications.
ease is admitted to the hospital for pneumonia. During

ic
the evening of his admission, his tachypnea worsens, 50. An 85-year-old man with hypertension, coronary
and he is placed on increased supplemental oxygen. artery disease, and chronic kidney disease is admitted
Several hours later, he becomes hypotensive and more
lethargic. He is found soon thereafter without a pulse.
ed after an overdose of verapamil. His initial blood pres-
sure is 130/80 mm Hg with a pulse of 75 beats per
he cardiac arrest team is called. minute. His mental status is normal, and his serum
sm
Which of the following statements about his resus- chemistries and complete blood count are unchanged
citation is most accurate? from baseline. His electrocardiogram shows a pro-
A. Inadequate bag-and-mask ventilation is a com- longed PR interval. He is given activated charcoal
mon cause of failure to recover from an arrest not and intravenous calcium and is admitted to the ICU.
ok

due to ventricular tachycardia or ventricular ibril- Over the next several hours, he develops bradycardia
lation. and hypotension. He is given additional calcium, atro-
B. A rapid response team would not have reduced the pine, and glucagon with some transient improvement.
bo

likelihood of cardiac arrest. However, his bradycardia and hypotension continue


C. A pulse check should be performed immediately to worsen, and he requires intubation and escalating
after administration of medications. doses of vasopressors.
e

D. A pulse check should be performed immediately Which of the following therapies should be initiated?
://

after deibrillation. A. Hemodialysis


E. End-tidal CO2 detection may help deter- B. Bicarbonate infusion
tp

mine the quality of cardiopulmonary resus- C. Physostigmine


citation (CPR) and return of spontaneous D. Intravenous corticosteroids
circulation. E. High-dose insulin therapy
ht

Chapter 4 Answers
1. ANSWER: D. Respiratory failure and death times in the placebo-treated group; deaths resulting
In the Salmeterol Multicenter Asthma Research from asthma occurred 13 times in the salmeterol-
Trial (SMART), approximately 26,000 patients were treated group versus 3 times in the placebo-treated
randomly assigned to receive either inhaled salmeterol group (both diferences were statistically signiicant).
or placebo for 6 months, added to their “usual care.” he explanation for these adverse outcomes remains
Study outcomes included deaths and near-deaths uncertain. he possibility that this efect would be
(respiratory failure requiring admission to an inten- eliminated if all patients used an inhaled corticoste-
sive care unit) resulting from asthma. Although these roid at the same time that they received a long-acting
events were rare, they occurred more commonly in beta-agonist bronchodilator or placebo is currently
the group randomized to receive salmeterol than in a being investigated. he SMART study of adverse
placebo group. Episodes of respiratory failure occurred outcomes related to salmeterol revealed no increase
37 times in the salmeterol-treated group versus 22 in serious atrial or ventricular arrhythmias (Answers
88 C HA P T E R 4 Pulmonary and Critical Care Medicine

A and B), cardiovascular events (heart attacks or would be expected in a patient with a long smok-
strokes) (Answer C), or drug-induced allergic events ing history and evidence of airlow obstruction on
(angioedema or anaphylaxis) (Answer E). Of note, the examination.
increased mortality found with use of long-acting beta Vestbo J, Hurd SS, Agustí AG, et  al. Global strategy for the
agonists alone has not been found in studies compar- diagnosis, management, and prevention of chronic obstruc-
ing long-acting beta agonist/inhaled steroid combina- tive pulmonary disease: GOLD executive summary. Am J
tion inhalers with inhaled steroids alone. Respir Crit Care Med. 2013;187(4):347–365.
Nelson HS, Weiss ST, Bleecker ER, et al. he Salmeterol Multi-
center Asthma Research Trial: a comparison of usual pharmaco- 3. ANSWER: A. he diagnosis is established by a blood
therapy for asthma or usual pharmacotherapy plus salmeterol. test.
[Published erratum appears in Chest. 2006;129(5):1393.] he presence of widespread (panlobular) emphysema
Chest. 2006;129(1):15–26. in a nonsmoker should raise the possibility of alpha-1
Stempel DA, Raphiou IH, Kral KM, et  al. Serious asthma antitrypsin deiciency; other suggestive indings include
events with luticasone plus salmeterol versus luticasone COPD that develops in a cigarette smoker at an unusu-
alone. N Engl J Med. 2016;374(19):1822–1830. ally young age (e.g., before age 50) or a strong family

t
history of emphysema. However, many patients with

ne
2. ANSWER: A. FEV1 2.0 L (66% of predicted), FVC 4.0 homozygous alpha-1 antitrypsin deiciency do not have
L (100% of predicted), FEV1/FVC 0.5 (66% of pre- these “classic” characteristics, and some experts recom-
dicted)

e.
mend widespread testing for alpha-1 antitrypsin dei-
Let us ofer a simple strategy for interpreting spirom- ciency among patients with COPD. Testing for alpha-1
etry results. After you have checked to see if the test has antitrypsin begins with measurement of serum alpha-1

in
been properly performed, with a smooth graphic dis- antitrypsin level, a routine blood test done in the chemis-
play (volume–time plot) and at least 6 seconds of forced try laboratory. Patients who are homozygous for alpha-1

ic
expiratory time, look at the FEV1/FVC ratio. If this antitrypsin deiciency will have a very low blood level of
ratio is reduced (as in example A), there is an obstructive ed the protein, on the order of 15% of normal. his ind-
pattern. A reduced ratio is typically deined as a value ing relects the fact that most patients with alpha-1 anti-
below the 95% conidence interval (not given in these trypsin deiciency make an alpha-1 antitrypsin protein
examples). he Global Initiative for Chronic Obstruc- that is transported inefectively out of the liver, rather
sm
tive Lung Disease (GOLD) guidelines attempt to sim- than making no alpha-1 antitrypsin protein (the homo-
plify this deinition by using a cutof for a reduced ratio zygous null/null genotype is rare). Further testing of the
of <0.7. his cutof is generally accurate except for all patient with a very low serum alpha-1 antitrypsin level
ok

but the oldest of patients, in whom an FEV1/FVC ratio can be performed by protein electrophoresis or genetic
<0.7 may still be normal. (he FEV1/FVC ratio declines analysis to determine the speciic genetic abnormality.
with age because of loss of lung elastic recoil during the Patients with heterozygous alpha-1 antitrypsin abnor-
bo

normal aging process.) If the FEV1/FVC ratio is normal mality will have low serum alpha-1 antitrypsin levels
or increased, there is no obstruction. (often on the order of approximately 50% of normal).
Having established the presence of airlow obstruc- here is controversy regarding whether heterozygous
e

tion based on a reduced FEV1/FVC ratio, one can judge persons are at a modest increased risk for developing
the severity of the obstruction by looking at the FEV1
://

COPD. In any case, a blood level less than the lower lim-
expressed as a percentage of normal. By widely accepted its of normal does not establish homozygous deiciency
convention, in this context, an FEV1 80%–99% of nor- (Answer B). It is estimated that as many as 30% of non-
tp

mal indicates mild obstruction; 50%–79% of normal smoking persons with alpha-1 antitrypsin deiciency
indicates moderate obstruction; 35%–49% of normal will never develop obstructive lung disease (Answer C).
ht

indicates severe obstruction; and <35% of normal indi- he percentage of adult patients with alpha-1 antitryp-
cates very severe obstruction. he patient in example A sin deiciency who develop clinical evidence of liver dis-
has moderate airlow obstruction. he degree to which ease (speciically those with ZZ phenotype and other
the FEV1 is reduced (in the absence of an acute exacerba- forms in which abnormal protein accumulates in the
tion) correlates directly with disease morbidity and mor- liver) is estimated to be approximately 20% and varies
tality in chronic obstructive pulmonary disease (COPD). with age (Answer D). Emphysema due to alpha-1 anti-
he pattern in Answer B suggests severe restric- trypsin deiciency, such as emphysema due to smoking
tion; Answer C shows normal spirometry; and in in persons with a normal alpha-1 antitrypsin level, is
Answer D, the results suggest a pattern of moder- associated with a reduced difusing capacity of the lung
ate restriction. Restriction is conirmed with full for carbon monoxide due to impaired transfer of carbon
measurement of lung volumes, including total lung monoxide across decreased alveolar-capillary membrane
capacity, functional residual capacity, and residual surface area (Answer E).
volume, as measured by the helium dilution tech- Stoller JK, Aboussouan LS. A review of α1-antitrypsin dei-
nique or plethysmography. Obstructive lung disease ciency. Am J Respir Crit Care Med. 2012;185(3):246–259.
CHAPTER 4 Pulmonary and Critical Care Medicine 89

4. ANSWER: D. “Given your history, a leukotriene Vestbo J, Hurd SS, Agustí AG, et  al. Global strategy for the
modiier such as montelukast, zairlukast, or zileu- diagnosis, management, and prevention of chronic obstruc-
ton might be particularly helpful as treatment for tive pulmonary disease: GOLD executive summary. Am J
your asthma.” Respir Crit Care Med. 2013;187(4):347–365.
Aspirin-sensitive asthma (also called aspirin-intoler-
ant asthma and, most recently, aspirin-exacerbated respi- 6. ANSWER: B. Chest tube drainage of pleural space,
ratory disease) occurs in approximately 3%–5% of adults with thoracoscopic lysis of pleural adhesions if nec-
with asthma. It is exceedingly rare in children and does essary
not have a familial predisposition. Although the precise he clinical scenario of fever, productive cough, chest
biochemical abnormality leading to asthmatic exacer- pain, and leukocytosis is consistent with an acute pneu-
bations after aspirin ingestion is unknown, it involves monia and parapneumonic efusion. he efusion is
some aspect of the cyclooxygenase pathway, and spe- moderate to large and appears to be loculated, based on
ciically cyclooxygenase-1. hus any cyclooxygenase-1 diiculty aspirating more than 60 mL of luid during tho-
inhibitor, including ibuprofen, naproxen, and other racentesis. he pleural luid is serous without frank pus or
nonsteroidal antiinlammatory drugs (NSAIDs), can bacteria in the pleural space; that is, there is no evidence

t
of an empyema. In this context, the pleural luid pH is

ne
precipitate an attack and need to be avoided (Answer
B). Acetaminophen and speciic cyclooxygenase-2 helpful. he “normal” pleural luid pH is >7.4. A value of
inhibitors (e.g., celecoxib) have weak cyclooxygenase-1 7.2 or less indicates a “complicated” parapneumonic efu-

e.
inhibition and are safe to use in most patients. he sion, one that is likely to behave like an empyema, with
dose of aspirin that triggers an asthmatic reaction varies formation of adhesions and diiculty clearing the infec-
tion without drainage of the pleural space. Together with

in
among aspirin-sensitive asthmatics, but some patients
will respond to 81 mg or less (Answer A). the rest of the clinical picture, the need for chest tube
drainage of the pleural space and possibly surgical lysis of

ic
A feature of patients with aspirin-sensitive asthma is
that they synthesize more than the normal amount of adhesions and decortication of the pleura are indicated. It
cysteinyl leukotrienes at baseline and markedly excess ed is thought that in this setting the low pleural luid pH is
amounts after ingestion of aspirin or other cyclooxygen- caused by anaerobic metabolism of glucose by infection
ase-1 inhibitors. As a result, it is logical to try antileu- (not found in the sampled luid) and inlammation in
the pleural space, leading to generation of carbon dioxide
sm
kotriene therapy (either a leukotriene receptor antagonist
or lipoxygenase inhibitor) to treat their asthma (but not and lactate.
to make aspirin or other NSAID ingestion safe) (Answer Like managing an empyema, antibiotics alone with-
D). he reaction to aspirin/NSAIDs is not IgE mediated out pleural drainage will have a low likelihood of suc-
ok

and is not associated with IgE-mediated allergic reactions, cess (Answer A). Pleural biopsy might be considered if
such as food allergy to nuts (Answer C). Aspirin sensitiv- malignancy or tuberculosis were high in the diferential
ity is not associated with an increased risk of adverse reac- diagnosis, which they are not (Answer C). A history
bo

tions to beta-agonist bronchodilators (Answer E). of recurrent vomiting or recent upper endoscopy with
severe retrosternal or upper abdominal pain might sug-
5. ANSWER: B. Stage 1C gest esophageal rupture with spillage of gastric luid into
e

he latest GOLD classiication of COPD uses the pleural space. A high pleural luid amylase is typical
://

severity of FEV1 impairment, number of exacerba- of this syndrome (Answer D). A low pleural luid pH
tions, and dyspnea scores to classify patients. may be seen in pleural efusions due to collagen vascular
disease, especially in rheumatoid efusions, but the clin-
tp

Group A: Mild (GOLD 1, FEV1 >80% predicted)


to moderate (GOLD 2, FEV1 50%–80% predicted) ical history does not support this diagnosis (Answer E).
airlow limitation, 0 to 1 exacerbation per year, and
ht

low scores on dyspnea scales (either CAT or mMRC 7. ANSWER: A. Lymphangioleiomyomatosis (LAM)
scales). Chylothorax is typically milky in appearance (white
Group B: Mild or moderate airlow limitation, 0 to to beige in color) and (in the absence of cirrhosis) has
1 exacerbation per year, but higher scores on dyspnea a high triglyceride concentration (>110 mg/dL). he
scales. diagnosis of chylothorax can be conirmed by identiica-
Group C: Severe (GOLD 3, FEV1 <50% predicted) tion of chylomicrons in the pleural luid on lipoprotein
or very severe (GOLD 4, <30% predicted) airlow lim- electrophoresis. Chylothorax results from disruption of
itation or >2 exacerbations per year or one hospitaliza- the low of chyle from the small intestines through the
tion, with low dyspnea scale scores. thoracic duct into the left brachiocephalic vein. Causes
Group D: Severe or very severe airlow limitation, of thoracic duct obstruction or rupture include the pro-
>2 exacerbations per year or one hospitalization, plus liferation of tumor-like smooth muscle cells in LAM;
high dyspnea scale scores. malignancy such as lymphoma; radiation ibrosis; and
Based on his exacerbation history, this patient direct surgical trauma. he other choices above—para-
would fall into GOLD stage 1C. pneumonic efusion (Answer B), malignant efusion
90 C HA P T E R 4 Pulmonary and Critical Care Medicine

(Answer C), pulmonary infarction (Answer D), and Cryptogenic organizing pneumonia (Answer D),
sarcoidosis (Answer E)—typically cause an exudative also referred to as bronchiolitis obliterans organizing
pleural efusion with low lipid content. pneumonia (BOOP), mimics infectious pneumonia,
with areas of consolidation on chest imaging. Trans-
8. ANSWER: C. Idiopathic pulmonary ibrosis bronchial lung biopsy may identify areas of organiz-
his patient presents with a typical history for chronic ing pneumonia and occasionally the endobronchiolar
interstitial lung disease. He has progressive dyspnea, a polypoid tissue of bronchiolitis obliterans. Finally,
nonproductive cough, crackles on chest examination, Langerhans cell histiocytosis (Answer E) presents with
and restriction on pulmonary function testing. His chest cystic lung disease and associated small lung nodules;
x-ray is conirmatory, with bilateral linear and nodular on immunohistochemical staining of bronchoalveolar
opacities, the radiographic correlate of an interstitial pro- lavage luid, one may be able to identify the character-
cess. he chest CT scan shows a pattern that is suggestive istic Langerhans cells.
of the pathologic process called usual interstitial pneumo-
nitis (UIP). In particular, there are dense bands of opacity 9. ANSWER: A. Normal oxygen saturation at rest that
suggesting ibrosis with associated traction bronchiecta- falls to 82% with exertion

t
ne
sis (bronchial walls pulled apart by the retraction of scar Characteristic of the interstitial lung diseases is oxy-
formation) and honeycombing (dilated alveolar spaces, gen desaturation with exercise. he alveolar-to-arterial
likewise enlarged by the retractive forces from surround- gradient for oxygen widens with exertion for two

e.
ing scar formation). he peripheral and basilar predomi- reasons. In part, there is likely worsened ventilation/
nance are consistent with UIP. perfusion mismatching when minute ventilation and

in
Potential causes of this pathologic process include cardiac output increase. In part, difusion impairment
collagen vascular diseases, pneumoconioses, and medi- is made manifest when cardiac output increases, cir-

ic
cations such as bleomycin and nitrofurantoin. In the culation time decreases, and the transit time for red
absence of an identiiable cause or association, the blood cells in alveolar capillaries shortens, limiting the
diagnosis is idiopathic pulmonary ibrosis (IPF). Of
note, clubbing is found in as many as 50% of patients
ed amount of oxygen that can difuse across thickened
alveolar-capillary membranes. In the patient with idio-
with IPF. In this case example, bronchoscopy was per- pathic pulmonary ibrosis or asbestosis, as described in
sm
formed to exclude a potential infectious etiology, with this case, one might ind a normal oxygen saturation at
results that were nonspeciic, consistent with but not rest, but oxygen saturation characteristically decreases
diagnostic of UIP. with exertion (Answer A). In IPF and other difuse
Neither the radiographic indings nor the trans- interstitial lung diseases, the difusion capacity of the
ok

bronchial lung biopsy results were suggestive of alter- lung for carbon monoxide (DLCO) is decreased; lungs
native diagnoses. Sarcoidosis (Answer A) is typically are stifer than normal and so exhibit reduced lung
bronchocentric in location, generally with an upper compliance (a measure of the change in lung volume
bo

lobe predominance, and often associated with hilar and for any change in transpulmonary pressure); and lung
mediastinal lymphadenopathy. Transbronchial lung volumes are reduced, characteristic of restrictive pul-
biopsy will often reveal noncaseating granulomas. Like monary processes
e

IPF, nonspeciic interstitial pneumonia (Answer B) is an


://

idiopathic chronic inlammatory lung disease. It usually 10. ANSWER: E. Pirfenidone


presents with areas of ground-class opacities on a chest Recent studies suggest that both pirfenidone, an
tp

CT scan (Fig. 4.10), and transbronchial lung biopsy antiibrotic agent, and nintedanib, a tyrosine kinase
indings are often nonspeciic, as in this case. inhibitor, may slow disease progression in IPF. Supple-
mental oxygen should be given to chronically hypox-
ht

emic patients, and some patients may be eligible for


lung transplant.
High-dose steroids (Answer A) are often used as a
“therapeutic trial” to treat chronic idiopathic inlam-
matory lung disease when the diagnosis is uncertain.
However, there is no role for high-dose systemic
steroids when the diagnosis of idiopathic pulmo-
nary ibrosis has been established. A randomized,
controlled clinical trial of high-dose steroids, aza-
thioprine, and N-acetylcysteine (Answer B) versus
placebo in patients with idiopathic pulmonary ibro-
sis, referred to as the PANTHER-IPF trial, was ter-
minated early when it was found that mortality was
greater in the group randomized to receive steroids,
• Fig. 4.10 Ground-class opacities on chest CT scan (Answer 8). azathioprine, and N-acetylcysteine than in the placebo
CHAPTER 4 Pulmonary and Critical Care Medicine 91

group. Rituximab (Answer C), a monoclonal antibody Gottlieb DJ, Yenokyan G, Newman AB, et al. Prospective study
directed at the CD20 protein found on the surface of of obstructive sleep apnea and incident coronary heart dis-
B cells, is used to treat granulomatosis with polyan- ease and heart failure: the sleep heart health study. Circula-
giitis (Wegener granulomatosis) and may be efective tion. 2010;122(4):352–360.
in interstitial lung disease associated with rheumatoid Somers VK, White DP, Amin R, et  al. Sleep apnea and car-
diovascular disease: an American Heart Association/Ameri-
arthritis; it is not known to be efective in the treat-
can College of Cardiology Foundation Scientiic Statement
ment of idiopathic pulmonary ibrosis. Hydroxychlo- from the American Heart Association Council for High
roquine (Answer D) is used to treat collagen vascular Blood Pressure Research Professional Education Commit-
disease, especially rheumatoid arthritis, but it is not a tee, Council on Clinical Cardiology, Stroke Council, and
therapy for chronic inlammatory lung disease. Council on Cardiovascular Nursing. J Am Coll Cardiol.
Idiopathic Pulmonary Fibrosis Clinical Research Network. 2008;52(8):686–717.
Prednisone, azathioprine, and N-acetylcysteine for pulmo- Tregear S, Reston J, Schoelles K, Phillips B. Obstructive sleep
nary ibrosis. N Engl J Med. 2012;366(21):1968–1977. apnea and risk of motor vehicle crash: systematic review and
King Jr TE, Bradford WZ, Castro-Bernardini S, et al. A phase meta-analysis. J Clin Sleep Med. 2009;5(6):573–581.
3 trial of pirfenidone in patients with idiopathic pulmonary

t
ibrosis. [Published erratum appears in N Engl J Med. 2014;

ne
13. ANSWER: C. Relapsing polychondritis
371(12):1172.] N Engl J Med. 2014;370(22):2083–2092. Relapsing polychondritis, the rarest of the collagen
Richeldi L, du Bois RM, Raghu G, et  al. Eicacy and safety vascular diseases listed, is associated with airway dis-
of nintedanib in idiopathic pulmonary ibrosis. [Published

e.
ease, including tracheomalacia and subglottic stenosis,
erratum appears in N Engl J Med. 2015;373(8):782.] N Engl
J Med. 2014;370(22):2071–2082.
but not with difuse interstitial lung disease. he other
choices are all associated with interstitial lung disease.

in
In addition, scleroderma is associated with pulmonary
11. ANSWER: A. Retrognathia hypertension and with esophageal and sometimes oro-

ic
Persons with a narrowed posterior pharyngeal pharyngeal dysfunction with an increased risk of aspi-
opening are at increased risk for critical narrowing and ed ration pneumonia. Systemic lupus erythematosus can
occlusion of the upper airway during sleep. he poste- cause pleural efusions, difuse alveolar hemorrhage,
rior pharynx may be narrowed due to a large tongue, and “shrinking lung syndrome,” possibly due to dia-
posteriorly extending soft palate, large tonsils, or pos- phragmatic weakness. Mixed connective tissue disease
sm
teriorly positioned jaw and tongue (retrognathia). A may also cause pulmonary hypertension.
large neck circumference (>16 inches in women and
>17 inches in men) correlates with excess fatty deposi- 14. ANSWER: C. Central apneas
ok

tion in the upper airway (Answer B). Nasal obstruc- he case history is a classic description of obesity
tion, such as with large nasal polyps (Answer C), can hypoventilation syndrome (OHS), previously referred to
aggravate preexisting obstructive sleep apnea, but it is as pickwickian syndrome, deined as the combination of
bo

not a risk factor for the presence of obstructive sleep morbid obesity (BMI >30 kg/m2), Pco2 >45 mm Hg,
apnea. Short stature in the absence of obesity is not and sleep-disordered breathing in the absence of other
a risk factor for sleep-disordered breathing (Answer causes of alveolar hypoventilation. his morbidly obese
e

D). Insomnia (Answer E) may be a manifestation of young man has evidence of chronic hypoxemia causing
obstructive sleep apnea syndrome, but it is nonspe-
://

secondary polycythemia, pulmonary hypertension, and


ciic and not closely associated with the presence of cor pulmonale. He also has daytime hypercapnia, which,
obstructive sleep apnea. in the absence of known lung disease, is likely due to cen-
tp

tral hypoventilation. In some cases of OHS, prolonged


12. ANSWER: D. Motor vehicle accidents obstructive apneas (with associated acute hypercapnia)
ht

Persons with obstructive sleep apnea (OSA) have a without adequate compensatory hyperventilation follow-
sevenfold increased risk of being involved in a motor ing these apneas can lead to daytime hypercapnia. Con-
vehicle accident as a consequence of their associated tinuous positive airway pressure (CPAP) may successfully
daytime hypersomnolence. Strong epidemiologic evi- treat OHS in these instances. In other patients, central
dence indicates that OSA is also an independent risk apneas persist despite CPAP therapy, relecting a primary
factor for adverse cardiovascular events. hese include abnormality of ventilatory drive (Answer A).
heart attacks and strokes as well as heart failure. In addi- Orthodeoxia (Answer B) refers to a decrease in arterial
tion, OSA is a cause of hypertension, likely as a result of oxygenation in the upright position that improves when
repetitive sympathetic stimulation associated with epi- lying supine; it is the objective manifestation of the symp-
sodic hypoxemia and sudden awakenings, and evidence tom of platypnea (dyspnea in the upright position that
indicates that treatment of OSA results in a meaningful lessens when supine). Intracardiac right-to-left shunts and
lowering of blood pressure. OSA does not increase the intrapulmonary arteriovenous shunts at the lung bases
risk of aspiration (Answer A), laryngospasm (Answer may cause orthodeoxia. Morbid obesity is more likely to
B), diabetic ketoacidosis (Answer C), or hypoxemic cause the opposite positional efects, that is, orthopnea
respiratory failure (Answer E). and worsened hypoxemia when supine. Nothing in the
92 C HA P T E R 4 Pulmonary and Critical Care Medicine

history suggests the presence of restless legs syndrome seen in all difuse obstructive lung diseases and is not
(Answer D), characterized by repetitive readjustment of diagnostic of any speciic disease. On the other hand,
the positioning of the legs while awake and jerking move- a large tracheal tumor causing upper airway narrow-
ments of the legs while asleep. A shortened (rather than ing results in low at a single, ixed rate throughout
prolonged) sleep latency period (Answer E) is a nonspe- a large portion of exhalation and again during inha-
ciic marker of sleepiness and is found in many sleep- lation. he resulting appearance on the low–volume
related disorders, including narcolepsy. loop is distinctive: a horizontal portion (referred to as
Chau EH, Lam D, Wong J, et  al. Obesity hypoventila- a “plateau” pattern) of the expiratory and inspiratory
tion syndrome: a review of epidemiology, pathophysi- curves. Although not true of Answer E, often the rate
ology, and perioperative considerations. Anesthesiology. of low is similar on expiration and inspiration at the
2012;117(1):188–205. midportion of the curves (50% of the vital capacity),
unlike in difuse intrathoracic airlow obstruction, in
15. ANSWER: B. Blood gas on room air which inspiratory low at 50% of the vital capacity
Obesity hypoventilation syndrome is marked by typically exceeds expiratory low (e.g., Fig. 4.11). Nor-
daytime hypoventilation and is often associated with mal low–volume curves (Answer A) or low–volume

t
ne
nocturnal sleep apnea. he daytime hypoxemia in an curves with a pattern suggesting restriction (Answer
obese patient without signiicant evidence of intrinsic B) would not be expected in the presence of a large
lung disease suggests obesity hypoventilation, which is tracheal tumor.

e.
diagnosed with a room air blood gas showing hyper-
capnia. A chest x-ray or chest CT might be done, but 17. ANSWER: B. Serum immunoglobulin G
with a normal pulmonary exam and only mild restric-

in
Many cases of bronchiectasis are “idiopathic,” pre-
tion on pulmonary function tests (likely from body sumably the result of a prior necrotizing infection
habitus), the yield will be lower. A CBC might show

ic
that caused irreversible damage to the airway walls. In
an elevated hemoglobin resulting from chronic hypox- other instances, however, an underlying predisposition
emia, but it cannot conirm the diagnosis. A 6-minute ed can be identiied. Among the potential conditions that
walk test would demonstrate the degree of desatura- may predispose to the development of bronchiectasis
tion with exercise but would again not be diagnostic. are hypogammaglobulinemia and primary ciliary dys-
Bilevel positive airway pressure with a manda-
sm
kinesia (also called immotile cilia syndrome). he former
tory backup respiratory rate applied during sleep in is typically treated with intravenous gamma-globulin
patients with obesity hypoventilation syndrome that replacement therapy, which would be expected to
fails to improve with CPAP can correct or amelio- slow the progression of the disease. he latter (pri-
ok

rate sleep-related hypercapnia and improve daytime mary ciliary dyskinesia), which is diagnosed by elec-
gas exchange as well. he latter “spillover” efect may tron microscopic examination of nasal or bronchial
be due to a reduction in the renal compensation for epithelial tissue and associated ciliary ultrastructure
bo

hypercapnia with less blunting of the ventilatory drive (Answer D), does not have therapeutic options that
by metabolic alkalosis. Daytime hypersomnolence will would alter treatment of lares at this time. Pulmonary
also improve. Supplemental oxygen will address the vasculitides, such as granulomatosis with polyangiitis
e

hypoxemia and its consequences, but it may worsen


://

the patient’s hypoventilation and does not address the


underlying disease pathophysiology. 8
tp

7
16. Answer: E. Fig. 4.6 Flow–volume loop
he low–volume loop displays the results of the 6
ht

same maximal forced expiratory maneuver used to 5


generate the spirogram. Instead of displaying the
4
results as a plot of volume versus time as in spirometry, FEV1
Flow liters/s

it displays low (vertical axis) versus volume (horizon- 3


tal axis). It also typically adds display of low versus 2
volume during completion of a maximal inspiration
1 FVC
performed immediately after maximal exhalation. he
expiratory limb of the low–volume curve is displayed 0
above the horizontal “zero low” line; the inspira- 1
1 2 3 4 5
tory limb is displayed below the “zero low” line; and
together they form the low–volume loop. 2
Characteristic of the low–volume curves in asthma 3
and COPD is the scooped, concave appearance of the
expiratory curve (Answers C and D), relecting difuse 4 Volume liters
intrathoracic airway narrowing. his appearance is • Fig. 4.11 Diffuse intrathoracic airlow obstruction (Answer 16).
CHAPTER 4 Pulmonary and Critical Care Medicine 93

(Wegener syndrome) or eosinophilic granulomatosis (Fig. 4.14), honeycombing (Answer C) (Fig. 4.15),
with polyangiitis (Churg-Strauss syndrome), do not typi- and atelectasis (Answer E) (Fig. 4.16) are shown, each
cally cause bronchiectasis, making assay for antinuclear with a distinctive and very diferent appearance than
cytoplasmic antibodies (ANCA) associated with vascu- “tree-in-bud” nodules.
litis unhelpful (Answer A). Celiac disease (Answer C) Eisenhuber E. he tree-in-bud sign. Radiology. 2002;222(3):
is associated with idiopathic pulmonary hemosiderosis 771–772.
(called Lane-Hamilton disease) but not bronchiectasis.
he etiology of bronchiectasis cannot be discerned by
detailed characterization of its physiologic consequences
on pulmonary function testing (Answer E).
Knowles MR, Daniels LA, Davis SD, et al. Primary ciliary dys-
kinesia. Recent advances in diagnostics, genetics, and char-
acterization of clinical disease. Am J Respir Crit Care Med.
2013;188(8):913–922.
McShane PJ, Naureckas ET, Tino G, Strek ME. Non-cystic ibro-

t
sis bronchiectasis. Am J Respir Crit Care Med. 2013;188(6):

ne
647–656.

18. ANSWER: D. Tree-in-bud nodules

e.
he CT image displayed in Question 18 has the
following abnormality: small lung nodules (2–3 mm

in
in diameter) grouped around peripheral blood ves-
sels in the lung periphery. he image is suggestive of

ic
some trees in early spring, with their buds (the “nod-
ules”) sprouting along the distal branches before leaves ed
appear (Fig. 4.12).
Pathologically, the nodules on CT imaging cor- • Fig. 4.13 Ground-glass opacity (Answer 18).
respond to an inlammatory reaction, often granu-
sm
lomatous, around bronchioles (which run in parallel
to peripheral vessels). he nodules are the pathologic
correlate of a bronchiolitis or peribronchiolitis, such
ok

as may be seen in sarcoidosis, mycobacterial infection,


aspiration, and other conditions.
Examples of ground-glass opacity (Answer A) (Fig.
bo

4.13), consolidation with air bronchograms (Answer B)


e
://
tp
ht

• Fig. 4.14 Consolidation with air bronchograms (Answer 18).

• Fig. 4.12 Tree-in-bud nodules (Answer 18). • Fig. 4.15 Honeycombing (Answer 18).
94 C HA P T E R 4 Pulmonary and Critical Care Medicine

19. ANSWER: A. Alpha-1 antitrypsin deiciency for myocardial infarction based on his negative cardiac
he radiographic appearance of alpha-1 antitrypsin stress test). he next step in his management should
deiciency is that of emphysema, with hyperinlation (as be referral to a thoracic surgeon for surgical excision of
demonstrated by large lung volumes and lattened dia- the lung nodule. Surgical resection not only provides
phragms) and hyperlucency (due to a decrease in the nor- deinitive diagnosis but also, if this nodule is indeed a
mal vascular markings) of the lung ields. In some cases lung cancer, ofers the best chance for cure.
of emphysema due to alpha-1 antitrypsin deiciency, a he argument against attempting to establish a pre-
distinctive inding is predominance of emphysema in the operative diagnosis with iberoptic bronchoscopy and
lower lung ields (rather than the typical predisposition transbronchial lung biopsy (Answer B) or transthoracic
for the upper lobes). In the chest x-ray shown in Ques- needle aspirate/biopsy (Answer C) is the following. If a
tion 19, one can see blood vessels in the upper halves diagnosis of lung cancer is conirmed, the patient will
of the lungs bilaterally but few in the lower halves. he then be referred for surgical resection of the nodule.
cause of the basilar predominance of emphysema in some If the sample returns with nonspeciic indings (“no
patients with alpha-1 antitrypsin deiciency is unknown. malignant cells seen”), there remains an unacceptably
he chest x-ray does not have the bilateral upper high chance (10% or greater) of a false-negative result.

t
ne
lobe opacities consistent with ankylosing spondylitis he likelihood of the sample providing a speciic diag-
(Answer B); upper lobe-predominant linear mark- nosis of a benign lesion, such as a histoplasmoma, is
ings in the orientation of bronchi, with oval cystic small, estimated at 5% or less.

e.
spaces and hyperinlation, suggestive of cystic ibrosis In the absence of suggestive symptoms or a known
(Answer C); multiple lung nodules, potentially some extrathoracic primary malignancy, the likelihood that

in
with cavitation, as may be seen in granulomatosis with this nodule represents a solitary pulmonary metasta-
polyangiitis (Wegener granulomatosis) (Answer D); or sis is low, especially with the PET scan indicating no

ic
hyperinlation with difuse, thin-walled cystic spaces abnormal areas of increased glucose uptake outside the
typical of lymphangioleiomyomatosis. chest. Workup for an alternative primary malignancy

20. ANSWER: D. Surgical resection of the lung nodule


ed from which this nodule might be a solitary pulmonary
metastasis (Answer A) is unnecessary beyond the results
A new (over the course of 1 year), PET-positive of the PET scan. Monitoring for growth of the nod-
sm
lung nodule in a middle-aged cigarette smoker is ule before proceeding with surgical excision (Answer E)
highly suspicious for lung cancer. Benign etiologies may be appropriate for small or PET-negative nodules,
are possible, including benign tumors, infections, and but in this example, the probability of lung cancer is
inlammatory reactions, but the risk of lung cancer is suiciently high that delay in excision poses an unneces-
ok

high, especially given the relatively large size of the sary risk of growth and spread of malignancy.
nodule (>1 cm in diameter). he evaluation described Gelbman BD, Cham MD, Kim W, et  al. Radiographic and
in the case history serves to exclude obvious evidence clinical characterization of false negative results from CT-
bo

for metastatic disease (based on the absence of other guided needle biopsies of lung nodules. J horac Oncol.
areas of increased uptake on the PET scan) and contra- 2012;7(5):815–820.
indications to surgical resection (adequate ventilatory
e

reserve based on his spirometry results and low risk 21. ANSWER: B. Fluticasone
://

A common side efect of inhaled corticosteroids is


hoarse voice or dysphonia (a change in voice quality).
tp

he precise pathophysiology is uncertain, but it likely


is the consequence of deposition of corticosteroid onto
the larynx, with consequent irritative efect, mucosal
ht

thinning, or muscular atrophy. he dysphonia tends


to be intermittent and fully reversible with cessation
of medication. It cannot be prevented by rinsing the
posterior pharynx or hypopharynx after each use of the
medication. Sometimes this unpleasant side efect can
be ameliorated by addition of a valved holding cham-
ber (“spacer”) to the metered-dose inhaler being used to
deliver the inhaled steroid or changing the delivery sys-
tem (e.g., from a dry-powder inhaler to a metered-dose
inhaler with spacer). he inhaled steroid ciclesonide is
released as a prodrug that is activated by esterases along
the bronchial mucosa. It is said to have fewer oropha-
ryngeal (and possibly laryngeal) side efects as a result
• Fig. 4.16 Atelectasis (Answer 18). of this unique mechanism of activation.
CHAPTER 4 Pulmonary and Critical Care Medicine 95

Although any inhaled medication can potentially such as with bilevel positive airway pressure, can reduce
cause occasional hoarseness, none of the other choices the need for intubation with mechanical ventilation in
(salmeterol, tiotropium, albuterol, or ipratropium) severe exacerbations of COPD (Answer C).
commonly cause this problem. de Jong YP, Uil SM, Grotjohan HP, et  al. Oral or IV pred-
nisolone in the treatment of COPD exacerbations: a
22. ANSWER: C. She has ulcerative colitis. randomized, controlled, double-blind study. Chest.
Bronchiolitis obliterans is a potential cause of severe 2007;132(6):1741–1747.
obstructive lung disease and should be considered in a Leuppi JD, Schuetz P, Bingisser R, et  al. Short-term vs con-
ventional glucocorticoid therapy in acute exacerbations of
patient without other common causes of chronic air-
chronic obstructive pulmonary disease: the REDUCE ran-
low obstruction, such as asthma, cigarette smoking, domized clinical trial. JAMA. 2013;309(21):2223–2231.
or genetic predisposition to emphysema (alpha-1 anti- Niewoehner DE, Erbland ML, Deupree RH, et  al.
trypsin deiciency). Physical indings may include evi- Efect of systemic glucocorticoids on exacerbations of
dence for pulmonary hyperinlation and sometimes an chronic obstructive pulmonary disease. N Engl J Med.
inspiratory squeak. Findings on chest CT scan may be 1999;340(25):1941–1947.
few, sometimes with areas of mosaic oligemia or tree-

t
Quon BS, Gan WQ, Sin DD. Contemporary management

ne
in-bud nodularity. Airlow obstruction is often poorly of acute exacerbations of COPD: a systematic review and
reversible to bronchodilators or corticosteroids. metaanalysis. Chest. 2008;133(3):756–766.
Bronchiolitis obliterans may be immune mediated,

e.
as a pulmonary manifestation of ulcerative colitis, 24. ANSWER: E. No therapy
rheumatoid arthritis, or graft-versus-host disease fol- his case history strongly points to a diagnosis of stage 1

in
lowing bone marrow transplant or rejection following pulmonary sarcoidosis. he patient has a mild, dry cough
lung transplant. A poorly ventilated oice building but is otherwise asymptomatic. No treatment is neces-

ic
is not a cause of bronchiolitis obliterans (Answer A). sary. he rate of spontaneous remission of his disease over
Chronic exposure to parrots (Answer B) may cause the next 2 years is approximately 80%. Treatment with
hypersensitivity pneumonitis, a restrictive lung dis-
ease. Mixed connective tissue disease (Answer D) can
ed systemic steroids (Answers A and B) may hasten resolu-
tion, but at an unacceptably high cost of medication side
cause interstitial lung disease with ibrosis, likewise a efects. Importantly, there is no evidence to suggest that
sm
cause of restriction on pulmonary function testing. early treatment of sarcoidosis prevents long-term progres-
he peripheral lung scarring that results from radia- sion or sequelae of the disease. Consequently, there is no
tion therapy for localized breast cancer (Answer E) justiication for treating an asymptomatic or minimally
typically has no impact on pulmonary function. symptomatic patient, whose disease has a high rate of
ok

spontaneous remission, with a medication that entails


23. ANSWER: D. Start antibiotics because the patient frequent short-term and long-term adverse side efects.
has a change in sputum quantity and character. Hydroxychloroquine (Answer C) has been used to
bo

he bacterial pathogens causing exacerbations of treat cutaneous sarcoidosis, including erythema nodo-
COPD are most often Streptococcus pneumoniae, Hae- sum, but it is not efective in most cases of pulmonary
mophilus inluenzae, and Moraxella catarrhalis, bacteria sarcoidosis. Methotrexate (Answer D) is a potential
e

that can be treated with a variety of antibiotics cho- steroid-sparing agent in the treatment of severe pul-
://

sen empirically (including macrolides, cephalosporins, monary sarcoidosis.


amoxicillin/clavulanate, trimethoprim-sulfamethoxa- Baughman RP, Lower EE. Treatment of sarcoidosis. Clin Rev
tp

zole, doxycycline, and quinolones). Empiric treatment Allergy Immunol. 2015;49(1):79–92.


of exacerbations of COPD without need for sputum
Gram stain and culture has proven efective in numer- 25. ANSWER: B. Acute metabolic alkalemia due to
ht

ous clinical trials. Although many lares of COPD are “unmasking” of compensatory chronic metabolic
triggered by events other than bacterial infection, omis- alkalosis
sion of antibiotics when there is evidence for increased In patients with chronic hypercapnia, such as from
cough, sputum purulence, and/or fever results in less COPD with severe airlow obstruction, the sudden
frequent resolution and more frequent deterioration reversal of hypercapnia with mechanical ventilation
than treatment with a short course of antibiotics. Oral leads to a rapid change in extracellular pH (from aci-
steroids given for 2 weeks result in identical outcomes demic to alkalemic). he resulting electrolyte shifts
(in terms of resolution of the exacerbation and risk of across myocardial cell membranes can result in seri-
recurrence) to 8 weeks (Answer A), and recent evidence ous, potentially life-threatening cardiac arrhythmias.
suggests that 5 days of therapy may suice in many By a similar mechanism, seizures may develop. he
cases. Oral steroids are equally efective compared with target arterial Pco2 during mechanical ventilation for
intravenous steroids in the treatment of COPD exacer- exacerbations of COPD in patients with acute-on-
bations with no greater incidence of gastrointestinal side chronic hypercapnia should be values close to base-
efects (Answer B). Noninvasive mechanical ventilation, line hypercapnia rather than a normal arterial Pco2. If
96 C HA P T E R 4 Pulmonary and Critical Care Medicine

lowering the arterial Pco2 to this goal involves exces- concerns for complications, in particular neuromus-
sively high peak inlation pressures or auto-PEEP, one cular weakness related to critical illness (neuropathy,
may choose to accept persistently higher arterial Pco2 myopathy, or polyneuromyopathy). However, patient
values until lung function improves, the ventilatory respiratory eforts and ventilator dyssynchrony may
strategy referred to as permissive hypercapnia. result in lung injury that may be mitigated by paraly-
Reversing chronic hypoxemia with high inspired oxy- sis (Answer C). A recent randomized controlled trial
gen concentrations (Answer A) may lead to worsened demonstrated a reduction in organ failure (Answer E)
hypercapnia in the spontaneously breathing patient with and an overall mortality beneit for 48 hours of paralysis
chronic hypercapnia, but it will have no adverse conse- in relatively severe ARDS (Pao2/Fio2 ratio <150 with
quences in a patient receiving mechanical ventilation. at least 5 cm H2O of PEEP). Despite the concern for
Inadequate time for exhalation during mechanical venti- weakness, there was no increase in weakness noted at 28
lation of patients with severe expiratory airlow obstruc- days or ICU discharge compared with placebo (Answer
tion may lead to the development of high airway and A). Neuromuscular blockade requires deep sedation to
transpulmonary pressures at the end of exhalation (auto- avoid anesthesia awareness and would not be expected
PEEP) (Answer C). Auto-PEEP can cause hypotension to decrease sedation requirements. Indeed, sedation was

t
ne
and increased risk of barotrauma, but cardiac arrhyth- similar in the two groups in this trial (Answer D).
mias would not be expected in the absence of hypo- Papazian L, Forel JM, Gacouin A, et al. Neuromuscular block-
tension. his patient does not have respiratory acidosis ers in early acute respiratory distress syndrome. N Engl J

e.
(Answer D); his Paco2 is 36 mm Hg, and his arterial pH Med. 2010;363(12):1107–1116.
is 7.56. One cannot exclude a chronic metabolic alkalo-
28. ANSWER: D. Volume targeted, tidal volume 410 mL

in
sis (Answer E) based on the blood gas results. However,
corticosteroids and inhaled beta-agonist bronchodilators his patient has acute respiratory distress syn-
drome (ARDS). His Pao2/Fio2 ratio is <100, which

ic
are not likely to cause a metabolic alkalosis of this sever-
ity, and chronic metabolic alkalosis does not precipitate meets criteria for severe ARDS. Low tidal volume
ventricular arrhythmias. ed ventilation results in decreased mortality and is the
standard of care for ARDS. he goal tidal volume of
26. ANSWER: B. Dopamine is more likely to cause car- 6 mL/kg is calculated based on the ideal rather than
sm
diac arrhythmias. actual body weight. For a male of 5 feet 8 inches, the
In a double-blind, randomized, controlled trial of ideal body weight is 68.4 kg; thus 6 mL/kg would
initial treatment of shock, treatment with dopamine equal 410 mL. he protocol used in the ARDSNet
versus norepinephrine demonstrated a signiicant trial started at 8 mL/kg, or 550 mL; thus tidal vol-
ok

increase in arrhythmias with dopamine. No statisti- umes of 650 mL, 700 mL, and 800 mL (Answers
cally signiicant diference in overall mortality was A, B, and C) are too large. Mechanical ventilation
breaths can be delivered either by setting a volume
bo

seen, but a subgroup analysis demonstrated increased


mortality in patients with cardiogenic shock treated or by setting a pressure, and plateau pressures are of
with dopamine (Answer D). he trend in other sub- importance in ARDS. hus pressure-targeted assist-
control mode (pressure control) is a reasonable ven-
e

groups did not favor dopamine, and norepinephrine


was not associated with more adverse efects (Answer tilator mode for ARDS. However, without knowing
://

C). Dopamine has been studied for its renal protective the respiratory mechanics and tidal volume achieved
efects; this trial did not show improvement in renal by this setting, it is impossible to know whether this
tp

function, and previous trials in early renal dysfunction ventilator setting is more—or even potentially less—
also demonstrated no beneit of dopamine (Answer injurious (Answer E).
Acute Respiratory Distress Syndrome Network. Ventilation
ht

A). Based on data from this trial and others, the Sur- with lower tidal volumes as compared with traditional tidal
viving Sepsis guidelines recommend norepinephrine volumes for acute lung injury and the acute respiratory dis-
as the irst-line vasopressor in septic shock. tress syndrome. N Engl J Med. 2000;342(18):1301–1308.
De Backer D, Biston P, Devriendt J. Comparison of dopamine
and norepinephrine in the treatment of shock. N Engl J
Med. 2010;362(9):779–789. 29. ANSWER: E. Hyperkalemia
Dellinger RP, Levy MM, Rhodes A, et  al. Surviving Sepsis Succinylcholine is a neuromuscular blocking agent
Campaign: international guidelines for management of that is frequently used for intubation due to its rapid
severe sepsis and septic shock, 2012. Intensive Care Med. onset, brief duration, and predictable response. Suc-
2013;39(2):165–228. cinylcholine acts as a depolarizing agent and thus can
result in hyperkalemia. In the setting of preexisting
27. ANSWER: B. A reduction in mortality hyperkalemia, particularly if severe, this side efect can
Neuromuscular blockade in severe acute respira- be life threatening. Succinylcholine is also contraindi-
tory distress syndrome (ARDS) has traditionally been cated in patients with a history of malignant hyper-
used for refractory cases and with great caution due to thermia and in cases where acetylcholine receptors are
CHAPTER 4 Pulmonary and Critical Care Medicine 97

expected to be upregulated (e.g., denervation from time of this patient’s ingestion is unknown (Answer
stroke, inherited myopathies). Neuroleptic malignant C). Gastric lavage and induced vomiting are not
syndrome is not related to malignant hyperthermia recommended (Answer E). Sodium bicarbonate is a
and is not a contraindication to succinylcholine or therapy for other ingestions (e.g., tricyclic and salicy-
propofol (Answer A). Propofol may cause hypoten- late poisoning). Whereas it may be helpful for cases of
sion, and thus hypertension is not a contraindication severe acidosis, in this case acidosis is not mentioned
(Answer D). Hypernatremia and corticosteroid use are (Answer B).
not contraindications to sedatives and neuromuscular
blockers in general (Answer B and C). Etomidate is 32. ANSWER: C. Noninvasive ventilation
an alternative sedative commonly used at the time of he arterial blood gases are consistent with acute-
intubation because of its general lack of hemodynamic on-chronic respiratory acidosis. In the setting of a
side efects; it can be associated with adrenal suppres- COPD exacerbation with hypercapnia (Pco2 >45 mm
sion. Hg), several studies have demonstrated that noninva-
sive positive pressure ventilation decreases mortality
30. ANSWER: D. Measurement of bladder pressure and need for intubation. Her oxygen saturation is ade-

t
ne
his patient is at high risk for abdominal compart- quate, and increasing supplemental oxygen is unlikely
ment syndrome. Initially recognized in surgical condi- to result in signiicant improvement in her dyspnea
tions (e.g., trauma, abdominal surgery), this syndrome and may worsen hypercapnia (Answer A). Conversely,

e.
is increasingly being recognized in medical settings, decreasing supplemental oxygen is likely to result in
particularly in the context of large-volume resuscita- suboptimal oxygen saturation and is unlikely to result

in
tion and intraabdominal pathology. Among the con- in improvement in the patient’s dyspnea (Answer D).
sequences of increased intraabdominal pressure are a Intubation may eventually be needed. However, this

ic
decrease in venous return with associated hypoten- patient is not in severe distress and does not have any
sion and extrinsic compression of the lung, resulting clear contraindications to noninvasive ventilation
in increased atelectasis and dead space. he normal
intraabdominal pressure in a nonobese, critically ill
ed (e.g., impaired consciousness, high aspiration risk, or
severe respiratory failure). herefore noninvasive ven-
patient, traditionally measured by transducing blad- tilation should be attempted irst (Answer B).
sm
der pressure, is <7 mm Hg. Intraabdominal hyperten- Plant PK, Owen JL, Elliott MW. Early use of non-invasive ven-
sion is deined as a sustained intraabdominal pressure tilation for acute exacerbations of chronic obstructive pul-
>12 mm Hg. Abdominal compartment syndrome is monary disease on general respiratory wards: a multicentre
deined as organ dysfunction with intraabdominal randomised controlled trial. Lancet. 2000;355(9219):1931–
ok

1935.
hypertension and is common with intraabdominal
pressures >20 mm Hg.
Conservative treatment includes bowel decompres- 33. ANSWER: B. Spontaneous breathing trial
bo

sion and patient positioning; surgical decompression Increased duration of mechanical ventilation is
may be required for severe cases. Although imaging associated with an increased risk of complications,
may be helpful in identifying an alternative etiol- especially ventilator-associated pneumonia. Patients
e

ogy for his abdominal distention, it would diagnose should be assessed daily for their readiness for discon-
://

abdominal compartment syndrome. hus abdominal tinuation of mechanical ventilation. In most patients
ultrasound and portable radiographs are not the best who are not requiring high levels of support, a spon-
tp

choice (Answers C and E). Given his instability, trans- taneous breathing trial is a reasonable next step. A
portation to a CT scanner may be risky (Answer A). spontaneous breathing trial has the patient breathe
Empiric antibiotics may be considered in severe pan- on minimal to no support for at least 30 minutes.
ht

creatitis with necrosis or if the etiology of his worsen- Many ICUs also include a rapid shallow breathing
ing is due to sepsis, but these processes are less likely index (RSBI) prior to, or as a part of, the spontane-
than intraabdominal hypertension (Answer B). ous breathing trial. he RSBI is performed with the
patient breathing spontaneously without pressure
31. ANSWER: D. N-acetylcysteine support or PEEP and is the ratio of respiratory rate
Acetaminophen poisoning remains a major cause of divided by tidal volume (in liters). An RSBI <105
acute liver failure in the United States. Treatment with identiies patients more likely to be ready to be extu-
N-acetylcysteine is the standard of care and should bated. However, the decision to extubate should not
begin as soon as possible, ideally within 8 hours after be based solely on the RSBI. he RSBI has only mod-
ingestion. he suspicion for overdose is high, and one erate speciicity to predict readiness to extubate; can
should not wait for the results of toxicology screening be artiicially low in the setting of sedation; and does
(Answer A). Administration of activated charcoal can not take into account mental status, airway secre-
be helpful, but it is most efective if given within 4 tions, and other factors (Answer A). In several ventila-
hours (preferably within 1 hour) of ingestion, and the tor “weaning” trials, a substantial number of patients
98 C HA P T E R 4 Pulmonary and Critical Care Medicine

were able to be extubated immediately after a success- controlled trial of early goal-directed therapy (EGDT)
ful spontaneous breathing trial. hus the decision to demonstrated improved mortality with early and
proceed with decreasing amounts of pressure support aggressive resuscitation in septic shock. Adequate
should ideally be initiated only after the results of the resuscitation in this trial was demonstrated by a CVP
spontaneous breathing trial are known and the deci- from 8 to 12 cm H2O. his speciic numeric target
sion regarding immediate extubation is made (Answer has been controversial because a static measurement of
C). Weaning via SIMV results in a longer duration of CVP is poorly predictive of luid responsiveness. None-
mechanical ventilation than daily spontaneous breath- theless, the 2.5 L of luid that this patient received is a
ing trials or pressure support (Answer E). Decreasing relatively low amount, particularly in someone who is
PEEP and Fio2 to levels required to adequately sup- obese, and other indicators, such as respiratory varia-
port oxygenation is reasonable if the patient is unable tion in CVP, may also serve as indicators of volume
to be extubated, but it is not the most appropriate next responsiveness. Debate persists about which aspect
step in this patient’s care (Answer D). of the Rivers protocol may have led to improved out-
Brochard L, Rauss A, Benito S, et  al. Comparison of three comes. A more recent trial comparing protocolized
methods of gradual withdrawal from ventilatory support care in the emergency room versus usual care (which

t
ne
during weaning from mechanical ventilation. Am J Respir now include early antibiotic therapy) did not show a
Crit Care Med. 1994;150(4):896–903. decrease in mortality, but patients in the control arm
Esteban A, Frutos F, Tobin MJ, A comparison of four methods received 2.3 L in the irst 6 hours, relecting a change
of weaning patients from mechanical ventilation. N Engl J

e.
in general practice toward earlier resuscitation.
Med. 1995;332(6):345–350.
Adequate resuscitation is preferred over increasing

in
vasopressors (Answer A). Fluid should be given as a
34. ANSWER: A. Continuous renal replacement ther-
bolus and not as a continuous infusion, both to treat
apy (CRRT) does not have a mortality beneit com-

ic
hypotension quickly and to be able to gauge respon-
pared with intermittent hemodialysis (IHD).
siveness (Answer B). Although colloid is likely safe in
Acute renal failure in the setting of septic shock ed this setting, there is no clear justiication for its added
is a frequent and morbid problem. Although only a
expense (Answer C). Although the patient is anemic,
minority of patients go on to require renal replace-
a baseline hematocrit is not given, and in the absence
ment therapy, the appropriate modality has been
sm
of active cardiac ischemia or ongoing blood loss, a
controversial. CRRT and IHD have been compared
transfusion would not be worth the risks (e.g., trans-
in several studies; overall, the evidence is most con-
fusion reaction or increased susceptibility to ARDS)
sistent with CRRT being equivalent to IHD (Answer
(Answer D).
ok

A). Although CRRT may be easier to tolerate in the ProCESS Investigators. A randomized trial of protocol-based care
setting of vasopressors (or mechanical ventilation), a for early septic shock. N Engl J Med. 2014;370(18):1683–
study by Vinsonneau and colleagues did not demon- 1693.
bo

strate any mortality beneit in a population where the Rivers E, Nguyen B, Havstad S, et al. Early goal-directed ther-
vast majority required both vasopressor and mechani- apy in the treatment of severe sepsis and septic shock. N
cal ventilator support (Answers C and D). In this study, Engl J Med. 2001;345(19):1368–1377.
e

despite the use of vasopressors, most patients were


able to be treated with IHD (Answer E). he available
://

36. ANSWER: B. Initiate argatroban


data on dialysis intensity suggest that this factor may
his patient has a high likelihood of having heparin-
be important. However, study results have been con-
tp

induced thrombocytopenia with thrombosis (HITT).


licting. Harm may be associated with low-intensity
he pretest probability of HITT is often given as the
therapy, and whereas some studies have demonstrated
“4 Ts”: thrombocytopenia, timing, thrombosis, and
ht

improved mortality with high-intensity renal replace-


exclusion of other causes of thrombocytopenia. Her
ment, several others have not demonstrated a beneit
thrombocytopenia has no other clear etiology; the
over standard-intensity dialysis (Answer B).
Vinsonneau C, Camus C, Combes A, et al. Continuous veno-
timing is within 5–10 days after exposure; her platelet
venous haemodiailtration versus intermittent haemodi- count has fallen by over 50%; and she has evidence
alysis for acute renal failure in patients with multiple-organ suggesting a new deep venous thrombosis (DVT) of
dysfunction syndrome: a multicentre randomised trial. Lan- her right leg. Treatment is indicated for most patients
cet. 2006;368(9533):379–385. with HIT, and certainly for patients with HITT. Addi-
RENAL Replacement herapy Study Investigators. Intensity tional testing should be done, but therapy should not
of continuous renal-replacement therapy in critically ill be delayed (Answers A and E). Of the available agents,
patients. N Engl J Med. 2009;361(17):1627–1638. only argatroban is indicated (alternatives may include
fondaparinux or bivalirudin). Low-molecular-weight
35. ANSWER: E. Intravenous crystalloid bolus heparin is also associated with a risk of HIT, albeit
Adequate luid resuscitation is a mainstay of hemo- lower than unfractionated heparin, and therefore is
dynamic support in septic shock. A randomized still contraindicated in HIT (Answer C). Warfarin
CHAPTER 4 Pulmonary and Critical Care Medicine 99

rapidly lowers protein C levels and thus is associated hemoglobin <7 g/dL) was associated with decreased
with increased risk of thrombosis during initiation mortality. Although patients with a primary cardiac
(Answer D). Transition to warfarin should be consid- diagnosis appeared less likely to be enrolled in this
ered when the patient’s platelet count has improved trial, analysis of patients with coronary disease dem-
and she has been stably anticoagulated. onstrated no increased risk of complications in the
Greinacher A. Heparin-induced thrombocytopenia. N Engl J restrictive transfusion group (Answer A) and no ben-
Med. 2015;373(3):252–261. eit from transfusions in terms of duration of mechani-
cal ventilation in the liberal transfusion group (Answer
37. ANSWER: E. Stress ulcer prophylaxis C). he patient’s gastrointestinal bleed has stabilized.
Ventilator-associated pneumonias (VAP) (or more Particularly in the absence of active blood loss, there
broadly, ventilator-associated complications) are an is no compelling reason to transfuse, and a random-
important cause of morbidity and mortality in the ized trial has demonstrated a beneit for a restrictive
ICU. Several practices have been shown to reduce the (hemoglobin <7 g/dL) transfusion strategy for most
incidence of VAP. Many of these therapies have been patients with upper gastrointestinal bleeding (Answer
included in ventilator “bundles” (grouped practices B). Transfusions are associated with several electrolyte

t
ne
to reduce complications in mechanically ventilated abnormalities, including hypocalcemia and hyperkale-
patients). hese therapies include elevation of the mia, but not hypokalemia (Answer E).
head of the hospital bed, oropharyngeal decontami- Hébert PC, Wells G, Blajchman MA, et al. A multicenter, ran-

e.
nation, and daily assessment of readiness to extubate domized, controlled clinical trial of transfusion requirements
(to reduce the duration of mechanical ventilation) in critical care. [Published erratum appears in N Engl J Med.
(Answers A and B). Continuous drainage of subglot- 1999;340(13):1056.] N Engl J Med. 1999;340(6):409–417.

in
tic secretions has been shown in several studies to Villanueva C, Colomo A, Bosch A, et al. Transfusion strategies
decrease VAP. However, this approach requires spe- for acute upper gastrointestinal bleeding. [Published erra-

ic
tum appears in N Engl J Med. 2013;368(24):2341.] N Engl
cially designed endotracheal or tracheostomy tubes
ed J Med. 2013;368(1):11–21.
(Answer D). Selective decontamination of the gastro-
intestinal tract has also been shown to decrease VAP,
though use has been limited in part due to concerns 39. ANSWER: B. Discontinue sertraline and fentanyl
he diferential diagnosis for fever in the ICU is
sm
about promotion of antibiotic resistance (Answer C).
Whereas stress ulcer prophylaxis is usually included as broad. Although infectious etiologies are the most
part of a ventilator “bundle,” acid suppression actu- common causes, certain toxidromes are important
ally results in an increased, not decreased, risk of VAP. to recognize. he serotonin syndrome is a life-threat-
ok

he mechanism is thought to be bacterial overgrowth ening condition associated with increased seroto-
in the stomach in the absence of suppressive gastric nergic activity and is characterized by mental status
acid, with risk of relux and aspiration of these bacte- changes, autonomic hyperactivity, and neuromuscular
bo

rial pathogens. changes. In addition to agents traditionally associated


Dodek P, Keenan S, Cook D, et  al. Evidence-based clinical with serotonin reuptake, many other medications can
practice guideline for the prevention of ventilator-associated increase serotonin, such as metoclopramide, carbi-
e

pneumonia. Ann Intern Med. 2004;141(4):305–313. dopa-levodopa, meperidine, and fentanyl. he sero-
tonin syndrome shares many features with neuroleptic
://

Koeman M, van der Ven AJ, Hak E, et  al. Oral decontami-
nation with chlorhexidine reduces the incidence of ven- malignant syndrome (NMS), including fever, mental
tilator-associated pneumonia. Am J Respir Crit Care Med. status change, and increased muscle tone. However, in
tp

2006;173(12):1348–1355. contrast to NMS (Answer D), serotonin syndrome is


characterized by hyperrelexia, not bradyrelexia, and
ht

38. ANSWER: D. Transfusion may be associated with clonus, which can be inducible, spontaneous, and ocu-
an increase in mortality. lar. In addition, this patient’s agitation began before
Anemia results in decreased oxygen delivery. How- administration of haloperidol; the onset of serotonin
ever, correction of anemia via transfusion of packed syndrome usually occurs within 24 hours of initiation
red blood cells is associated with risks, including of medication and is dose related, whereas the onset
hemolytic transfusion reactions, immunosuppression, of NMS begins within days to weeks. Management of
transfusion-associated circulatory overload (TACO), serotonin syndrome consists of discontinuation of all
and transfusion-related acute lung injury (TRALI). serotonergic agents (thus Answer A is incorrect), seda-
In the Transfusion Requirements in Critical Care tion with benzodiazepines, and consideration of cypro-
trial, over 800 critically ill patients were randomized heptadine, a serotonergic antagonist. Propofol would
to a restrictive or liberal transfusion strategy. here not treat serotonin syndrome (Answer C). Whereas
was no beneit to the liberal transfusion group. In a haloperidol should probably not be continued because
subgroup analysis of patients less acutely ill and those of anticholinergic efects, its discontinuation would
<55 years of age, a restrictive strategy (transfusion of not address the life-threatening serotoninergic efects
100 C HA P T E R 4 Pulmonary and Critical Care Medicine

of the sertraline and fentanyl (Answer D). here is no with relative hypotension and risk of barotrauma
evidence for an infectious etiology for this patient’s from high inlation pressures (Answer A). Increasing
fever, so vancomycin is not appropriate (Answer E). the respiratory rate or increasing the I:E ratio would
worsen hyperinlation (Answers C and E). Increasing
40. ANSWER: E. herapeutic hypothermia should be PEEP in the setting of obstructive lung disease with
continued for 24 hours. auto-PEEP could improve triggering and patient-
In patients who survive an out-of-hospital cardiac ventilator dyssynchrony, but these are not problems in
arrest, anoxic-ischemic encephalopathy is a major cause this case (Answer B).
of morbidity and mortality. he beneit of induced
hypothermia in ventricular ibrillation and pulseless 42. ANSWER: C. Use of midazolam instead of propofol
ventricular tachycardia was demonstrated in two ran- Providing appropriate control of discomfort dur-
domized controlled trials, which both demonstrated ing mechanical ventilation is an important goal in
a signiicant improvement in favorable neurologic the intensive care unit. Although keeping patients
outcomes. Although the data are less robust, recom- heavily sedated may appear to be most appropri-
mendations for therapeutic hypothermia have been

t
ate, excessive sedation is associated with prolonged

ne
extended to include all causes of cardiac arrest. Risks mechanical ventilation and other adverse outcomes,
of therapeutic hypothermia include increased risk of such as greater likelihood of delirium. Random-
infection, coagulopathy, hypokalemia (not hyperkale- ized trials have demonstrated more rapid extuba-

e.
mia, Answer B), and bradycardia. Although these risks tion with daily interruption of sedation (Answer B),
must be weighed against the beneits of hypothermia, daily spontaneous breathing trials (Answer B), and a

in
there was no statistically signiicant increase in sepsis combination of daily interruption of sedation with
or lethal arrhythmias in these two trials (Answer C). In spontaneous breathing trials. he type of sedation

ic
the larger of the two trials, development of infection used may also be important. Trials have demon-
was not associated with increased mortality (Answer strated decreased duration of mechanical ventilation
C). Based on results from these trials and other obser-
vational data, hypothermia should be initiated early
ed with propofol versus midazolam (thus Answer C is
associated with an increased duration of mechanical
(that is, within 4–6 hours) and not delayed for reassess- ventilation), as well as dexmedetomidine, an alpha-2
sm
ment of neurologic status (Answer D), and it should agonist with analgesic and sedative properties, versus
be continued for 12–24 hours. Of note, in a more midazolam (Answer D). Avoidance of routine seda-
recent trial, targeted temperature management with a tives altogether can be achieved in many intubated
goal of 36°C led to outcomes equivalent to a target patients, and this approach has also been associated
ok

of 33°C, suggesting that it may be active temperature with decreased duration of mechanical ventilation
management and possibly prevention of fever, rather (Answer E).
than hypothermia per se, that leads to improved out- Brochard L, Rauss A, Benito S, et  al. Comparison of three
bo

comes after cardiac arrest. methods of gradual withdrawal from ventilatory support
Bernard SA, Gray TW, Buist MD, et  al. Treatment of coma- during weaning from mechanical ventilation. Am J Respir
tose survivors of out-of-hospital cardiac arrest with induced Crit Care Med. 1994;150(4):896–903.
e

hypothermia. N Engl J Med. 2002;346(8):557–563. Girard TD, Kress JP, Fuchs BD, et  al. Eicacy and safety of
://

Hypothermia after Cardiac Arrest Study Group. Mild thera- a paired sedation and ventilator weaning protocol for
peutic hypothermia to improve the neurologic outcome mechanically ventilated patients in intensive care (Awak-
after cardiac arrest. N Engl J Med. 2002;346(8):549–556. ening and Breathing Controlled trial): a randomised con-
tp

Nielsen N, Wetterslev J, Cronberg T, et al. Targeted tempera- trolled trial. Lancet. 2008;371(9607):126–134.
ture management at 33°C versus 36°C after cardiac arrest. N Kress JP, Pohlman AS, O’Connor MF, Hall JB. Daily interrup-
ht

Engl J Med. 2013;369(23):2197–2206. tion of sedative infusions in critically ill patients undergoing
mechanical ventilation. N Engl J Med. 2000;342(20):1471–
41. ANSWER: C. Decrease respiratory rate 1477.
Ventilator support for obstructive lung disease
should allow for longer exhalation times to avoid 43. ANSWER: C. Fomepizole
hyperinlation and barotrauma (i.e., pneumothorax his patient’s presentation is highly suspicious for
and pneumomediastinum). In this case, the measured ethylene glycol intoxication. His osmolar gap is 15 if
end-expiratory pressure is above the set PEEP (i.e., one includes ethanol in the calculation of the expected
auto-PEEP is present), and the patient’s blood pressure serum osmolarity: expected osmolarity = 2 × (sodium
is borderline low. hus decreasing the respiratory rate in mEq/L) + (glucose in mg/dL)/18 + (BUN in mg/
and allowing adequate exhalation time should reduce dL)/2.8 + (ethanol in mg/dL)/3.7. Ingestion of a toxic
auto-PEEP and, as a result of decreased intratho- alcohol should be considered when the osmolar gap
racic pressure, improve hemodynamics. Without any is >10. However, it is important to note that this gap
change in ventilator settings, the patient will continue relects the presence only of the parent alcohols and
CHAPTER 4 Pulmonary and Critical Care Medicine 101

may be normal in late presentations. he combination Hypoventilation is another important cause of hypox-
of an osmolar gap and anion-gap metabolic acidosis emia, particularly in the setting of sedation. In this
should raise concern for ethylene glycol toxicity. he situation, another important cause of hypoxemia to
presence of crystals in the urine is consistent with consider is methemoglobinemia. Methemoglobin is
calcium oxalate, which is a consequence of ethylene formed when the ferrous (Fe2+) iron in heme is oxi-
glycol metabolism. he treatment for ethylene glycol dized to the ferric (Fe3+) state; the ferric state is unable
ingestion is fomepizole (Answer C). Fomepizole is an to bind oxygen. Increases in methemoglobin can be
inhibitor of alcohol dehydrogenase and thereby inhib- due to genetic conditions resulting in deiciencies of
its the metabolism of ethylene glycol to its toxic com- cytochrome b5 reductase or globin mutations, but
pounds. Administration of ethanol would also help they are more commonly due to medications. he list
prevent ethylene glycol metabolism, but it is associated of medications potentially causing methemoglobin-
with additional side efects (Answer A). Alcohol with- emia includes nitrates; dapsone; metoclopramide; and
drawal is a concern in this patient, for which benzodi- topical anesthetics, particularly benzocaine.
azepines would be appropriate, but this intervention Standard pulse oximeters use photodetectors to
would not treat ethylene glycol poisoning (Answer B). identify oxyhemoglobin and deoxyhemoglobin; met-

t
ne
Bicarbonate would be more appropriate therapy for a hemoglobin cannot be speciically detected and instead
non–anion gap acidosis (Answer E). Hemodialysis is causes an artifactual change (usually decrease) in oxygen
indicated for severe ingestions with refractory acidosis, saturation with increasing concentrations of methe-

e.
renal failure, deterioration in vital signs, or a markedly moglobin, generally to around 85%–87%. An arterial
elevated level of ethylene glycol (Answer D). blood gas with cooximetry can quantify the degree of

in
methemoglobinemia, if present. In severe cases, met-
44. ANSWER: E. Cardiogenic shock hemoglobinemia is treated with methylene blue. An

ic
he central venous pressure is elevated, inconsistent arterial blood gas alone can provide the actual oxygen
with hemorrhagic or hypovolemic shock (Answers C saturation, but without cooximetry, it cannot diagno-
and D). he central venous oxygen saturation relects
oxygen extraction for blood returning via the superior
ed sis methemoglobinemia (Answer D). A chest x-ray and
electrocardiogram are reasonable initial tests for this
vena cava and approximates the mixed venous oxygen patient, but they would not be diagnostic of this patient’s
sm
saturation. In septic shock following adequate luid condition (Answers A and E). Pulmonary embolism can
resuscitation, this value is typically elevated, relecting certainly cause a sudden reduction in oxygen saturation,
impaired tissue oxygen extraction. A low value indi- but chest CT angiography would not be the irst test of
cates elevated tissue extraction and is consistent with choice (Answer B).
ok

impaired cardiac output. Impaired cardiac output


can be due to inadequate illing (e.g., hypovolemia), 46. ANSWER: C. Endotracheal suctioning
obstruction of blood low (e.g., massive pulmonary he pressure required to deliver a breath to a
bo

embolus), or intrinsic cardiac causes (e.g., systolic dys- mechanically ventilated patient can be simpliied into
function). In this case, the low venous oxygen satu- two components: airway resistance and lung compli-
ration and elevated central venous pressure, together ance (or conversely, lung elastance). Problems with air-
e

with his physical examination (cool extremities), are way resistance and lung compliance can increase peak
://

more consistent with cardiogenic shock. Although inspiratory pressures, but only problems afecting lung
myocardial dysfunction is not uncommon in the set- compliance will increase the plateau pressure. he pla-
tp

ting of severe sepsis, it is also possible that his bilat- teau pressure is calculated by performing a breath-hold
eral radiographic opacities relect pulmonary edema. at end-inspiration and subtracting PEEP from the
Although central venous oxygen saturation is typically measured pressure. Because this pressure is calculated
ht

high in the setting of sepsis, it can at times be low, at zero low, it relects the pressure required to distend
particularly in settings of inadequate luid resuscita- the lung parenchyma by the given tidal volume, inde-
tion or concomitant impaired cardiac output (Answer pendent of airway resistance. In this case, although the
A). A high central venous oxygen saturation can also plateau pressure before the event is not given, it is not
be seen in end-stage liver disease, although shock is likely to be signiicantly increased, because the plateau
usually due to superimposed infection or bleeding pressure must be lower than the peak pressure and
(Answer B). because in COPD, one anticipates a low peak pressure
due to high lung compliance. hus these indings (a
45. ANSWER: C. Arterial blood gas with cooximetry high peak inspiratory pressure with a low plateau pres-
he most common causes of hypoxemia are sure) are consistent with an airway problem. Increases
ventilation–perfusion mismatch and shunt; these in ventilator pressures due to airway resistance can be
abnormalities can be due to a variety of parenchy- due to mucous plugging, obstructed or kinked endo-
mal and vascular causes, such as atelectasis, aspira- tracheal tubes, or bronchospasm. hus endotracheal
tion, pulmonary edema, and pulmonary embolism. suctioning is a reasonable irst step. A chest x-ray may
102 C HA P T E R 4 Pulmonary and Critical Care Medicine

be helpful, but it should not delay a potentially benei- cerebral perfusion. hey have also discouraged empha-
cial intervention (Answer A). Needle decompression sis on establishing an advanced airway with assisted
and chest tube placement would treat a pneumotho- ventilation, which not only interrupts chest compres-
rax or pleural efusion (Answers B and D), and diure- sions but also may lead to overventilation, increased
sis would treat pulmonary edema (Answer E), all of intrathoracic pressure, and decreased venous return.
which typically would be expected to increase the pla- Although hypoxemia and hypercapnia can cause
teau pressures. cardiac arrest, inadequate bag-and-mask ventilation is
usually not the cause of failure to recover from an arrest
47. ANSWER: D. Lorazepam infusion followed by fos- (Answer A). In this case, overventilation is a particu-
phenytoin is a reasonable treatment. lar concern, given the patient’s COPD. ACLS guide-
Status epilepticus is deined as seizures that persist lines have recognized a detrimental efect on efective
or recur frequently enough so that recovery between CPR from frequent interruptions for pulse checks
attacks does not occur. Management of status epilepti- and consequently recommend immediately resuming
cus includes assessment of the underlying etiology and compressions for 2 minutes after administration of
supportive care of respiratory and circulatory status, drugs (such as epinephrine) or a deibrillation attempt

t
ne
which may include intubation. Initial management is (Answers C and D). Rapid response teams have not
generally through administration of benzodiazepines, convincingly been shown to decrease mortality, but
most often lorazepam, although others can be used. several studies have demonstrated that they can reduce

e.
In a randomized controlled trial comparing several the incidence of in-hospital cardiac arrest (Answer B).
treatments, lorazepam alone was found to be superior Chan PS, Jain R, Nallmothu BK, et al. Rapid response teams:
a systematic review and meta-analysis. Arch Intern Med.

in
to phenytoin alone (Answer A). Fosphenytoin is pre-
ferred over phenytoin because it is water soluble, may 2010;170(1):18–26.

ic
be infused more rapidly, and results in less local irri-
tation. Although the absence of the propylene glycol 49. ANSWER: C. Corticosteroids will likely improve
carrier in fosphenytoin should in theory lead to fewer
cardiovascular side efects, several studies suggest that
ed his response to vasopressors.
Despite decades of study, the use of corticosteroids
the risk of hypotension does not difer between the two in sepsis and septic shock remains controversial. Older
sm
drugs (Answer B). Diazepam has higher lipid solubility studies using high-dose steroids found no beneicial
and likely faster, not slower, onset of action than loraz- efect and a possible increase in mortality (Answer E).
epam (Answer C). For some cases of status epilepticus, A study of moderate-dose corticosteroids (200 mg
treatment with lorazepam alone is suicient. However, of hydrocortisone) appeared to demonstrate beneit,
ok

in many cases, addition of phenytoin or fosphenytoin limited to those who were deemed to have inadequate
can help provide a prolonged antiseizure efect and thus adrenal reserve after an ACTH stimulation test. How-
represents a reasonable treatment in this case. Although ever, this favorable inding was not conirmed in a
bo

propofol has not been as well studied as benzodiazepines subsequent trial (Answer A). Experts continue to rec-
or phenytoin, it has been used successfully to treat status ommend moderate-dose steroids for septic shock with
epilepticus in several small trials (Answer E). continued need for vasopressors, based on a consistent
e

Rossetti AO, Reichhart MD, Schaller MD, et al. Propofol treat- improvement in hemodynamics and response to vaso-
://

ment of refractory status epilepticus: a study of 31 episodes. pressors.


Epilepsia. 2004 Jul;45(7):757–763. he use of corticosteroids in ARDS, as in septic
tp

Treiman DM, Meyers PD, Walton NY, et al. A comparison of shock, is controversial. More data are available for the
four treatments for generalized convulsive status epilepticus.
use of steroids late in the course of ARDS. In an ARD-
N Engl J Med. 1998;339(12):792–798.
ht

SNet trial of late rescue (after 7 days), use of steroids


was associated with fewer days in shock and shorter
48. ANSWER: E. End-tidal CO2 detection may help duration of mechanical ventilation but a higher rate of
determine the quality of cardiopulmonary resus- reintubation. Mortality was increased in the subgroup
citation (CPR) and return of spontaneous circu- that received therapy after 14 days (Answer D). Assess-
lation. ment of adrenal insuiciency in critically ill patients is
Carbon dioxide is detected in exhaled breath in the challenging, owing to wide variations in serum corti-
presence of circulation. hus end-tidal carbon diox- sol levels and in free versus bound cortisol. However,
ide detection has been advocated as a method both to in the absence of true adrenal insuiciency, neither
assess for adequacy of CPR and to detect the return of the absolute serum cortisol level nor the response to
spontaneous circulation. Over the past several years, ACTH appears reliably to identify patients who will
advanced cardiac life support (ACLS) guidelines have beneit from corticosteroids (Answer B).
been updated to relect studies highlighting the det- Annane D, Bellissant E, Bollaert PE, et al. Corticosteroids in
rimental efects of inadequate and interrupted chest the treatment of severe sepsis and septic shock in adults: a
compressions, which result in decreased cardiac and systematic review. JAMA. 2009;301(22):2362–2375.
CHAPTER 4 Pulmonary and Critical Care Medicine 103

Sprung CL, Annane D, Keh D, et al. Hydrocortisone therapy treatment, intravenous lipid emulsion, has been advo-
for patients with septic shock. N Engl J Med. 2008;358(2): cated for use in several types of severe overdoses,
111–124. including calcium channel blocker overdose. Although
Steinberg KP, Hudson LD, Goodman RB, et al. Eicacy and this therapy has not been well studied, results from
safety of corticosteroids for persistent acute respiratory dis- animal studies and case reports of severe toxicity in
tress syndrome. N Engl J Med. 2006;354(16):1671–1684.
humans have demonstrated evidence of beneit. he
mechanism of action is unclear, but it may work by
50. ANSWER: E. High-dose insulin therapy acting as a “lipid sink” for the toxic medication.
Calcium channel blocker overdose is associated Hemodialysis is not efective for removing vera-
with substantial morbidity and mortality. he efects pamil, because verapamil is highly protein bound
of overdose depend on the type and dose of calcium (Answer A). Bicarbonate infusion may be indicated in
channel blocker. In addition to vasodilation and the setting of severe acidemia or a widened QRS com-
decreased inotropy, diltiazem and verapamil, but not plex due to sodium channel blockade, as in tricyclic
the dihydropyridines, typically result in bradycardia. overdose (Answer B). Physostigmine is a therapy for
Recognition of the overdose is important because anticholinergic overdose (Answer D). Intravenous cor-

t
hemodynamic deterioration can be delayed by hours

ne
ticosteroids are not helpful in calcium channel blocker
after the ingestion. In addition to supportive and overdose (Answer E).
symptomatic care, treatment usually includes intrave- Greene SL, Gawarammana I, Wood DM, et al. Relative safety of
nous calcium and glucagon. Whole-bowel irrigation

e.
hyperinsulinaemia/euglycaemia therapy in the management
may be appropriate for ingestion of extended-release of calcium channel blocker overdose: a prospective observa-
preparations. Temporary pacemaker support may be tional study. Intensive Care Med. 2007;33(11):2019–2024.

in
indicated for persistent bradycardia. High-dose insu- Doepker B, Healy W, Cortez E, Adkins EJ. High-dose insu-
lin therapy has been increasingly advocated for cases lin and intravenous lipid emulsion therapy for cardiogenic

ic
of calcium channel blocker (and beta-blocker) over- shock induced by intentional calcium-channel blocker
dose. he mechanisms of action are unclear, but cal- ed and beta-blocker overdose: a case series. J Emerg Med.
cium channel blockers may cause insulin resistance 2014;45(4):486–490.
in the myocardium that is overcome by high doses
of insulin. Although this therapy is usually instituted Acknowledgment
sm

with a concomitant glucose infusion, calcium chan-


nel blocker overdose itself can cause impaired glucose he authors and editors gratefully acknowledge the con-
metabolism, and patients are often hyperglycemic and tributions of the previous authors, Christopher Fanta and
Michael Cho.
ok

may not require additional glucose. Recently, another


e bo
://
tp
ht
5
Endocrinology
OLE-PETTER R. HAMNVIK

1. An 89-year-old Caucasian woman is seen at your hormone (PTH) 76 pg/mL (normal 10–65). Vita-

t
ne
clinic for a routine physical examination. She is min D levels are normal. Screening bone density
known to have prediabetes for many years, and you shows osteoporosis at the femoral neck. She gen-
have recently diagnosed her with new-onset type 2 erally feels well and denies any history of kidney

e.
diabetes. She is asymptomatic, but her fasting blood stones. She refuses to consider parathyroid surgery
glucose was 145 mg/dL, and her recent hemoglobin at this time.

in
A1c (HbA1c) was 7.5%. Her past medical history What do you recommend for her?
is signiicant for hypertension, hypercholesterol- A. 24-hour urine to measure calcium excretion
emia, and coronary artery disease with ischemic B. Alendronate 70 mg orally weekly

ic
cardiomyopathy (ejection fraction 40%). Medi- C. Annual calcium, creatinine, and bone mineral
cations include amlodipine, aspirin, atorvastatin,
and metoprolol. On physical exam, she weighs
ed density testing
D. Cinacalcet 30 mg orally twice daily
39 kg; her blood is pressure 99/64 mm Hg; and E. Neck ultrasound and sestamibi scanning
sm
her heart rate is 67 beats per minute. Her jugular
venous pulse is estimated at 5 cm. She has normal 3. A 20-year-old woman presents to the clinic because
heart sounds with no added sounds or murmurs. of oligomenorrhea. She went through menarche
Her lungs are clear to auscultation with no wheeze and adrenarche at the same age as her peers. She is
ok

or crackles. She has normal pedal pulses and no currently obese and has struggled with her weight
edema. Her abdominal and neurologic exams were since her teens. She is concerned about several years
normal. Serum creatinine was measured at 1.3 mg/ of hair thinning, male pattern balding, and facial
bo

dL. You refer the patient for diabetes education and and chest terminal hair growth. She has minimal
to a dietitian for nutritional counseling. She wishes acne. She has the following laboratory test results:
to start metformin because her husband has been total testosterone 42.4 ng/dL (mildly elevated), sex
e

taking this for diabetes for the past 15 years with hormone–binding globulin (SHBG) 12 nmol/L,
://

good glycemic control. prolactin 9.6 ng/mL, follicle-stimulating hormone


How would you advise her regarding metformin (FSH) 5.2 mIU/mL, luteinizing hormone (LH)
tp

therapy? 11.8 mU/mL, and HbA1c 5.2%. She also has a


A. An extended-release formulation of metformin normal dexamethasone suppression test result and
should be started at 2000 mg once daily. normal 17-hydroxyprogesterone. You diagnose her
ht

B. Metformin should be started at 500 mg once daily with polycystic ovarian syndrome (PCOS). She is
and slowly increased to a target daily dose of 2000 not currently interested in pregnancy, and you start
mg per day. treatment with an estrogen-progestin contraceptive.
C. Metformin should be started at 500 mg twice After 6 months her hirsutism persists, and she is
daily with no further dose escalation. very concerned about this.
D. Metformin therapy is contraindicated on the basis In addition to continuing the oral contraceptive
of renal dysfunction. pill, what is the next best treatment strategy for her
E. Metformin therapy is contraindicated due to her hirsutism?
history of congestive heart failure. A. Encourage weight loss.
B. Reassure her that her hirsutism will improve with
2. A 60-year-old woman is found to have mild primary prolonged treatment.
hyperparathyroidism on routine chemistry screen- C. Start letrozole.
ing with serum calcium 10.9 mg/dL (normal 8.6– D. Start metformin.
10.4), albumin 4.0 g/dL, and intact parathyroid E. Start spironolactone.

104
CHAPTER 5 Endocrinology 105

4. A 48-year-old man had a history of hypertension for the What is the best initial treatment for this patient?
past 7 years. His treatment included lisinopril, amlodip- A. Bromocriptine 2.5 mg daily
ine, and hydrochlorothiazide. Despite taking these three B. Hydrocortisone 50 mg IV every 8 hours
medications, his blood pressure remained at 170/100 C. Levothyroxine 100 µg orally once daily
mm Hg. On a routine physical examination, his serum D. Urgent surgical resection of the mass
potassium was found to be 2.7 mEq/L. Previously, while E. Vitamin K 10 mg daily
on hydrochlorothiazide, his potassium had been between
3.3 and 3.7 mEq/L. He was given potassium chloride 7. A 31-year-old woman presents with a 5-week history
supplementation, but his serum potassium remained of weight loss and symptomatic palpitations. Labora-
less than 3.5 mEq/L. To further address his hypokale- tory tests demonstrate thyrotoxicosis. She is diagnosed
mia, his hydrochlorothiazide was stopped, and instead with Graves disease on the basis of clinical indings,
he was treated with spironolactone. With the addition of including goitrous enlargement of her thyroid, mild
spironolactone for 4 weeks, his blood pressure improved proptosis, and periorbital swelling. She is efectively
to 140/80 mm Hg, and his potassium stabilized at 3.7 treated with methimazole for 12 months, and then
mEq/L. At that time, a biochemical workup for hyper- treatment is held to see if she may have entered a state

t
ne
aldosteronism revealed a serum aldosterone of 42 ng/dL of remission. Laboratory tests checked 2 months after
and a plasma renin activity of 4.2 ng/mL/h. stopping methimazole show thyroid-stimulating hor-
What is the best interpretation of these laboratory mone (TSH) <0.001 mU/L, free thyroxine (T4) 1.9

e.
results? ng/dL, T4 12.1 µg/dL, and triiodothyronine (T3)
A. Essential hypertension 243 ng/dL. After options for further management are

in
B. Insuicient information to conirm a diagnosis reviewed, the patient elects to proceed with radioactive
C. Primary hyperaldosteronism iodine treatment.

ic
D. Renal tubular acidosis What should you tell her?
E. Secondary hyperaldosteronism A. She should wait until she is 6 months out from

5. A 39-year-old man is admitted to the intensive care


ed treatment before trying to conceive.
B. She will need to be strictly isolated from family
unit (ICU) for severe community-acquired pneu- members for 2 weeks.
sm
monia requiring intubation. He is otherwise healthy C. She will not have to take any more medication
except for obesity, and he is not known to have dia- after treatment.
betes. Random blood glucose levels obtained during D. he treatment may improve her thyroid eye dis-
the irst day in the ICU are 183, 195, and 182 mg/dL. ease.
ok

HbA1c was 5.5%. E. he treatment should work immediately.


What would be your next step in glucose manage-
ment of this patient? 8. A 50-year-old woman with multiple sclerosis was
bo

A. Initiation of intravenous insulin with glucose tar- treated for over 1 year with high-dose prednisone. Her
get of 140–180 mg/dL prednisone dose was as high as 60 mg daily, and over
B. Initiation of sliding-scale insulin, with a rapid- the course of the last year, as her neurologic symptoms
e

acting insulin every 4 hours for ingerstick glucose improved, her prednisone dose was decreased gradu-
://

values above 180 mg/dL ally and ultimately stopped. Two weeks after her last
C. Initiation of subcutaneous agonist of the gluca- prednisone dose, she was scheduled for an elective
tp

gon-like peptide 1 receptor with a target glucose orthopedic surgical procedure.


of 80–180 mg/dL What empiric hormone treatment should be admini-
D. Initiation of subcutaneous long-acting basal insu- stered perioperatively?
ht

lin and titration of dose until glucose values are A. Cosyntropin


80–110 mg/dL B. Dehydroepiandrosterone (DHEA)
E. Monitoring of ingerstick glucose values every 4 C. Epinephrine
hours only D. Fludrocortisone
E. Hydrocortisone
6. A 55-year-old woman is admitted with a severe head-
ache. She also complains of double vision. On physical 9. A 50-year-old obese woman with a history of gesta-
exam, her blood pressure is 89/48 mm Hg; her heart tional diabetes and a family history of diabetes and
rate is 99 beats per minute; and she is afebrile. She is coronary artery disease was diagnosed with type 2 dia-
slightly drowsy, and she has partial palsy of left cranial betes during a visit with her primary care physician, at
nerves III, IV, and VI. A CT scan of the head shows which time she was found to have an HbA1c level of
a large pituitary mass with extension into the left cav- 8.7% on routine screening blood tests. Despite imple-
ernous sinus and with an area of likely hemorrhage. menting dietary changes under the supervision of a
Blood test results have not yet been returned. dietitian, engaging in moderate exercise several times
106 C HA P T E R 5 Endocrinology

weekly, and complying with treatment with metfor- A. Karyotyping


min 1000 mg twice daily, her HbA1c remains above B. Measure a morning total testosterone and sex hor-
target at 7.7%. You wish to add a second medication mone–binding globulin.
to intensify her treatment, but she is resistant due to C. Pituitary MRI
concern for hypoglycemia. You learn that her brother- D. Testicular ultrasound
in-law died after a motor vehicle accident thought to E. Start testosterone replacement therapy.
be caused by insulin-induced hypoglycemia.
Which of the following would be the best option 13. A 46-year-old previously healthy woman is admitted
for this patient, given the above considerations? to the hospital after presenting with lethargy, polyuria,
A. Add glipizide polydipsia, and 2-kg weight loss. On physical exami-
B. Add insulin degludec nation, her vital signs showed a respiratory rate of 32
C. Add nateglinide breaths per minute, heart rate of 132 beats per minute,
D. Add pioglitazone blood pressure of 92/50 mm Hg, and temperature of
E. No change in treatment 37.0°C. She is noted to be obese with a BMI of 34 kg/
m2. Her cardiac exam is notable for tachycardia but no

t
ne
10. A 75-year-old man with a remote history of papillary murmurs. Her lungs are clear to auscultation bilater-
thyroid cancer is diagnosed with metastatic prostate ally. Her neurologic exam is normal. Her skin exam
cancer. He undergoes prostate surgery, local radiation, shows acanthosis nigricans of the neck and axillae.

e.
and androgen deprivation therapy. He also receives 4 Her laboratory tests conirmed diabetic ketoacido-
mg of zoledronic acid intravenously to prevent bone sis with glucose 623 mg/dL, bicarbonate 10 mmol/L,

in
loss. Five days later, he presents to the emergency and an anion gap of 24 mmol/L. he patient is treated
room with severe paresthesias and muscle cramping. with intravenous normal saline, potassium, and insu-

ic
On examination, he has positive Chvostek and Trous- lin and transitioned to a subcutaneous basal-bolus reg-
seau signs and a well-healed neck scar. Laboratory data imen (NPH insulin 10 U twice daily and insulin lispro
include TSH 0.54 µIU/mL (normal), alkaline phos-
phatase 120 U/L (normal), 25-hydroxyvitamin D 25
ed 5 U with each meal). he test for GAD65 antibodies
was negative.
ng/mL, serum calcium 6.5 mg/dL (normal 8.6–10.4), She returns 4 weeks later to her primary care physi-
sm
albumin 4.0 g/L, and phosphate 6.0 mg/dL. cian for follow-up. She reports that she has a strong
What is the most likely etiology of the hypocalcemia? family history of type 2 diabetes in two sisters and
A. Hungry bone syndrome both her parents. She has been compliant with the
B. Hypomagnesemia insulin that she was prescribed on discharge, but she
ok

C. Hypoparathyroidism has had increasing problems with hypoglycemia, now


D. Osteoblastic bone metastases occurring almost every day. She is frustrated with the
E. Vitamin D deiciency frequent injections and also upset that she has gained
bo

2.5 kg since before her admission.


11. A 65-year-old Caucasian woman is noted to have What is the next best step in the treatment of this
osteopenia on bone mineral density screening. When patient’s hyperglycemia?
e

you calculate her FRAX score, you ind that she has a A. Advise her that she can likely stop insulin therapy;
://

22% chance of sustaining a major osteoporotic frac- initiate metformin and reduce insulin doses to
ture over the next 10 years. According to the National help achieve this.
tp

Osteoporosis Foundation, antiresorptive therapy B. Advise her that she does not need antidiabetic
should be considered when the 10-year absolute risk therapy; stop insulin now and monitor her inger-
of a major osteoporotic fracture is above what level? stick glucose values three times per day.
ht

A. 5% C. Advise her that she will need lifelong insulin treat-


B. 10% ment; switch her basal insulin to insulin degludec
C. 15% to reduce hypoglycemic episodes.
D. 20% D. Advise her that she will need to remain on insulin
E. 25% for the rest of her life and reduce the doses to pre-
vent the frequent hypoglycemic episodes.
12. A 48-year-old man with a history of obesity and type 2
diabetes mellitus presents with low serum total testos- 14. A 48-year-old woman presents for follow-up of her
terone. He notes decreased libido and erectile dysfunc- type 2 diabetes mellitus. She was diagnosed 1 year ago
tion. An afternoon total testosterone was 200 ng/dL and has been taking metformin, her only medication,
(normal range 220–1000 ng/dL). On examination, he since then. She checks ingerstick glucose values at
has a BMI of 34 kg/m2 but is normally virilized with home before breakfast and always has values above 230
testicular size of 15 mL bilaterally. mg/dL. She has checked her glucose later in the day
What is the next best step? only twice over the past month, and her glucose values
CHAPTER 5 Endocrinology 107

were 289 mg/dL and 300 mg/dL. A recent HbA1c sodium increased to 148 mEq/L, and his fasting glu-
was 6.5%, and a recent glucose value sent as part of a cose was elevated at 106 mg/dL.
basic metabolic panel was 274 mg/dL. he remainder What diagnosis are you suspecting in this patient?
of her menstrual history is notable for menorrhagia, A. Central diabetes insipidus
a hemithyroidectomy for a benign nodule, and over- B. Nephrogenic diabetes insipidus
weight (BMI 28.9 kg/m2). C. Psychogenic polydipsia
What is the best intervention for this patient? D. Syndrome of inappropriate antidiuretic secretion
A. Evaluate for mutations of the HBB gene. (SIADH)
B. Provide her with a new glucose meter. E. Type 2 diabetes mellitus
C. Provide the patient with a continuous glucose
monitor. 17. A 25-year-old woman with hypothyroidism presented
D. Reassure her that her HbA1c is at her goal of <7%. to the emergency room complaining of weakness. She
E. Send for a complete blood count. was feeling dizzy and lightheaded while out on a hot
summer day. In the preceding weeks, her friends had
15. A 55-year-old woman is noted to be tachycardic with noticed that she had lost weight and was increasingly

t
ne
a resting pulse of 104 beats per minute during a rou- fatigued during the day. In the emergency room, the
tine physical examination. Laboratory tests checked to patient appeared lethargic and weak. Her sodium was
evaluate this show TSH 0.002 mU/L with follow-up 133 mEq/L, and her potassium was 5.7 mEq/L. At

e.
laboratory tests showing free T4 2.3 ng/dL, T4 14.7 2:00 p.m., her plasma adrenocorticotropic hormone
µg/dL, and T3 289 ng/dL. She denies any history of (ACTH) level was 550 pg/mL (10–60) and serum cor-

in
anterior neck discomfort, weight loss, palpitations, tisol was 2.4 µg/dL (2.3–19.4). he patient was given
anxiety, tremor, heat intolerance, or insomnia. Physi- 250 µg of ACTH for a stimulation test, and 60 min-

ic
cal examination reveals tachycardia with a regular utes later her cortisol was 4.1 µg/dL.
rhythm, a slightly enlarged thyroid without any dis- What is the most likely diagnosis?
crete nodularity, and no evidence of proptosis or ocu-
lar irritation. A radioiodine uptake scan shows difuse
ed A. Primary adrenal insuiciency
B. Secondary adrenal insuiciency
uptake consistent with Graves disease. She is started C. Ectopic ACTH syndrome
sm
on methimazole with normalization of her thyroid D. Cushing disease
function within 2 months; she is currently taking it E. Cushing syndrome
at a dose of 5 mg per day without any adverse efects.
Eighteen months later, she asks you if she can stop 18. A 56-year-old woman presents complaining of genital
ok

the methimazole because she is concerned about side discomfort manifesting as daily itching and stinging,
efects. Her last TSH was 4.1 mU/L. with more severe symptoms upon attempted inter-
What do you recommend as the next step? course, including dyspareunia and slight bleeding. Her
bo

A. Continue methimazole indeinitely. last menstrual period was 3 years prior. She has hot
B. Measure erythrocyte sedimentation rate (ESR). lashes once or twice per week. A pelvic exam reveals
C. Refer for radioiodine ablation. pale vaginal mucosa. he cervix appears normal and
e

D. Refer to a thyroid surgeon for a thyroidectomy. a Pap smear was negative for cytologic changes sug-
://

E. Stop methimazole. gestive of cervical cancer. he patient otherwise has a


history of atypical hyperplasia of the breast, and her
tp

16. A 22-year-old man presents with headache and mother had breast cancer.
peripheral vision loss. Brain imaging reveals a large sel- What is the best management of this patient’s cli-
lar mass with both solid and cystic components that is macteric symptoms?
ht

compressing the optic chiasm. Humphrey visual ield A. Bazedoxifene with conjugated equine estrogens 1
testing demonstrates bitemporal hemianopsia. He has tablet daily
no previous medical history, his BMI is 20 kg/m2, B. Estradiol 10-µg vaginal pill
and he is taking no medications. His initial pituitary C. Gabapentin 600 mg PO at bedtime
functional evaluation revealed central hypogonadism D. Lubricant with intercourse
and central hypothyroidism. His adrenal function E. Weekly estradiol patch with daily medroxyproges-
was normal. MRI characteristics suggested the mass terone acetate
was a craniopharyngioma. He was taken to surgery to
remove the pituitary mass in an attempt to decompress 19. A 56-year-old woman presented to the emergency room
the optic chiasm and restore his vision. He did well with right lower quadrant abdominal pain. An abdomi-
in the immediate postoperative period, but within 24 nal CT scan was performed to evaluate for appendicitis.
hours, he began to develop polyuria. His urine output No abnormal bowel indings or other abnormalities to
increased to 400 mL/h with a urine speciic gravity account for the pain were identiied. However, a 1.2-
<1.001. He complained of extreme thirst. His serum cm adrenal mass was incidentally detected in the medial
108 C HA P T E R 5 Endocrinology

limb of the left adrenal gland. he mass was described Based on the guidelines from the American Diabe-
as homogeneous, round, with smooth borders and a tes Association, what is the best strategy for cardiovas-
lipid-rich density (6 Hounsield units). She denied any cular risk reduction in this patient?
episodic adrenergic symptoms, weight gain, or muscle A. Initiate aspirin 81 mg daily.
weakness. She had no history of hypertension, impaired B. Initiate rosuvastatin 20 mg daily.
glucose handling, or low bone density. C. Initiate chlorthalidone 25 mg daily.
Which of the following would you advise? D. Initiate insulin glargine 10 U at night.
A. Corticotropin stimulation test E. Obtain a coronary artery calcium score.
B. Dexamethasone suppression test
C. MRI of the adrenal gland 22. A 33-year-old man is noted to have palpable enlarge-
D. No further testing needed ment of the right side of his thyroid on a routine
E. Serum catecholamines physical examination. A thyroid ultrasound reveals a
solitary 3.1-cm right-sided nodule with smooth bor-
20. A 44-year-old man is referred to your clinic for diabetes ders. Laboratory tests show TSH 0.1 mU/L and T4
management. Following a routine screening 6 months 11.5 µg/dL. He reports a history of occasional symp-

t
ne
ago, he was found to have a fasting glucose level of 129 tomatic palpitations and weight loss of 5 lb over the
mg/dL and HbA1c of 6.8%. He was started on metfor- course of 3 months despite an increase in his appetite.
min 1000 mg twice daily, which was efective for sev- He is not taking any medications and has not noted

e.
eral months in controlling his blood sugar. For the last any problems with dysphagia or dysphonia.
month, the patient’s fasting blood glucose levels have What should you do next?
A. Administer a 15 mCi dose of 131I.

in
been elevated again at 150–180 mg/dL. After meals,
most of his readings are in the 200–300 mg/dL range, B. Obtain a thyroid scan.

ic
and his most recent HbA1c was 8.6%. he patient has C. Perform a ine-needle aspiration biopsy of the
no other medical conditions. Blood pressure is 125/76 right-sided nodule.
mm Hg, and body mass index is 23.4 kg/m2. he rest
of the physical examination is unremarkable, with no
ed D. Refer the patient to a thyroid surgeon.
E. Start methimazole at a dose of 5 mg daily.
acanthosis nigricans or skin tag and no other physical
sm
exam features of endocrinopathies. Blood tests were 23. A 47-year-old woman has a long history of having
signiicant for HDL cholesterol of 46 mg/dL and fast- diiculty losing weight. She has multiple members
ing triglycerides of 132 mg/dL. he patient has no in her extended family who are overweight but states
other family members with diabetes. that her parents and siblings are all of normal weight.
ok

How would you further evaluate this patient? She states that she has always struggled to maintain
A. Obtain a 24-hour urine collection for cortisol levels. her weight, but in the last 2 years she has gained 20
B. Obtain a serum insulin-like growth factor-1 level. kg despite exercising daily and watching her caloric
bo

C. Obtain MRI of the pancreas. intake. She has recently been diagnosed with both
D. Test for maturity-onset diabetes of youth (MODY) type 2 diabetes mellitus and hypertension. Last year,
mutations. she fell while riding her bicycle, broke her elbow, and
e

E. Test for the presence of antiglutamic acid decar- was found to have osteoporosis. She reports men-
://

boxylase (GAD) antibodies. arche at age 12 with regular menses until the last
year, when they have become increasingly unpredict-
tp

21. A 55-year-old Caucasian woman has recently been able. She has been distressed by her weight gain and
diagnosed with type 2 diabetes mellitus. Her only recently started taking an antidepressant. On physi-
past medical history is gastritis treated with Helico- cal examination, she is found to be obese with a BMI
ht

bacter pylori eradication therapy 3 years ago. Her cur- of 35 kg/m2, blood pressure of 160/100 mm Hg,
rent medications are metformin and a multivitamin. and heart rate of 72 beats per minute. She has a very
Her family history is notable for a myocardial infarc- round face with ine, thin hair on her lateral cheeks.
tion in her father at age 83. Physical examination is She has supraclavicular fat accumulation and multi-
signiicant for a body mass index of 27.1 kg/m2 and ple pigmented striae on her abdomen. She has several
blood pressure of 132/84 mm Hg. Her laboratory tests bruises on her lower extremities. You are concerned
show fasting blood glucose 118 mg/dL, HbA1c 6.3%, that she may have Cushing syndrome.
total cholesterol 205 mg/dL, low-density lipoprotein Which of these is an appropriate irst-line screening
(LDL) cholesterol 127 mg/dL, high-density lipopro- test for hypercortisolism?
tein (HDL) cholesterol 46 mg/dL, and triglycerides A. 8:00 a.m. ACTH level
165 mg/dL. Although the patient denies chest pain or B. 8:00 a.m. serum cortisol level
shortness of breath, she is concerned about her risk of C. Overnight 8-mg dexamethasone suppression test
cardiovascular disease and asks you how she can reduce D. Cosyntropin stimulation test
this risk. E. Late-night salivary cortisol test
CHAPTER 5 Endocrinology 109

24. A 53-year-old man with a history of acromegaly pre- 26. A 55-year-old man presents to your oice as a new
sented to establish routine care with a primary care patient to establish care after he recently moved to the
provider. He underwent surgical resection of a pitu- area. His past medical history is signiicant for hyper-
itary macroadenoma 6 years prior to your visit and has tension, which is well controlled with hydrochlo-
been lost to endocrine follow-up. rothiazide 25 mg once daily, as well as dyslipidemia
He has a history of diabetes mellitus, hyperten- treated with atorvastatin 80 mg once daily. He also
sion, and obstructive sleep apnea. He has been feel- has a history of type 2 diabetes diagnosed at age 36,
ing poorly the last few months and presents with which was initially poorly controlled due to diiculty
fatigue, difuse joint pain, and excessive sweating. with afording his medications, although his HbA1c
He feels his hands and feet have been swollen. He has been at goal of 6.5%–7.0% for the past 5 years on
denies headaches or visual changes. His medications metformin 1000 mg twice daily and exenatide once
include metformin and lisinopril. On examination, weekly. He also takes aspirin 81 mg daily. He reports
he has the classic coarse facial features of acromegaly: that a recent eye exam showed background diabetic
enlarged tongue and spaces between his teeth. His retinopathy. He is allergic to lisinopril, which caused
blood pressure is 152/80 mm Hg, and his heart rate a cough; the remainder of his history was unremark-

t
ne
is 64 beats per minute. His visual ields are normal on able. His physical exam is notable for blood pressure
confrontation testing. He has multiple skin tags, and of 120/70 mm Hg and heart rate 72 beats per minute.
his hands are very large and sweaty. he remainder of He has intact sensation to monoilament and nor-

e.
his examination is unremarkable. Laboratory testing mal ankle jerk relexes; the remainder of the physical
of his pituitary reveals normal thyroid, adrenal, and examination is normal. Laboratory data reveal normal

in
gonadal function. However, his insulin-like growth electrolytes, serum creatinine 0.8 mg/dL, fasting glu-
factor-1 (IGF-1) and growth hormone (GH) levels are cose 140 mg/dL, normal liver function tests, total cho-

ic
elevated at twice the upper limit of normal. Suspecting lesterol 170 mg/dL, HDL 45 mg/dL, triglycerides 135
recurrence of his acromegaly, you obtain a pituitary mg/dL, LDL 90 mg/dL, and HbA1c 7.2%. A urine
MRI, which does not show obvious signiicant tumor
burden. An oral glucose tolerance test performed to
ed test shows a ratio of urine albumin to creatinine of 89
mg/g (normal <30 mg/g). hese results are all similar
assess GH suppression conirms that his acromegaly is to blood test results obtained by his prior physician 5
sm
not in biochemical remission. months earlier.
Which of the following is the initial treatment of What is the best intervention to reduce this patient’s
choice for this patient? risk of developing complications of his diabetes?
A. Medical therapy with a dopamine agonist A. Initiate therapy with labetalol.
ok

B. Medical therapy with a somatostatin analogue B. Prescribe niacin.


C. Observation only C. Refer for intravitreal injection of bevacizumab.
D. Radiation therapy to the pituitary D. Replace metformin with insulin glargine.
bo

E. Repeat transsphenoidal pituitary tumor resection E. Start therapy with telmisartan.

25. A 76-year-old woman presenting with a 1-week history 27. A 38-year-old woman is evaluated for progressive
e

of a cough, fever, and audible stridor is diagnosed with shortness of breath, decreased exercise tolerance, and
://

community-acquired pneumonia. A chest x-ray does not extreme fatigue. Heart and lung examination results
show evidence of an iniltrate, but it does reveal marked are negative, but chest x-ray shows bilateral hilar
tp

rightward tracheal deviation with a visible mediastinal lymphadenopathy with reticular opacities in the lower
soft tissue mass. A noncontrast chest CT scan reveals lung zones. Routine serum chemistries show serum
multiple bilateral thyroid nodules with an 8.5-cm left calcium 11.8 mg/dL, albumin 4.0 g/dL, and intact
ht

lower pole nodule extending below the clavicle and ster- PTH <10 pg/mL.
num with compression and narrowing of the trachea. Which test is most likely to explain her hypercal-
Laboratory tests show TSH <0.001 mU/L, free T4 2.8 cemia?
ng/dL, T4 12.9 µg/dL, and T3 245 ng/dL. A thyroid A. 1,25-dihydroxyvitamin D
uptake and scan reveals 24-hour uptake of 37% with B. 24-hour urinary calcium excretion
tracer accumulation localized to two right-sided thyroid C. 25-hydroxyvitamin D
nodules and the substernal left-sided thyroid nodule. D. Parathyroid hormone–related peptide (PTHrP)
What is the next appropriate step in management? E. Technetium-99m bone scan
A. Administer a 30-mCi dose of 131I.
B. Check pulmonary function tests with low–vol- 28. A 19-year-old man with an unremarkable medical his-
ume loops. tory presents because of delayed puberty. His height is
C. Refer the patient to a thyroid surgeon. 171.5 cm, arm span is 178 cm, and weight is 92 kg.
D. Start levothyroxine at a dose of 137 µg daily. On physical examination, he has Tanner stage 1 pubic
E. Start methimazole at a dose of 10 mg daily. hair, gynecomastia, and small testes. Sense of smell is
110 C HA P T E R 5 Endocrinology

intact. He has no history of testicular injury or mumps index 31.2 kg/m2) and asthma; he has no complications
orchitis. Laboratory evaluation shows total testoster- of his diabetes. He is not taking any medications other
one 88 ng/dL (normal 220–1000; conirmed by repeat than metformin, with which he is compliant. Liver and
testing), LH 30 mU/mL (normal 1.7–8.6), FSH 32 kidney function are normal. He wishes to improve his
mIU/mL (normal 1.5–12.4), normal prolactin, and glucose control, but it is important for him that he does
HbA1c 6.6%. not regain any of the weight that he lost.
What other testing would be warranted? What would be the next best step in the manage-
A. Echocardiogram ment of this patient’s hyperglycemia?
B. Ferritin, iron/total iron-binding capacity A. Initiate glipizide 5 mg twice daily.
C. Karyotyping B. Initiate insulin glargine 10 U at bedtime.
D. Pituitary MRI C. Initiate linagliptin 5 mg daily.
E. Renal ultrasound D. Initiate pioglitazone 30 mg daily.
E. Initiate repaglinide 0.5 mg before each meal.
29. A 35-year-old Asian American man with history of
advanced melanoma was diagnosed with panhypopi- 32. A 54-year-old man who was previously healthy is

t
ne
tuitarism secondary to ipilimumab-related hypophysi- being worked up for weakness and generalized body
tis. His biochemical testing included an undetectable aches, progressively worsening over 10 months. He
total testosterone level and very low LH and FSH lev- denies any change in food intake or bowel habits.

e.
els. You start him on testosterone replacement therapy Other than acetaminophen, he has not taken any
with a daily topical gel. hree months later, he returns medications. He does not smoke or consume alco-

in
for follow-up and feels greatly improved. hol. Physical examination was remarkable for bony
What tests are recommended? tenderness throughout. A plain x-ray of a particularly

ic
A. Hematocrit painful right humerus showed severe osteopenia and
B. LH metaphysial changes consistent with osteomalacia.
C. Liver function tests
D. Prolactin
ed Laboratory testing found phosphorus of 0.9 mg/dL
(reference range 3.5–5.0 mg/dL), alkaline phosphatase
E. Prostate-speciic antigen (PSA) 234 IU/L, 25-hydroxyvitamin D 35 ng/mL, and nor-
sm
mal basic metabolic panel and complete blood count.
30. A 45-year-old woman presents to the clinic for her Which of the following laboratory tests is most
annual examination. She is doing well overall but likely to be elevated?
reports new-onset fatigue, constipation, and a 3-kg A. 1,25-dihydroxyvitamin D
ok

weight gain over the last year. She attributes these to B. 24-hour urine calcium
work-related stress. On further questioning, she reports C. Calcitonin
oligomenorrhea. Her mother went through menopause D. Fibroblast growth factor 23 (FGF23)
bo

at the age of 50, and the patient believes she may be E. Intact PTH
perimenopausal. Laboratory results show the following:
sodium 130 mEq/L, potassium 3.5 mEq/L, creatinine 33. A 28-year-old woman presents to clinic with new-onset
e

0.80 mg/dL, glucose 90 mg/dL, FSH 5.7 IU/L, TSH amenorrhea of 6 months’ duration. She had menarche
://

60 (mIU/L), prolactin 35 ng/mL, total cholesterol 276 at the age of 13. She has had normal 28-day menstrual
mg/dL, HDL 47 mg/dL, and LDL 169 mg/dL. cycles for the majority of her life until approximately
tp

What is the best treatment option for this patient? 6 months ago. She speciically denies acne, hirsutism,
A. 1 liter luid restriction galactorrhea, headaches, visual changes, and heat or
B. Treatment with a statin cold intolerance. She does, however, report that she
ht

C. Treatment with cabergoline has recently started running competitively and is cur-
D. Treatment with levothyroxine rently training for her third marathon. She reports
E. Treatment with menopausal hormone replace- that after experiencing a stress fracture during train-
ment therapy ing for her second marathon, she was disappointed by
her marathon results. She is now following a calorie-
31. A 44-year-old man is referred to your clinic for the man- restricted diet and training 6 days per week to improve
agement of type 2 diabetes. his was diagnosed 3 years her results at an upcoming marathon.
prior and has been treated with metformin and nutri- What is the most likely etiology of this patient’s sec-
tion referral. he patient lost 4 kg by dieting with the ondary amenorrhea?
help of a smartphone app; despite this, his HbA1c values A. Cushing disease
have remained in the 7.2%–8.1% range; the most recent B. Hyperprolactinemia
value was 7.6%, and his recent fasting ingerstick glu- C. Hypothalamic amenorrhea
cose values have been in the range of 130–150 mg/dL. D. PCOS
Past medical history is signiicant for obesity (body mass E. Pregnancy
CHAPTER 5 Endocrinology 111

34. A 74-year-old man with metastatic prostate cancer 126/82 mm Hg, and his pulse is 76 beats per minute.
developed weakness of his legs and incontinence. He does not have lid lag, and his visual ields by con-
Imaging studies revealed metastatic foci of cancer in frontation are normal. His neurologic examination is
his spine with evidence of spinal cord compression. He otherwise normal. No tremor is noted. He does not
was started on dexamethasone 2 mg every 6 hours to look cushingoid. His thyroid function tests show the
reduce spinal cord edema, and radiation therapy was following: TSH 0.1 mIU/L (normal 0.5–5 mIU/L)
initiated. he following day, he experienced transient and free T4 0.6 ng/dL (normal 0.9–1.7 ng/dL).
tachycardia and lightheadedness with mild orthostatic What adjustments should be made to his levothy-
hypotension. A morning cortisol level was 0.5 µg/dL. roxine dose?
What is the best interpretation of this cortisol value? A. Decrease levothyroxine dose.
A. Primary adrenal insuiciency B. Discontinue levothyroxine completely.
B. Secondary adrenal insuiciency due to metastatic C. Increase levothyroxine dose.
lesion to the pituitary D. No change; continue current levothyroxine dose.
C. Secondary adrenal insuiciency due to radiation
therapy 37. A 78-year-old woman is seen for type 2 diabetes diag-

t
ne
D. his cortisol value cannot be adequately inter- nosed over 25 years ago. She also has hypertension and
preted. had an anterior wall myocardial infarction diagnosed
2 years ago. A recent eye examination is signiicant for

e.
35. A 62-year-old man with a history of poorly controlled a mild nonproliferative diabetic retinopathy, which
type 2 diabetes is hospitalized for treatment of acute has been stable for many years. She has no evidence of

in
pyelonephritis after presenting with a 5-day history of any other diabetes-related complications. For the last
fever, dysuria, and left-sided lank pain. Blood cultures 5 years, her HbA1c has been consistently around 6.5%.

ic
obtained on admission grow Escherichia coli. Labora- She is treated with metformin, liraglutide, and 22 U
tory tests checked on intake reveal TSH 0.22 mU/L, of insulin glargine once daily. Following her repeated
T4 3.1 µg/dL, and T3 64 ng/dL. He is treated with
broad-spectrum intravenous antibiotics with minimal
ed episodes of asymptomatic hypoglycemia as low as 39
mg/dL, you have decreased her insulin dose to 18 U.
improvement. An abdominal CT scan checked 6 days She has come today to review her blood glucose read-
sm
after admission reveals indings consistent with a left- ings. Morning glucose levels are between 90 and 125
sided perinephric abscess that requires management mg/dL, and before lunch and dinner, her glucose levels
with percutaneous drainage. he patient is discharged range between 125 and 165 mg/dL. She has had no
to a rehabilitation facility 23 days after admission and hypoglycemic events since the adjustment of the insu-
ok

has laboratory tests checked on intake that show TSH lin dose. However, her HbA1c has increased to 7.4%.
16.4 mU/L. His current weight is 218 lb. What would he patient is concerned about the recent increase in
you recommend? HbA1c, and she would like to optimize her regimen to
bo

A. Obtain a pituitary MRI scan. meet treatment goals.


B. Obtain a thyroid uptake and scan. What is the best next step in management of this
C. Start levothyroxine 50 µg daily. patient’s diabetes?
e

D. Start liothyronine 5 µg twice daily. A. Add glyburide.


://

E. Wait 6 weeks and recheck a full proile of thyroid B. Discontinue liraglutide and start insulin aspart
hormone levels. with her meals.
tp

C. Increase insulin glargine.


36. A 61-year-old man who has a history of a nonfunc- D. No further adjustment of her regimen.
tioning pituitary macroadenoma presents for routine E. Provide the patient with a continuous glucose-
ht

follow-up. He underwent surgical resection a number monitoring device.


of years ago for a large tumor and has had no evi-
dence of recurrence. He has hypopituitarism and is 38. A 45-year-old man with a history of dyslipidemia was
on hormone replacement therapy for hypothyroidism, recently found to have an HbA1c of 6.3% on routine
adrenal insuiciency, and hypogonadism. He takes screening blood tests. He has no known history of
levothyroxine, hydrocortisone, and a topical testoster- heart disease but has a strong family history of pre-
one gel. He returns for routine follow-up. He denies mature coronary artery disease. His only medications
headaches or visual changes. He complains of fatigue are aspirin 81 mg daily and atorvastatin 80 mg daily.
and inability to lose weight. He has noted very dry skin he patient states that he tries to eat a healthy diet
but attributes it to the weather. He denies tremor, pal- but admits to eating large portion sizes. His wife feels
pitations, insomnia, or change in bowel movements, that there is certainly much room for improvement as
although he tends toward constipation. He denies far as the patient’s diet. He exercises on a treadmill
sexual dysfunction. On physical examination, he has for 20–30 minutes twice weekly. He is overweight and
gained 2.5 kg since his last visit. His blood pressure is reports that his weight has been stable for many years.
112 C HA P T E R 5 Endocrinology

he patient feels well overall and has no speciic com- new-onset fatigue and has found it diicult to keep
plaints today. up with her children. She had no previous history of
On examination, his blood pressure was 126/78 headaches, but over the last 6 months, she has noted
mm Hg, heart rate was 70 beats per minute, and BMI several episodes of headaches. On examination, her
was 27 kg/m2. His physical examination was otherwise BMI is 31 kg/m2, and she has a round, plethoric face
unremarkable. You obtain screening laboratory tests; with supraclavicular fullness. She has evidence of
the results show a fasting blood glucose level of 110 abdominal striae. She has no expressible galactorrhea.
mg/dL, and a repeat HbA1c is 6.2%. In addition, you Which laboratory test would most likely help
receive copies of his old medical records, which indi- determine the cause of this woman’s secondary amen-
cate that his HbA1c was 6% about 1 year ago. orrhea?
What would you recommend as the irst step in the A. 24-hour urine free cortisol
treatment of this patient’s dysglycemia? B. HbA1c
A. Dietary/lifestyle interventions C. LH-to-FSH ratio
B. Start metformin. D. Serum prolactin
C. Start insulin therapy. E. TSH, FT4

t
ne
D. Start a dipeptidyl peptidase-4 (DPP-4) inhibitor.
42. A 45-year-old woman had a 5-year history of hyper-
39. A 50-year-old man presents with a wrist fracture after tension and anxiety. She was treated with lisinopril

e.
a minor fall. He is generally in good health other than and venlafaxine. She reports a new increase in anxi-
a history of irritable bowel syndrome and some weight ety along with palpitations in the preceding 5 weeks,

in
loss. He takes 1200 mg calcium and 800 IU vitamin and her blood pressure has been trending higher than
D3 daily because his mother had osteoporosis. Bone usual at approximately 145/80 mm Hg (typically

ic
mineral density measurement shows a T-score of −2.5 135–140/80 mm Hg). She has been under a lot of
at the hip and −3.0 at the 1/3 distal radius of the non- pressure at work, which has heightened her anxiety.
fractured wrist. On examination, he has a scaly skin
rash on his arms and trunk. Laboratory test results
ed She denies any illicit drug use. Plasma metanephrines
were ordered and revealed plasma metanephrines
reveal serum calcium 8.7 mg/dL, 25-hydroxyvitamin <0.20 nmol/L (0–0.49) and plasma normetanephrines
sm
D 24 ng/dL, intact PTH 120 pg/mL, and 24-hour 1.6 nmol/L (0–0.89).
urine calcium excretion of 38 mg with creatinine What is the most likely cause of the elevated
1000 mg. normetanephrine levels in this patient?
What intervention is most likely to improve his A. Extraadrenal paraganglioma
ok

skeletal health? B. Lisinopril use


A. Doubling his calcium intake to 2400 mg daily C. Methamphetamine use
B. Doubling his vitamin D intake to 1600 IU daily D. Pheochromocytoma
bo

C. Oral alendronate 70 mg weekly E. Venlafaxine use


D. Parathyroidectomy
E. Starting a gluten-free diet 43. A 55-year-old man with type 2 diabetes mellitus,
e

coronary artery disease, hypertension, dyslipidemia,


://

40. A 28-year-old man presents for evaluation of infertil- and obesity returns for follow-up after a recent hospi-
ity. His serum LH, FSH, and testosterone levels are all talization for an acute myocardial infarction. He was
tp

normal. Semen analysis reveals azoospermia. diagnosed with diabetes 5 years ago and was treated
Which of the following gene mutations is most initially with metformin alone and subsequently a
likely related to the above indings? combination of metformin 1000 mg twice daily and
ht

A. CFTR gene glipizide 10 mg twice daily. Although he reports com-


B. FGFR1 gene pliance with his medications, his HbA1c at the time of
C. KAL1 gene his hospital admission was 9.1%. He was discharged
D. KISS1 gene to home on a regimen of insulin glargine 40 U once
E. TAC3 gene daily, insulin aspart 10 U before each meal, and met-
formin 1000 mg twice daily. He has been monitoring
41. A 40-year-old woman presents to the clinic for new- his glucose levels at home and has worked with a phar-
onset amenorrhea of 12 months’ duration. She has macist in your clinic to optimize his insulin dose; he is
two children and had previously had regular 28-day currently taking insulin glargine 80 U twice daily and
menstrual cycles for her reproductive life. Her mother aspart 30 U with each meal using insulin pens. His
went through menopause at the age of 50, and she ingerstick glucose values at home are in the 200s at all
is concerned that she is now going through early times, and his HbA1c has decreased to 8.6%.
menopause. On further questioning, she reports a What is the most appropriate treatment of his
10-kg weight gain over the last year. She also reports hyperglycemia at this point?
CHAPTER 5 Endocrinology 113

A. Add a DPP-4 inhibitor such as sitagliptin. very lipid-rich density (−50 Hounsield units). He had a
B. Add quick-release bromocriptine. normal blood pressure and denied any episodic adrener-
C. Increase the dose of insulin glargine and insulin gic symptoms, weight gain, muscle weakness, alopecia, or
aspart by 15%. abnormal hair growth. He denied melena or blood in his
D. Switch from glargine to NPH insulin. stools, and he had recently had a normal colonoscopy and
E. Switch to regular insulin, U-500. prostate-speciic antigen test. He had smoked one pack of
cigarettes per day for more than 25 years, but he denied
44. A 74-year-old woman is complaining of left hip cough, bloody sputum, shortness of breath, or fever.
pain for the past year, which has been progressively What is the most likely diagnosis?
worsening. She denies early morning stifness, fevers, A. Adrenal adenoma
chills, weight loss, or trauma. On physical examina- B. Adrenal carcinoma
tion, her left hip has intact range of motion with no C. Adrenal myelolipoma
pain on active or passive range of motion. Routine D. Adrenal pheochromocytoma
laboratory work reveals an elevated alkaline phospha- E. Lung cancer metastases to the adrenal gland
tase of 205 IU/L and a 25-hydroxyvitamin D level

t
ne
of 32 ng/mL; the remainder of her laboratory panel 47. A 74-year-old man with a history of hypertension and
was normal, including calcium, phosphate, creati- hypercholesterolemia is hospitalized after presenting with
nine, γ-glutamyltransferase, and aminotransferases. A a 3-month history of progressive fatigue and dyspnea on

e.
plain x-ray of the hip and pelvis shows patchy areas of exertion. His weight on admission is 74 kg; his pulse is
osteolysis with multifocal sclerotic patches of the left 44 beats per minute; and laboratory tests show creatine

in
hemipelvis, including near the hip joint. A bone scan phosphokinase (CPK) 528 U/L, TSH 65 mU/L, free T4
shows uptake of radiotracer in the pelvic lesion. 0.2 ng/dL, and T4 2.3 µg/dL. A pharmacologic nuclear

ic
Which of the following is the best management stress test reveals indings consistent with difuse isch-
strategy at this time? emia. Subsequent coronary angiography reveals difuse
A. Monitor alkaline phosphatase levels with no treat-
ment unless it exceeds 400 IU/L.
ed three-vessel disease that is not amenable to percutaneous
stenting. A consulting cardiologist has recommended
B. Prescribe alendronate 70 mg by mouth once that he undergo coronary artery bypass surgery.
sm
weekly. What would you recommend?
C. Prescribe salmon calcitonin, 50 units subcutane- A. Administer 70 µg of levothyroxine intravenously
ously once daily. once daily.
D. Refer for zoledronic acid infusion 5 mg intrave- B. Check antithyroid peroxidase and antithyroglobu-
ok

nously. lin antibodies.


C. Defer further evaluation and treatment until after
45. A 68-year-old woman presents for follow-up after her he has undergone revascularization.
bo

irst bone mineral density (BMD) scan showed osteo- D. Start levothyroxine at a dose of 25 µg daily.
porosis, with T-scores of −2.6 at the lumbar spine and E. Start liothyronine 5 µg three times daily.
−2.4 at the femoral neck. She has had no fractures and
e

no other relevant past medical history. She underwent 48. A 27-year-old woman presents with a 3-month history
://

menarche at age 11 and menopause at age 49, with of amenorrhea. She reports menarche at age 12 and
normal menstrual cycles throughout her life except regular menses. She states she is healthy and denies
tp

during her two pregnancies. Her physical examination other medical problems; she takes no medications.
was normal with no features of endocrinopathies, no She denies galactorrhea. She is not currently talking
kyphosis or spinal tenderness, and normal dentition. any medications except a birth control pill. She noted
ht

Which of the following laboratory tests is indicated a 10-pound weight gain over the last 6 months and
at this time? occasionally feels cold. She attributes the weight gain
A. 24-hour urine free cortisol to dietary changes and lack of exercise. Her examina-
B. Complete blood count tion is unremarkable, except that she had expressible
C. Insulin-like growth factor-1 (IGF-1) galactorrhea on breast examination. A prolactin level is
D. Serum protein electrophoresis ordered and comes back elevated at 52 ng/dL (normal
E. Tryptase <22 ng/dL); her TSH level was normal at 3.1 mU/L.
In addition to repeating the prolactin test, what is
46. A 55-year-old man had an abdominal CT scan to evalu- the next test that should be ordered in this patient?
ate the source of abdominal pain. No acute abnormali- A. Humphrey visual ield testing
ties to explain his abdominal pain were found, and his B. Pelvic ultrasound
pain spontaneously resolved. However, a 3-cm adrenal C. Pituitary MRI
mass was noted in his left adrenal gland. he mass was D. Serum dopamine level
described as round, without calciications, and with a E. Urine pregnancy test
114 C HA P T E R 5 Endocrinology

49. A 28-year-old woman with a 6-year history of hypo- galactorrhea. She notes that her menses have been
thyroidism due to Hashimoto thyroiditis is treated more irregular over the last year and that her last
with levothyroxine at a dose of 125 µg daily; her last menstrual period was approximately 6 weeks ago.
TSH was 1.6 mIU/L. She is currently planning preg- She reports occasional headaches but otherwise feels
nancy and is asking for your advice about manage- well. She does not take any medications or herbal
ment of her hypothyroidism in this setting. supplements. Laboratory testing reveals a negative
How do you advise her? pregnancy test, TSH 2.2 mIU/L, and prolactin of
A. Increase the dose of levothyroxine to 137 µg while 360 ng/mL. You suspect a prolactinoma and order
attempting pregnancy. a pituitary MRI, which reveals a 10-mm × 12-mm
B. Increase the dose of levothyroxine to nine pills per pituitary lesion. here is no cavernous sinus inva-
week once she is pregnant. sion or compression of the optic nerves by the pitu-
C. Obtain monthly TSH levels until she is pregnant. itary tumor.
D. Recommend fetal thyroid ultrasound. What is the initial treatment of choice?
E. Switch from levothyroxine to a preparation of A. Medical therapy with a dopamine agonist
combined T4 and T3. B. Medical therapy with a somatostatin analogue

t
ne
C. Observation only
50. A 35-year-old woman with a history of regu- D. Radiation therapy to the pituitary
lar menses presents with complaints of bilateral E. Transsphenoidal pituitary tumor resection

e.
Chapter 5 Answers

in
ic
1. ANSWER: D. Metformin therapy is contraindi- 2. ANSWER: B. Alendronate 70 mg orally weekly
cated on the basis of renal dysfunction. Learning objective: Choose appropriate therapies to
Learning objective: Determine suitability of metfor-
min therapy in a patient on the basis of comorbidities.
ed improve bone strength in a patient with primary hyper-
parathyroidism who is refusing parathyroidectomy.
he irst-line treatment for most patients with type his patient has osteoporosis and therefore meets
sm
2 diabetes is metformin. However, there is a concern criteria for parathyroid surgery. However, because
of using metformin in patients with decreased glo- she refuses surgery, there is no reason to proceed with
merular iltration rate (GFR) due to the potential for imaging procedures to localize an adenoma. Similarly,
lactic acidosis. he Food and Drug Administration a 24-hour urine test will not change management if
ok

(FDA) recently changed the label information for she refuses surgery. Cinacalcet will reduce both serum
metformin-containing products. Prior recommen- calcium and PTH levels but does not improve bone
dations considered metformin contraindicated in mineral density (BMD) in patients with primary
bo

men and women with serum creatinine concentra- hyperparathyroidism. Annual monitoring would
tions ≥1.5 and ≥1.4 mg/dL, respectively. he new be appropriate in the absence of already established
recommendation suggests that metformin use is osteoporosis. Given that primary hyperparathyroidism
e

safe in patients with an estimated GFR as low as 30 is associated with bone loss and increased fracture risk,
://

mL/min/1.73 m2. However, the FDA recommends the best answer is to treat the patient with an antire-
against initiating metformin in patients whose esti- sorptive agent for osteoporosis, such as alendronate.
tp

mated GFR is below 45 mL/min/1.73 m2. his She should, however, be advised that recent data show
recommendation is also endorsed by the Ameri- much better results with surgical management of pri-
can Diabetes Association (ADA) and the European mary hyperparathyroidism compared with treatment
ht

Association for the Study of Diabetes. In this elderly with antiresorptive agents.
woman, the estimated GFR ranges from 17 to 41 mL/ Marcocci C, Bollerslev J, Khan AA, Shoback DM. Medical
min/1.73 m2, depending on the calculation used. It management of primary hyperparathyroidism: Proceedings
is therefore not appropriate to start metformin in this of the fourth international workshop on the management of
patient at any dose. Her heart failure appears stable, asymptomatic primary hyperparathyroidism. J Clin Endo-
crinol Metab. 2014;99(10):3607–3618.
with no evidence of decompensation, and therefore
it is not a contraindication. In this patient, lifestyle
interventions alone are appropriate because recent 3. ANSWER: E. Start spironolactone.
diabetes guidelines suggest that less-stringent HbA1c Learning objective: Select appropriate treatment for
goals (such as <8%) may be appropriate for patients hirsutism in PCOS.
with a limited life expectancy. Although there are several diferent treatment
Lipska KJ, Bailey CJ, Inzucchi SE. Use of metformin in the set- strategies for PCOS, the best treatment strategy
ting of mild-to-moderate renal insuiciency. Diabetes Care. should be tailored to the patient’s presenting symp-
2011;34(6):1431–1437. toms and goals. his patient is not interested in
CHAPTER 5 Endocrinology 115

pregnancy and is most concerned about symptoms of Funder JW, Carey RM, Fardella C, et al. Case detection, diag-
hirsutism. Although metformin and weight loss may nosis, and treatment of patients with primary aldosteron-
eventually help ameliorate her symptoms, they are ism: an Endocrine Society clinical practice guideline. J Clin
unlikely to help in the short term. After 6 months of Endocrinol Metab. 2008;93(9):3266–3281.
treatment on an oral contraceptive, it is unlikely that
continued treatment would provide further improve- 5. ANSWER: A. Initiation of intravenous insulin with
ment. Spironolactone, an antiandrogenic agent, has glucose target of 140–180 mg/dL
been shown to help with hirsutism. However, it has Learning objective: Determine appropriate targets and
teratogenic efects and should be used in conjunc- management strategies for hyperglycemia in inpatients.
tion with an oral contraceptive in young women of he current standards of care and guidelines, includ-
reproductive age. ing the American Diabetes Association guidelines
Legro RS, Arslanian SA, Ehrmann DA, et  al. Diagnosis and published in 2012 for the medical care of patients with
treatment of polycystic ovary syndrome: an Endocrine diabetes, recommend that in critically ill patients in
Society clinical practice guideline. J Clin Endocrinol Metab. the hospital, insulin therapy be initiated for the treat-
2013;98(12):4565–4592. ment of persistent hyperglycemia starting at a thresh-

t
ne
old of no greater than 180 mg/dL, and once insulin
4. ANSWER: B. Insuicient information to conirm a therapy is started, a glucose range of 140–180 mg/dL
diagnosis is recommended for the majority of these patients. he

e.
Learning objective: Identify medications that interfere optimal target range is a topic that has been debated.
with testing for primary hyperaldosteronism. Van den Berghe et  al. conducted the irst prospec-

in
his patient’s progressive and resistant hypertension tive randomized trial comparing tight blood glucose
and newly worsening hypokalemia both likely suggest that control (target 80–110 mg/dL) with intensive insu-
lin therapy with conventional blood glucose control

ic
he has underlying primary hyperaldosteronism. However,
there is insuicient information at this time to conirm (180–200 mg/dL) in critically ill surgical patients. In
that diagnosis. he initial screening test for hyperaldo-
steronism involves obtaining a serum aldosterone level
ed this study, which included over 1500 patients, 63%
of whom had undergone cardiac surgery before ICU
and plasma renin activity together. hese should be con- admission, researchers found that tight blood glucose
sm
sidered in the following settings: moderate to severe or control resulted in a signiicant reduction in mortal-
resistant hypertension, spontaneous or diuretic-induced ity (10.6% with intensive treatment vs. 20.2% with
hypokalemia in hypertensive individuals, and when there conventional treatment, p = .005) in patients who
is hypertension in the setting of an incidentally discov- required ≥5 days of ICU care with multiorgan fail-
ok

ered adrenal mass. An aldosterone-to-renin ratio (ARR) ure and sepsis. In addition, cardiac surgical mortality
that is greater than at least 20–30 should be considered was reduced in those patients requiring ≥5 days of
as a positive screen for primary hyperaldosteronism that ICU care with other conditions. After this study, for a
bo

requires further conirmatory testing (for example, a salt/ while, it was widely accepted that tight blood glucose
saline suppression test). It is expected that in addition to control (80–110 mg/dL) is better than conventional
a high ARR, the plasma renin activity will also be very control in surgery or in the ICU. However, since then,
e

low or suppressed in primary hyperaldosteronism as the several randomized trials have failed to show a beneit
://

expected physiologic response to the volume retention of tight blood glucose control with intensive insulin
induced by the high aldosterone levels. therapy, and authors of a meta-analysis of 29 random-
tp

Many antihypertensive medications alter the func- ized studies focusing on the beneits and risks of tight
tion of the renin-angiotensin-aldosterone system; glucose control (very tight ≤110 mg/dL or moderately
therefore, the patient’s medications must be carefully tight <150 mg/dL) in critically ill adult patients con-
ht

considered before interpreting the ARR. Spirono- cluded that tight glucose control was not associated
lactone, a mineralocorticoid inhibitor, functions to with signiicantly reduced hospital mortality but was
inhibit the negative feedback of aldosterone on renin associated with an increased risk of hypoglycemia.
release and therefore will cause a rise in plasma renin In addition, a recent large prospective randomized
activity. his patient has an ARR <20 and unsup- multicenter trial (the NICE-SUGAR study) demon-
pressed plasma renin activity, both of which are more strated that intensive blood glucose control with a
consistent with secondary hyperaldosteronism and target of 81–108 mg/dL increased mortality among
likely relect the efect of spironolactone. herefore, adults in the ICU compared with conventional blood
to obtain a reliable screening ARR to diagnose pri- glucose control with a target of ≤180 mg/dL. here-
mary hyperaldosteronism, spironolactone should be fore, although stricter glycemic control (110–140 mg/
stopped and substituted for another medication to dL) may be appropriate in selected patients if it can
control blood pressure that does not interfere with the be achieved without signiicant hypoglycemia, on the
ARR (such as an alpha-antagonist), and measurement basis of currently available evidence and guidelines,
of ARR should be repeated after several weeks. maintaining a blood glucose target between 140 and
116 C HA P T E R 5 Endocrinology

180 mg/dL would be the appropriate/recommended Pregnancy should be delayed until at least 6 months
target for this patient. he patient will need to be tran- after radioactive iodine treatment to minimize the risk
sitioned to a subcutaneous insulin regimen before the of fetal exposure. Patients treated with radioactive
intravenous insulin infusion is discontinued, but he iodine need to take some precautions to avoid expos-
may be discharged on no antidiabetic agents or on an ing family members to excreted 131I for the irst 2–4
oral regimen only. days after treatment, but strict, prolonged isolation
Subcutaneous insulin regimens are not recom- is not necessary. Radioactive iodine treatment may
mended in critically ill patients, owing to the many exacerbate thyroid eye disease. Pretreatment with glu-
variables that impact blood glucose levels, includ- cocorticoids may help minimize this risk in patients
ing hypermetabolic neurohormonal stress responses with moderate to severe changes. Most patients with
in the setting of critical illness/surgery, the patient’s Graves disease who are treated with radioactive iodine
nutritional status (e.g., taking nothing or very little will progress to develop postablative hypothyroidism
by mouth), potential requirement for pressors, and so that will require treatment with levothyroxine replace-
forth. In this setting, a continuous intravenous infu- ment. It may take up to 2–6 months to see the full
sion of insulin yields better blood glucose control and efects of treatment after administration of a dose of

t
ne
is the optimum and recommended method for man- radioactive iodine.
agement of hyperglycemia. Glucagon-like peptide 1 American hyroid Association Taskforce on Radioiodine
receptor agonists are not recommended in critically ill Safety. Radiation safety in the treatment of patients with

e.
patients. thyroid diseases by radioiodine 131I: practice recommen-
NICE-SUGAR Investigators. Intensive versus conventional dations of the American hyroid Association. hyroid.
glucose control in critically ill patients. N Engl J Med. 2011;21(4):335–346.

in
2009;360(13):1283–1297.

ic
8. ANSWER: E. Hydrocortisone
6. ANSWER: B: Hydrocortisone 50 mg IV every Learning objective: Implement empiric steroid
8 hours
Learning objective: Recognize and treat pituitary apo-
ed treatment in a patient at high risk of adrenal insuf-
iciency.
plexy. he patient has recently ceased use of chronic glu-
sm
his patient has pituitary apoplexy: ischemia and/ cocorticoids. She likely still has an element of chronic
or hemorrhage into the pituitary gland. It may occur secondary adrenal insuiciency. Exogenous glucocor-
in a normal gland, but often it occurs in a gland that ticoids suppress pituitary ACTH release and thereby
is abnormal due to a preexisting adenoma or difuse cause a decline in adrenal cortisol production. With
ok

enlargement such as that seen in pregnancy. She is at chronic glucocorticoid therapy and chronic ACTH
high risk for central adrenal insuiciency, and she is inhibition, endogenous ACTH and cortisol produc-
hypotensive. She should therefore be given a gluco- tion may require many days, weeks, or even months
bo

corticoid such as hydrocortisone as soon as possible to normalize following the cessation of glucocorti-
(ideally after blood has been drawn for cortisol and coid therapy. It is for this reason that glucocorticoids
adrenocorticotropic hormone [ACTH]). Bromocrip- are typically tapered gradually after prolonged use.
e

tine is appropriate management for a prolactinoma Although exogenous prednisone use in this patient
://

but is not indicated in the emergent management of likely resulted in ACTH suppression and thereby
pituitary apoplexy. he patient may have central hypo- cortisol deiciency, the stimulation and production of
tp

thyroidism, but this does not manifest for several days, endogenous adrenal mineralocorticoids remain intact
owing to the long half-life of thyroxine, and therefore secondary to stimulation by the renin-angiotensin
treatment with levothyroxine is not indicated at this system and potassium. herefore, this patient should
ht

point. he patient may require surgical resection, but receive empiric hydrocortisone therapy to prevent
empiric treatment for adrenal insuiciency is more precipitating an adrenal crisis in the setting of the
important at this point. Although the patient had physiologic stress of surgery. She does not need treat-
hemorrhage into the pituitary gland, there is no indi- ment for mineralocorticoid deiciency (with agents
cation that she is coagulopathic and therefore no indi- such as ludrocortisone), androgen deiciency (with
cation for vitamin K administration. DHEA), or catecholamine deiciency (epinephrine).
Rajasekaran S, Vanderpump M, Baldeweg S, et al. UK guide- A preoperative assessment of the patient’s adrenal
lines for the management of pituitary apoplexy. Clin Endo- function with a baseline morning cortisol level fol-
crinol (Oxf ). 2011;74(1):9–20. lowed by a stimulation test with synthetic ACTH
(cosyntropin) would also be reasonable, but these
7. ANSWER: A. She should wait until she is 6 months should be done several days in advance of the surgery
out from treatment before trying to conceive. and not perioperatively.
Learning objective: Counsel a patient on the risks and Oelkers W. Adrenal insuiciency. N Engl J Med. 1996;335(16):
beneits of radioactive iodine treatment for Graves disease. 1206–1212.
CHAPTER 5 Endocrinology 117

9. ANSWER: D. Add pioglitazone in those with osteoporosis by bone mineral density


Learning objective: Select appropriate antidiabetic criteria (T-score below −2.5), most fractures occur
treatment to avoid hypoglycemia when metformin mono- in patients with osteopenia because the prevalence
therapy has failed. of osteopenia in the population is much higher. As a
his patient’s HbA1c of 7.7% is still higher than result, several clinical prediction rules have been devel-
goal (<7%) despite her maximum eforts with lifestyle oped to identify those patients with osteopenia at high-
interventions and metformin, and therefore adding a est risk of fractures, who would presumably beneit the
second agent would be indicated. Of the choices pro- most from therapy. he most validated risk prediction
vided, given this patient’s concerns for hypoglycemia, tool is the FRAX calculator, which gives the 10-year
pioglitazone (a thiazolidinedione) would be the best absolute risk of major osteoporotic fractures and of hip
option because all the other agents can cause hypo- fractures. he current recommendation in the United
glycemia. Glipizide, a sulfonylurea, and nateglinide, a States is to initiate therapy in osteopenic individuals
meglitinide, both act on the beta-cells of the pancreas when the risk of a major osteoporotic fracture exceeds
to stimulate insulin release, whereas insulin degludec 20% or when the risk of a hip fracture exceeds 3%.
is a long-acting insulin. Cosman F, de Beur SJ, LeBof MS, et al. Clinician’s guide to

t
ne
Inzucchi SE, Bergenstal RM, Buse JB, et  al. Management of prevention and treatment of osteoporosis. Osteoporos Int.
hyperglycemia in type 2 diabetes, 2015: a patient-centered 2014;25(10):2359–2381.
approach: update to a position statement of the American

e.
Diabetes Association and the European Association for the 12. ANSWER: B. Measure a morning total testosterone
Study of Diabetes. Diabetes Care. 2015;38(1):140–149.
and sex hormone–binding globulin.

in
Learning objective: Choose diagnostic tests for male
10. ANSWER: C. Hypoparathyroidism hypogonadism.

ic
Learning objective: Distinguish between causes of he diagnosis of male hypogonadism requires mea-
hypocalcemia on the basis of other laboratory indings. surement of early-morning serum total testosterone
Osteoblastic bone metastases from prostate cancer
can lead to hypocalcemia from deposition of calcium
ed levels. A low value should be conirmed by repeating
the measurement along with a serum sex hormone–
in the bone metastases, particularly in patients with binding globulin (SHBG) level. Men with obesity
sm
widespread disease along with calcium and vitamin and type 2 diabetes mellitus often have low SHBG,
D deiciencies. his patient has a normal alkaline resulting in low total testosterone but normal “free”
phosphatase and no evidence of widely metastatic dis- (“biologically active”) testosterone levels. Because the
ease. His vitamin D, though on the low end, should reliability of “free testosterone” assays varies greatly, it
ok

not result in this degree of hypocalcemia. Hungry is best to measure total testosterone plus SHBG and
bone syndrome generally presents after surgical cure then calculate the free testosterone level using a widely
of hyperparathyroidism and would result in both available calculation tool. Prior to conirmation of
bo

hypocalcemia and hypophosphatemia because both hypogonadism, it is too early to perform other tests or
electrolytes are rapidly deposited into previously to begin treatment.
unmineralized bone. Hypomagnesemia can lead to Bhasin S, Cunningham GR, Hayes FJ, et al. Testosterone ther-
e

functional hypoparathyroidism, but there is nothing apy in men with androgen deiciency syndromes: an Endo-
://

in the history to suggest that this patient would be at crine Society clinical practice guideline. J Clin Endocrinol
risk for low magnesium. he most likely etiology is Metab. 2010;95(6):2536–2559.
tp

hypoparathyroidism resulting from previous thyroid


cancer surgery. Patients with decreased parathyroid 13. ANSWER: A. Advise her that she can likely stop
reserve may manifest overt hypocalcemia when given insulin therapy; initiate metformin and reduce
ht

a potent antiresorptive agent such as zoledronic acid insulin doses to help achieve this.
or denosumab for osteoporosis or cancer. In this situa- Learning objective: Recognize ketosis-prone diabetes
tion, the sudden loss of calcium resorption from bone and initiate appropriate therapy.
and inadequate PTH response can lead to profound his patient presented with diabetic ketoacidosis
hypocalcemia and hyperphosphatemia, as seen in this but has several features of type 2 diabetes: family his-
case. tory of type 2 diabetes, obesity and acanthosis nigri-
Shoback D. Hypoparathyroidism. N Engl J Med. 2008;359 cans on physical exam, and negative anti-GAD65
(4):391–403. antibodies. She therefore is unlikely to have type 1 dia-
betes (which would require lifelong insulin therapy);
11. ANSWER: D. 20% she is more likely to have type 2 diabetes. Certain
Learning objective: Initiate antiresorptive therapy in individuals with type 2 diabetes present with diabetic
patients with osteopenia based on current guidelines. ketoacidosis, thought to be secondary to acute insu-
hough the risk of a major osteoporotic fracture lin deiciency due to toxic efects of hyperglycemia on
(spine, forearm, hip, or shoulder fractures) is highest beta-cells leading to reduced insulin secretion. Upon
118 C HA P T E R 5 Endocrinology

correction of the hyperglycemia, beta-cell function for agranulocytosis and liver abnormalities. An ESR
often recovers, and patients can be managed with is not helpful in the determination of management of
standard therapies for type 2 diabetes. herefore, this Graves disease.
patient who is experiencing weight gain and hypo- Bahn RS, Burch HB, Cooper DS, et  al. Hyperthyroidism
glycemia as a result of her insulin regimen should be and other causes of thyrotoxicosis: management guide-
started on metformin as the standard irst-line therapy lines of the American hyroid Association and American
for diabetes, and her total daily dose of insulin should Association of Clinical Endocrinologists. Endocr Pract.
2011;17(3):456–520.
be reduced with a goal of stopping it completely if she
does well on metformin alone.
Balasubramanyam A, Nalini R, Hampe CS, Maldonado M. 16. ANSWER: A. Central diabetes insipidus
Syndromes of ketosis-prone diabetes mellitus. Endocr Rev. Learning objective: Diagnose disorders of water bal-
2008;29(3):292–302. ance after pituitary surgery.
he symptoms and presentation in this case are
14. ANSWER: E. Send for a complete blood count. most concerning for the development of postopera-
Learning objective: Indicate appropriate workup of a tive central diabetes insipidus. Diabetes insipidus is a

t
ne
discrepant HbA1c value. syndrome of hypotonic polyuria and is due to either
his patient has an HbA1c value that is discrepant a deiciency of arginine vasopressin, also known as
from her glucose measurements; the irst step in man- antidiuretic hormone (ADH), or an inadequate renal

e.
agement should be to identify the etiology of the dis- response to ADH. Patients with diabetes insipidus
crepancy. Common causes of inaccurate HbA1c values often complain of extreme thirst and will excrete large

in
include conditions that change erythrocyte survival amounts of very dilute urine. Without this hormone,
(the shortened survival in patients with blood loss or individuals cannot adequately concentrate their urine

ic
hemolytic anemias will falsely lower HbA1c values; and, if not allowed access to liquids, may develop
patients with B12/folate anemias can have falsely ele- severe, life-threatening hypernatremia. Central dia-
vated HbA1c values). In addition, certain hemoglobin
variants can interfere with the HbA1c measurement.
ed betes insipidus is due to deiciency, either partial or
complete, of ADH. Central diabetes insipidus can
In this patient, the most likely cause of a falsely low occur after any pituitary surgery. It can be transient
sm
HbA1c is a blood loss anemia related to her menor- or permanent and is seen more frequently in patients
rhagia, and a complete blood count will diagnose this. with larger tumors or those that tend to invade the
Her HbA1c is not accurate, and the patient should pituitary stalk, such as craniopharyngiomas. Given the
therefore not be reassured. Although glucometer clinical presentation of a pituitary lesion, this patient’s
ok

errors could also cause discrepancies, her glucose value diabetes insipidus would be much more likely to be
as measured in the laboratory corroborates the values central than nephrogenic. Psychogenic polydipsia, a
from her glucometer. Mutations in the HBB gene primary disorder of increased water intake, is associ-
bo

could cause hemoglobinopathies such as sickle cell ated with polyuria and dilute urine, but it would be
disease, but this could also be diagnosed on the basis associated with a low or low-normal serum sodium
of the complete blood count, so genetic testing is not level, not hypernatremia as seen in this patient. he
e

yet indicated. A continuous glucose monitor will just diagnosis is not likely SIADH, because this condition
://

show the same indings as her ingerstick glucose val- is typically associated with hyponatremia. Type 2 dia-
ues, namely persistent hyperglycemia, and it is there- betes mellitus would also not be a likely cause of this
tp

fore not the next best intervention. patient’s acute severe polyuria and polydipsia, because
Saudek CD, Derr RL, Kalyani RR. Assessing glycemia in dia- the fasting glucose is only mildly elevated.
betes using self-monitoring blood glucose and hemoglobin Lamas C, del Pozo C, Villabona C; Neuroendocrinology Group
ht

A1c. JAMA. 2006;295(14):1688–1697. of the SEEN. Clinical guidelines for management of dia-
betes insipidus and syndrome of inappropriate antidiuretic
15. ANSWER: E. Stop methimazole hormone secretion after pituitary surgery. Endocrinol Nutr.
2014;61(4):e15–e24.
Learning objective: Develop a long-term management
strategy for Graves disease.
his patient with Graves disease appears to be in 17. ANSWER: A. Primary adrenal insuiciency
remission on the basis of a normal TSH while taking Learning objective: Interpret hormone levels in the
only a low dose of methimazole. At this point, it is patient with adrenal insuiciency.
reasonable to stop her methimazole and monitor her his patient has primary adrenal insuiciency
TSH for any evidence of recurrence of hyperthyroid- (Addison disease) that is most likely secondary to an
ism. Should she have recurrence, radioiodine ablation autoimmune process, given her history of concomi-
or thyroidectomy may be reasonable options. Long- tant hypothyroidism at a young age. Primary adrenal
term therapy with methimazole is also an option, but insuiciency is characterized by the destruction of the
often it is not the preferred option, owing to concerns entire adrenal cortex, thereby resulting in deiciencies
CHAPTER 5 Endocrinology 119

in cortisol, the mineralocorticoid aldosterone, and assess the mass for malignant potential and hormone
adrenal sex hormones. he resulting clinical mani- functionality because either of these features can result
festations relect mineralocorticoid deiciency (hypo- in adverse health sequelae. he vast majority of inci-
volemia, salt wasting, hyponatremia, hyperkalemia), dental adrenal masses are benign adenomas that are
glucocorticoid deiciency (fatigue, weight loss, nau- characterized by a lipid-rich density (<10 Hounsield
sea), and adrenal androgen deiciency (loss of pubic units on CT scan), such as in the case of this patient;
and axillary hair). Characteristic laboratory examina- repeat imaging with MRI does not add to the ind-
tion results include very low cortisol and aldosterone ings described on the basis of the CT scan. Despite the
and very high ACTH and plasma renin activity to presence of reassuring radiographic features, a benign
relect physiologic compensation. Patients may also adenoma may still be functional; excess cortisol, aldo-
display signiicant hyperpigmentation. Administra- sterone, sex hormones, or catecholamines can all result
tion of cosyntropin results in minimal or no cortisol in long-term health complications.
increase, given the destruction of the adrenal cortex. Even when overt features of hypercortisolism
hese patients warrant immediate treatment with glu- (Cushing syndrome) or catecholamine excess (pheo-
cocorticoids, saline expansion, and ultimately miner- chromocytoma) are absent, the prevalence of subclini-

t
ne
alocorticoids, to prevent hemodynamic collapse and cal cortisol or catecholamine excess is suiciently high
life-threatening electrolyte abnormalities. that it is recommended that all patients with a newly
Secondary adrenal insuiciency refers to a disrup- discovered adrenal mass be assessed for these hormone

e.
tion at the level of pituitary production of ACTH. In excess states. he most eicient and speciic testing
the absence of ACTH, patients manifest glucocorti- involves performing a 1-mg dexamethasone suppres-

in
coid insuiciency; however, mineralocorticoid produc- sion test (to assess for cortisol excess) and plasma
tion (and therefore maintenance of sodium/volume metanephrines (for catecholamine excess). Serum

ic
homeostasis and potassium balance) remains intact. catecholamines are not reliable indicators, given their
When ACTH deiciency is prolonged, the adrenal variance in the blood; therefore, measurement of
glands will gradually atrophy and display a diminished
response to exogenous ACTH stimulation.
ed intermediates of catecholamine metabolism (meta-
nephrines) is preferred. When high blood pressure,
Bornstein SR, Allolio B, Arlt W, et  al. Diagnosis and treat- hypokalemia, or hirsutism is present, a serum aldoste-
sm
ment of primary adrenal insuiciency: an Endocrine Soci- rone and plasma renin activity, or adrenal androgen
ety clinical practice guideline. J Clin Endocrinol Metab. proile, should also be assessed.
2016;101(2):364–389. Young WF Jr. he incidentally discovered adrenal mass. N Engl
J Med. 2007;356(6):601–610.
ok

18. ANSWER: B. Estradiol 10-µg vaginal pill


Learning objective: Choose appropriate treatment for 20. ANSWER: E. Test for the presence of antiglutamic
genitourinary menopausal symptoms. acid decarboxylase (GAD) antibodies.
bo

his patient has genitourinary menopausal symp- Learning objective: Recognize features that should
toms. hese are best managed with local therapies. raise the suspicion for type 1 diabetes in adults and choose
A lubricant with intercourse may be suicient if the the correct test to make this diagnosis.
e

patient has symptoms only with intercourse, but An underlying secondary etiology of diabetes
://

because this patient has symptoms on a daily basis, should always be considered in patients with newly
a local low-dose estradiol formulation will be most diagnosed diabetes, especially if there are features that
tp

efective. Systemic estrogen or gabapentin would be would be atypical for type 2 diabetes, such as acro-
helpful to control the vasomotor symptoms of meno- megalic or cushingoid indings on exam, rapid failure
pause, but her symptoms are quite mild and probably of oral antidiabetic agents, lean body habitus with no
ht

do not warrant systemic treatment, especially because other indings of insulin resistance such as acanthosis
her risk of breast cancer is elevated due to her prior nigricans or skin tags, and no other metabolic abnor-
breast disease and her family history. malities (hypertension, dyslipidemia). he patient
Stuenkel CA, Davis SR, Gompel A, et al. Treatment of symptoms described has no physical exam features to suggest
of the menopause: an Endocrine Society clinical practice Cushing syndrome or acromegaly; thus, a 24-hour
guideline. J Clin Endocrinol Metab. 2015;100(11):3975– urinary free cortisol or IGF-1 level is not needed. he
4011. patient does not have symptoms to suggest a pancreatic
tumor or chronic pancreatitis as a cause of his hyper-
19. ANSWER: B. Dexamethasone suppression test glycemia; therefore, a pancreatic MRI is not indicated.
Learning objective: Outline the hormonal evaluation Maturity-onset diabetes of youth (MODY) is a mono-
of an incidentally discovered adrenal mass. genic form of diabetes with an autosomal dominant
he incidental detection of adrenal masses is increas- pattern of inheritance. Patients with MODY are usu-
ing as the frequency of abdominal imaging expands. ally diagnosed in adolescence and have a strong family
Once an adrenal mass is discovered, it is important to history of diabetes, which this patient does not have.
120 C HA P T E R 5 Endocrinology

In addition, in the absence of signiicant weight gain, Pharmacologic therapy for hypertension (such as
glycemic control in patients with MODY is unlikely chlorthalidone) is not indicated unless the blood pres-
to deteriorate. Testing for MODY is therefore not sure exceeds 140/90 mm Hg. Additional interven-
needed. hat leaves the most likely diagnosis of type 1 tions such as insulin to decrease glucose levels that are
diabetes. his form of diabetes accounts for 5%–10% already very well controlled have not been shown to
of diabetes cases and is a gradually progressive disease have a signiicant efect on cardiovascular outcomes.
that may be diagnosed on the basis of routine screen- Routine screening for cardiovascular disease in asymp-
ing while patients have only mild hyperglycemia. Type tomatic individuals with, for example, a coronary
1 diabetes is usually diagnosed from early childhood artery calcium score is not recommended.
to early adulthood, but it may occur at any age. It American Diabetes Association. Standards of medical care in
should therefore be considered even in adult patients, diabetes—2016 abridged for primary care providers. Clin
especially those who are lean, who do not have the Diabetes. 2017;35(1):5–26.
common metabolic abnormalities usually observed
in type 2 diabetes (such as hypertension and dyslip- 22. ANSWER: B. Obtain a thyroid scan.
idemia), and who lack a family history of diabetes. In Learning objective: Incorporate radionuclide imaging

t
ne
this patient, the rapid failure of oral medications to in the workup of thyrotoxicosis.
control blood glucose should increase the suspicion When a patient presenting with a thyroid nodule
for type 1 diabetes. Autoantibodies such as anti-GAD who is not taking levothyroxine is noted to have a sup-

e.
antibodies, islet cell autoantibodies, and insulin auto- pressed TSH level, a thyroid scan should be checked
antibodies may be used to diferentiate type 1 from to determine if the nodule itself is an autonomously

in
type 2 diabetes. functioning toxic adenoma. If there is radioiodine
Laugesen E, Østergaard JA, Leslie RD; Danish Diabetes Acad- uptake in the nodule on the thyroid scan, it is most
emy Workshop, Workshop Speakeers. Latent autoimmune

ic
likely benign and does not need to be biopsied. If there
diabetes of the adult: current knowledge and uncertainty. is no uptake into the nodule, ine-needle aspiration
Diabet Med. 2015;32(7):843–852. ed biopsy of the nodule should be performed to guide
further management. Treatment of a toxic adenoma or
21. ANSWER: B. Initiate rosuvastatin 20 mg daily. Graves disease with methimazole or radioactive iodine
sm
Learning objective: Assess and intervene on cardiovas- may be indicated, but it should be considered only
cular risk factors in patients with type 2 diabetes. after it has been determined whether a biopsy is neces-
his patient has diabetes and one risk factor for ath- sary. Referral for thyroid surgery would be indicated
erosclerotic cardiovascular disease (ASCVD), namely only if a biopsy of a cold nodule revealed suspicious or
ok

an LDL level >100 mg/dL. he ADA recommends malignant cytopathology or if there were contraindi-
high-intensity statin treatment, such as rosuvastatin cations to treatment of a toxic adenoma with radioac-
20 mg, for such individuals. he American Heart tive iodine or methimazole.
bo

Association (AHA) also recommends statin therapy Bahn RS, Burch HB, Cooper DS, et  al. Hyperthyroidism
but suggests moderate-intensity statin therapy in and other causes of thyrotoxicosis: management guide-
patients with diabetes aged 40–75 years with an LDL lines of the American hyroid Association and American
e

level of 70–189 mg/dL. A high-intensity statin can be Association of Clinical Endocrinologists. Endocr Pract.
://

2011;17(3):456–520.
considered if the 10-year risk of ASCVD (as estimated
using the AHA ASCVD risk calculator) is above 7.5%
tp

or if LDL cholesterol level exceeds 190 mg/dL. 23. ANSWER: E. Late-night salivary cortisol
Aspirin for primary prevention is recommended in Learning objective: Understand the tests available for
patients whose 10-year-risk of cardiovascular disease diagnosing cortisol excess.
ht

(as estimated using the ASCVD risk calculator) exceeds his patient should be screened for Cushing syn-
10% and can be considered on the basis of individual drome/hypercortisolism because of a number of con-
patient factors for patients whose risk is 5%–10%. In cerning clinical signs, including rapid weight gain and
those at lower risk, the risks exceed the beneits. In a new diagnoses of diabetes mellitus, hypertension, and
woman with diabetes, treatment with aspirin is gener- osteoporosis, as well as notable physical examination
ally not indicated unless she is over 60 years of age and indings (round face, hirsutism, supraclavicular fat
has at least one other major cardiovascular risk factor accumulation, pigmented striae, and bruising). After
(family history of premature atherosclerotic cardiovas- ruling out exogenous glucocorticoid exposure, there
cular disease, hypertension, smoking, dyslipidemia, are three acceptable irst-line screening tests for endog-
or albuminuria). In this patient, the ASCVD 10-year enous hypercortisolism: late-night salivary cortisol,
risk is 4.9%; aspirin is therefore not indicated. Even if 1-mg overnight dexamethasone suppression test, and
her risk were >5%, her prior history of gastritis (albeit 24-hour urine free cortisol with creatinine. he late-
appropriately treated) should be taken into account night salivary cortisol test takes advantage of the fact
before initiating aspirin therapy. that cortisol is secreted in a diurnal rhythm such that
CHAPTER 5 Endocrinology 121

nadir levels are usually observed late in the evening somatotrope cellular proliferation and GH secretion,
in a person without hypercortisolism. Patients with leading to a reduction in IGF-1 levels. Biochemi-
hypercortisolism lose the diurnal variation and show cal remission, deined as normalization of IGF-1 and
high cortisol levels when they should be at the nadir. GH levels, is associated with improvement or resolu-
he 1-mg overnight dexamethasone suppression test tion of acromegaly symptoms. Long-acting somatosta-
takes advantage of the fact that individuals with nor- tin analogues, such as lanreotide and octreotide LAR,
mal hypothalamic-pituitary-adrenal (HPA) axis physi- are typically considered the initial medical option in
ology will suppress cortisol secretion when given the uncontrolled acromegaly. he dopamine agonist caber-
synthetic steroid dexamethasone. Individuals with a goline can be used, especially if there are also elevations
normal HPA axis and thus normal inhibitory feed- in prolactin levels, but it is less eicacious than soma-
back mechanisms should suppress their cortisol level tostatin analogues. he GH antagonist pegvisomant is
to <1.8 µg/dL at 8:00 a.m. the day following ingestion also an option for this patient. his drug tends to be a
of 1-mg dexamethasone at 11:00 p.m. the day before. second-line therapy due to the need for frequent sub-
Although cortisol levels vary from minute to minute, cutaneous injections and its lack of inhibition of tumor
patients with Cushing syndrome show generally ele- growth. Observation would not generally be advised,

t
ne
vated levels over a 24-hour period. herefore, patients because the patient is symptomatic, and biochemical
with Cushing syndrome will have high 24-hour urine remission is associated with improved clinical outcomes
free cortisol levels that relect their overall high con- and reduced mortality. Repeat surgery would not be

e.
tinuous cortisol secretion over a 24-hour period. Uri- optimal in this case, because a clear surgical target was
nary creatinine should always be checked at the same not identiied. Finally, radiation is generally reserved

in
time as the 24-hour urinary free cortisol to ensure for patients with very aggressive tumors or those for
adequacy of the urine collection. hese three tests con- whom medical therapy fails.

ic
irm only hypercortisolism/Cushing syndrome and do Katznelson L, Laws ER Jr, Melmed S, et  al. Acromegaly: an
not indicate the cause of the cortisol hypersecretion. Endocrine Society clinical practice guideline. J Clin Endo-
Additional testing with an ACTH level is required to ed crinol Metab. 2014;99(11):3933–3951.
diferentiate between ACTH-dependent (most com-
monly pituitary Cushing disease or ectopic source) 25. ANSWER: C. Refer the patient to a thyroid sur-
sm
and ACTH-independent (likely adrenal source of cor- geon.
tisol overproduction) Cushing syndrome. Learning objective: Recommend appropriate manage-
An 8:00 a.m. ACTH or cortisol level is not appro- ment of a goiter causing tracheal compression.
priate for screening, because cortisol levels are dynamic A multinodular goiter that has extended subster-
ok

over time and inluenced by a number of factors, such nally to the point of causing tracheal compression
as acute pain and stress. he 8-mg dexamethasone sup- should be resected by an experienced thyroid surgeon,
pression test (as opposed to the 1-mg dexamethasone regardless of its functional status. he presence of
bo

suppression test) is used to help distinguish a pituitary audible stridor and evidence of tracheal narrowing on
source of excess ACTH from an ectopic source; it is radiographic images obviates the need for pulmonary
not an appropriate initial screening test. A cosyntro- function testing with low–volume loops. Treatment
e

pin stimulation test is used to diagnose adrenal insuf- with methimazole might help control hyperthyroid-
://

iciency rather than cortisol excess. ism caused by autonomously functioning thyroid
Nieman LK, Biller BMK, Findling JW, et al. he diagnosis of nodules, but it would not help shrink the dominant
tp

Cushing’s syndrome: an Endocrine Society clinical practice nodule to any extent. Treatment with 131I might help
guideline. J Clin Endocrinol Metab. 2008;93(5):1526–1540. shrink the dominant nodule over time, but it might
cause radiation-induced thyroiditis with expansion of
ht

24. ANSWER: B. Medical therapy with a somatostatin afected tissue that could severely compromise respira-
analogue tory function. Treatment with levothyroxine to try to
Learning objective: Describe the medical management suppress further enlargement of thyroid tissue may be
options for acromegaly. marginally efective at best in euthyroid patients, and
Multiple treatment modalities exist for recurrent it would be completely inefective in a patient present-
acromegaly. In a case such as this one, medical therapy ing with hyperthyroidism.
would be the initial treatment recommendation, par- Newman E, Shaha AR. Substernal goiter. J Surg Oncol.
ticularly because signiicant residual/recurrent pituitary 1995;60(3):207–212.
tumor is not visualized by MRI. here are a number
of medical treatment options, but somatostatin ana- 26. ANSWER: E. Start therapy with telmisartan.
logues are typically the initial treatment of choice in Learning objective: Initiate therapy for diabetic
this setting. GH-secreting pituitary tumor cells express nephropathy in a patient with allergy to ACE inhibitors.
somatostatin receptors, and treatment with soma- his patient has established diabetic nephropathy, as
tostatin analogues acting via these receptors inhibits evidenced by an elevated urine albumin-to-creatinine
122 C HA P T E R 5 Endocrinology

ratio. he beneit of inhibitors of angiotensin, either evaluation, typically causes hypogonadotropic hypo-
in the form of angiotensin-converting enzyme (ACE) gonadism, not primary gonadal failure.
inhibitors or angiotensin receptor blockers (ARBs) Dwyer AA, Phan-Hug F, Hauschild M, et  al. Transition in
such as telmisartan, has been well established and endocrinology: hypogonadism in adolescence. Eur J Endo-
proven to delay the progression to end-stage renal dis- crinol. 2015;173(1):R15–R24.
ease, even in patients with normal blood pressure. he
patient’s cough when treated with an ACE inhibitor 29. ANSWER: A. Hematocrit
is not a contraindication to ARB therapy. Additional Learning objective: Create an appropriate monitoring
blood pressure control with labetalol is not indicated, plan for a patient receiving hormone replacement therapy
because his blood pressure is already at goal. Niacin with testosterone.
could raise the HDL but has not been proven to have According to the Endocrine Society clinical guide-
beneit in preventing complications of diabetes and line on male hypogonadism, serum total testosterone
has side efects that limit its use. he patient only had and hematocrit should be monitored 3–6 months after
background retinopathy on the last eye exam; intra- initiating testosterone treatment. Testosterone levels
vitreal bevacizumab is used of-label in the treatment are measured to allow titration of the testosterone

t
ne
of proliferative diabetic retinopathy. Although the dose to achieve a serum testosterone level in the mid-
patient should have intensiication of his antidiabetic normal range. he hematocrit is measured to monitor
therapy, metformin should be continued unless a con- for erythrocytosis, a known side efect of testosterone

e.
traindication develops. therapy. If hematocrit is >54%, testosterone therapy is
American Diabetes Association. Standards of medical care in stopped until the hematocrit decreases to a safe level.
diabetes—2016 abridged for primary care providers. Clin

in
Measurement of PSA levels is not indicated in men
Diabetes. 2016;34(1):3–21. younger than 40 years of age. In men 40 years of age or

ic
older, check PSA level before initiating treatment and
27. ANSWER: A. 1,25-dihydroxy-vitamin D at 3–6 months. here is no recommendation to order
Learning objective: Describe the mechanism of hyper-
calcemia in granulomatous diseases.
ed liver function tests or measure prolactin levels.
Bhasin S, Cunningham GR, Hayes FJ, et al. Testosterone ther-
his patient has sarcoidosis, a granulomatous dis- apy in men with androgen deiciency syndromes: an Endo-
crine Society clinical practice guideline. J Clin Endocrinol
sm
order in which activated macrophages can express the
enzyme that converts vitamin D to its active form, cal- Metab. 2010;95(6):2536–2559.
citriol (1,25-dihydroxyvitamin D). his leads to both
hypercalcemia and hypercalciuria. he other choices are 30. ANSWER: D. Treatment with levothyroxine
ok

all reasonable to check in cases of non–PTH-mediated Learning objective: Identify hypothyroidism as a cause
hypercalcemia but would not explain the underlying of menstrual and laboratory abnormalities.
pathophysiology of hypercalcemia associated with granu- his woman presents for her annual examination
bo

lomatous disease. Treatment of the hypercalcemia in sar- with fatigue, constipation, weight gain, and oligomen-
coidosis would start with hydration and glucocorticoids. orrhea. Although it may be possible that the symptoms
Carroll MF, Schade DS. A practical approach to hypercalcemia. are a result of work-related stress and perimenopause,
e

Am Fam Physician. 2003;67(9):1959–1966. one should keep in mind that hypothyroidism may
://

also present in a similar manner. Her elevated TSH


28. ANSWER: C. Karyotyping is diagnostic of hypothyroidism. Given that her oli-
tp

Learning objective: Recognize the clinical phenotype of gomenorrhea, hyponatremia, hyperprolactinemia, and
Klinefelter syndrome and obtain appropriate diagnostic hyperlipidemia are most likely manifestations of her
evaluation. hypothyroidism, it would be best to treat her hypo-
ht

his patient has clinical symptoms and signs of thyroidism irst with levothyroxine before considering
male hypogonadism. His biochemical tests reveal treatment for these other clinical abnormalities.
primary gonadal failure as the cause of the hypogo- Garber JR, Cobin RH, Gharib H, et  al. Clinical practice
nadism. Klinefelter syndrome is relatively common guidelines for hypothyroidism in adults: cosponsored
among young males with primary hypogonadism. by the American Association of Clinical Endocrinolo-
Karyotyping will identify the chromosomal abnormal- gists and the American hyroid Association. Endocr Pract.
2012;18(6):988–1028.
ity (XXY). Echocardiogram and renal ultrasound will
not uncover the etiology of primary hypogonadism;
these may be indicated in a phenotypic woman with 31. ANSWER: C. Initiate linagliptin 5 mg daily.
Turner syndrome. Pituitary MRI is helpful in patients Learning objective: Incorporate patient preference in
with hypogonadotropic hypogonadism, but in this the choice of antidiabetic agent after failure of metformin
case, the FSH and LH would be low (or inappropri- monotherapy.
ately normal). Hemochromatosis, which would be here are various antidiabetic medications avail-
expected if there were signs of iron excess on laboratory able, and the American Diabetes Association, as
CHAPTER 5 Endocrinology 123

well as other professional associations, promotes 33. ANSWER: C. Hypothalamic amenorrhea


the individualization of the treatment approach, Learning objective: Identify excessive exercise as a
taking into account patient preferences, cost, and cause of amenorrhea.
potential side efects of each class of medications. his woman is experiencing secondary amenor-
In the absence of contraindications, metformin is rhea, deined as an absence of menses for more than
the preferred initial agent. here is a long-standing three cycles or 6 months in women who had previ-
evidence base for both the eicacy and safety of ously been menstruating. he most likely etiology in
metformin; it is not associated with weight gain or this case is hypothalamic amenorrhea. his description
hypoglycemia; and it may reduce the risk of car- its the classic description of the “female athlete triad,”
diovascular events and cancers. When avoiding which consists of an eating disorder, amenorrhea, and
weight gain is important to the patient, initiation osteoporosis. Although the exact pathophysiology of
of insulin therapy, sulfonylureas (such as glipizide), this disorder is unknown, it is believed that amenor-
meglitinides (such as repaglinide), and thiazolidin- rhea occurs because of a mismatch between nutritional
ediones (such as pioglitazone) should be avoided intake and energy expenditure.
because these are all associated with weight gain. Gordon CM, Ackerman KE, Berga SL, et al. Functional hypo-

t
ne
Linagliptin, an inhibitor of dipeptidyl peptidase-4 thalamic amenorrhea: an Endocrine Society Clinical Practice
(DPP-4), is weight neutral; that is, it is not associ- Guideline. J Clin Endocrinol Metab. 2017;102(5):1413–1439.
ated with weight loss or weight gain and is therefore

e.
the best option listed. Other options that could pro- 34. ANSWER: D. his cortisol value cannot be ade-
mote weight loss include glucagon-like peptide-1 quately interpreted.

in
(GLP-1) agonists, sodium/glucose cotransporter 2 Learning objective: Understand the efect of exogenous
(SGLT-2) inhibitors, or pramlintide. glucocorticoids on hypothalamic-pituitary-adrenal axis

ic
Inzucchi SE, Bergenstal RM, Buse JB, et  al. Management of function.
hyperglycemia in type 2 diabetes, 2015: a patient-centered Metastatic solid tumors have been known to metas-
approach: update to a position statement of the American ed tasize to the hypothalamus or pituitary and result in
Diabetes Association and the European Association for the secondary adrenal insuiciency. Solid tumors can also
Study of Diabetes. Diabetes Care. 2015;38(1):140–149.
metastasize to bilateral adrenal glands, although this
sm
scenario does not necessarily result in primary adrenal
32. ANSWER: D. Fibroblast growth factor 23 (FGF23) insuiciency. Symptoms of orthostatic hypotension
Learning objective: Recognize the role of FGF23 in are not pathognomonic for adrenal insuiciency, but
phosphate homeostasis. this entity should certainly be considered. In the set-
ok

his patient has severe hypophosphatemia caus- ting of dexamethasone use, cortisol values are reliable
ing osteomalacia. In the absence of a history of poor only for detecting inappropriately high cortisol states
oral intake, diarrhea, or antacid use, this is most (Cushing syndrome), not when investigating inap-
bo

likely resulting from urinary losses, which can be propriately low cortisol states (adrenal insuiciency).
conirmed by a high (or inappropriately normal) Dexamethasone is a synthetic glucocorticoid that is
urinary phosphate. Vitamin D deiciency and not detected by the conventional assay for cortisol;
e

primary hyperparathyroidism are the most com- however, like all glucocorticoids, it inhibits the pro-
://

mon causes of a mild hypophosphatemia, but this duction of ACTH and therefore suppresses endog-
patient’s vitamin D was normal, as was his calcium. enous cortisol production. herefore, a low cortisol
tp

Fanconi syndrome (type 2, or proximal, renal tubu- level in this patient most likely relects the expected
lar acidosis) is another cause of hypophosphatemia, action of dexamethasone. Whether he may also have
especially in patients with a paraproteinemic dis- concomitant brain metastases that have resulted in
ht

order or taking culprit medications, but it is asso- secondary adrenal insuiciency by compromising the
ciated with metabolic acidosis and hypokalemia, pituitary apparatus, or concomitant adrenal metasta-
which were not described here. Hence, this patient ses that have resulted in primary adrenal insuiciency,
is most likely to have tumor-induced osteomalacia, cannot be ascertained by this single testing performed
which is caused by FGF23 secretion from a mesen- in the setting of dexamethasone intake.
chymal tumor. FGF23 causes phosphate loss in the Bornstein SR, Allolio B, Arlt W, et  al. Diagnosis and treat-
urine with an elevated urine phosphate. 1,25-dihy- ment of primary adrenal insuiciency: an Endocrine Soci-
droxyvitamin D level is generally low or low-nor- ety clinical practice guideline. J Clin Endocrinol Metab.
mal in this disorder because the renal tubular defect 2016;101(2):364–389.
causing the phosphaturia also impairs synthesis of
1,25-dihydroxyvitamin D. Calcitonin and urinary 35. ANSWER: E. Wait 6 weeks and recheck a full pro-
calcium are not high. ile of thyroid hormone levels.
Hamnvik OP, Becker CB, Levy BD, Loscalzo J. Wasting away. Learning objective: Describe changes in thyroid func-
N Engl J Med. 2014;370(10):959–966. tion with acute nonthyroidal illness.
124 C HA P T E R 5 Endocrinology

he shifts in proiles of thyroid hormone levels 37. ANSWER: D. No further adjustment of her regimen.
seen in this patient are characteristic of physiologic Learning objective: Individualize glycemic targets in
changes that may occur during acute illness and sub- a patient at high risk of complications from hypoglycemia
sequent recovery in patients with normal endogenous he 2012 position statement of the American
thyroid function. Initial suppression of TSH, T4, and Diabetes Association (ADA) and the European Asso-
T3 levels during progression of a nonthyroidal illness ciation for the Study of Diabetes (EASD) suggests
may be misinterpreted as evidence of underlying cen- that an HbA1c goal as high as 7.5%–8.0% or even
tral hypothyroidism caused by hypothalamic or pitu- slightly higher is appropriate for patients with a his-
itary dysfunction. Subsequent transient elevation of tory of severe hypoglycemia; limited life expectancy;
the TSH level during recovery may be misinterpreted advanced complications; extensive comorbid condi-
as evidence of underlying primary hypothyroidism. tions; and those in whom the target is diicult to
In such cases, it is usually prudent to wait until the attain despite intensive self-management education,
patient has made a full recovery before attempting repeated counseling, and efective doses of multiple
to determine whether any thyroid dysfunction is glucose-lowering agents, including insulin. he
present. A thyroid uptake and scan would not pro- patient described in the question has a long dura-

t
ne
vide any useful information in this setting. Start- tion of disease with a history of myocardial infarc-
ing treatment with any form of thyroid hormone at tion and appears to have hypoglycemia unawareness.
this stage would be premature, and doing so would Intensifying therapy in this patient with a sulfonyl-

e.
make it diicult to interpret subsequent proiles of urea, prandial insulin, or increased basal insulin is
thyroid hormone levels. A pituitary MRI scan would likely to be associated with greater risks than bene-

in
be informative only if serial measurement of thyroid its. herefore, maintaining an HbA1c between 7.0%
hormone levels showed persistent low TSH levels and and 7.5% without hypoglycemia is appropriate. A

ic
low free T4 levels consistent with possible central continuous glucose monitor is unlikely to provide
hypothyroidism. any further information.
DeGroot LJ. he non-thyroidal illness syndrome [updated
2015 Feb 1]. In: De Groot LJ, Chrousos G, Dungan K,
ed Inzucchi SE, Bergenstal RM, Buse JB, et  al. Management of
hyperglycemia in type 2 diabetes, 2015: a patient-centered
et  al., eds, Endotext [Internet]. South Dartmouth, MA: approach: update to a position statement of the American
MDText.com, Inc.; 2000. https://www.ncbi.nlm.nih.gov/ Diabetes Association and the European Association for the
sm
books/NBK285570/ Study of Diabetes. Diabetes Care. 2015;38(1):140–149.

36. ANSWER: C. Increase levothyroxine dose. 38. ANSWER: A. Dietary/lifestyle interventions


ok

Learning objective: Titrate levothyroxine dose in Learning objective: Recommend appropriate manage-
patients with secondary hypothyroidism based on levels of ment of prediabetes.
free T4 and clinical indings. Prediabetes is a condition in which normal glu-
bo

his patient has central hypothyroidism due to cose homeostasis is compromised. It is characterized
his pituitary macroadenoma. Clinically, he has some by impaired fasting glucose (fasting plasma glucose of
symptoms suggestive of inadequately treated hypo- 100–125 mg/dL) and/or impaired glucose tolerance
e

thyroidism, including fatigue, weight gain, and (IGT) (plasma glucose of 140–199 mg/dL 2 hours
://

dry skin. He does not have symptoms or physical after a 75-g glucose load), and/or HbA1c levels of
examination indings suggestive of hyperthyroidism. 5.7%–6.4%. Prediabetes confers a three- to sevenfold
tp

His laboratory tests are classic for central hypothy- increase in the risk of developing overt type 2 diabe-
roidism—a low free T4 in combination with either tes compared with individuals with normal glucose
an inappropriately low or normal TSH. his case values. Evidence from numerous studies also suggests
ht

illustrates how TSH alone cannot be used reliably that the chronic complications of type 2 diabetes start
to assess thyroid hormone replacement in patients to develop during the prediabetic state. herefore, to
with central hypothyroidism and/or hypothalamic/ minimize the burden of complications associated with
pituitary dysfunction. Clinical indings and periph- hyperglycemia, early intervention, before overt diabe-
eral thyroid hormone levels must be used to guide tes develops, is prudent. However, there are currently
levothyroxine replacement therapy. herefore, in this no approved pharmacotherapies for prediabetes. Evi-
case, despite the low TSH, the levothyroxine dose dence derived from prevention studies supports the
should be increased with the goal of normalizing the hypothesis that early intervention with lifestyle modi-
free T4. ication or pharmacotherapy may slow the progression
Garber JR, Cobin RH, Gharib H, et  al. Clinical practice to diabetes by delaying the underlying pathophysiol-
guidelines for hypothyroidism in adults: cosponsored ogy of the disease. In the Diabetes Prevention Program
by the American Association of Clinical Endocrinolo- (DPP), in which 3234 individuals with impaired fast-
gists and the American hyroid Association. Endocr Pract. ing glucose or IGT were enrolled, intensive lifestyle
2012;18(6):988–1028.
modiication, aiming to achieve at least a 7% weight
CHAPTER 5 Endocrinology 125

loss and 150 minutes of physical activity per week, 40. ANSWER: A. CFTR gene
reduced the incidence of type 2 diabetes by 58% Learning objective: Distinguish genetic causes of infer-
compared with placebo after 2.8 years of follow-up. tility.
Patients randomized to treatment with metformin in About 1%–2% of infertile men have azoospermia
the DPP had a 31% reduction in the incidence of type due to congenital absence of the vas deferens, and
2 diabetes after 2.8 years of follow-up. here is also most of these patients have mutations in the CFTR
evidence derived from randomized clinical trials sug- gene. Mutations in this gene also causes cystic ibrosis,
gesting potential beneit of other oral hypoglycemic but men with mutations leading to absent vas defer-
agents, namely acarbose and thiazolidinediones, in ens do not need to manifest the full clinical picture
preventing the progression from prediabetes to overt of cystic ibrosis (such as pulmonary and pancreatic
diabetes, but these agents are either less efective or dysfunction). Mutations in the KISS1, KAL1, FGFR1,
have safety and tolerability issues. he most recent and TAC3 genes would cause hypogonadotropic
position statement issued by the ADA regarding stan- hypogonadism with low LH, FSH, and testosterone.
dards of medical care in diabetes, as well as a consensus Chillon M, Casals T, Mercier B, et al. Mutations in the cystic
statement by the American College of Endocrinol- ibrosis gene in patients with congenital absence of the vas

t
ne
ogy (ACE) and the American Association of Clinical deferens. N Engl J Med. 1995;332(22):1475–1480.
Endocrinologists (AACE), recommends lifestyle inter-
vention as the preferred treatment option for predia- 41. ANSWER: A. 24-hour urine free cortisol

e.
betes because it has been shown to be safe and highly Learning objective: Recognize the clinical features of
efective. Starting metformin therapy, although likely Cushing syndrome and select an appropriate diagnostic

in
to be efective, is not yet indicated, because current test.
guidelines and available evidence would prioritize Given this patient’s presentation, the most likely

ic
initiation/intensiication of lifestyle interventions. In etiology for her amenorrhea is cortisol excess, possibly
this patient, interventions would be directed toward resulting from Cushing disease. herefore, a 24-hour
dietary modiications, increased exercise, and weight
loss eforts. here is no evidence that insulin or DPP-4
ed urine free cortisol would be a good screening test to
evaluate for cortisol excess. She has a classic presenta-
inhibitors have beneit in prediabetes. tion for cortisol excess, including progressive weight
sm
Knowler WC, Barrett-Connor E, Fowler SE, et al. Reduction in gain, fatigue, weakness, abdominal striae, and second-
the incidence of type 2 diabetes with lifestyle intervention ary amenorrhea. Cushing disease (pituitary ACTH-
or metformin. N Engl J Med. 2002;346(6):393–403. dependent Cushing syndrome) is more common in
women and typically presents between 25 and 45 years
ok

39. ANSWER: E. Starting a gluten-free diet of age. An LH-to-FSH ratio could be helpful in the
Learning objective: Recognize calcium malabsorption diagnosis of PCOS, but given her clinical presenta-
as a secondary cause of hyperparathyroidism and osteo- tion, cortisol excess is more likely. Hyperprolactinemia
bo

porosis. and hypothyroidism may cause secondary amenorrhea


It is important to recognize secondary hyperpara- but are less likely, given her clinical presentation.
thyroidism due to malabsorption and to note that in Nieman LK, Biller BM, Findling JW, et  al. he diagnosis of
e

some disorders, such as celiac disease, calcium malab- Cushing’s syndrome: an Endocrine Society clinical practice
://

sorption can occur even in the presence of reasonably guideline. J Clin Endocrinol Metab. 2008;93(5):1526–1540.
normal vitamin D levels. he clues to the diagnosis
tp

of celiac disease are the irritable bowel syndrome, 42. ANSWER: E. Venlafaxine use
weight loss, very low urinary calcium excretion, and Learning objective: Recognize factors that afect mea-
a rash that is consistent with dermatitis herpetifor- surements of plasma metanephrines.
ht

mis on physical examination. Increasing calcium he assessment of plasma metanephrines should


and vitamin D will not necessarily correct the sec- take into account medications that can raise their lev-
ondary hyperparathyroidism, and oral alendronate els. By the time a catecholamine-producing tumor,
is unlikely to be absorbed. Parathyroidectomy is such as a pheochromocytoma or paraganglioma, is
clearly inappropriate for secondary hyperparathy- capable of causing systemic adrenergic symptoms,
roidism because the elevated parathyroid hormone is plasma metanephrine levels are typically greater than
an appropriate response to calcium malabsorption. fourfold higher than the upper limit of normal. It
A gluten-free diet, after conirming the diagnosis is not uncommon to see mild elevations in plasma
of celiac disease, will improve calcium and nutri- normetanephrines that may represent high sympa-
ent absorption and is the irst step in improving this thetic nervous system tone or medication-induced
patient’s skeletal health. efects. Medications that inhibit the reuptake of cat-
Cosman F, de Beur SJ, LeBof MS, et al. Clinician’s guide to echolamines in a synapse can increase circulating
prevention and treatment of osteoporosis. Osteoporos Int. plasma normetanephrines. hese medications include
2014;25(10):2359–2381. serotonin-norepinephrine reuptake inhibitors (such as
126 C HA P T E R 5 Endocrinology

venlafaxine), selective serotonin reuptake inhibitors, his patient has Paget disease, a bone condition of
and tricyclic antidepressants. he discontinuation of unknown etiology characterized by disordered bone
venlafaxine is likely to result in normalization of this resorption with a secondary increase in osteoblast activ-
patient’s values. ity, leading to disorganized bone architecture. Paget dis-
Neary NM, King KS, Pacak K. Drugs and pheochromocy- ease can be asymptomatic and diagnosed during workup
toma—don’t be fooled by every elevated metanephrine. N of an incidentally noted increased alkaline phosphatase
Engl J Med. 2011;364(23):2268–2270. level. In these asymptomatic cases, if a bone scan does
not show lesions in the skull or near joints, treatment
43. ANSWER: E. Switch to regular insulin, U-500. may be deferred, and the patient can be monitored for
Learning objective: Adjust antidiabetic management the development of signs and symptoms of progressive
in patients with severe insulin resistance. disease. However, in this symptomatic patient with
Severe insulin resistance (deined as the need for lesions close to the hip joint, treatment should be initi-
≥200 U of insulin per day to achieve glycemic control) ated. First-line treatment is a bisphosphonate, but usu-
is commonly seen with obesity and can complicate ally at higher doses than for osteoporosis. Oral options
diabetes management. he management of patients are alendronate at a dose of 40 mg by mouth once

t
ne
with diabetes who have severe insulin resistance is dif- daily or risedronate 30 mg by mouth once daily. Intra-
icult and at times frustrating, and it requires a multi- venous options include pamidronate (diferent dosing
faceted approach. regimens; a commonly used regimen is 60 mg every 3

e.
For obese patients, weight loss is the best treatment months) or zoledronic acid 5 mg (redosed when there is
option, but weight loss can be a challenging task for sign of recurrence after 1–2 years). Calcitonin is no lon-

in
patients to achieve and maintain. Medications that ger recommended as irst-line therapy for Paget disease.
decrease insulin needs, such as metformin, thiazolidin- Ralston SH. Paget’s disease of bone. N Engl J Med. 2013;
368(7):644–650.

ic
ediones (TZDs), and GLP agonists, might help, but
many patients still need high doses of insulin. In addi-
tion, treatment with TZDs is generally associated with
weight gain, particularly when combined with insulin.
ed
45. ANSWER: B. Complete blood count
Learning objective: Describe the routine laboratory
Bariatric surgery is highly efective for obese patients evaluation for secondary causes of osteoporosis in a patient
sm
with severe insulin resistance. with newly diagnosed low bone mass.
Delivering an appropriate insulin volume to these All patients with osteoporosis should be evaluated for
patients can be diicult and inconvenient. Most insu- secondary causes of osteoporosis. A medical history and
lin pens can deliver a maximal dose of 60–80 U per a physical exam should be focused on identifying risk
ok

injection, whereas patients using a syringe and vial can factors and underlying causes of bone loss. In addition,
take up to 100 U per injection. If doses exceed this all patients should have a laboratory evaluation for sec-
amount, the patient needs to take several consecutive ondary causes. Although the extent of laboratory testing
bo

injections. Furthermore, absorption of insulin from is unclear, the National Osteoporosis Foundation rec-
large-volume injections may be less eicient. his ommends in all patients consideration of sending for a
has led to the development of concentrated insulins complete blood count; routine chemistry levels, includ-
e

with 200–500 (U-200–U-500) U/mL. his allows ing calcium, phosphorus, magnesium, and renal/liver
://

for delivery of higher doses of insulin per injection. function tests; a thyroid-stimulating hormone level;
Improved control may occur because of better compli- a 25-hydroxyvitamin D level; bone turnover mark-
tp

ance with dosing (fewer total daily injections) or bet- ers; and a 24-hour urinary calcium. Men should also
ter insulin action and absorption. U-500 kinetics are have a testosterone level measured. In selected patients,
similar to premixed or NPH insulin. evaluation for primary hyperparathyroidism, multiple
ht

A DPP-4 inhibitor or bromocriptine will not be myeloma, celiac disease, Cushing syndrome, and sys-
suiciently potent to control this patient’s hypergly- temic mastocytosis may be indicated. In this patient,
cemia. Increasing the dose of insulin or switching to there are no unusual indings on the physical exam to
NPH insulin is not likely to be as efective as switching suggest any of these disorders, and therefore laboratory
to regular insulin, U-500. evaluation beyond the routine tests is not indicated.
Lane WS, Cochran EK, Jackson JA, et  al. High-dose insu- Cosman F, de Beur SJ, LeBof MS, et al. Clinician’s guide to
lin therapy: is it time for u-500 insulin? Endocr Pract. prevention and treatment of osteoporosis. Osteoporos Int.
2009;15(1):71–79. 2014;25(10):2359–2381.

44. ANSWER: D. Refer for zoledronic acid infusion 5 46. ANSWER: C. Adrenal myelolipoma
mg intravenously. Learning objective: Distinguish causes of adrenal
Learning objective: Describe the appropriate treat- masses on the basis of imaging characteristics.
ment for Paget disease based on symptoms and anatomic he radiographic phenotype of an adrenal mass
location of lesions. can provide valuable clues in distinguishing benign
CHAPTER 5 Endocrinology 127

from malignant adrenal processes. Benign adrenal 48. ANSWER: E. Urine pregnancy test
masses are typically lipid-rich and therefore have a Learning objective: Consider physiologic causes of
low density as measured on noncontrast CT scan hyperprolactinemia.
by x-ray attenuation (Hounsield units [HU]). Adi- Hyperprolactinemia is a relatively common cause
pose tissue tends to have a density of approximately of amenorrhea and is certainly part of the diferential
−20 to −150 HU. A density of <10 HU (lipid-rich diagnosis when a woman presents with missed men-
entity) on noncontrast CT scan almost always rep- strual cycles. It is important to replicate the labora-
resents a benign adrenal adenoma, except when the tory abnormality because performance of the breast
density is very low (less than −40 HU), in which examination immediately prior to blood sampling
case it almost always represents a myelolipoma. can cause a transient elevation in prolactin. Pregnancy
Myelolipomas are benign, lipid-rich growths that must be excluded as a cause of hyperprolactinemia in
are not hormonally active. all women of childbearing potential because prolac-
Some benign adrenal adenomas may have a lower tin levels rise as a normal physiologic response dur-
lipid content and therefore may have a more dense ing pregnancy. Another physiologic cause is primary
phenotype (>10 HU). Higher-density adrenal masses hyperparathyroidism because the elevated thyrotropin-

t
ne
increase the concern for pheochromocytomas, carci- releasing hormone (TRH) level seen in this condition
nomas, and other malignant entities. stimulates both TSH and prolactin release. Medica-
Young WF Jr. he incidentally discovered adrenal mass. N Engl tion-induced hyperprolactinemia is another cause to

e.
J Med. 2007;356(6):601–610. consider. A pituitary MRI to assess for the presence
of a pituitary adenoma is indicated only after other

in
47. ANSWER: D. Start levothyroxine at a dose of 25 µg causes of hyperprolactinemia have been excluded. If
daily. a mass is seen that impinges on the optic apparatus,

ic
Learning objective: Safely initiate thyroid hormone visual ields should be formally assessed. Although
replacement therapy in a patient with coronary artery dopamine is a regulator of prolactin release, serum lev-
disease.
his patient has primary hypothyroidism based
ed els are not helpful, because they do not relect levels
in the hypothalamo-hypophyseal portal system. A pel-
on the elevated TSH and low free T4 and total T4. vic ultrasound does not have a role in the workup of
sm
he treatment is levothyroxine. In view of his coro- hyperprolactinemia.
nary artery disease, levothyroxine should be started Melmed S, Casanueva FF, Hofman AR, et al. Diagnosis and
at a low dose (12.5–25 µg daily) to avoid trigger- treatment of hyperprolactinemia: an Endocrine Soci-
ing further ischemia as the metabolic rate starts to ety clinical practice guideline. J Clin Endocrinol Metab.
ok

increase. 2011;96(2):273–288.
His full replacement dose of 125 µg orally cor-
responds to an intravenous dose of 60–80 µg. How- 49. ANSWER: B. Increase the dose of levothyroxine to
bo

ever, starting this dose could exacerbate cardiac nine pills per week once she is pregnant.
ischemia. Similarly, liothyronine (synthetic T3) can Learning objective: Advise appropriate dose adjust-
rapidly increase metabolic rate and thus worsen car- ment of thyroid hormone replacement in a woman pursu-
e

diac ischemia. Checking antithyroid peroxidase ing pregnancy.


://

and antithyroglobulin antibodies would not pro- Levothyroxine requirements increase in pregnancy
vide any additional information. In the absence due to the increased levels of thyroid-binding proteins
tp

of any known history of thyroid surgery or exter- under the inluence of estrogen. A similar efect is
nal radiation treatment to the head and neck, it seen when initiating the combined oral contraceptive
can be presumed that his severe hypothyroidism is pill. In pregnancy, a dose increase of 30% is usually
ht

due to autoimmune thyroiditis. If the patient had required even early in pregnancy. herefore, women
needed an emergent cardiac intervention, it could with hypothyroidism who are of childbearing age
be performed without waiting for thyroid hormone should be advised to increase the dose of levothyrox-
replacement therapy. However, because this patient ine from seven pills per week to nine pills per week (so
does not need the procedure acutely, it would be take two pills on 2 days of the week) as soon as they are
safer to initiate treatment before revascularization pregnant. here is no need to increase the dose before
because severe hypothyroidism can increase the risk the patient is pregnant or to measure monthly TSH
of complications related to general anesthesia and values while she is attempting pregnancy, because her
the use of sedatives. last TSH was normal. Women attempting pregnancy
Garber JR, Cobin RH, Gharib H, et  al. Clinical practice should not take agents that contain T3, because T3
guidelines for hypothyroidism in adults: Cosponsored does not cross the placenta, and therefore the fetus
by the American Association of Clinical Endocrinolo- may be relatively hypothyroid. Fetal thyroid ultra-
gists and the American hyroid Association. Endocr Pract. sound is recommended in certain cases if the mother
2012;18(6):988–1028.
has had Graves disease.
128 C HA P T E R 5 Endocrinology

Alexander EK, Pearce EN, Brent GA, et al. 2017 Guidelines patients who do not tolerate or are not responsive to
of the American hyroid Association for the Diagnosis and the dopamine agonists. Radiation would be consid-
Management of hyroid Disease During Pregnancy and the ered only in very rare cases of advanced or progressive
Postpartum. hyroid. 2017;27(3):315–389. prolactinomas for which medical or surgical interven-
tions have failed. Observation alone would not be the
50. ANSWER: A. Medical therapy with a dopamine preferred approach in a patient with a macroadenoma
agonist (size >10 mm), because control of tumor growth
Learning objective: Choose appropriate therapies for a would be desired.
prolactin-secreting pituitary adenoma. Melmed S, Casanueva FF, Hofman AR, et al. Diagnosis and
Medical therapy with a dopamine agonist is con- treatment of hyperprolactinemia: an Endocrine Soci-
sidered the irst-line initial therapy for symptomatic ety clinical practice guideline. J Clin Endocrinol Metab.
prolactinomas. Dopamine agonists have both anti- 2011;96(2):273–288.
proliferative and antisecretory efects on prolactin-
producing pituitary cells and typically result in both Acknowledgment
decreased prolactin secretion and tumor shrinkage.

t
ne
Cabergoline is preferred over bromocriptine for most he author and editors gratefully acknowledge the contribu-
patients due to its increased eicacy and tolerability. tions of the previous authors, Carolyn Becker, Amir Tirosh,
Somatostatin analogues are used in the treatment of Le Min, Klara Rosenquist, Anand Vaidya, Whitney W.

e.
acromegaly. Transsphenoidal surgery is reserved for Woodmansee, Bindu Chamarthi, and Matthew Kim.

in
ic
ed
sm
ok
e bo
://
tp
ht
6
Nephrology and Hypertension
MEGAN PROCHASKA AND ERNEST I. MANDEL

1. A 55-year-old man with a history of stage 3 chronic kid- disoproxil, efavirenz, lisinopril, and amlodipine. His

t
ne
ney disease secondary to hypertensive nephropathy pres- laboratory values are notable for sodium 135 mEq/L,
ents with ongoing hypertension and mild bilateral lower potassium 2.5 mEq/L, chloride 110 mEq/L, bicarbonate
extremity swelling. His blood pressure was previously 19 mEq/L, blood urea nitrogen 24 mg/dL, creatinine 1.0

e.
controlled with hydrochlorothiazide. Laboratory results mg/dL, glucose 95 mg/dL, and phosphorus 1.5 mg/dL.
reveal an estimated glomerular iltration rate (GFR) of 30 Urinalysis reveals pH 6.0, 2+ glucose, and 2+ protein.

in
mL/min/1.73 m2. Complete blood count, transaminases, Which of the following is the likely cause of his
albumin, and coagulation studies are all normal. Urinaly- clinical presentation?
sis is normal. A recent echocardiogram was normal. A. Lamivudine

ic
What is the next step in managing the patient’s B. Tenofovir disoproxil
hypertension?
A. Start amlodipine.
ed C. Efavirenz
D. Lisinopril
B. Start lisinopril. E. Amlodipine
sm
C. Change hydrochlorothiazide to chlorthalidone.
D. Change hydrochlorothiazide to furosemide. 4. A 65-year-old woman with a history of stage 4 chronic
E. Low-salt diet kidney disease, hypertension, stroke, and recently
diagnosed lung cancer presents to the clinic for evalu-
ok

2. A 30-year-old man presents to the emergency depart- ation of asymptomatic anemia. Hemoglobin is 9 g/dL.
ment with a 3-week history of headaches and blurry Iron stores are normal. Vitamin B12 and folate levels
vision and is found to have a blood pressure of are found to be normal.
bo

270/140 mm Hg. Physical examination is notable What is the next step?


for retinal hemorrhages. he complete blood count A. Start an erythropoietin-stimulating agent.
reveals a white blood cell count of 5500/mm3, hemo- B. Transfuse packed red blood cells.
e

globin of 8 g/dL, and platelets of 50,000/µL. Serum C. Transfuse packed red blood cells and start an
://

creatinine is 19 mg/dL. Coagulation studies are nor- erythropoietin-stimulating agent.


mal. Fibrinogen is elevated. Lactate dehydrogenase D. Start oral iron therapy.
tp

(LDH) is elevated, reticulocyte index is calculated to E. Counsel the patient on vascular access for dialysis.
be elevated, and haptoglobin is low. A review of the
peripheral smear reveals schistocytes. he urine sedi- 5. A 72-year-old woman with three-vessel coronary artery
ht

ment is remarkable for several red blood cell casts. disease returns from the operating room after undergo-
Electrocardiography reveals the presence of left ven- ing coronary artery bypass grafting with the intraopera-
tricular hypertrophy. tive use of cardiopulmonary bypass, with postoperative
What is the diagnosis? laboratory tests revealing acute kidney injury and hyper-
A. Atypical hemolytic uremic syndrome kalemia with a potassium of 6.5 mEq/L. EKG shows
B. Glomerulonephritis peaked T waves and widened QRS complexes. Cal-
C. Hypertensive emergency cium, insulin, and dextrose are administered.
D. hrombotic thrombocytopenic purpura What is the next best management step for deini-
E. Disseminated intravascular coagulation tive treatment of hyperkalemia?
A. Furosemide
3. A 42-year-old man with a history of HIV on antiret- B. Sodium polystyrene sulfonate
roviral therapy (ART) and with hypertension presents C. Sodium polystyrene sulfonate with sorbitol
with a 1-week history of polyuria and progressive muscle D. Sodium bicarbonate
weakness. Medications include lamivudine, tenofovir E. Hemodialysis

129
130 C HA P T E R 6 Nephrology and Hypertension

6. A 47-year-old African American man with recently doses. here is no peripheral edema on examina-
diagnosed HIV infection in the setting of esophageal tion. Secondary hypertension workup reveals an
candidiasis is referred for evaluation of bilateral lower aldosterone-to-plasma renin activity ratio of 35,
extremity edema associated with a serum creatinine of with a serum aldosterone level of 30 ng/dL. After
2 mg/dL and 3+ albumin by urine dipstick. A 24-hour conirmatory testing is performed to demonstrate
urine collection reveals 6 g of proteinuria. Testing for inappropriately high aldosterone secretion, CT
hepatitis B and C is negative. After conirming the imaging of the abdomen and pelvis with contrast
suspected diagnosis by kidney biopsy, which of the is performed and reveals no evidence of adrenal
following treatment options should be pursued in pathology.
addition to an angiotensin-converting enzyme (ACE) What is the next step?
inhibitor? A. Adrenal vein sampling
A. Steroids alone B. MRI with gadolinium contrast
B. ART alone C. Addition of spironolactone
C. ART + steroids D. Renal artery stenting
D. Steroids + mycophenolate mofetil E. Renal sympathetic denervation

t
ne
E. Plasmapheresis
10. A 48-year-old woman with rheumatoid arthri-
7. A 71-year-old man with chronic kidney disease tis and Crohn disease presents to the clinic with

e.
presents for a routine visit. He is currently asymp- bloody diarrhea and abdominal pain, suspected to
tomatic. On physical examination, his jugular vein be related to Crohn lare. She is initiated on steroids

in
is not distended and he has no peripheral edema. in the clinic and sent to the emergency department.
Laboratory data are remarkable for serum sodium Laboratory tests reveal sodium 135 mEq/L, potas-

ic
of 130 mEq/L, potassium 4 mEq/L, chloride 104 sium 3.0 mEq/L, chloride 110 mEq/L, bicarbon-
mEq/L, bicarbonate 24 mEq/L, blood urea nitro- ate 16 mEq/L, blood urea nitrogen 20 mg/dL, and
gen 28 mg/dL, creatinine 3 mg/dL, and glucose 90
mg/dL. An osmolality gap is present. Urine dipstick
ed creatinine 1.2 mg/ dL. Urinalysis is within normal
limits. Urine pH is 6.5. he urine anion gap is posi-
analysis is negative for protein. Of the following, tive.
sm
which is the most likely explanation of the patient’s What is the likely cause of the patient’s hypokalemia?
hyponatremia? A. Loss of potassium and bicarbonate in stool
A. Hypovolemia B. Hypokalemia resulting from corticosteroid admin-
B. Low solute intake istration
ok

C. Multiple myeloma C. Distal (type 1) renal tubular acidosis (RTA)


D. Syndrome of inappropriate antidiuretic hormone D. Proximal (type 2) RTA
(SIADH) E. Type 4 RTA
bo

E. Hypothyroidism
11. A 29-year-old woman with lupus presents with a new
8. A 26-year-old woman is referred to the emergency onset of 3+ proteinuria on urine dipstick. A spot urine
e

department for evaluation of acute kidney injury. protein-to-creatinine ratio is 5. Serum creatinine is
://

Physical examination is remarkable for a blood pres- stable at 0.9 mg/dL. he patient has low C3 and C4,
sure of 130/80 mm Hg, difuse skin thickening, and elevated dsDNA titers, and evidence of red blood cell
tp

sclerodactyly. Laboratory tests reveal an acute rise in casts on urine microscopy. Renal biopsy is performed,
creatinine from 1 mg/dL to 4.5 mg/dL. White blood and histopathology demonstrates focal lupus nephritis
cell count is 4500/µL, hemoglobin is 7 g/dL, and (WHO class III).
ht

platelets are 110,000/µL. LDH is elevated and hap- What is the next step?
toglobin is low. Schistocytes are present on peripheral A. No therapy, continue close monitoring.
smear. B. ACE inhibitor and prednisone
What is the next best step? C. ACE inhibitor and rituximab
A. Renal biopsy D. ACE inhibitor, prednisone, and mycophenolate
B. Plasmapheresis mofetil
C. Captopril E. ACE inhibitor, prednisone, and cyclophospha-
D. Intravenous immunoglobulin therapy (IVIG) mide
E. Methylprednisolone
12. An 88-year-old man with a history of uncontrolled
9. A 41-year-old man presents with resistant hyper- hypertension and stroke was recently seen in the clinic
tension and hypokalemia. Physical examination is and started on amlodipine 5 mg daily and benazepril
remarkable for a blood pressure of 180/110 mm Hg 20 mg daily. He now presents to the clinic with blood
despite three antihypertensive agents at maximal pressure of 140/70 mm Hg.
CHAPTER 6 Nephrology and Hypertension 131

What is the next step in management? 16. A 44-year-old woman with a history of asthma and
A. Add chlorthalidone 12.5 mg daily. gastroesophageal relux disease (GERD) and chronic
B. Increase benazepril to 40 mg daily. cough presents with a fever, rash, and a newly noted
C. Increase amlodipine to 10 mg once daily. eosinophilia, associated with acute kidney injury. Uri-
D. Counsel on low-salt diet. nalysis is remarkable for cellular casts. She has been
E. Perform a workup for secondary causes of hyper- on inhaled steroids for treatment of her asthma and
tension. was recently initiated on treatment for GERD in the
setting of a persistent cough. he patient monitors her
13. A 52-year-old woman presents to the emergency room peak lows, and these have been normal. She otherwise
after a motor vehicle accident in which she sustained a has no symptoms.
crush injury and is found to have acute kidney injury What is the likely cause of the patient’s acute kid-
3 days after admission to the hospital. Laboratory data ney injury?
reveal sodium 137 mEq/L, potassium 4.3 mEq/L, A. Acute tubular necrosis
chloride 105 mEq/L, bicarbonate 25 mEq/L, creati- B. Acute interstitial nephritis
nine 3 mg/dL, calcium 6.5 mg/dL, and phosphorus 6 C. Acute glomerulonephritis

t
ne
mg/dL. Creatine kinase is found to be 10,000 U/L. D. Henoch-Schönlein purpura
What is the most appropriate treatment? E. Drug reaction with eosinophilic and systemic
A. Isotonic bicarbonate symptoms (DRESS) syndrome

e.
B. Isotonic saline
C. Furosemide 17. A 70-year-old man with a history of congestive heart

in
D. Hemodialysis failure, coronary artery disease, hypertension, and
E. Mannitol total body volume overload presents with recurrent

ic
admissions for pulmonary edema. His outpatient
14. An 18-year-old man presents with acute kidney injury blood pressures are well controlled on a combination
3 days after an upper respiratory tract infection, with
rise in creatinine from 0.9 to 1.6 mg/dL. he patient’s
ed of furosemide, carvedilol, amlodipine, and lisinopril.
He is compliant with his medications. Secondary
urinalysis reveals 2+ blood and 2+ protein. C3 and C4 workup of the patient’s hypertension reveals unilateral
sm
are normal. Serologic workup is negative for antineu- atherosclerotic renal artery stenosis.
trophil cytoplasmic antibody (ANCA) and antibodies What is the next step?
against glomerular basement membrane (anti-GBM). A. Percutaneous renal artery angioplasty with stent
Serum protein electrophoresis (SPEP) and urine pro- placement
ok

tein electrophoresis (UPEP) are within normal limits. B. Renal artery surgical revascularization
What is the likely diagnosis? C. Adding a thiazide diuretic
A. IgA nephropathy D. Increasing the dose of furosemide
bo

B. Postinfectious glomerulonephritis E. Referring for heart transplant


C. hin basement membrane disease
D. Alport syndrome 18. A 55-year-old woman with a history of long-standing
e

E. Granulomatous polyangiitis diabetes with macroalbuminuria and hypertension


://

presents to the clinic with ongoing hypertension. She


15. A 71-year-old man with a history of chronic obstruc- takes lisinopril 40 mg daily for management of hyper-
tp

tive pulmonary disease (COPD), smoking, and tension, but her blood pressure remains at 150/85
chronic hyponatremia attributed to SIADH pres- mm Hg. She reports compliance with a low-salt diet.
ents to the emergency department with altered men- Urine microalbumin-to-creatinine ratio reveals ongo-
ht

tal status, tongue biting, and urinary incontinence. ing macroalbuminuria.


Serum sodium is found to be 105 mEq/L, down from What is the next best agent to add?
125 mEq/L, which was noted on laboratory testing A. Add valsartan.
approximately 2 weeks prior to presentation. herapy B. Add chlorthalidone.
is initiated with hypertonic saline, and a repeat serum C. Add metoprolol.
sodium in 10 hours is found to be 120 mEq/L. His D. Add aliskiren.
mental status has now improved back to baseline. E. Add benazepril.
What is the next step in therapy?
A. Continue hypertonic saline. 19. A 32-year-old man returns from travel to India with
B. Stop hypertonic saline and start D5W and symptoms of productive cough, night sweats, chill,
DDAVP. and unintentional weight loss. He does not initially
C. Stop hypertonic saline and monitor closely. seek medical care and develops worsening malaise,
D. Switch luids to isotonic saline. nausea, and lightheadedness. When he presents
E. Switch luids to half-normal saline. to the emergency department, his blood pressure
132 C HA P T E R 6 Nephrology and Hypertension

is 84/45 mm Hg. Laboratory data are remarkable 23. A 60-year-old patient with autosomal dominant poly-
for sodium 122 mEq/L and potassium 6.1 mEq/L. cystic kidney disease underwent a living unrelated kid-
Serum creatinine is 1.1 mg/dL. ney transplant 10 years ago. He had been on stable
What is the next step in management? doses of tacrolimus, mycophenolate mofetil, and pred-
A. Fluid restriction nisone, but he accidentally took double the dose of
B. Hypertonic saline tacrolimus over the past 2 weeks.
C. Isotonic saline alone Which of these indings is associated with calcineu-
D. Isotonic saline with hydrocortisone rin inhibitor toxicity?
E. Tolvaptan A. Hypokalemia
B. Acute interstitial nephritis
20. A 30-year-old woman presents to the clinic for evalua- C. Hypomagnesemia
tion of an enlarging jaw mass. Laboratory results reveal D. Hypercalcemia
potassium 5.5 mEq/L, calcium 6.5 mg/dL, phospho- E. Rhabdomyolysis
rus 11 mg/ dL, and creatinine 1.4 mg/dL. Uric acid is
15 mg/dL. 24. A 38-year-old woman presents with newly diagnosed

t
ne
In addition to intravenous luids, what is the next Hodgkin lymphoma associated with bilateral lower
step in the appropriate management of this condition? extremity edema. Laboratory workup reveals 10 g of
A. Rasburicase proteinuria on a 24-hour urine collection.

e.
B. Allopurinol Which of the following pathologic entities most
C. Febuxostat likely explains the presence of proteinuria in this

in
D. Sodium polystyrene sulfonate patient?
E. Hemodialysis A. Membranous nephropathy

ic
B. Minimal change disease
21. A 62-year-old man with a history of type 2 diabetes C. Focal segmental glomerulosclerosis
mellitus, coronary artery disease, peripheral artery
disease, and hypertension presents to the clinic for
ed D. IgA nephropathy
E. Amyloidosis
routine follow-up. He is currently on three anti-
sm
hypertensive agents. His blood pressure is 130/70 25. A 49-year-old man with untreated hepatitis C viral
mm Hg. Serum creatinine is 0.7 mg/dL. There is infection develops persistent proteinuria.
no evidence of microalbuminuria. Serum aldoste- Which of the following is a renal manifestation
rone and plasma renin activity are mildly elevated. associated with hepatitis C viral infection?
ok

A renal duplex study reveals bilateral renal artery A. Acute interstitial nephritis
stenosis. B. Focal segmental glomerulosclerosis
What is the next step in managing this patient? C. Polyarteritis nodosa
bo

A. Percutaneous renal artery angioplasty with stent D. Nephrogenic diabetes insipidus


placement E. Fanconi syndrome
B. Renal artery surgical revascularization
e

C. Adding an additional antihypertensive 26. A 43-year-old man presents with bilateral lower
://

D. Stopping an antihypertensive to permissively raise extremity edema associated with 3 g/d of proteinuria
the systolic blood pressure closer to 140 mm Hg with preserved renal function. He undergoes renal
tp

E. Do nothing biopsy, which demonstrates membranous nephropa-


thy. Infectious workup for hepatitis B and C virus and
22. A 56-year-old man presents for evaluation of anxi- syphilis is negative. Complements are normal. Lupus
ht

ety, depression, and peripheral neuropathy. He has serologies are unremarkable. Age-appropriate cancer
been employed as a painter for most of his adult screening is unremarkable. In addition to an ACE
life. Laboratory testing reveals an estimated GFR inhibitor, what therapy should be ofered?
of 25 mL/min/1.73 m2, nongap metabolic acido- A. Nothing
sis associated with a positive urine anion gap, and B. Steroids
microcytic anemia. A serum lead level was found to C. Steroid
be elevated. D. Steroids and cyclophosphamide
What would a renal biopsy be expected to demon- E. Rituximab
strate in this patient?
A. Acute interstitial nephritis 27. A 92-year-old woman presents to the emergency
B. Acute tubular necrosis department with symptoms of right-sided weakness
C. Glomerulonephritis and slurred speech that were noted upon awakening
D. Glomerular hypertrophy from sleep. MRI conirms an acute ischemic stroke.
E. Chronic interstitial nephritis Her initial blood pressure on presentation was 240/120
CHAPTER 6 Nephrology and Hypertension 133

mm Hg, but it has since come down to 200/100 mm What is the appropriate treatment?
Hg. She continues to have slurred speech and right- A. Steroids
sided weakness but appears to be alert and following B. Steroids and cyclophosphamide
commands. C. Plasmapheresis, steroids, and cyclophosphamide
Which of the following is the most appropriate D. Rituximab
blood pressure goal in this setting? E. Dialysis
A. Less than 220/120 mm Hg
B. Less than 180/105 mm Hg 32. A 65-year-old man with a family history of autoso-
C. Less than 150/90 mm Hg mal dominant polycystic kidney disease is diagnosed
D. Less than 140/90 mm Hg with polycystic kidney disease on the basis of CT
E. Less than 120/80 mm Hg imaging.
Which of the following treatments could be
28. A 77-year-old man with hypertension, restrictive expected to reduce the annual increase in kidney
cardiomyopathy, recently diagnosed diabetes, and size?
peripheral neuropathy develops 10 g/d of proteinuria A. Vasopressin receptor antagonism

t
ne
as measured by a 24-hour urine collection. he onset B. Antihypertensive therapy
of peripheral neuropathy preceded the diagnosis of C. Statin therapy
diabetes. Physical examination is remarkable for peri- D. Low-protein diet

e.
orbital purpura and lateral scalloping of the tongue. E. Steroid therapy
What is the likely explanation for the patient’s

in
nephrotic range proteinuria? 33. A 54-year-old woman with a history of stage 4 chronic
A. AA amyloid kidney disease secondary to uncontrolled diabetes mel-

ic
B. AL amyloidosis litus presents with the following laboratory values: cal-
C. Focal segmental glomerulosclerosis (FSGS) cium 7.1 mg/dL, phosphorus 6.3 mg/dL, PTH 1000
D. Diabetic nephropathy
E. Hypertensive nephropathy
ed pg/mL, and normal 25-hydroxy-vitamin D level. In
addition to starting a phosphorus binder, what is the
next step in management?
sm
29. An 18-year-old man presents with hemolytic ane- A. Start 25-hydroxyvitamin D supplementation.
mia, thrombocytopenia, and acute kidney injury, in B. Start 1,25-hydroxyvitamin D supplementation.
the absence of a diarrheal prodrome. C3 is low, and C. Start cinacalcet, a calcimimetic.
C4 is normal. CH50 is <10%, and AH50 is <10%. D. Start calcium supplementation.
ok

ADAMTS13 activity is somewhat low at 40%. E. Refer for parathyroidectomy.


How should this condition be treated?
A. Plasmapheresis 34. A 25-year-old man presents to clinic for follow-up
bo

B. IVIG after a recent attack of gout. His serum uric acid is 6.7
C. Steroids mg/dL, and his creatinine is 1.4 mg/dL. His brother
D. Rituximab also has gout and was diagnosed with end-stage kidney
e

E. Eculizumab disease at the age of 41; his mother died at age 59 after
://

10 years on dialysis.
30. A 36-year-old woman with a history of lupus nephritis What is the most likely diagnosis?
tp

presents with a rising creatinine level and worsening pro- A. Uromodulin kidney disease
teinuria. She recently completed a course of treatment B. Lead nephropathy
with steroids and mycophenolate mofetil for Interna- C. Acute uric acid nephropathy
ht

tional Society of Nephrology/Renal Pathology Society D. Chronic urate nephropathy


(ISN/RPS) class IV and class V lupus nephritis. A repeat E. Diabetic nephropathy
biopsy is performed and reveals class VI nephropathy.
What is the most appropriate treatment? 35. A 42-year-old man with a diagnosis of hyperten-
A. Rituximab sion, coronary artery disease, congestive heart
B. Steroids and mycophenolate mofetil failure, and obstructive sleep apnea presents for
C. Steroids and cyclophosphamide evaluation of ongoing hypertension. he patient
D. Steroids and tacrolimus is already on three antihypertensives. A secondary
E. Discuss dialysis and transplant workup for causes of hypertension is performed.
Endocrine workup is negative. Serum aldosterone
31. A 62-year-old woman presents with a rise in creatinine and plasma renin activity are within normal limits.
from 1 to 6 mg/L in the span of 1 week, associated A renal ultrasound duplex study reveals no evidence
with a new onset of hemoptysis. Serologic workup of renal artery stenosis.
reveals positive c-ANCA and anti-PR3 antibody. What is the best next step in management?
134 C HA P T E R 6 Nephrology and Hypertension

A. Add a fourth antihypertensive medication. chemistries reveal sodium 138 mEq/L, potassium 3.6
B. Recommend continuous positive airway pressure mEq/L, chloride 98 mEq/L, bicarbonate 22 mEq/L,
(CPAP) at night. blood urea nitrogen 13 mg/dL, creatinine 0.9 g/dL,
C. Refer for renal sympathetic denervation. and glucose 90 mg/dL. Ethanol level is undetectable.
D. Counsel on DASH diet. Measured serum osmolality is 298 mOsm/kg. Serum
E. Counsel on low-salt diet. ketones are negative. Serum lactic acid is within nor-
mal limits. Urinalysis is within normal limits.
36. A 68-year-old man with a history of coronary artery What is the likely cause of the patient’s anion gap
disease with multiple prior myocardial infarctions, acidosis?
peripheral artery disease, and diabetes mellitus pres- A. Lorazepam
ents with ongoing hypertension despite the use of B. Rhabdomyolysis related to seizure
benazepril. C. Alcoholic ketoacidosis
Which of the following agents should be added D. Uremia
next for the management of this patient’s hyperten- E. d-Lactic acidosis
sion?

t
ne
A. Hydrochlorothiazide 40. A 35-year-old woman presents with a new onset of
B. Amlodipine hypertension, and a secondary workup of hypertension
C. Furosemide is performed. Renal ultrasound with duplex is notable

e.
D. Amiloride for an increased peak velocity and tortuosity in the mid-
E. Minoxidil dle and distal renal arteries in the left kidney.

in
Which of the following statements is true about
37. A 58-year-old woman with no known medical history this condition?

ic
is sent to the emergency department for evaluation of A. he condition is more common in men.
hypertension with a blood pressure of 210/110 mm B. Extracranial cerebrovascular involvement is com-
Hg. he patient currently has no complaints. Labora-
tory test results are unremarkable, with the exception
ed mon.
C. MR angiogram is the preferred diagnostic study.
of urinalysis that reveals trace protein. Electrocardio- D. Surgical management is the preferred treatment
sm
gram reveals left ventricular hypertrophy. for the renal lesions.
What is the next step in management? E. Atherosclerosis is the primary cause of the disease.
A. Oral chlorthalidone and lisinopril and follow-up
as an outpatient 41. A 55-year-old man with a history of cirrhosis result-
ok

B. Intravenous labetalol ing from fatty liver disease presents with a gastrointes-
C. Intravenous nitroprusside tinal bleed, followed by the development of oliguric
D. Intravenous nitroglycerin acute kidney injury with no proteinuria and a normal
bo

E. Intravenous hydralazine and transition to oral urine sediment. Urine sodium is less than 10 mEq/L.
hydralazine Volume repletion with blood products and crystalloid
solution results in no improvement in the patient’s
e

38. A 64-year-old woman with a long-standing history of renal function.


://

bipolar disorder managed on lithium for 20 years is Which of the following is true about the likely clin-
referred to the nephrology department for evaluation ical diagnosis?
tp

of chronic kidney disease (CKD) and concern for lith- A. Octreotide and midodrine can be used for deini-
ium nephrotoxicity. In addition to diabetes insipidus, tive management of this condition.
renal tubular acidosis (RTA), and chronic interstitial B. Liver transplant is contraindicated.
ht

nephritis, which of the following is a manifestation of C. Hemodialysis, if indicated, is typically well toler-
lithium nephrotoxicity? ated.
A. Hypercalciuria D. Renal biopsy would be expected to reveal a normal
B. Hypoparathyroidism kidney
C. Hyperphosphatemia E. If evaluating the patient for a liver transplant, a
D. Hypocalcemia kidney transplant would also be indicated.
E. Nephrotic syndrome
42. A 28-year-old man presents with arm and leg cramps
39. A 42-year-old man with a history of alcohol abuse and polyuria and is found to have a blood pressure of
presents to the emergency department with a seizure approximately 90/50 mm Hg associated with a serum
in the setting of alcohol withdrawal. He is initiated potassium level of 3.1 mEq/L and a magnesium level
on a lorazepam intravenous drip and transferred to of 0.9 mEq/L. Urinary calcium is low, and urinary
the intensive care unit. Weaning the drip results in chloride is elevated.
recurrent seizures, so lorazepam is restarted. Serum What is the likely diagnosis?
CHAPTER 6 Nephrology and Hypertension 135

A. Surreptitious vomiting 47. A 53-year-old woman with a history of alcohol abuse


B. Bartter syndrome presents to the emergency department in an inebri-
C. Gitelman syndrome ated state after drinking antifreeze solution. Labora-
D. Primary hyperaldosteronism tory test results are remarkable for a negative ethanol
E. Cushing syndrome level, sodium 137 mEq/L, bicarbonate 8 mEq/L, chlo-
ride 101 mEq/L, glucose 110 mg/dL, BUN 18 mg/
43. An 18-year-old man with a history of deafness and dL, and serum osmolality 312 mOsm/kg. Urinalysis
hematuria presents with a progressively rising creati- reveals calcium oxalate crystals.
nine level. His sister, mother, and father are all healthy, What is the next step in management?
though he does recall a maternal grandfather who died A. Start isotonic saline.
as a result of complications of kidney disease. B. Perform gastric lavage.
What is the likely diagnosis? C. Administer furosemide.
A. hin basement membrane disease D. Administer activated charcoal.
B. Alport syndrome E. Administer fomepizole and prepare for dialysis.
C. Fabry disease

t
ne
D. Amyloidosis 48. A 56-year-old woman with type 2 diabetes and hyper-
E. Tubulointerstitial nephritis with uveitis tension currently on losartan and amlodipine presents
for follow-up. Blood pressure is 130/80 mm Hg, and

e.
44. A 47-year-old woman with a history of Roux-en-Y gas- physical exam is otherwise notable for clear lungs and
tric bypass surgery 6 years ago presents with acute onset trace lower extremity edema. Laboratory test results

in
of right-lank pain and gross hematuria. Abdominal are notable for creatinine 2.4 mg/dL, potassium 5.2
imaging with a CT scan demonstrates a kidney stone. mEq/L, and bicarbonate 20 mmol/L. Urine studies are

ic
Which of the following is the most likely type of notable for pH 5.5 and microalbumin-to-creatinine
kidney stone? ratio of 64 mg/g.
A. Calcium phosphate
B. Calcium oxalate
ed What is the most appropriate next step in manage-
ment?
C. Uric acid A. Add spironolactone 12.5 mg daily.
sm
D. Cystine B. Discontinue losartan and replace with chlorthali-
E. Struvite done 12.5 mg daily.
C. Start oral sodium bicarbonate therapy.
45. A 19-year-old man with a history of stroke in the D. Start oral potassium citrate therapy.
ok

absence of hypertension, left ventricular hypertrophy, E. Start ludrocortisone 0.1 mg daily.


and angiokeratomas (vascular cutaneous lesions) pres-
ents for evaluation of chronic kidney disease. Renal 49. A 65-year-old man with a history of diabetes melli-
bo

imaging demonstrates multiple renal sinus cysts. tus, hypertension, and gout presents with new-onset
What is the likely diagnosis? abdominal pain. He takes metformin, lisinopril, and
A. hin basement membrane disease allopurinol. His electrolytes are within the reference
e

B. Alport syndrome range. His uric acid is 5.7 mg/dL, and his creatinine is
://

C. Fabry disease 1.2 mg/dL. He has a noncontrast CT scan that shows


D. Amyloidosis a 9-mm kidney stone on the right and multiple smaller
tp

E. Tubulointerstitial nephritis with uveitis kidney stones on the left. He passes one of the stones,
and the stone analysis reveals 100% uric acid.
46. A 39-year-old man with newly diagnosed Hodgkin Which of the following is the target urinary pH for
ht

lymphoma develops dyspnea and is noted to have treatment of uric acid kidney stones?
Kussmaul respirations. Serum chemistries and blood A. pH 5.0–5.5
gas reveal anion gap acidosis. Blood, urine, and spu- B. None; pH is not a factor in the management of
tum cultures are sent, and he is started on empiric uric acid kidney stones.
broad-spectrum antimicrobial therapies. Despite neg- C. pH greater than 7.5
ative culture data and several days of antibiotics, the D. pH 6.5–7.0
anion gap acidosis is persistent. E. pH less than 5.0
How would you classify this patient’s anion gap
acidosis? 50. An 84-year-old woman presents for follow-up of
A. Type A lactic acidosis CKD. Since her last visit 6 months ago, she has
B. Type B lactic acidosis become increasingly dependent after experiencing an
C. d-Lactic acidosis inluenza-like illness, now with a 10-lb weight loss
D. Ketoacidosis and poor nutrition. hree months ago, she moved
E. Ethylene glycol acidosis into a nursing home. Past medical history includes
136 C HA P T E R 6 Nephrology and Hypertension

CKD stage 3 with baseline creatinine 1.3 mg/dL pre- dL, bicarbonate 17 mmol/L, and albumin 2.9 mg/dL.
sumed to be secondary to long-standing hypertension Lactate and ketones are normal.
as well as past NSAID use for osteoarthritis. Current Which of the following is the most appropriate
medications include amlodipine 10 mg daily, meto- next step in management of the patient’s acidosis?
prolol 50 mg daily, furosemide 40 mg daily, lisinopril A. Start oral bicarbonate therapy.
10 mg daily, and acetaminophen 975 mg three times B. Discontinue acetaminophen.
daily. Blood pressure is 142/74 mm Hg, and physi- C. Start parenteral nutrition.
cal exam reveals a thin woman with clear lungs and D. Discontinue lisinopril.
no lower extremity edema. Laboratory testing reveals E. Start midodrine to optimize perfusion.
Cr 1.4 mg/dL, sodium 134 mEq/L, chloride 99 mg/

Chapter 6 Answers

1. ANSWER: D. Change hydrochlorothiazide to furo- intravenous coagulopathy (DIC) is less likely in the

t
ne
semide. setting of normal coagulation studies and an elevated
Amlodipine is incorrect because it may worsen ibrinogen level.
the patient’s lower extremity edema. hough angio-

e.
tensin-converting enzyme (ACE) inhibitors such as 3. ANSWER: B. Tenofovir disoproxil
lisinopril are recommended in patients with protein- his patient’s clinical presentation is consistent

in
uric chronic kidney disease, they do not provide a with Fanconi syndrome resulting from tenofovir
preferential beneit in treating patients with nonpro- disoproxil therapy. Fanconi syndrome is a disorder

ic
teinuric chronic kidney disease. hough chlorthali- of the proximal tubule characterized by low serum
done has been purported to be more efective than bicarbonate resulting from reduced proximal bicar-
hydrochlorothiazide on the basis of meta-analyses
(due to a longer duration of action), thiazide diuret-
ed bonate reabsorption, low serum phosphate resulting
from phosphaturia, glucosuria with normal serum
ics are less efective than loop diuretics in chronic glucose, and proteinuria. Fanconi syndrome can be
sm
kidney disease. For this reason, the correct answer is inherited or acquired. Tenofovir in general is associ-
to switch the thiazide to a loop diuretic (furosemide) ated with acquired Fanconi syndrome, which is sig-
to treat the patient’s apparent volume expansion. A niicant because an estimated 84% of U.S. residents
low-salt diet should always be recommended because on therapy for HIV infection take a formulation of
ok

a high-salt diet can lead to diuretic resistance, but in tenofovir, and this drug is also used to treat chronic
this patient it is not likely the reason for the patient’s hepatitis B infection. Since 2015 there have been two
diuretic resistance. diferent formulations of tenofovir available as part
bo

of antiretroviral regimens for HIV infection: teno-


2. ANSWER: C. Hypertensive emergency fovir disoproxil (which was approved by the FDA in
he presence of markers of hemolysis (anemia, ele- 2001) and tenofovir alafenamide (which was approved
e

vated LDH, low haptoglobin, and elevated reticulocyte by the FDA in 2015). Both are prodrugs of the same
://

index), combined with the presence of schistocytes active antiviral medication, but tenofovir alafenamide
in the peripheral smear, raises concern for microan- achieves high intracellular concentrations of the active
tp

giopathic hemolytic anemia. Although these indings, drug, thereby allowing for lower doses of the prodrug
in combination with thrombocytopenia, raise con- and diminished renal toxicity. As the use of tenofo-
cern for hemolytic uremic syndrome and thrombotic vir alafenamide rises, the occurrence of Fanconi syn-
ht

thrombocytopenic purpura, the renal disease seen is drome associated with tenofovir would be expected to
usually much milder, and hypertension of this sever- decline.
ity is not typical. On the other hand, hypertensive Sax PE, Wohl D, Yin MT, et al. Tenofovir alafenamide versus
emergency can cause thrombotic microangiopathy tenofovir disoproxil fumarate, coformulated with elvitegra-
and is the likely explanation for this patient’s clinical vir, cobicistat, and emtricitabine, for initial treatment of
presentation. he presence of left ventricular hypertro- HIV-1 infection: two randomised, double-blind, phase 3,
non-inferiority trials. Lancet. 2015;385(9987):2606–2615.
phy on an electrocardiogram (EKG) suggests that the
Walensky RP, Horn TH, Paltiel AD. he Epi-TAF for tenofovir
hypertension has been long-standing. Atypical hemo-
disoproxil fumarate? Clin Infect Dis. 2016;62(7):915–918.
lytic uremic syndrome describes a variant of hemolytic
uremic syndrome (HUS) that occurs in the absence
of diarrhea and can be explained by other infections 4. ANSWER: E. Counsel the patient on vascular
or complement dysregulation. hough red blood cell access for dialysis.
casts are seen in glomerulonephritis, they can also be On the basis of the 2012 Kidney Disease Improv-
seen in thrombotic microangiopathies. Disseminated ing Global Outcomes (KDIGO) guidelines, there
CHAPTER 6 Nephrology and Hypertension 137

should be careful consideration of whether erythro- HIV can be associated with a thrombotic microan-
poietin-stimulating agent (ESA) therapy should be giopathy, the suspected diagnosis in this patient is
started for hemoglobin values >10 g/dL. he risks and HIVAN. Furthermore, plasmapheresis should not be
beneits should be weighed. Risks of ESAs include combined with an ACE inhibitor, because the com-
hypertension, thrombosis, stroke, and the observation bination can result in a syndrome of abdominal pain,
of increased mortality in patients with malignancy. In lushing, and hypotension.
patients with hemoglobin values <10 g/dL, ESA ther-
apy needs to be individualized. he absence of anemic 7. ANSWER: C. Multiple myeloma
symptoms and the presence of lung cancer would both he irst step in analyzing a patient with hypona-
argue against the use of ESA therapy. hough packed tremia is to diferentiate pseudohyponatremia from
red blood cell transfusion would be safe to administer true hyponatremia by evaluating the serum osmolality.
from the standpoint of malignancy, there is no clear hough this patient’s calculated serum osmolality ([2
indication for transfusion in the absence of symptoms. × Na] + [BUN/2.8] + [glucose/18]) is on the lower end
Iron therapy is not warranted in the setting of normal of normal at 275 mOsm/kg, the presence of an osmo-
iron stores. All patients with stage 4 CKD should be lality gap suggests that the measured osmolality is

t
ne
counseled on vascular (and peritoneal) access for dialy- actually normal to elevated, raising concern for pseu-
sis, so this is the correct answer. dohyponatremia. Combined with a low anion gap,
Kidney Disease: Improving Global Outcomes (KDIGO) which can be seen with hypercalcemia or a positively

e.
Chronic Kidney Disease Work Group. KDIGO 2012 Clini- charged immunoglobulin, these indings are strongly
cal Practice Guideline for the Evaluation and Management of suggestive of a diagnosis of multiple myeloma. Urine
Chronic Kidney Disease. Kidney Int (Suppl). 2013;3:1–150.

in
protein by dipstick is speciic for albumin and would
not be expected to reveal the presence of a paraprotein.

ic
5. ANSWER: E. Hemodialysis All other answer choices relect causes of true hypona-
In this patient with hyperkalemia with peaked T tremia.
waves and widened QRS complexes, temporizing
measures and deinitive management of hyperkalemia
ed
8. ANSWER: C. Captopril
are strongly indicated. Sodium polystyrene sulfonate Scleroderma renal crisis (SRC) develops in 10%–
sm
with or without sorbitol carries a risk of intestinal 20% of patients with difuse systemic sclerosis. Clini-
necrosis, which is greatest in the postoperative setting cally, it resembles thrombotic thrombocytopenic
and thus should be avoided in this patient. Sodium purpura–hemolytic-uremic syndrome (TTP-HUS)
bicarbonate may aid in shifting potassium intracellu- and can present with hemolytic anemia, mild throm-
ok

larly, but it does not remove potassium from the body. bocytopenia, and thrombotic microangiopathy on
Although a loop diuretic could be considered with renal biopsy, but the physical examination features of
less severe hyperkalemia, the evidence of cardiotoxic- scleroderma help distinguish this entity from TTP-
bo

ity on an EKG warrants hemodialysis for correction of HUS. Renal biopsy is not helpful, because it will not
hyperkalemia. Indeed, furosemide takes several hours distinguish SRC from TTP-HUS. he use of high-
to be efective and will not resolve this patient’s EKG dose steroids is a risk factor for the development of
e

changes acutely. SRC. he mainstay of therapy is ACE inhibition


://

Kovesdy CP. Management of hyperkalemia: an update for the (captopril), even in advanced renal failure requiring
internist. Am J Med. 2015;128(12):1281–1287. dialysis, because there is a much greater chance of
tp

renal recovery with discontinuation of dialysis and


6. ANSWER: B. ART alone improved mortality observed in patients treated with
HIV-associated nephropathy (HIVAN) is seen ACE inhibitors.
ht

most commonly in African Americans, especially in Denton CP, Lapadula G, Mouthon L, Müller-Ladner U. Renal
the setting of advanced HIV with a low CD4 count complications and scleroderma renal crisis. Rheumatology
associated with acute kidney injury and nephrotic- (Oxford). 2009;48(Suppl 3):iii32–iii35.
range proteinuria. Renal biopsy is required to conirm
the diagnosis because nearly 40% of individuals with 9. ANSWER: A. Adrenal vein sampling
suspected HIVAN will have an alternate diagnosis by In this patient with an elevated aldosterone to
biopsy. Antiretroviral therapy (ART) is the mainstay plasma renin activity (PRA) ratio (>30) and elevated
of treatment. Steroids are not typically indicated, serum aldosterone (>20 ng/dL) with positive conir-
but they may be considered if renal function wors- matory testing, the diagnosis is primary hyperaldoste-
ens despite the initiation of ART along with renin- ronism. In cases of hyperaldosteronism with normal
angiotensin system (RAS) inhibition. Steroid-sparing adrenal imaging, the possibilities include a nonvisu-
agents are not indicated. Steroids plus mycopheno- alized adrenal mass and bilateral adrenal hyperplasia.
late mofetil are also not indicated. Plasmapheresis he only way to distinguish between these in the
plays no role in the management of HIVAN. hough face of normal CT imaging is to pursue adrenal vein
138 C HA P T E R 6 Nephrology and Hypertension

sampling. CT imaging has superior spatial resolution management of adult and paediatric lupus nephritis. Ann
compared with MRI in the visualization of adrenal Rheum Dis. 2012;71(11):1771–1782.
glands, so an MRI study would not be helpful. Spi- Rovin BH, Furie R, Latinis K, et al. Eicacy and safety of ritux-
ronolactone would be appropriate only if bilateral imab in patients with active proliferative lupus nephritis:
adrenal hyperplasia were conirmed by adrenal vein the Lupus Nephritis Assessment with Rituximab study.
Arthritis Rheum. 2012;64(4):1215–1226.
sampling. he other answers are not treatments for
primary hyperaldosteronism.
12. ANSWER: D. Counsel on low-salt diet.
10. ANSWER: C. Distal (type 1) renal tubular acidosis In the very elderly (patients older than 80 years of
(RTA) age), the beneits of controlling blood pressure aggres-
A positive urine anion gap (urine Na+ + K+ − Cl) sively are not known. he SPRINT trial demonstrated
suggests that impaired renal excretion of ammonium a decreased risk of the composite cardiovascular out-
(which is an unmeasured cation in the urine) is the come when targeting systolic blood pressures less than
cause of the patient’s metabolic acidosis, as opposed 120 mm Hg compared with less than 140 mm Hg.
However, SPRINT excluded individuals with prior

t
to loss of bicarbonate in the stool, which would result

ne
in a negative urine anion gap. he presence of a low stroke, so it cannot be directly applied in this case.
serum bicarbonate with a normal anion gap favors On the basis of the HYVET trial, it is known that
a diagnosis of RTA as opposed to a steroid-induced treating systolic blood pressure to less than 150 mm

e.
hypokalemia, which would be expected to be associ- Hg decreases the risk of fatal stroke, heart failure, and
ated with a metabolic alkalosis from mineralocorticoid all-cause mortality. he ACCOMPLISH trial dem-

in
receptor binding by the corticosteroid. Type 4 RTA onstrated that nonobese individuals at higher risk for
is associated with hyperkalemia. Absence of glycosuria cardiovascular events may particularly beneit from

ic
on urinalysis argues against Fanconi syndrome, which the combination of a dihydropyridine calcium chan-
often accompanies a proximal RTA from injury to the nel blocker and ACE inhibitor therapy. Because this
proximal tubule. Distal RTA (seen with urine pH >5.5
and hypokalemia) related to rheumatoid arthritis is
ed patient’s systolic blood pressure is already under 150
mm Hg, medications should not be intensiied, and
the likely explanation of the hypokalemia. a secondary workup should not be performed. It is
sm
always reasonable to counsel hypertensive patients on
11. ANSWER: D. ACE inhibitor, prednisone, myco- a low-salt diet.
phenolate mofetil Jamerson K, Weber MA, Bakris GL, et al. Benazepril plus amlo-
Renal biopsy is required in most patients with dipine or hydrochlorothiazide for hypertension in high-risk
ok

patients. N Engl J Med. 2008;359(23):2417–2428.


known lupus and evidence of renal involvement
SPRINT Research Group. A randomized trial of intensive
because the severity on clinical presentation may be
versus standard blood-pressure control. N Engl J Med.
bo

discordant with the true degree of renal involvement 2015;373(22):2103–2116.


by biopsy, and because the histologic subtype along
with indices of activity and chronicity help guide
13. ANSWER: B. Isotonic saline
e

the treatment. Once the pathological classiication is


determined, therapy may be initiated. ACE inhibi- Isotonic saline is the mainstay of therapy for man-
://

tor is an important adjunctive therapy for patients agement of rhabdomyolysis. Isotonic bicarbonate
with proteinuria. he Joint European League Against should not be used in the setting of hypocalcemia,
tp

Rheumatism and European Renal Association-Euro- because the use of bicarbonate can reduce the ionized
pean Dialysis and Transplant Association (EULAR/ fraction of calcium as a result of increased albumin
binding of calcium, leading to symptomatic hypocal-
ht

ERA-EDTA) guideline recommends immunosup-


pression for patients with class III lupus nephri- cemia. Loop diuretics such as furosemide can be used
tis. Prednisone along with mycophenolate mofetil if there is volume overload, but they can also worsen
is the recommended initial treatment. Prednisone hypocalcemia and should not be used as irst-line
and cyclophosphamide is an appropriate treatment therapy. he role of mannitol in the management of
option but poses a greater risk of infertility in this rhabdomyolysis remains uncertain.
patient. Glucocorticoid monotherapy is not recom-
mended. On the basis of the LUNAR trial, there is 14. ANSWER: A. IgA nephropathy
insuicient evidence to support the use of rituximab IgA nephropathy can present as a rapidly progress-
as an initial immunosuppressive therapy for class III ing glomerulonephritis (biopsy with crescentic IgA
lupus nephritis. nephropathy), as a latent inding of hematuria and
Bertsias GK, Tektonidou M, Amoura Z, et  al. Joint Euro- proteinuria, or in the setting of an upper respiratory
pean League Against Rheumatism and European Renal tract infection (“synpharyngitic hematuria”), as is the
Association-European Dialysis and Transplant Asso- case in this patient. IgA nephropathy typically occurs
ciation (EULAR/ERA-EDTA) recommendations for the 1–3 days after an upper respiratory tract infection, as
CHAPTER 6 Nephrology and Hypertension 139

opposed to postinfectious glomerulonephritis (GN), heart failure or recurrent lash pulmonary edema is an
which occurs 7–21 days after an infection. he pres- ACC/AHA class I indication for renal artery revascu-
ence of normal complements also helps distinguish larization. he safest way to achieve this is via percu-
IgA nephropathy from postinfectious GN. hin base- taneous angioplasty and stent placement, with surgical
ment membrane disease can be associated with some revascularization reserved for patients with complex
hematuria but does not it the clinical picture. Alport anatomy. he other options are not appropriate at this
syndrome can be associated with proteinuria, hearing time.
loss, and progression to end-stage renal disease, but it ASTRAL Investigators. Revascularization versus medi-
does not it the clinical picture. Granulomatous poly- cal therapy for renal-artery stenosis. N Engl J Med.
angiitis (formerly known as Wegener granulomatosis) is 2009;361(20):1953–1962.
unlikely with a negative ANCA.
18. ANSWER: B. Add chlorthalidone.
15. ANSWER: B. Stop hypertonic saline and start In patients with diabetes, compelling medica-
D5W and DDAVP. tions for hypertensive management include thia-
Rapid overcorrection of chronic hyponatremia (rise zides, beta-blockers angiotensin-converting-enzyme

t
ne
in Na+ >10 mEq/24 h) can result in osmotic demy- inhibitors (ACEIs), angiotensin receptor blockers
elination syndrome. he symptoms of osmotic demy- (ARBs), and calcium channel blockers. hough the
elination typically do not occur for 2–6 days after COOPERATE trial suggested a beneit of dual RAS

e.
the sodium has been overcorrected. Animal models blockade with ACEIs and ARBs (such as valsartan),
show a decreased incidence of osmotic demyelination it was later found that the authors had engaged in

in
when sodium that has been overcorrected is relowered. serious scientiic misconduct, resulting in retraction
Human studies show that relowering is safe. he best of the paper from he Lancet. he ONTARGET trial

ic
answer in this patient would be to stop the hypertonic demonstrated no beneit to combination therapy
saline and to relower the sodium using D5W and with an ACEI and an ARB. Diabetic proteinuria
DDAVP.
Sterns RH. Disorders of plasma sodium—causes, consequences,
ed tends to improve with lowering of blood pressure,
and the addition of chlorthalidone would be a rea-
and correction. N Engl J Med 2015;372(1):55–65. sonable choice for a second antihypertensive. Meto-
sm
prolol is unlikely to lower the patient’s blood pressure
16. ANSWER: B. Acute interstitial nephritis as much as a diuretic.
hough the triad of fever, eosinophilia, and rash is Mann JF, Schmieder RE, McQueen M, et  al. Renal out-
seen in only a small percentage of patients with allergic comes with telmisartan, ramipril, or both, in people at
ok

interstitial nephritis (AIN), any combination of these high vascular risk (the ONTARGET study): a multicen-
signs should raise suspicion for it. Allergic interstitial tre, randomised, double-blind, controlled trial. Lancet.
2008;372(9638):547–553.
nephritis (causing acute interstitial nephritis) is the
bo

likely diagnosis in this patient, and a proton pump


inhibitor for treatment of the patient’s GERD is the 19. ANSWER: D. Isotonic saline with hydrocortisone
likely culprit. Cellular casts raise concern for glomeru- he patient’s symptomatology is highly concern-
e

lonephritis, but the type of cells was not speciied in ing for tuberculosis, and the acute development of
://

the question stem. White blood cell casts can be seen hypotension, hyponatremia, and hypokalemia are
in AIN, whereas typically red blood cell casts are seen all concerning for adrenal crisis likely resulting from
tp

in glomerulonephritis. DRESS can result from a drug tuberculous iniltration of the adrenal glands. hough
hypersensitivity as well, but the absence of other sys- the patient will ultimately beneit from multidrug
temic symptoms makes this diagnosis unlikely. therapy for tuberculosis, the acute management should
ht

Geevasinga N, Coleman PL, Webster AC, Roger SD. Proton be focused on volume resuscitation and steroid admin-
pump inhibitors and acute interstitial nephritis. Clin Gas- istration for management of acute adrenal crisis. Fluid
troenterol Hepatol. 2006;4(5):597–604. restriction is inappropriate in a patient who appears
hypovolemic. Hypertonic saline should be reserved for
17. ANSWER: A. Percutaneous renal artery angioplasty patients with symptomatic hyponatremia and sodium
with stent placement values less than 120 mEq/L. Tolvaptan is contraindi-
hough the largest trial comparing renal artery cated in hypovolemic patients because it induces a free
stenting with medical management (ASTRAL trial) water diuresis that may worsen hypovolemia.
found that stenting conferred little beneit and much
risk in the management of renal artery stenosis, the 20. ANSWER: A. Rasburicase
enrollment criteria for the study excluded all patients his patient has spontaneous tumor lysis syn-
whose physicians felt they would “deinitely” beneit drome secondary to Burkitt lymphoma. Intra-
from stenting. hough stenting should not generally venous luids are important in preventing this
be performed in asymptomatic patients, refractory condition when administered with chemotherapy,
140 C HA P T E R 6 Nephrology and Hypertension

but rasburicase is the safest and fastest way to break Peces R, Sánchez L, Gorostidi M, et  al. Minimal change
down uric acid (to the water-soluble compound nephrotic syndrome associated with Hodgkin’s lymphoma.
allantoin) to enhance its clearance. Allopurinol and Nephrol Dial Transplant. 1991;6(3):155–158.
febuxostat would not be helpful in the acute setting.
he hyperkalemia is being driven by breakdown 25. ANSWER: C. Polyarteritis nodosa
of tumor cells and is not likely to respond signii- Hepatitis C–associated renal diseases include mixed
cantly to sodium polystyrene sulfonate. Indications cryoglobulinemia, membranoproliferative glomerulo-
for hemodialysis in tumor lysis syndrome include nephritis (MPGN), membranous nephropathy, and
severe oliguria, persistent hyperkalemia, and hyper- polyarteritis nodosa (PAN). Membranous nephropa-
phosphatemia-induced symptomatic hypocalcemia, thy, MPGN, and PAN can also be observed in hepa-
none of which are present at this time. titis B.
Alakel N, Middeke JM, Schetelig J, et al. Prevention and treat-
ment of tumor lysis syndrome, and the eicacy and role of 26. ANSWER: A. Nothing
rasburicase. Onco Targets her. 2017;10:597–605. Phospholipase A2 receptor (PLA2R) has been identi-
Criscuolo M, Fianchi L, Dragonetti G, et al. Tumor lysis syn- ied as a key pathogenic antigen in idiopathic membra-

t
drome: reviewof pathogenesis, risk factors and management

ne
nous nephropathy, which appears to be the diagnosis in
of a medical emergency. Expert Rev Hematol. 2016;9(2): this patient without a clear secondary cause. Treatment
197–208.
of idiopathic membranous nephropathy is stratiied by

e.
the clinical severity. Patients with normal renal function
21. ANSWER: E. Do nothing. and proteinuria <4 g/d (such as in the patient in this
In the absence of renovascular disease (given case) can be safely observed without immunosuppres-

in
normal creatinine) and refractory hypertension, sion because they often undergo spontaneous partial or
there is no acute indication to pursue renal artery complete remission. Glucocorticoids in combination

ic
revascularization. here are considerable risks with with either cyclophosphamide or a calcineurin inhibi-
both percutaneous angioplasty and surgical revas- ed tor may be efective in treating patients at moderate and
cularization, so these procedures should be reserved higher risk for progression of renal disease. Rituximab
for patients with compelling indications. A blood may be used in refractory cases.
pressure of 130/70 mm Hg in a patient with dia-
sm
Beck Jr LH, Bonegio RG, Lambeau G, et al. M-type phospholi-
betes is appropriate and requires no adjustment of pase A2 receptor as target antigen in idiopathic membranous
antihypertensives. nephropathy. N Engl J Med. 2009;361(1):11–21.
ok

22. ANSWER: E. Chronic interstitial nephritis 27. ANSWER: A. Less than 220/120 mm Hg
Lead nephropathy can present acutely with Fan- In a patient experiencing an acute ischemic stroke,
coni syndrome resulting from injury to the proximal the blood pressure goal is determined by candidacy
bo

tubules with intranuclear inclusion bodies made up of for thrombolytic therapy. Because the patient’s stroke
lead–protein complexes. Chronic injury is character- symptoms occurred while she was asleep, the timing of
ized by the inding of chronic interstitial nephritis. symptom onset cannot be determined. hus systemic
e

Glomerulosclerosis may also be seen on biopsy. thrombolytic therapy cannot be safely ofered. In a
://

patient who is not a candidate for thrombolytic ther-


23. ANSWER: C. Hypomagnesemia apy (such as the patient in this case), most consensus
guidelines suggest lowering the blood pressure only if
tp

Acute kidney injury, thrombotic microangioathy,


hyperkalemia, hypomagnesemia, and hyperlipidemia it is above 220/120 mm Hg. his strategy is described
are all associated with the use of calcineurin inhibi- as permissive hypertension. In candidates for throm-
ht

tors such as tacrolimus. Hypercalcemia, hypokalemia, bolytic therapy, blood pressure must be maintained
acute interstitial nephritis, and rhabdomyolysis are not at <180/105 mm Hg to minimize the risk of hemor-
typically associated with tacrolimus toxicity. rhagic conversion. After the acute stroke episode has
resolved, a systolic blood pressure goal of 150 mm Hg
24. ANSWER: B. Minimal change disease may be appropriate in this very elderly patient (>80
he most common cause of nephrotic syndrome in years of age) as per the HYVET trial.
patients with Hodgkin lymphoma is minimal change Beckett NS, Peters R, Fletcher AE, et al. Treatment of hyper-
disease, though focal segmental glomerulosclerosis has tension in patients 80 years of age or older. N Engl J Med.
2008;358(18):1887–1898.
also been described. he other conditions listed do not
share an association with Hodgkin disease. Approxi-
mately 0.4% of patients with Hodgkin lymphoma 28. ANSWER: B. AL amyloidosis
develop minimal change disease, and the degree of Amyloidosis typically presents with a combination of
proteinuria typically parallels the course of the malig- symptoms, signs, and laboratory abnormalities, includ-
nancy. ing nephrotic syndrome, restrictive cardiomyopathy,
CHAPTER 6 Nephrology and Hypertension 141

peripheral neuropathy, hepatomegaly, macroglossia, prednisolone, and cyclophosphamide was superior to


purpura, and abnormal bleeding. he presence of lateral an approach of a methylprednisolone pulse followed
scalloping of the tongue as seen in this patient can be a by prednisolone and cyclophosphamide with regard to
result of the lateral edges of the tongue being lattened mortality and renal function. All of the patients in the
by the teeth in the setting of macroglossia. AA amyloid MEPEX trial were switched to azathioprine for mainte-
tends to occur secondarily to chronic inlammation, nance therapy after 3 months. Dialysis may ultimately
which does not appear to be present in this patient. AL be needed, but based on the information provided in
amyloidosis refers to a monoclonal gammopathy com- the question, it is not emergently indicated.
plicated by the formation of ibrils made up of light Jayne DR, Gaskin G, Rasmussen N, et  al. Randomized trial
chain fragments. he presence of purpura, especially in of plasma exchange or high-dosage methylprednisolone
the periorbital area, is characteristic of AL amyloidosis. as adjunctive therapy for severe renal vasculitis. J Am Soc
Diabetes mellitus can cause both a peripheral neuropa- Nephrol. 2007;18(7):2180–2188.
thy and nephrotic-range proteinuria but usually needs
to be present for several years prior to onset of these 32. ANSWER: A. Vasopressin receptor antagonism
complications. Hypertensive nephropathy is not asso- Chronic kidney disease (CKD) is a risk factor for

t
ne
ciated with nephrotic-range proteinuria. FSGS can be cardiovascular disease, so control of hypertension and
associated with nephrotic syndrome but cannot explain lipids is certainly important. However, the only ther-
the patient’s associated symptoms. apy listed above that has been shown (in the TEMPO

e.
Wechalekar AD, Gillmore JD, Hawkins PN. Systemic amy- trial) to reduce the annual increase in total kidney vol-
loidosis. Lancet. 2016;387(10038):2641–2654. ume and slow the decline in kidney function is tolvap-

in
tan, a vasopressin receptor antagonist. A low-protein
29. ANSWER: E. Eculizumab diet can slow the progression of CKD. Steroid therapy
he patient’s diagnosis is hemolytic uremic syndrome

ic
has no role in the treatment of polycystic kidney dis-
(HUS) without diarrhea, also known as atypical HUS. ease.
Atypical HUS is characterized by a low C3 level. When
it is caused by factor H or I deiciency, it is associated
ed Torres VE, Chapman AB, Devuyst O, et  al. Tolvaptan in
patients with autosomal dominant polycystic kidney dis-
with low CH50 and AH50. ADAMTS13 activity may ease. N Engl J Med. 2012;367(25):2407–2418.
sm
be low but is typically greater than 10%, which helps
distinguish this condition from thrombotic thrombo- 33. ANSWER: B. Start 1,25-hydroxyvitamin D supple-
cytopenic purpura. Blocking the terminal complement mentation
cascade is the most efective therapy for this condition, his patient has secondary hyperparathyroidism
ok

which can be achieved with eculizumab. associated with chronic kidney disease, in which the
Legendre CM, Licht C, Muus P, et al. Terminal complement elevation in PTH results from chronically high phos-
inhibitor eculizumab in atypical hemolytic-uremic syn- phorus and low calcium concentrations. he goal of
bo

drome. N Engl J Med. 2013;368(23):2169–2181. reducing PTH is to prevent high-turnover renal bone
disease. Overreduction of PTH may result in a low-
30. ANSWER: E. Discuss dialysis and transplant. turnover bone disease, so PTH is not typically lowered
e

ISN/RPS class VI lupus nephritis denotes a biopsy all the way down to normal. he irst line of therapy in
://

with global sclerosis noted on more than 90% of the lowering PTH is to control high phosphorus via insti-
glomeruli. In this setting, the kidney is “burned out,” tution of a low-phosphorus diet and a phosphorus-
tp

and immunosuppressive therapy is not warranted. he binding agent. he next line of therapy is to correct for
appropriate course of action in this setting would be to deiciencies in 25-hydroxyvitamin D levels. he next
discuss dialysis and transplant, which are likely in the line of therapy would be to use 1,25-hydroxyvitamin
ht

patient’s near future, even if no acute indications for D to suppress PTH. Cinacalcet is reserved for refrac-
dialysis exist at this time. tory cases, often as a bridge to parathyroidectomy.
Bertsias GK, Tektonidou M, Amoura Z, et  al. Joint Euro- Cunningham J, Locatelli F, Rodriguez M. Secondary hyper-
pean League Against Rheumatism and European Renal parathyroidism: pathogenesis, disease progression, and ther-
Association-European Dialysis and Transplant Association apeutic options. Clin J Am Soc Nephrol. 2011;6(4):913–921.
(EULAR/ERA-EDTA) recommendations for the manage-
ment of adult and paediatric lupus nephritis. Ann Rheum
Dis. 2012;71(11):1771–1782.
34. ANSWER: A. Uromodulin kidney disease
Uromodulin kidney disease (UKD), sometimes
referred to as familial juvenile hyperuricemic nephropathy
31. ANSWER: C. Plasmapheresis, steroids, and cyclo- (FJHN), is the most common subtype of a broader cat-
phosphamide egory of autosomal dominant tubulointerstitial kidney
Based on the results of the MEPEX trial, in patients diseases (ADTKD). UKD is characterized by mutations
with ANCA-associated vasculitis and a creatinine level in the UMOD gene encoding uromodulin (also known
greater than 5.8 mg/dL, an approach of plasma exchange, as Tamm-Horsfall protein). Clinical manifestations
142 C HA P T E R 6 Nephrology and Hypertension

include gout, hyperuricemia, and progressive renal dis- diuretic combined with an ACE inhibitor would be
ease. he early onset of renal disease distinguishes UKD a reasonable regimen. Hydralazine is not a irst-line
from gout and other uric acid renal diseases. Acute uric antihypertensive, and intravenous hydralazine is not
acid nephropathy is typically associated with overpro- indicated.
duction and underexcretion of uric acid from rapid Kessler CS, Joudeh Y. Evaluation and treatment of severe asymp-
cell lysis as occurs in the treatment of certain myelo- tomatic hypertension. Am Fam Physician. 2010;81(4):470–
proliferative disorders, leukemias, and lymphomas. 476.
Lead nephropathy can occur in association with gout, Staford EE, Will KK, Brooks-Gumb AN. Management
though it usually includes some history of workplace, of hypertensive urgency and emergency. Clin Rev.
2012;22(10):20.
water supply, or moonshine exposure. Chronic urate
nephropathy is associated with impaired renal function,
but the early onset of renal disease and family history 38. ANSWER: E. Nephrotic syndrome
make UKD more likely. Nephrogenic diabetes insipidus (and central diabe-
Eckardt KU, Alper SL, Antignac C, et al. Autosomal dominant tes insipidus), chronic interstitial nephritis, and distal
tubulointerstitial kidney disease: diagnosis, classiication, RTA can occur in patients with lithium nephrotoxic-

t
ne
and management—a KDIGO consensus report. Kidney Int. ity. Nephrotic syndrome (related to minimal change
2015;88(4):676–683. disease and focal segmental glomerulosclerosis) has
also been described in patients with lithium nephro-

e.
35. ANSWER: B. Recommend continuous positive air- toxicity. Hyperparathyroidism with resultant hyper-
way pressure (CPAP) at night calcemia has been described in patients taking lithium
Obstructive sleep apnea (OSA) is a known sec-

in
therapy. Other associated indings in this setting
ondary cause of hypertension. he severity of sleep include normal serum phosphorus level and hyperma-
apnea correlates with the severity of hypertension, and

ic
gnesemia.
approximately three-fourths of patients referred with
resistant hypertension who were referred for sleep ed
39. ANSWER: A. Lorazepam
studies were found to have OSA. Treating the OSA he salient features in this patient’s clinical pre-
with CPAP may result in a modest improvement in sentation include an anion gap acidosis and an osmo-
blood pressure. A DASH diet and a low-salt diet may
sm
lality gap (measured osmolality 298 mOsm/kg vs.
also be helpful, but the OSA needs to be addressed calculated osmolality 285 mOsm/kg). his suggests
irst. an unmeasured osmole. his occurs most commonly
with ethylene glycol and methanol poisoning. Pro-
ok

36. ANSWER: B. Amlodipine pylene glycol toxicity can also present this way, with
Based on the results of the ACCOMPLISH trial most reported cases occurring due to administration
published in the New England Journal of Medicine in of medications that contain a propylene glycol sol-
bo

2008, the combination of benazepril and amlodipine vent, including lorazepam and phenobarbital. he
was superior to the combination of benazepril and other answer choices would not cause an osmolality
hydrochlorothiazide in reducing cardiovascular events gap. hough acute alcohol poisoning can result in an
e

in patients with hypertension with a high risk for such osmolality gap, the negative ethanol level suggests
://

events. he other agents would not be irst- or sec- that the ethanol is not directly responsible for the
ond-line therapies for hypertension. Furosemide can osmolality gap.
be used as a diuretic in the presence of chronic kid-
tp

Tsao YT, Tsai WC, Yang SP. A life-threatening double gap


ney disease when thiazide diuretics lose their potency, metabolic acidosis. Am J Emerg Med. 2008;26(3):385.e5–6.
which is not the case in this patient.
ht

Jamerson K, Weber MA, Bakris GL, et al. Benazepril plus amlo- 40. ANSWER: B. Extracranial cerebrovascular involve-
dipine or hydrochlorothiazide for hypertension in high-risk ment is common.
patients. N Engl J Med. 2008;359(23):2417–2428.
In patients with established renal ibromuscular
dysplasia (FMD), as many as 65% of patients can be
37. ANSWER: A. Oral chlorthalidone and lisinopril found to have carotid and vertebral artery involve-
and follow-up as an outpatient ment. FMD is more common in women. hough CT
Hypertensive urgency is deined by severe hyperten- angiogram has excellent diagnostic utility, MR angiog-
sion in the relative absence of symptoms, which is the raphy is poorly sensitive for the diagnosis of FMD due
diagnosis in this patient. As opposed to hypertensive to poor spatial resolution. In patients with newly diag-
emergency, the management of hypertensive urgency nosed hypertension as a result of FMD, percutaneous
involves the use of oral hypertensives to achieve a transluminal angioplasty is the preferred approach if
reduction of blood pressure to goal over hours to days. opting for deinitive therapy. Unlikely atherosclerotic
Given the severe elevation of blood pressure, a two- renal artery stenosis, the pathology of the lesion most
drug regimen should be started, and a thiazide-type commonly found in FMD is medial ibroplasia.
CHAPTER 6 Nephrology and Hypertension 143

41. ANSWER: D. Renal biopsy would be expected to Kruegel J, Rubel D, Gross O. Alport syndrome—insights from
reveal a normal kidney. basic and clinical research. Nat Rev Nephrol. 2013;9(3):170–
Hepatorenal syndrome (HRS) is a feared complica- 178.
tion of end-stage liver disease that carries a great degree
of morbidity and mortality. Hepatorenal syndrome is a 44. ANSWER: B. Calcium oxalate
disorder marked by splanchnic vasodilation and renal Calcium oxalate is the most common stone type for
artery vasoconstriction, with a decrease in renal perfu- all patients who form kidney stones. Patients with a
sion causing reductions in glomerular iltration rate and history of gastric bypass surgery are at increased risk of
sodium excretion. It can be precipitated by infections or calcium oxalate stones, primarily due to increased risk
bleeding. he renal parenchyma would be expected to of hyperoxaluria. Bariatric surgery can lead to malab-
be preserved early in the diagnosis. If a liver transplant sorption of free fatty acids. Fatty acids in the intestinal
were being considered, a kidney transplant would not lumen bind to free calcium, resulting in an increase
be indicated early in the process, because there should in free oxalate, which is absorbed and subsequently
be no irreversible injury present. Octreotide and mid- excreted in the urine. In addition, patients with mal-
odrine are temporizing measures at best because liver absorption may have lower urine volume due to luid

t
ne
transplant is the only cure for type 1 HRS unless there losses resulting from diarrhea and low urinary citrate
is spontaneous improvement in the underlying liver due to chronic metabolic acidosis resulting from diar-
function. Hemodialysis may be diicult to perform in rhea. Both low urine volume and low urinary citrate

e.
patients with HRS because they are frequently hypoten- are risk factors for kidney stone formation. Low urine
sive at baseline. Continuous venovenous hemoiltration citrate is a risk factor for calcium oxalate stones because
citrate is an inhibitor of calcium oxalate formation.

in
may be needed as a bridge to liver transplant if renal
replacement therapy is required. Gonzalez, RD, Canales BK. Kidney stone risk following mod-
ern bariatric surgery. Curr Urol Rep. 2014;15(5):401.

ic
Ginès P, Schrier RW. Renal failure in cirrhosis [Published erra-
tum appears in N Engl J Med. 2011 Jan 27;364(4):389]. N
Engl J Med 2009;361:1279–1290. 45. ANSWER: C. Fabry disease
ed Fabry disease is an X-linked lysosomal storage dis-
42. ANSWER: C. Gitelman syndrome order that may be associated with renal dysfunction in
half of patients by the age of 35 years. Fabry disease
sm
Gitelman syndrome is the most likely diagnosis
in this patient. Although both Gitelman and Bartter should be suspected in young adult patients with the
syndromes are associated with hypotension and hypo- following indings: chronic kidney disease of unclear
kalemia, the presence of low urine calcium favors a etiology, multiple renal sinus and parapelvic cysts,
ok

diagnosis of Gitelman syndrome. If one can remember decreased perspiration, cutaneous vascular lesions, left
that Gitelman syndrome resembles the administration ventricular hypertrophy of unclear etiology, and stroke
of thiazide diuretics (whereas Bartter syndrome resem- of unclear etiology.
bo

bles loop diuretics), it is easy to remember that Gitel- Zarate YA, Hopkin RJ. Fabry’s disease. Lancet. 2008;
man syndrome is associated with a low urine calcium 372(9647):1427–1435.
because it is precisely this property of thiazide diuretics
e

that makes them useful for the treatment of calcium 46. ANSWER: B. Type B lactic acidosis
://

oxalate stone disease by minimizing calcium excretion Type A lactic acidosis is the type most com-
in the urine. Surreptitious vomiting would be expected monly seen and occurs as a result of cell breakdown
resulting from tissue hypoperfusion, as is present
tp

to result in a low urinary chloride resulting from vol-


ume depletion. Both primary hyperaldosteronism and in septic shock. his patient has no clinical signs of
Cushing syndrome are associated with hypokalemia, septic shock, which is documented on the basis of
ht

and these patients are typically hypertensive. In the negative culture data and lack of response to broad-
case of Cushing syndrome, physical examination ind- spectrum antibiotics. Type B lactic acid can be seen
ings may also be readily apparent. with medications (e.g., metformin and certain HIV
medications), malignancy, and alcoholism. d-Lactic
43. ANSWER: B. Alport syndrome acidosis may be seen in patients with short gut syn-
Alport syndrome is an inherited form of glomerular drome as a result of bacterial metabolism in the gut.
dysfunction associated with sensorineural hearing loss his patient most likely has type B lactic acidosis
that is most frequently X-linked in transmission. he related to Hodgkin lymphoma, which may respond to
syndrome afects males out of proportion to females. chemotherapy.
Symptoms may begin with episodes of gross hema-
turia but ultimately progress to proteinuria, hyper- 47. ANSWER: E. Administer fomepizole and prepare
tension, chronic kidney disease, and end-stage renal for dialysis
disease. he other diagnoses would not account for the his patient is likely experiencing ethylene gly-
patient’s hearing loss. col intoxication, which causes a large anion gap and
144 C HA P T E R 6 Nephrology and Hypertension

osmolality gap. Calcium oxalate crystaluria can be chronic diarrhea, diabetes, and metabolic syndrome.
found in ethylene glycol toxicity because oxalate is he most important biochemical risk factor for uric
a metabolite of ethylene glycol. Treatment of eth- acid stones is low urinary pH. In addition to increased
ylene glycol toxicity includes stabilization of the luid intake, management of uric acid stones should
airway and circulatory systems. Inhibition of alco- be focused on urine alkalinization. he target urinary
hol dehydrogenase with fomepizole (or ethanol if pH for management is 6.5–7.0. At a urine pH of
fomepizole is not available) will block ethylene glycol 6.75, greater than 90% of the urinary uric acid will
from being metabolized to toxic metabolites. Dialy- be present as the soluble urate salt. For this reason,
sis will remove ethylene glycol and toxic metabolites uric acid stones can be dissolved by increasing uri-
and is fundamental to management of severe intoxi- nary volume and using therapies (such as potassium
cation. Sodium bicarbonate can be used to correct citrate) to increase urine pH to 6.5–7.0. A urine
severe acidemia, which will also reduce the difusion pH less than 5.5 would increase the risk of uric acid
of toxic metabolites across cell membranes and into stones. A urine pH greater than 7.5 will add little
end-organ tissues. here is a limited role for gastric additional risk reduction for uric acid stones but may
lavage or activated charcoal because ethylene glycol is increase the risk of calcium phosphate stones (which

t
ne
absorbed quickly. form in alkaline urine). Urine pH is not a factor for
calcium oxalate stones, but it is an important factor
48. ANSWER: C. Start oral sodium bicarbonate therapy for uric acid stones.

e.
his patient has hyporeninemic hypoaldosteron- Coe FL, Evan A, Worcester E. Kidney stone disease. J Clin
ism, which is a common renal manifestation of dia- Invest. 2005;115(10):2598–2608.

in
betes mellitus. It manifests as mild hyperkalemia that
leads to decreased ammonia production, resulting in 50. ANSWER: B. Discontinue acetaminophen

ic
decreased urinary acid excretion and metabolic acido- his elderly patient likely has 5-oxoprolinemia (also
sis; this is also known as type IV RTA. Management known as pyroglutamate), which is a metabolite of acet-
of type IV RTA can include both hyperkalemia and
acidosis management. Hyperkalemia can be man-
ed aminophen that can accumulate and cause an anion
gap metabolic acidosis. Risk factors include malnutri-
aged by diuretic therapy, especially when the patient tion and CKD, both of which this patient has. Owing
sm
has volume overload, or by ludrocortisone when vol- to concerns regarding NSAID and opiate use in the
ume overload is not present and blood pressure is not elderly, acetaminophen use has become more common,
elevated. In this patient with relatively mild hyperka- with standing regimens introduced especially in nurs-
lemia and otherwise controlled blood pressure with ing home patients in an efort to manage pain proac-
ok

signs of mild volume overload, one could consider a tively. Discontinuing acetaminophen, assuming such a
diuretic, though not in place of an angiotensin recep- decision would be acceptable to the patient from a pain
tor blocker, which is indicated for diabetic nephropa- management perspective, is the correct choice in this
bo

thy. With underlying hypertension and mild volume case to resolve the acidosis, with some experts recom-
overload, ludrocortisone would not be indicated. Oral mending N-acetylcysteine despite a lack of evidence of
bicarbonate therapy could be considered to increase eicacy. Eliminating the cause of the acidosis is prefer-
e

bufering capacity and prevent bone demineralization. able to starting alkali therapy. Because the patient’s lac-
://

Additionally, there is some evidence that treating met- tate is normal and blood pressure is stable, midodrine is
abolic acidosis in CKD may prevent progression; in not indicated. Although her albumin is low, parenteral
tp

that case, the goal bicarbonate would be >22 mmol/L. nutrition would not be irst-line therapy for malnutri-
Likewise, spironolactone could enhance the hypoaldo- tion. Lisinopril is not relevant to the patient’s acidosis.
sterone state and exacerbate hyperkalemia. Although Fenves AZ, Kirkpatrick HM 3rd, Patel VV, et  al. Increased
ht

potassium citrate would provide base supplementa- anion gap metabolic acidosis as a result of 5-oxoproline
tion, it would not be appropriate to administer potas- (pyroglutamic acid): a role for acetaminophen. Clin J Am
sium salts to this patient. An alternative option not Soc Nephrol. 2006;1(3):441–447.
ofered could also be dietary potassium restriction.
Acknowledgment
49. ANSWER: D. pH 6.5–7.0
Although most kidney stones are calcium oxalate he authors and editors gratefully acknowledge the contri-
stones, uric acid stones still account for 5%–10%. butions of the previous authors, Karandeep Singh and Ajay
Clinical risk factors for uric acid stones include gout, K. Singh.
7
Gastroenterology
MUTHOKA L. MUTINGA, MOLLY PERENCEVICH, AND ROBERT BURAKOFF

1. A 54-year-old man with a long history of gastroesoph- C. his disorder has the highest mortality rate of the

t
ne
ageal relux disease but no other medical conditions ischemic gut disorders.
undergoes an esophagogastroduodenoscopy (EGD), D. he diagnosis can be deinitively made on the basis
which reveals a 6-cm-long segment of salmon-colored of CT imaging.

e.
mucosa extending proximally from the gastroesopha- E. Serum lactate levels are markedly elevated in this
geal junction. Biopsies demonstrate the presence of disorder.

in
Barrett esophagus (intestinal metaplasia of the esopha-
gus), but no dysplasia is noted. Surveillance biopsies 4. A 46-year-old man with a long history of intravenous
obtained 1 year later reveal no dysplasia. drug abuse is noted to have persistent elevation of liver

ic
What is the current recommended surveillance transaminases despite a long period of sobriety from
guideline for this patient?
A. Biopsy of the Barrett segment every 3 months
ed alcohol and illicit drug use. An extensive workup for
causes of liver blood test abnormalities, including a
B. Biopsy of the Barrett segment every 6–12 months hepatitis C antibody test, is negative. However, in light
sm
C. Biopsy of the Barrett segment every 3–5 years of his risk factors, you obtain a hepatitis C viral load,
D. Biopsy of the Barrett segment every 10 years which conirms that he does actually have hepatitis C
E. No further biopsies are necessary. infection.
Which of the following conditions could result in
ok

2. A 24-year-old woman with a long history of non- a false-negative hepatitis C virus (HCV) antibody test
bloody diarrhea and bloating, but with no systemic result?
symptoms such as weight loss, is seen in your oice A. Autoimmune hepatitis
bo

for further management. Her quality of life has been B. Lupus


adversely afected by these symptoms. Prior evalua- C. HIV infection
tion including stool and laboratory tests, imaging, and D. Rheumatoid arthritis
e

colonoscopic biopsies as well as inspection of the ter-


://

minal ileum have been unrevealing. 5. A 57-year-old man with cirrhosis due to alcoholism,
Which of the following treatment options is most who is currently undergoing biweekly therapeutic para-
tp

likely to result in signiicant improvement of symptoms? centesis to manage refractory ascites, is seen in follow-
A. Hyoscyamine up. He is compliant with a low-sodium diet and diuretic
B. Prednisone therapy. Recently, ultrasound imaging revealed patent
ht

C. Rifaximin hepatic and portal veins, and there is no evidence of


D. Mesalamine hepatocellular carcinoma. Ascites luid analysis is not
E. Psyllium suggestive of spontaneous bacterial peritonitis.
Which of the following statements is true regard-
3. A 67-year-old man with a history of congestive heart ing nonselective beta-blocker therapy in patients with
failure and peripheral vascular disease presents with 48 cirrhosis?
hours of left lower quadrant pain, diarrhea with inter- A. It is indicated for management of acute variceal
mittent bleeding, and low-grade fever. Stool cultures bleeding.
are negative for infection, and ischemic colitis is con- B. It does not alter the risk of developing spontane-
sidered as a possible cause of his symptoms. ous bacterial peritonitis.
Which one of the following is true of this disorder? C. It increases mortality in patients with decompen-
A. Digoxin may predispose patients to bowel ischemia. sated cirrhosis.
B. Urgent angiography is useful for identifying a cul- D. It is indicated to prevent variceal formation in
prit blood vessel. patients with early cirrhosis.

145
146 C HA P T E R 7 Gastroenterology

6. A 52-year-old woman presents for evaluation of symp- Which of the following statements regarding gastric
toms of dry eyes and mouth. She is not taking any polyps is true?
medications that could cause these symptoms and is A. Fundic gland polyps and adenomas are the most
otherwise asymptomatic. A thorough history and phys- common epithelial gastric polyps.
ical examination are unrevealing. Results of laboratory B. Hyperplastic gastric polyps have no malignant
tests for primary Sjögren syndrome are negative. potential.
Which of the following gastrointestinal disorders C. Fundic gland polyps in familial adenomatous
is often associated with secondary Sjögren syndrome? polyposis syndrome (FAP) have no malignant
A. Gastroesophageal relux disease potential.
B. Small intestinal bacterial overgrowth D. PPI-associated fundic gland polyps are not associ-
C. Gallstone pancreatitis ated with an increased risk of cancer.
D. Primary biliary cirrhosis
E. Ulcerative colitis 10. A 22-year-old man returned early from a volun-
teer trip to Mexico 3 weeks ago, where he and his
7. A 34-year-old with intermittent postprandial bloating two friends developed watery diarrhea and a fever.

t
ne
and pain undergoes abdominal ultrasound imaging to Although the diarrhea and fever resolved, the man
evaluate for gallstones. No gallstones are found, and subsequently developed painful swelling of his left
the gallbladder appears normal. However, she is noted knee. Testing for gonorrhea is negative. Reactive

e.
to have a 2.5-cm solid lesion in the right lobe of the arthritis associated with the recent enteric infection
liver. She has no history of malignancy and no risk is suspected.

in
factors for chronic hepatitis B. She has been on oral Which of the following is true about this condition?
contraceptives for 10 years. A. Intestinal infection with ameba has been associ-

ic
Which of the following liver lesions is most strongly ated with this syndrome.
associated with long-term use of oral contraceptives? B. There is a higher prevalence of HLA-DQ2
A. Cavernous hemangioma
B. Focal nodular hyperplasia
ed antigen.
C. It is more common in women than in men.
C. Gastrointestinal stromal tumor D. It may be associated with a triad of arthritis, con-
sm
D. Hepatic adenoma junctivitis, and esophagitis.
E. Nodular regenerative hyperplasia E. Synovial luid shows an elevated leukocyte count.

8. A 62-year-old farmer presents for a second opinion 11. A 57-year-old man is referred to your clinic for
ok

after unrevealing extensive workup of migrating joint evaluation of symptoms of dysphagia. He was
pain for the past 2 years and chronic diarrhea, post- asymptomatic until 6 months ago, when he began
prandial abdominal cramping, and weight loss for the developing proximal upper and lower extremity
bo

past 6 months. Your suspicion for Whipple disease is muscle weakness, double vision, and drooping of
high. his eyelids, more noticeable at the end of the day
Which of the following statements regarding this than at other times. He was recently admitted to the
e

disease is true? hospital when he developed a cough and fever and


://

A. Chronic diarrhea is common, but extraintestinal was found to have a right middle lobe pneumonia
symptoms are rare. thought to be likely due to aspiration. He endorses
tp

B. Prolonged antibiotic treatment is required to erad- a sensation of diiculty chewing and swallowing.
icate the causative organism. He denies a sensation of food getting stuck after he
C. he causative organism, Tropheryma whipplei, is swallows.
ht

easily cultured. What is his likely diagnosis?


D. he disease is more common in women than in A. Goiter
men. B. Early hypopharyngeal cancer
E. Colonoscopy with biopsy is the diagnostic test of C. Anxiety disorder
choice. D. Myasthenia gravis
E. Gastroesophageal relux disease
9. A 39-year-old man with long-standing gastroesopha-
geal relux disease on long-term proton pump inhibi- 12. A 33-year-old woman is seen in your oice for evalu-
tor (PPI) therapy undergoes an upper endoscopy for ation of fatigue, malaise, and mild right upper quad-
evaluation of recent exacerbation of symptoms. No rant discomfort, ongoing for the past 1 month. Her
Barrett esophagus, hiatal hernia, or relux esophagi- medical history is notable for hypothyroidism, and
tis is noted. However, many small to medium-sized her only medication is levothyroxine. Her examina-
gastric polyps are found and removed. he pathology tion result is notable only for mildly tender hepa-
report conirms that they are all fundic gland polyps. tomegaly. On further questioning, she reports no
CHAPTER 7 Gastroenterology 147

risk factors for viral hepatitis. She does not consume 15. A 22-year-old Caucasian man with end-stage cystic
alcohol or illicit drugs, nor does she take herbs or ibrosis is being evaluated for a lung transplant. He
supplements. Her mother has lupus, and a maternal was diagnosed with cystic ibrosis after presenting with
aunt has rheumatoid arthritis. Laboratory test results meconium ileus as a newborn. Since then, he has expe-
are notable for ALT 480 U/L, AST 211 U/L, total rienced a variety of gastrointestinal complications of
bilirubin 0.4 mg/dL, alkaline phosphatase 89 U/L, cystic ibrosis.
total protein 9.1 g/dL, albumin 4.1 g/dL, and TSH Which of the following are the most common gas-
2.1 µIU/mL. Her antismooth muscle antibody titer trointestinal complications of cystic ibrosis (CF)?
is 1:320. A. Small intestinal bacterial overgrowth and dysphagia
Which of the following statements about her likely B. Gastroesophageal relux disease and constipation
diagnosis is true? C. Gastroesophageal relux disease and diarrhea
A. his condition most often occurs in men. D. Pancreatic insuiciency and gallstones
B. he treatment of choice is prednisone with or E. Gallstones and dysphagia
without azathioprine.
C. he presence of other autoantibodies is uncom- 16. A 28-year-old man with eczema undergoes an urgent

t
ne
mon. upper endoscopy in the emergency room to dislodge
D. here is a very low risk of developing cirrhosis. an esophageal food impaction. Examination of his
E. It does not recur after liver transplant. esophagus after removal of the food bolus reveals

e.
multiple thin rings and linear furrows along most of
13. A 41-year-old man with ulcerative colitis, well con- the esophagus. No hiatal hernia, visible esophageal

in
trolled for several years with mesalamine, presents inlammation, mass, or luminal narrowing is seen.
with a 1-week history of up to 15 foul-smelling, Biopsies from the middle and distal esophagus are

ic
watery stools per day. Laboratory workup reveals mild obtained.
leukocytosis with mild elevation in the baseline serum Further questioning reveals a several years’ history
creatinine level. He tests positive for Clostridium dif-
icile and is started on oral vancomycin, which results
ed of intermittent dysphagia for solids with several prior
episodes of food impaction that resolved without need
in quick improvement in symptoms. for medical attention. he patient has not experienced
sm
Which of the following statements regarding weight loss or frequent heartburn. His hematocrit is
C. diicile infection is true? normal.
A. Stool testing for cure should be performed after What is his likely diagnosis?
completion of treatment. A. Esophageal lichen planus
ok

B. here is a hypervirulent strain associated with B. Schatzki ring


lower clinical cure rates and increased recurrence C. Eosinophilic esophagitis
rates. D. Peptic stricture
bo

C. Testing of all patients with inlammatory bowel E. Plummer-Vinson syndrome


disease (IBD) who are hospitalized with a disease
lare is not recommended. 17. A 64-year-old obese woman with a history of chronic
e

D. Repeat testing after a negative test should be per- Salmonella typhi is noted to have a 3-cm gallstone. Her
://

formed. daughter did some research about her mother’s condi-


E. Hand hygiene with an alcohol-based hand sani- tion and was quite alarmed about what she discovered.
tp

tizer soap is more efective than soap-and-water She decided to accompany her mother to her next
sanitizers. appointment and asked a number of questions. One
of these was related to concern about the increased risk
ht

14. A 79-year-old man presents with a 9-month history of for what type of cancer?
diiculty swallowing with a sensation of a lump in his A. Hepatocellular cancer
throat. His son, who accompanied him for the oice B. Sarcoma
appointment, also notes that he frequently experiences C. Breast cancer
regurgitation and coughing while eating and that he D. Gallbladder cancer
has noticed over the same period that his father has E. Colorectal cancer
had a foul breath odor.
Based on your clinical suspicion, what is the best 18. A 31-year-old man with AIDS and a CD4 count of 35
test to diagnose a Zenker diverticulum? undergoes an esophagogastroduodenoscopy (EGD)
A. Esophagogastroduodenoscopy (EGD) for evaluation of odynophagia that failed to respond
B. Barium esophagram to empiric therapy for candidal esophagitis. Several
C. CT scan small, shallow ulcers are noted in the middle and dis-
D. Esophageal motility study tal esophagus, and biopsies are obtained for further
E. Laryngoscopy evaluation.
148 C HA P T E R 7 Gastroenterology

Which of the following is the most likely diagnosis? diarrhea, and her parents have noted that she appears
A. Pill-induced esophageal ulcer disoriented at times. She denies use of laxatives, illicit
B. Idiopathic esophageal ulcer drugs, or over-the-counter medications. Her food
C. Herpes simplex virus (HSV)–associated ulcer intake has been very poor over the past few months.
D. Cytomegalovirus (CMV)-associated ulcer What vitamin deiciency is she likely to have?
E. Mycobacterium avium complex (MAC)–associated A. Vitamin C deiciency
ulcer B. Niacin deiciency
C. hiamine deiciency
19. A 52-year-old woman with a 30-year history of panco- D. Vitamin A deiciency
litis due to ulcerative colitis is seen for routine follow- E. Vitamin K deiciency
up in the oice. Her ulcerative colitis has been under
good control for most of the duration of the disease. 22. A 77-year-old man with a remote history of a transient
Which of the following statements regarding ischemic attack and atrial ibrillation, currently main-
screening for colonic dysplasia in patients with inlam- tained on warfarin, presents with brisk, painless hema-
matory bowel disease is true? tochezia began 4 hours earlier. He does not take aspirin

t
ne
A. Screening colonoscopy examinations for patients or nonsteroidal antiinlammatory drugs (NSAIDs) and
who have ulcerative proctitis should begin after has no history of gastrointestinal bleeding. He last had
10–12 years of disease. a colonoscopy at age 70 but does not recall the ind-

e.
B. Patients with isolated Crohn ileitis have an ings, and the procedure report is not accessible. He
increased risk of developing colorectal cancer. has no history of prior operations, including vascular

in
C. Patients with left-sided ulcerative colitis are not at surgery. On presentation he was normotensive, but he
an increased risk of developing colorectal cancer became lightheaded and orthostatic with mild tachy-

ic
compared with the general population. cardia when standing up. His hematocrit is 34% (base-
D. Patients who are noted to have high-grade dys- line hematocrit was 42% 3 months ago), INR is 1.4,
plasia, conirmed by a second pathologist, should
undergo more intensive surveillance colonoscopies
ed platelet count is 256,000/µL, BUN is 28 mg/dL, and
creatinine is 0.8 mg/dL. After luid resuscitation with 1
every 6 months. L of normal saline, he is no longer tachycardic or ortho-
sm
E. Patients with inlammatory bowel disease and pri- static, but he continues to pass blood from the rectum.
mary sclerosing cholangitis (PSC) should undergo Which of the following statements regarding his
surveillance colonoscopy every 1–2 years follow- management is correct?
ing the diagnosis of PSC. A. An unprepped colonoscopy should be performed
ok

next.
20. A 72-year-old man with a history of coronary artery B. Fresh frozen plasma should be administered while
disease and arthritis presents to the emergency depart- awaiting the next intervention.
bo

ment with melena. His wife reports that he takes both C. Angiography to localize and treat the source of
aspirin and ibuprofen. She also noticed that he seemed bleeding should be performed next.
confused today. A physical examination reveals that his D. An upper esophagogastroduodenoscopy should be
e

heart rate is 98 beats per minute and his blood pressure performed next.
://

is 85/43 mm Hg. His laboratory indings are notable


for a hematocrit of 26% (baseline, 42%), normal INR, 23. A 53-year-old man who underwent a Roux-en-Y gas-
tp

and normal platelet count, as well as a normal albumin tric bypass surgery 1 year ago is seen for evaluation
level and other components of the comprehensive met- of fatigue. He has not experienced melena, hemato-
abolic panel, except for an elevated blood urea nitrogen chezia, or any gastrointestinal symptoms, and he does
ht

(BUN) level. His calculated AIMS65 score is 3. not take aspirin or NSAIDs. He takes a proton pump
Which of the following is a criterion used to calcu- inhibitor once per day and an iron supplement, but he
late the AIMS65 score? has not been taking a multivitamin that had been pre-
A. Presence of melena scribed following the surgery. Mild muscle weakness is
B. Age >45 years noted on examination of his upper and lower extremi-
C. Systolic blood pressure <120 mm Hg ties. Laboratory tests are notable for microcytic ane-
D. INR >2.0 mia and mild neutropenia. A colonoscopy performed
E. Albumin <4.0 1 year ago was normal.
What is the likely cause of his anemia?
21. A 23-year-old woman with anorexia nervosa is seen A. Folate deiciency
in your oice for evaluation of a rash. Her examina- B. Iron deiciency
tion is notable for a symmetric erythematous, pruritic, C. Copper deiciency
blistering rash on exposed skin resembling a sunburn, D. Vitamin B12 deiciency
even though it is late winter. She reports intermittent E. Calcium deiciency
CHAPTER 7 Gastroenterology 149

24. A 19-year-old college student presents with a 6-month D. Allopurinol


history of abdominal bloating, diarrhea, and unex- E. Pentoxifylline
plained weight loss. She recently also noticed a rash
on her elbows. Her medical history is notable for type 27. A 23-year-old medical student with frequent heart-
1 diabetes mellitus. In addition, she was recently diag- burn since starting medical school 2 years ago is
nosed with vitamin D deiciency despite frequent sun seen in your oice for further management. His
exposure. Considering her likely diagnosis, which of symptoms are mostly nocturnal, and he reports
the following may also be noted? frequently eating fast food at night shortly before
A. Pericarditis going to bed.
B. Paget disease of the bone Which statement regarding gastroesophageal relux
C. Hypersplenism disease (GERD) is true?
D. Idiopathic pulmonary hypertension A. Surgical therapy should be considered in patients
E. Abnormal liver function test results who do not respond to proton pump inhibitor
(PPI) therapy.
25. A 62-year-old man with chronic hepatitis C–associated B. Routine biopsies from the distal esophagus are rec-

t
ne
cirrhosis presents to the emergency room with con- ommended to diagnose GERD.
fusion and abdominal pain. He is noted to have C. he treatment of choice for symptomatic relief
ascites, peripheral and scrotal edema, and asterixis. and healing of erosive esophagitis is an 8-week

e.
Which statement is true regarding care of patients course of a proton pump inhibitor.
with cirrhosis and potential complications? D. Barium radiographs are useful for the diagnosis of

in
A. Bare stents are preferred for transjugular intrahe- GERD.
patic portosystemic shunts (TIPS). E. Obesity is not associated with increased risk for

ic
B. Patients with ascites luid total protein <1.1 g/dL development of GERD.
and serum bilirubin >2.5 mg/dL should receive
prophylactic antibiotics.
C. Endoscopic variceal sclerotherapy is the preferred
ed
28. A 65-year-old woman is noted to have a 1.2-cm soli-
tary cyst with a central scar in the head of the pan-
option for secondary prophylaxis of variceal hem- creas on an abdominal CT scan obtained to evaluate
sm
orrhage. acute left lower quadrant pain. In addition to being
D. Ambulatory patients who have an ascites luid prescribed antibiotics for conirmed acute uncompli-
polymorphonuclear count >150 cells/mm3 should cated sigmoid diverticulitis, she is advised to follow
receive empiric antibiotics within 24 hours of their up with a gastroenterologist regarding the newly diag-
ok

test result. nosed pancreatic cyst. She has had no prior abdominal
E. he Model for End-Stage Liver Disease (MELD) imaging for comparison.
score is based on albumin, INR, and total Which of the following cystic pancreatic lesions has
bo

bilirubin. the lowest malignant potential?


A. Mucinous cystic neoplasm
26. A 29-year-old woman with a history of several epi- B. Main-branch intraductal papillary mucinous neo-
e

sodes of severe acute abdominal pain lasting several plasm


://

days is seen in your oice for further evaluation. C. Solid pseudopapillary neoplasm
Review of laboratory test results during episodes D. Serous cystadenoma
tp

of pain is notable for elevated lipase (greater than E. Side-branch intraductal papillary mucinous neo-
three times the upper limit of normal), but her liver plasm
biochemical test results and triglyceride level are
ht

normal. Results of toxic screens, including tests for 29. A 36-year-old man is referred for colonoscopy to
alcohol, are negative. Ultrasound imaging during at evaluate occasional rectal bleeding and mild iron dei-
least one episode of pain reveals a normal gallblad- ciency. A colonoscopy examination reveals 15 small
der without stones or sludge, as well as a normal to medium-sized polyps, which are all removed. he
bile duct size. She does not smoke or drink alco- pathology report indicates that all the polyps are tubu-
hol, and she has no family history of pancreatitis. lar adenomas. he patient has no family history of
She is referred for consideration of Oddi sphincter colon cancer or other malignancies.
manometry. Which of the following intestinal polyposis syndromes
Which of the following medications may reduce is characterized by multiple adenomatous polyps?
the risk of postendoscopic retrograde cholangiopan- A. Peutz-Jeghers syndrome
creatography (post-ERCP) pancreatitis? B. MUTYH-associated polyposis
A. N-acetylcysteine C. Juvenile polyposis syndrome
B. Octreotide D. Cowden syndrome
C. Rectal indomethacin E. Cronkhite-Canada syndrome
150 C HA P T E R 7 Gastroenterology

30. A 35-year-old man with idiopathic cardiomyopathy bilirubin and transaminases as well as mild coagulopa-
is noted to have mild hypoalbuminemia, mild liver thy. Neurologic examination reveals that he has grade
biochemical test abnormalities, and a small amount of I encephalopathy. he results of evaluation for infec-
ascites on ultrasound imaging of the abdomen. He has tious, toxic, autoimmune, and vascular causes of acute
no history of alcoholism, and test results for chronic liver failure are negative.
liver disease are negative. Which of the following metabolic liver diseases
Which of the following clinical or laboratory ind- may result in acute liver failure?
ings is typical of congestive heart failure (CHF)–asso- A. Nonalcoholic steatohepatitis
ciated liver disease? B. Hemochromatosis
A. Presence of portosystemic shunts such as esopha- C. Alpha-1 antitrypsin deiciency
geal varices D. Wilson disease
B. Normal total protein (>2.5 g/dL) in ascites E. Glycogenic hepatopathy
C. Presence of clinically overt jaundice
D. Serum-to-ascites albumin gradient (SAAG) <1.1 34. A 27-year-old man with ulcerative colitis was recently
g/dL diagnosed with primary sclerosing cholangitis (PSC)

t
ne
E. Absence of hepatojugular relux after undergoing evaluation of isolated elevation of
alkaline phosphatase. His transamines, albumin, and
31. A 26-year-old woman with prior history of IV drug bilirubin levels, as well as his prothrombin time, are all

e.
abuse resulting in HIV and hepatitis C coinfection is normal. He has done some research regarding PSC and
seen in a prenatal clinic for routine care. She is in the learned that cholestatic liver diseases such as primary

in
third trimester of pregnancy with her irst child. biliary cirrhosis (PBC) and PSC may result in fat-sol-
Which of the following statements regarding verti- uble vitamin deiciencies, especially in the setting of

ic
cal transmission of hepatitis C virus (HCV) is true? more advanced liver disease. Symptoms of which fat-
A. Cesarean section reduces the risk of transmission soluble vitamin may result in loss of proprioception?
to a greater degree than vaginal delivery.
B. HIV coinfection does not alter the risk of trans-
ed A. Vitamin A
B. Vitamin D
mission. C. Vitamin E
sm
C. Breastfeeding is not associated with increased risk D. Vitamin K
of transmission. E. Vitamin B12
D. Patients with HCV genotype 1 have a greater risk
of transmission than other genotypes. 35. You are asked for advice regarding consideration
ok

of transjugular intrahepatic portosystemic shunt


32. A 46-year-old man is referred for upper endoscopy (TIPS) placement in a 54-year-old man with cirrho-
for evaluation of a 1-month history of nausea, epi- sis complicated by refractory hepatic hydrothorax.
bo

gastric pain, diarrhea, and weight loss. He is noted to He has no history of hepatic encephalopathy and
have bilateral lower extremity edema. Marked difuse is compliant with diuretics and a low-sodium diet.
thickening of gastric folds is noted on upper endo- Your colleague wants to learn more about situations
e

scopic evaluation. Histologic examination of biopsies in which a TIPS procedure may be useful and when
://

of the gastric folds reveals extreme foveolar hypertro- it is contraindicated.


phy and glandular atrophy suggestive of Ménétrier In which of the following conditions should a TIPS
tp

disease. procedure be avoided?


Which of the following is true regarding Ménétrier A. Refractory ascites in a patient with right heart
disease in adults? failure
ht

A. he disease is generally self-limited. B. Acute refractory variceal bleeding not controlled


B. Hypoalbuminemia results primarily from dimin- with pharmacologic and endoscopic therapy
ished protein absorption. C. Hemorrhage from inaccessible gastric or intestinal
C. It occurs more frequently in women than in men. varices
D. Treatment with an H2 receptor blocker may pro- D. Bleeding from portal hypertensive gastropathy
vide symptomatic relief. E. Refractory hepatic hydrothorax
E. Supericial biopsies of the stomach are generally
suicient to establish the diagnosis. 36. A 52-year-old man with a history of heartburn and
cardiomyopathy presents with diiculty swallowing
33. A previously healthy 33-year-old man with no known solids and liquids for the past 7 months. An upper
underlying medical conditions presents with malaise endoscopy shows relux esophagitis but no stric-
and jaundice ongoing for the past week or more. Lab- tures or masses. Esophageal manometry shows low-
oratory test results are notable for marked elevation of amplitude contractions in the distal esophagus and
CHAPTER 7 Gastroenterology 151

incompetence of the lower esophageal sphincter. he addition to following her liver enzymes and provid-
skin of his hands has become thicker, which he thinks ing the infant with hepatitis B immune globulin
is from spending more time at work. He is originally (HBIG) and hepatitis B vaccination, which of the
from Brazil but has been living in the United States for following would you recommend with regard to her
the last 20 years and works as a mechanic. hepatitis B infection?
Which test result is most likely to be found in this A. No treatment at this time
patient? B. Tenofovir
A. Barium esophagram with distal “bird’s beak” nar- C. Pegylated interferon
rowing D. Cesarean delivery at term
B. Positive Trypanosoma cruzi antibody E. Lamivudine
C. Positive anti–Scl-70 antibody
D. Adenocarcinoma on biopsies of the stomach 40. A 32-year-old woman with ulcerative colitis comes to
E. Lung mass on chest CT scan your oice for evaluation of symptoms. She was diag-
nosed with mild to moderate pancolitis last year. She
37. A 45-year-old woman presents with postprandial initially responded well to induction and maintenance

t
ne
epigastric pain for the past 8 months. She does not therapy with mesalamine. However, 2 months ago, she
have dysphagia, anemia, nausea, or weight loss. A developed bloody diarrhea occurring six times per day,
Helicobacter pylori stool antigen test result is nega- as well as lower abdominal cramping. Treatment with

e.
tive. An empiric trial of a proton pump inhibitor for prednisone 40 mg daily resulted in improved symp-
8 weeks does not improve her symptoms. he result toms, but symptoms recurred when the prednisone

in
of esophagogastroduodenoscopy (EGD) is unremark- dose was tapered to 10 mg daily. A physical exami-
able, including gastric biopsies. Laboratory and imag- nation reveals her vital signs are normal, and she has

ic
ing indings are not suggestive of biliary or pancreatic minimal left lower quadrant tenderness. Laboratory
disease. studies reveal a white blood cell count of 11,000/µL
Which of the following is the most appropriate
next step in management?
ed and hematocrit of 34.5%, and the result of stool test-
ing for Clostridium diicile is negative.
A. A low-fat diet Which of the following is the most appropriate
sm
B. Ranitidine next step in treating this patient?
C. Metoclopramide A. Switch to balsalazide.
D. Amitriptyline B. Add budesonide.
E. Probiotics C. Increase prednisone to 20 mg daily and continue
ok

indeinitely.
38. A 69-year-old man with coronary artery disease status D. Increase prednisone to 20 mg/day and add aza-
after myocardial infarction and percutaneous coronary thioprine.
bo

intervention 2 years ago presents with melena and E. Add ciproloxacin.


anemia. Esophagogastroduodenoscopy (EGD) identi-
ies an 8-mm clean-based ulcer in the duodenal bulb. 41. A 61-year-old woman presented to the hospital with
e

In addition to his usual aspirin, he recently started new-onset abdominal distention and jaundice. She has
://

taking ibuprofen daily for back pain. In addition to a long history of drinking two or three glasses of wine
stopping the ibuprofen and prescribing omeprazole to per day and has been drinking more than this for the
tp

facilitate ulcer healing, what do you recommend at past few months in the context of social stressors. She
this time? denies abdominal pain, rectal bleeding, melena, con-
A. Start sucralfate fusion, fever, or chills. A physical examination reveals
ht

B. Lifelong use of omeprazole to prevent ulcer recur- her vital signs are normal. She is mildly jaundiced and
rence appears to have ascites, but she has no leg edema, spi-
C. Repeat EGD in 8 weeks der angiomas, or asterixis. Her laboratory studies show
D. Stopping his aspirin ALT 66 U/L, AST 424 U/L, alkaline phosphatase 285
E. Testing for H. pylori infection U/L, total bilirubin 6.3 mg/dL, albumin 3 g/dL, white
blood cell count 15,000/µL, hematocrit 35%, plate-
39. A 24-year-old woman from Algeria is pregnant with lets 345,000/µL, and INR 1.3. Her test result for viral
her irst child. Routine screening identiies her as hepatitis is negative. An abdominal ultrasound reveals
having a positive HBsAg. Additional testing shows moderate ascites and fatty liver with Doppler measure-
negative HBsAb, negative HBeAg, and positive ments suggesting portal hypertension, but she has no
HBeAb. Her hepatitis B viral load is 1186 IU/mL at splenomegaly or evidence of portosystemic shunts.
the end of the second trimester. Liver enzymes, INR, Paracentesis shows ascites luid albumin of 0.9 g/dL
and abdominal ultrasound indings are normal. In and WBC 195/mm3.
152 C HA P T E R 7 Gastroenterology

Which of the following is the most appropriate Which of the following tests is most likely to be
next step in management? diagnostic in this patient?
A. Cefotaxime A. Serum vasoactive intestinal peptide
B. Supportive care B. Serum calcitonin
C. Prednisolone C. Serum gastrin
D. Nadolol D. Urinary 5-hydroxyindoleacetic acid
E. Pentoxifylline E. Fat pad biopsy with Congo red staining

42. A 54-year-old man is hospitalized with pancreatitis 45. A 67-year-old man with cirrhosis due to hepatitis C
related to alcohol use. Five days after admission, he presents to the emergency department with hemateme-
continues to have epigastric pain and nausea, and sis and hematochezia. He is initially hypotensive, but
he has not been able to tolerate much oral intake. A he is stabilized with intravenous luids and blood trans-
physical examination reveals his vital signs are nor- fusions. A physical examination reveals he is jaundiced
mal, but he has difuse abdominal pain with pal- and moderately encephalopathic, has spider angiomas
pation. Laboratory studies show a white blood cell on his chest, and has moderate ascites. Concurrently

t
ne
count of 14,300/µL, ALT 89 U/L, AST 345 U/L, with intravenous octreotide infusion and PPI therapy,
alkaline phosphatase 176 U/L, and total bilirubin which of the follow treatments should be initiated at
2.3 mg/dL. Abdominal CT with intravenous con- this time?

e.
trast shows a difusely edematous pancreas with A. Penicillin
multiple small peripancreatic luid collections, no B. Intravenous albumin

in
pancreatic necrosis, and no gallstones or biliary C. Ceftriaxone
dilation. D. N-acetylcysteine

ic
Which of the following is the most appropriate E. Vasopressin
next step in management?
A. Parenteral nutrition
B. Enteral nutrition with a nasojejunal tube
ed
46. A 78-year-old man with hypertension and diabetes
presents for evaluation of ongoing anemia for the past
C. Imipenem 5 months. He denies abdominal pain, acid relux, nau-
sm
D. Pancreatic debridement sea, rectal bleeding, or melena. He has felt more tired
E. Interventional radiology drainage of luid collec- and short of breath with exertion, but he has not had
tion chest pain, cough, or lightheadedness. His medica-
tions are lisinopril, insulin, and aspirin.
ok

43. A 34-year-old woman with autoimmune thyroid- A physical examination shows his vital signs are
itis presents with recurrent episodes of pancreati- normal. He has no scleral icterus, lymphadenopa-
tis. Abdominal ultrasound shows no evidence of thy, or cardiac murmurs. His lungs are clear, and
bo

gallstones, and she denies signiicant alcohol use. his abdomen is soft, nontender, and nondistended
Abdominal CT shows a difusely enlarged pancreas with no organomegaly. Laboratory testing shows
with no focal lesions. A serum IgG4 test result is a hematocrit of 28% (his prior baseline was 40%)
e

elevated. with a mean corpuscular volume of 72 fL, as well


://

Which of the following therapies is most likely to as normal white blood cell count, normal platelet
prevent additional episodes of pancreatitis? count, and normal coagulation studies. His ferri-
tp

A. Prednisone tin level is 7 µg/L. The result of fecal occult blood


B. Alcohol cessation testing is positive. A colonoscopy and esophagogas-
C. Cholecystectomy troduodenoscopy (EGD) did not reveal a bleeding
ht

D. Pancreaticoduodenectomy (Whipple procedure) source. The result of CT enterography was unre-


E. Fibrate markable. He received 2 U of blood transfusion for
symptomatic anemia.
44. A 73-year-old woman presents with persistent diarrhea Which of the following is the next most appropri-
for the past 6 months. he diarrhea is watery, occurs ate step in evaluation?
8–10 times per day, including at night, and does not A. Push enteroscopy
improve with fasting. She also reports episodes of B. CT angiography
lushing as well as fatigue and dyspnea on exertion. C. Wireless capsule endoscopy
Stool study results are negative for infection, and her D. Double-balloon enteroscopy
white blood cell count is normal. An electrocardio- E. Tagged red blood cell scan
gram shows low-voltage QRS, a chest x-ray shows car-
diomegaly, and an echocardiogram shows moderate to 47. A 53-year-old woman with chronic pancreatitis and
severe tricuspid regurgitation. diabetes presents for evaluation of loose stool for the
CHAPTER 7 Gastroenterology 153

past 4 months. She reports four or ive bowel move- 49. A 61-year-old woman with a history of hypertension
ments per day, including occasional nocturnal symp- and low back pain presents to the emergency depart-
toms. She has bloating and chronic abdominal pain ment with epigastric pain for the past 36 hours. he
but no rectal bleeding or nausea. Her medical his- pain is constant and radiates to her back. She also
tory includes chronic pancreatitis due to prior alco- has nausea, vomiting, and decreased oral intake. Her
holism, diabetes mellitus, and seasonal allergies. She medications are hydrochlorothiazide and occasional
takes lisinopril, insulin, aspirin, oxycodone, pancreatic acetaminophen. A physical examination reveals her
enzyme supplementation, and loratadine. Her sister temperature is 100.3°F, heart rate 95 beats per min-
has celiac disease. She reports no change in diet or ute, blood pressure 136/84 mm Hg, respiration rate
increase in fatty food consumption. 14 per minute, and BMI 31 kg/m2. She has scleral
A physical examination reveals her vital signs are icterus and dry mucous membranes. Her abdomen is
normal. She has mild epigastric pain with palpation but tender in the epigastrium and right upper quadrant,
is otherwise not distended and has no organomegaly. but there is no rebound or guarding. She does not
She has no rashes. Laboratory studies reveal normal have ascites or spider angiomas. Laboratory testing
liver enzymes and lipase, hematocrit 33% with mean reveals a white blood cell count of 13,600/µL, lipase

t
ne
corpuscular volume 102 fL, and normal white blood 3578 U/L, alanine aminotransferase 347 U/L, aspar-
cell count and platelets. Her vitamin B12 level is 198 tate aminotransferase 266 U/L, alkaline phosphatase
pg/mL; her folate level is 31 ng/mL; and her tissue 272 U/L, and total bilirubin 5.2 mg/dL. Abdominal

e.
transglutaminase IgA antibody is normal (with normal ultrasonography shows cholelithiasis and a dilated
total serum IgA level). Test results for stool culture, ova common bile duct to 11 mm but no choledocholithi-

in
and parasite examinations, and Giardia antigen are neg- asis. Twelve hours later, her pain remains unchanged,
ative. he result of a colonoscopy including the termi- her total bilirubin is 5.8 mg/dL, and her temperature

ic
nal ileum is normal, including random biopsies of the is 101.1°F.
colon. he result of an esophagogastroduodenoscopy Which of the following is the most appropriate
(EGD) including biopsies of the duodenum is normal.
Abdominal imaging shows stable features of chronic
ed next step in management?
A. Abdominal CT
pancreatitis but no intestinal inlammation or stric- B. Magnetic resonance cholangiopancreatography
sm
tures. An increase in the dose of her pancreatic enzyme C. Cholecystectomy
supplementation does not improve her symptoms. D. Ursodiol
Which of the follow is the most likely diagnosis? E. Endoscopic retrograde cholangiopancreatography
A. Steatorrhea due to pancreatic exocrine dysfunc-
ok

tion 50. A 35-year-old woman is being evaluated for chronic


B. Irritable bowel syndrome abdominal pain and constipation for the past 3 years.
C. Celiac disease he pain is located in the lower abdomen, occurs
bo

D. Small intestinal bacterial overgrowth approximately once per week, and improves after
E. Crohn disease defecation. She reports having a bowel movement
approximately every 3 days, although the frequency
e

48. A 29-year-old woman with Crohn disease and prior varies. he stools are hard, and she often has a sense of
://

small bowel resections presented with severe left lank incomplete evacuation. She denies nausea, vomiting,
pain. Urinalysis reveals 100–200 red blood cells per rectal bleeding, weight loss, or neurologic symptoms.
tp

high-power ield and no bacteria. A kidney stone is She has one child. Her mother had colorectal cancer at
identiied on a CT scan. She reports no recent change age 59.
in her medications (inliximab, calcium, vitamin D), A physical examination reveals her vital signs are
ht

but she has been eating more spinach salads recently. normal. Her abdominal examination is unremarkable,
In addition to increasing her luid intake, what rec- and her rectal tone is normal. Laboratory evaluation
ommendations would you give this patient to prevent shows normal renal function, electrolytes, complete
additional stone formation? blood count, and thyroid-stimulating hormone. he
A. Low-oxalate diet and stop the calcium supplemen- results of a colonoscopy and radiopaque marker study
tation are normal.
B. Low-oxalate diet and continue the calcium supple- What is the most likely diagnosis for this patient?
mentation A. Colonic inertia
C. High-oxalate diet and stop the calcium supple- B. Dyssynergic defecation
mentation C. Constipation-predominant irritable bowel syn-
D. High-oxalate diet and continue the calcium drome
supplementation D. Multiple sclerosis
E. Potassium citrate E. Chronic intestinal pseudoobstruction
154 C HA P T E R 7 Gastroenterology

Chapter 7 Answers

1. ANSWER: C. Biopsy of the Barrett segment every in ischemic colitis and is more useful for evaluation
3–5 years of suspected acute mesenteric ischemia. Elevation of
Current Barrett esophagus surveillance guidelines serum lactate is uncommon in patients with ischemic
recommend EGD with biopsies every 3–5 years if colitis and, if present, suggests the presence of bowel
the initial biopsies followed by surveillance biopsies infarction and necrosis.
obtained 1 year later do not reveal dysplasia. Bar- Flynn AD, Valentine JF. Update in the diagnosis and manage-
rett esophagus surveillance every 3 months is recom- ment of colonic ischemia. Curr Treat Options Gastroenterol.
mended for patients with high-grade dysplasia who 2016;14(1):128–139.
do not wish to undergo ablative or surgical therapy.
A surveillance interval of every 6–12 months is rec- 4. ANSWER: C. HIV infection
ommended for patients with low-grade dysplasia. A false-negative HCV antibody test result may
No subsequent surveillance and surveillance every occur in patients who have immunocompromised

t
ne
10 years are not acceptable for this patient, who may states such as HIV or renal failure or who have malig-
be at long-term risk of developing adenocarcinoma of nancies such as lymphoma. Clariication of a sus-
the esophagus. pected false-positive or false-negative HCV antibody

e.
Spechler SJ, Sharma P, Souza RF, et  al. American Gastro- test result can be made by performing an HCV RNA
enterological Association medical position statement on (viral load) test. A false-positive HCV antibody test
the management of Barrett’s esophagus. Gastroenterology.

in
result may occur in patients with a variety of auto-
2011;140:1084–1091. immune disorders, such as autoimmune hepatitis,

ic
lupus, and rheumatoid arthritis, possibly related to the
2. ANSWER: C. Rifaximin hypergammaglobulinemia.
Irritable bowel syndrome (IBS) is a functional
bowel disorder characterized by episodic abdomi-
ed Smith BD, Teshale E, Jewett A, et al. Performance of premar-
ket rapid hepatitis C virus antibody assays in 4 National
nal discomfort and altered bowel habits. Bloating, Human Immunodeiciency Virus Behavioral Surveillance
System sites. Clin Infect Dis. 2011;53(8):780–786.
sm
nausea, and fatigue may also be present. Visceral
hypersensitivity is thought to play a role. Treatment
options, including stool bulking agents such as psyl- 5. ANSWER: C. It increases mortality in patients with
lium, dietary modiication, and antispasmodic agents decompensated cirrhosis.
ok

such as hyoscyamine, may result in variable improve- Researchers in recent studies have noted increased
ment of symptoms in some patients. Mesalamine, mortality in patients with decompensated cirrhosis
though efective for treatment of inlammatory bowel who are treated with nonselective beta blockers. Non-
bo

disease (IBD), is not useful for management of IBS. selective beta blockers are indicated for both primary
A 2-week course of rifaximin, a poorly absorbed, and secondary prophylaxis of variceal bleeding but not
topically acting antibiotic, has been shown to result in the acute management of variceal bleeding. Use of
e

in signiicant reduction in irritable bowel syndrome– nonselective beta blockers decreases the risk of spon-
://

related symptoms such as bloating, loose stools, and taneous bacterial peritonitis and bleeding from portal
abdominal pain. hypertensive gastropathy. Nonselective beta blockers
tp

Pimentel M, Lembo A, Chey WD, et al. Rifaximin therapy for are not indicated for prevention of variceal formation
patients with irritable bowel syndrome without constipa- in early cirrhosis.
tion. N Engl J Med. 2011;364(1):22–32. Ge PS, Runyon BA. he changing role of beta-blocker therapy
ht

in patients with cirrhosis. J Hepatol. 2014;60(3):643–653.


3. ANSWER: A. Digoxin may predispose patients to
bowel ischemia. 6. ANSWER: D. Primary biliary cirrhosis
Ischemic colitis is a form of gut ischemia most often Secondary Sjögren syndrome may develop in as
afecting the descending and sigmoid colon. It results many as 40%–65% of patients with primary biliary
from low vascular low to a colonic segment rather cirrhosis (PBC). Typical symptoms include dry mouth
than from an embolic process or vascular thrombo- (xerostomia) and dry eyes (keratoconjunctivitis) and
sis. It is generally associated with low mortality and may precede PBC-related symptoms. Gastroesopha-
rarely causes hemodynamically signiicant bleeding. geal relux disease, small intestinal bacterial over-
Digoxin, a splanchnic vasoconstrictor, can predispose growth, and gallstone pancreatitis are not associated
to both mesenteric ischemia and ischemic colitis. CT with Sjögren syndrome. Ulcerative colitis is associated
imaging may show colonic thickening, but it cannot with several extraintestinal manifestations, such as
distinguish ischemic colitis from other inlammatory uveitis, but secondary Sjögren syndrome is not one of
or infectious processes. Angiography is of limited value them.
CHAPTER 7 Gastroenterology 155

Uddenfeldt P, Danielsson A, Forssell A, et  al. Features of 70%–90% of gastric polyps. Dysplastic tissue can
Sjögren’s syndrome in patients with primary biliary cirrho- be found in 5%–19% of hyperplastic polyps. Some
sis. J Intern Med. 1991;230(5):443–448. guidelines recommend resection of gastric hyperplas-
tic polyps larger than 0.5–1 cm. Adenomatous gastric
7. ANSWER: D. Hepatic adenoma polyps should be resected due to risk of malignancy,
Hepatic adenomas are a common focal hepatic and a surveillance endoscopy should be performed 1
lesion seen predominantly in young women with a year after resection followed by surveillance every 3–5
history of prolonged oral contraceptive use, and they years thereafter.
sometimes undergo malignant transformation. hey Evans JA, Chandrasekhara V, Chathadi KV, et  al. he role
are typically found incidentally on imaging. Gastroin- of endoscopy in the management of premalignant and
testinal tumors (GISTs) are generally benign tumors, malignant conditions of the stomach. Gastrointest Endosc.
but they are most often found in the stomach and 2015;82(1):1–8.
small intestine, not in the liver. he remaining benign
liver lesions mentioned have not been deinitively 10. ANSWER: E. Synovial luid shows an elevated leu-
linked to the use of oral contraceptives. Cavernous kocyte count.

t
ne
hemangiomas are the most common benign focal Postenteric reactive arthritis (formerly known as
hepatic lesions. Focal nodular hyperplasia (FNH), Reiter syndrome) typically develops 2–4 weeks after an
90% of which occur in women, are the second most acute diarrheal illness. It is an immune-mediated syno-

e.
common benign liver tumors. FNH lesions may be vitis, so the synovial luid shows an elevated leukocyte
responsive to estrogen, but the development of FNH count despite the absence of infection. Gonococcal

in
has not been conclusively linked to oral contraceptive arthritis is important to exclude in a young person
use. Nodular regenerative hyperplasia is uncommon presenting with acute monoarticular arthritis. Shigella

ic
and is characterized by development of multiple small sp. is the most commonly associated enteric organism
regenerative nodules. associated with this syndrome, although Salmonella
Algarni AA, Alshuhri AH, Alonazi MM, et al. Focal liver lesions
found incidentally. World J Hepatol. 2016;8(9):446–451.
ed sp., Campylobacter jejuni, Yersinia enterocolitica, and
even Clostridium diicile have been implicated. Ame-
bic intestinal infections are not associated with reac-
sm
8. ANSWER: B. Prolonged antibiotic treatment is tive arthritis. People who develop this syndrome have
required to eradicate the causative organism. a higher prevalence of HLA-B27 antigen, not HLA-
Whipple disease requires prolonged antibiotic DQ2, which is associated with celiac disease. his
therapy, usually for 12 months. Most cases occur syndrome is more common in men and classically is
ok

in middle-aged men of European ancestry. Extrain- associated with the triad of arthritis, conjunctivitis,
testinal symptoms are common and may include and urethritis, not esophagitis.
rheumatologic, cardiac, and central nervous system Morris D, Inman RD. Reactive arthritis: developments and
bo

involvement, for example. he causative organism, challenges in diagnosis and treatment. Curr Rheumatol Rep.
Tropheryma whipplei, a gram-positive bacillus related 2012;14(5):390–394.
to actinomycetes, is diicult to culture. It is believed
e

to be acquired from soil and animals, making this rare 11. ANSWER: D. Myasthenia gravis
://

disease more common in farmers. Esophagogastrodu- his patient exhibits classic features of myasthenia
odenoscopy (EGD) with small bowel biopsy, not colo- gravis, including muscle fatigue and ocular-bulbar
tp

noscopy, is the diagnostic test of choice. he diagnosis symptoms. Myasthenia gravis can cause oropharyngeal
can usually be made by identiication of characteristic dysphagia due to muscle weakness, which can be a risk
PAS staining of macrophages in the lamina propria of for aspiration. he globus sensation, which refers to a
ht

the small intestine or with electron microscopy. sensation of a lump or foreign body in the throat in
Marth T, Moos V, Müller C, et al. Tropheryma whipplei infec- the absence of dysphagia and odynophagia, is not a
tion and Whipple’s disease. Lancet Infect Dis. 2016;16(3): feature of myasthenia gravis. Gastroesophageal relux,
e13–e22. goiter, early hypopharyngeal cancer, and anxiety disor-
ders all can be associated with globus sensation, unlike
9. ANSWER: D. PPI-associated fundic gland polyps myasthenia gravis.
are not associated with an increased risk of cancer. Binks S, Vincent A, Palace J. Myasthesia gravis: a clinical-
Fundic gland polyps may develop in people who immunological update. J Neurol. 2016;263(4):826–834.
are on long-term PPI therapy, and in this setting,
they are not associated with an increased risk of malig- 12. ANSWER: B. he treatment of choice is prednisone
nancy. However, in patients with FAP, fundic gland with or without azathioprine.
polyps are associated with an increased risk of cancer. he patient’s elevated hepatic enzymes, female
he most common types of gastric polyps are fundic sex, elevated globulin levels (total protein, albumin),
gland polyps and hyperplastic polyps, representing lack of risk factors for chronic viral hepatitis, personal
156 C HA P T E R 7 Gastroenterology

history of hypothyroidism that could be autoim- 15. ANSWER: B. Gastroesophageal relux disease and
mune in nature, family history of autoimmune dis- constipation
orders, and high-titer anti–smooth muscle antibody Cystic ibrosis (CF) is an inherited disease charac-
test are strong clues suggesting autoimmune hepatitis. terized by a mutation in the cystic ibrosis transmem-
Autoimmune hepatitis usually presents in the fourth brane conductance regulator (CFTR). CFTR is found
decade of life and is more common in women than in all epithelia of the gastrointestinal tract, includ-
in men. Several autoantibodies may be present, such ing the pancreas and liver. Constipation and obsti-
as antinuclear antibody (ANA), antisoluble liver anti- pation rather than diarrhea are common in patients
gen (SLA), antiliver kidney microsomal type 1 (anti- with CF, and they are related to intestinal dysmotil-
LKM-1) antibodies, and antineutrophil cytoplasmic ity and decreased water secretion due to the CFTR
antibodies (ASCA), for example. Treatment consists defect. GERD is reported in 30% of adult patients
of a glucocorticoid (such as prednisone) with or with- with CF, and it occurs primarily due to inappropriate
out azathioprine. Undiagnosed and untreated auto- lower esophageal sphincter relaxation. Intestinal dys-
immune hepatitis may lead to the development of motility, use of acid-suppressing agents, and chronic
cirrhosis. Autoimmune hepatitis recurs in 20%–30% antibiotic therapy predispose patients with CF to

t
ne
of patients after liver transplant despite the immuno- develop small intestinal bacterial overgrowth (SIBO).
suppression regimen. Although SIBO occurs in 30%–55% of patients with
Liberal R, Vergani D, Mieli-Vergani G. Update on autoimmune CF, dysphagia is not a typical symptom of CF. More

e.
hepatitis. J Clin Transl Hepatol. 2015;3(1):42–52. than 85% of patients with CF have pancreatic insui-
ciency related to pancreatic ductal obstruction caused

in
13. ANSWER: B. here is a hypervirulent strain asso- by thick pancreatic luid secretions. Gallstones are not
ciated with lower clinical cure rates and increased typically associated with CF. Liver disease in patients

ic
recurrence rates. with CF can include asymptomatic elevation in liver
A hypervirulent strain of C. diicile (NAP1/ enzyme tests, hepatosplenomegaly, steatosis, and cir-
BI/027) that is associated with lower clinical cure rates
and increased recurrence rates has been described. he
ed rhosis.
Assis DN, Freedman SD. Gastrointestinal disorders in cystic
C. diicile toxin and PCR assay results may remain ibrosis. Clin Chest Med. 2016;37(1):109–118.
sm
positive after resolution of symptoms; hence, testing
for cure after symptoms resolution is not advised. 16. ANSWER: C. Eosinophilic esophagitis
here has been a signiicant increase in the incidence Eosinophilic esophagitis typically occurs in young
of C. diicile infection in patients with IBD. Hence, men with a history of atopic disorders such as eczema
ok

all patients with IBD who are hospitalized with a dis- and asthma. Endoscopic indings include multiple
ease lare should undergo testing. Repeat C. diicile thin rings with linear furrows. Eosinophilic esophagi-
testing after a negative test result is positive <5% of tis is a chronic inlammatory, immune-mediated con-
bo

the time, and furthermore, repeat testing increases the dition in which dense eosinophilic iniltrates are seen
likelihood of a false-positive result. Hand hygiene with on esophageal biopsies (>15/high-power ield).
soap and water is more efective than alcohol-based Esophageal lichen planus can cause dysphagia
e

hand sanitizers for eradicating C. diicile spores and is due to stricture formation, but the disease occurs
://

therefore recommended when caring for a patient with mostly in middle-aged women, and strictures tend
C. diicile infection. to be located in the proximal esophagus. A Schatzki
tp

Ofosu A. Clostridium diicile infection: a review of current and ring is solitary and located in the distal esophagus,
emerging therapies. Ann Gastroenterol. 2016;29(2):147–154. often in association with a hiatal hernia. Peptic stric-
tures generally occur in the distal esophagus and
ht

14. ANSWER: B. Barium esophagram cause focal luminal narrowing. In addition, signs of
Zenker diverticulum is an outpouching of the acid-induced injury such as relux esophagitis and
mucosa immediately above the upper esophageal ulceration are commonly seen. Plummer-Vinson
sphincter. he best test to perform to diagnose a syndrome is characterized by proximal esophageal
Zenker diverticulum is a barium esophagram. EGD webs causing dysphagia, atrophic glossitis, and
may not identify them, and there is a risk of inadver- iron-deiciency anemia.
tent perforation of the diverticulum. A CT scan may Zhang M, Li Y. Eosinophilic gastroenteritis: a state-of-the-art
show a large Zenker diverticulum. Laryngoscopy is review. J Gastroenterol Hepatol. 2017;32(1):64–72.
not an ideal method for identifying a Zenker diver-
ticulum. An esophageal motility study would not 17. ANSWER: D. Gallbladder cancer
diagnose a Zenker diverticulum, but it may provide Risk factors for gallbladder cancer include obe-
information regarding the underlying pathogenesis. sity, gallstones, “porcelain” gallbladder, and chronic
Law R, Katzka DA, Baron TH. Zenker’s diverticulum. Clin Salmonella typhi infection. Gallstones are present in
Gastroenterol Hepatol. 2014;12(11):1773–1782. 70%–90% of patients with gallbladder cancer. Large,
CHAPTER 7 Gastroenterology 157

solitary stones (>2.5 cm) are associated with the great- 20. ANSWER: A. Presence of melena
est risk. A “porcelain” gallbladder is characterized by he AIMS65 score is used to predict inpatient mor-
intramucosal calciication due to chronic inlamma- tality in patients with upper gastrointestinal bleeding.
tion and is associated with a 2%–3% incidence of gall- he score is easy to calculate and relies on data readily
bladder cancer. obtainable in the emergency department.
Kanthan R, Senger JL, Ahmed S, et al. Gallbladder cancer in he presence of melena is not a variable used to
the 21st century. J Oncol. 2015;2015:967472. calculate the score. A score of 1 point is given to each
of the following ive risk factors: Albumin <3.0 mg/
18. ANSWER: C. Herpes simplex virus (HSV)–associated dL, INR >1.5, Altered mental status, Systolic blood
ulcer pressure <90 mm Hg, and age >65. A patient with an
Herpes simplex virus is usually associated with upper gastrointestinal bleed who has an AIMS65 score
multiple small, shallow ulcers rather than a solitary, of 3, such as in this case, has an estimated inpatient
large ulcer. Certain antiretroviral medications such mortality rate of 9%, compared with a mortality rate
as didanosine (ddI) and zidovudine (AZT) can cause of 0.3% for those with a score of 0.
pill esophagitis. hese ulcers are typically solitary and

t
Saltzman JR, Tabak YP, Hyett BH, et  al. A simple risk score

ne
are often found in the proximal to middle esophagus. accurately predicts in-hospital mortality, length of stay,
CMV and idiopathic ulcers are the most common and cost in acute upper GI bleeding. Gastrointest Endosc.
causes of esophageal ulceration in patients with AIDS 2011;74(6):1215–1224.

e.
and are usually large, solitary, and well circumscribed.
MAC can occasionally cause esophageal ulceration in 21. ANSWER: B. Niacin deiciency

in
patients with AIDS. his patient has pellagra, a common manifestation
O’Rourke A. Infective oesophagitis: epidemiology, cause, diag- of niacin deiciency. It is uncommon in the Western

ic
nosis and treatment options. Curr Opin Otolaryngol Head world except as a complication of anorexia nervosa,
Neck Surg. 2015;23(6):459–463. alcoholism, and some malabsorptive disorders. he

19. ANSWER: E. Patients with inlammatory bowel


ed classic clinical features are diarrhea, pigmented der-
matitis (as described in this patient), and dementia.
disease and primary sclerosing cholangitis (PSC) Vitamin C deiciency, known as scurvy, causes perifol-
sm
should undergo surveillance colonoscopy every 1–2 licular hemorrhage, gingivitis, anemia, and joint pain.
years following the diagnosis of PSC. hiamine deiciency may cause a variety of symptoms,
Colorectal cancer is one of the most dreaded such as polyneuropathy (dry beriberi) and high-output
complications of inlammatory bowel disease (IBD), cardiac failure (wet beriberi), as well as ataxia, nys-
ok

accounting for 15% of IBD-related deaths. Patients tagmus, confabulation, memory loss, and ophthal-
with IBD and PSC have a higher risk of develop- moplegia (Wernicke-Korsakof syndrome). Vitamin A
ing colorectal cancer, so it is recommended that they deiciency can cause night blindness, corneal drying,
bo

undergo surveillance colonoscopy every 1–2 years and follicular hyperkeratosis. Vitamin K deiciency
from the time of diagnosis of PSC. causes easy bruising and bleeding.
Patients with ulcerative proctitis (rectal involve- Barsell A, Norton SA. Pellagra’s three Ds: dermatology, death
e

ment only) are not considered at increased risk of and Dracula. JAMA Dermatol. 2015;151(9):951.
://

developing colorectal cancer. Long-standing Crohn


colitis is associated with an increased risk of colorec- 22. ANSWER: D. An upper esophagogastroduodenos-
tp

tal cancer. However, Crohn disease limited to the copy should be performed next.
ileum is not associated with an increased risk of An upper gastrointestinal source of bleeding should
developing colon cancer. Patients with long-stand- be considered in patients presenting with brisk hema-
ht

ing ulcerative colitis, including left-sided disease as tochezia and hemodynamic instability. he elevated
well as pancolitis, are at increased risk of developing blood urea nitrogen/creatinine ratio also suggests
colorectal cancer. Screening for dysplasia and cancer an upper gastrointestinal source of bleeding in this
should begin after 8 years of disease in patients who case. For further evaluation, an esophagogastroduo-
have pancolitis and after 10–12 years for those with denoscopy (upper endoscopy) should be performed
left-sided colitis, and who are surgical candidates, urgently prior to beginning the bowel preparation
per current guidelines. Colectomy is recommended for a colonoscopy examination because it is impor-
if high-grade dysplasia is detected and conirmed by tant to exclude an upper gastrointestinal source of
a second pathologist. Such patients have a 43% risk the signiicant bleeding, such as a peptic ulcer. An
of synchronous malignancy. unprepped lexible sigmoidoscopy or colonoscopy
Farraye FA, Odze RD, Eaden J, et al. AGA technical review has low diagnostic yield and is not recommended for
on the diagnosis and management of colorectal neo- evaluation of lower GI bleeding. Fresh frozen plasma
plasia in inlammatory bowel disease. Gastroenterology. (FFP) would not be indicated in this case, because
2010;138(2):746–774. endoscopic hemostasis can be adequately achieved
158 C HA P T E R 7 Gastroenterology

with an INR <1.5. In patients with an INR of 1.5– but pericarditis has not been reported to be associ-
2.5, endoscopic hemostasis may be considered before ated with celiac sprue. Hyposplenism, not hyper-
or concomitant with giving reversal agents such as splenism, has been described in some patients with
FFP. Reversal agents should be administered prior to celiac sprue, though the mechanisms are unknown.
endoscopy in patients with an INR >2.5. Angiogra- Patients with hyposplenism should receive pneumo-
phy and other radiographic interventions should be coccal vaccination. he Lane-Hamilton syndrome,
considered in patients with ongoing bleeding and a characterized by the coexistence of celiac sprue
negative upper endoscopy who also fail to adequately and idiopathic pulmonary hemosiderosis (not idio-
respond to hemodynamic resuscitation. In this set- pathic pulmonary hypertension), has been reported
ting, angiography may localize the source of bleeding in some patients. Adherence to a gluten-free diet
in 25%–70% of examinations. has led to resolution of pulmonary symptoms in a
Strate LL, Gralnek IM. ACG clinical guideline: management few patients.
of patients with acute lower gastrointestinal bleeding. Am J Kelly CP, Bai JC, Liu E, et al. Advances in diagnosis and man-
Gastroenterol. 2016;111:459–474. agement of celiac disease. Gastroenterology. 2015;148(6):
1175–1186.

t
ne
23. ANSWER: C. Copper deiciency
Bariatric surgery is the most common cause of 25. ANSWER: B. Patients with ascites luid total pro-
acquired copper deiciency. Microcytic, hypochro- tein <1.1 g/dL and serum bilirubin >2.5 mg/dL

e.
mic anemia and neutropenia are typical hematologic should receive prophylactic antibiotics.
manifestations. Muscle weakness, skin depigmenta- Patients with cirrhosis who have low total protein

in
tion, abnormally formed hair, hepatosplenomegaly, levels in ascites luid and advanced liver disease are
and even ataxia and neuropathy, mimicking vitamin at increased risk of developing spontaneous bacterial

ic
B12 deiciency, have been described. peritonitis and should therefore receive prophylac-
Folate, iron, and vitamin B12 deiciencies can tic antibiotics. Covered stents are preferred to bare
occur in up to 40% of patients who have under-
gone Roux-en-Y gastric bypass surgery and who do
ed stents for TIPS procedures, owing to lower risk of
shunt occlusion and dysfunction. A combination of
not take multivitamin with mineral supplements. a nonselective beta blocker plus endoscopic variceal
sm
Lack of gastric acid impairs iron and vitamin B12 band ligation (not sclerotherapy) is the preferred
absorption. Bypassing the duodenum and proxi- method for secondary prophylaxis of variceal hem-
mal jejunum impairs folate and iron absorption as orrhage. he threshold for empiric antibiotic treat-
well. Folate and vitamin B12 deiciency cause mac- ment for suspected spontaneous bacterial peritonitis
ok

rocytic anemia. Although iron deiciency may cause is a polymorphonuclear count >250. he Model for
a microcytic anemia, muscle weakness and neu- End-Stage Liver Disease (MELD) score is based on
tropenia are not typical. Poor calcium absorption creatinine, INR, and total bilirubin (albumin is not
bo

can occur after gastric bypass, but it usually causes included).


secondary hyperparathyroidism and metabolic bone Pericleous M, Sarnowski A, Moore A, et al. he clinical man-
disease. agement of abdominal ascites, spontaneous bacterial peri-
e

Gletsu-Miller N, Broderius M, Frediani JK, et al. Incidence and tonitis and hepatorenal syndrome: a review of current
://

prevalence of copper deiciency following Roux-en-Y gastric guidelines and recommendations. Eur J Gastroenterol Hepatol.
bypass surgery. Int J Obes (Lond). 2012;36:328–335. 2016;28(3):e10–e18.
tp

24. ANSWER: E. Abnormal liver function test results 26. ANSWER: C. Rectal indomethacin
his patient has celiac disease, a small intestinal Meta-analyses of several randomized controlled
ht

disorder resulting from a gluten allergy. his disease studies have demonstrated that rectally administered
is characterized by the gastrointestinal symptoms indomethacin signiicantly reduces the incidence of
described. In addition, some patients may develop post-ERCP pancreatitis. In addition, recent data sug-
a dermatologic manifestation—dermatitis herpeti- gest that rectally administered indomethacin alone
formis—as described in this case. Celiac disease has may be more efective than no prophylaxis and pan-
been associated with Down syndrome, selective IgA creatic duct stent placement in preventing post-ERCP
deiciency, type 1 diabetes mellitus, thyroid disease, pancreatitis.
autoimmune liver disease, and idiopathic liver enzyme N-acetylcysteine has not been shown to be benei-
elevations. cial in preventing post-ERCP pancreatitis. Octreotide
Osteoporosis may develop from calcium and may reduce serum amylase elevation, but the results of
vitamin D malabsorption. Paget disease of the bone studies evaluating its efect on the incidence of post-
has not been directly associated with celiac disease. ERCP pancreatitis are mixed. Studies of allopurinol
Some studies suggest that patients with celiac sprue and pentoxifylline have not supported their use for the
may be at increased risk of ischemic heart disease, prevention of post-ERCP pancreatitis.
CHAPTER 7 Gastroenterology 159

hiruvengadam NR, Forde KA, Ma GK, et  al. Rectal indo- Stark A, Donahue TR, Reber HA, et al. Pancreatic cyst disease:
methacin reduces pancreatitis in high- and low-risk patients a review. JAMA. 2016;315(17):1882–1893.
undergoing endoscopic retrograde cholangiopancreatogra-
phy. Gastroenterology. 2016;151(2):288–297.e4. 29. ANSWER: B. MUTYH-associated polyposis
MUTYH-associated polyposis (MAP) is an autoso-
27. ANSWER: C. he treatment of choice for symp- mal recessive disorder associated with biallelic muta-
tomatic relief and healing of erosive esophagitis is tions of the MUTYH gene and is characterized by
an 8-week course of a proton pump inhibitor. multiple adenomatous polyps. It is currently the only
PPI therapy is associated with superior healing known autosomal recessive hereditary colon cancer
rates and lower relapse rates compared with H2 block- syndrome. MAP may be phenotypically indistinguish-
ers in patients with erosive esophagitis. he greatest able from attenuated familial polyposis syndrome
response to antirelux surgery is seen primarily in (AFAP) and, in a few cases, classic familial adenoma-
patients with typical symptoms of heartburn and/or tous polyposis syndrome (FAP).
regurgitation, who demonstrate good response to PPI he other syndromes mentioned are characterized
therapy, or those who have abnormal ambulatory pH by hamartomatous polyps as detailed below.

t
ne
monitoring studies with good symptom correlation. Peutz-Jeghers syndrome is an autosomal domi-
On the basis of current literature, the use of routine nant disorder due to a germline mutation of a serine
biopsies of the esophagus to diagnose GERD cannot threonine kinase (STK11 or LKB1) characterized by

e.
be recommended. he overall sensitivity of a barium multiple hamartomatous polyps, predominantly in
esophagram to diagnose GERD is low; hence, it is not the small intestine. Patients with Peutz-Jeghers syn-

in
recommended for diagnosis. Last, obesity is a risk fac- drome have distinctive mucocutaneous pigmentation
tor for GERD, and indeed modest weight loss is one (brown macules) not only on the lips and perioral/

ic
of the more efective lifestyle modiications that may buccal mucosa but also on the hands and feet in some
be beneicial for GERD management. cases.
Anderson WD 3rd, Strayer SM, Mull SR. Common questions
about the management of gastroesophageal relux. Am Fam
ed Juvenile polyposis syndrome is an autosomal domi-
nant disorder associated with germline mutations of
Physician. 2015;91(10):692–697. one of three genes (BMPR1A, SMAD4, and ENG).
sm
Patients with juvenile polyposis syndrome have mul-
28. ANSWER: D. Serous cystadenoma tiple hamartomatous polyps in the colon and have an
Serous cystadenomas are more common in women, increased risk of developing colorectal cancer.
usually occur in people aged 60 or older, predominate Cowden syndrome is an autosomal dominant
ok

in the pancreatic head, and are almost always benign. disorder associated with a germline mutation of the
hey are usually small lesions, calcify more than any PTEN gene. Patients with Cowden syndrome have
other pancreatic neoplasm, and may contain a classic multiple gastrointestinal hamartomatous polyps as
bo

central stellate scar. well as some characteristic dermatologic manifesta-


Mucinous cystic neoplasms are found almost tions, including oral ibromas, cutaneous verrucous
exclusively in women, tend to occur at a younger age, papules known as trichilemmomas, and punctate
e

predominate in the pancreatic tail, are solitary, and palmoplantar keratosis.


://

have a moderate malignant potential. Intraductal Cronkhite-Canada syndrome is a rare nonfamilial


papillary cystic neoplasms (IPMNs) occur with simi- disorder of unknown etiology with features includ-
tp

lar frequency in men and women and usually arise ing hamartomatous gastrointestinal polyposis. Other
in the head of the pancreas. hey appear to be more features of this syndrome include alopecia, onycho-
common in cigarette smokers, patients with famil- dystrophy, cutaneous hyperpigmentation, diarrhea,
ht

ial adenomatous polyposis (FAP) and Peutz-Jegher weight loss, and abdominal pain.
syndromes, and in patients with familial pancre- Jasperson K, Burt RW. he genetics of colorectal cancer. Surg
atic carcinoma. Main-branch IPMNs have a higher Oncol Clin N Am. 2015;24(4):683–703.
malignant potential than side-branch IPMNs. Solid
pseudopapillary neoplasms predominantly afect 30. ANSWER: B. Normal total protein (>2.5 g/dL) in
women and occur at a young age (median age of ascites
30–38 years). hey contain solid and cystic com- Patients with CHF-associated liver disease gener-
ponents and may be diicult to diferentiate from ally have preserved hepatic synthetic function and thus
cystic pancreatic endocrine neoplasms on ine-needle higher protein levels (usually >2.5 g/dL) than patients
aspiration (FNA) cytologic analysis. Although most with primary liver disease. In addition, patients with
solid pseudopapillary neoplasms are benign, some CHF-associated liver disease rarely have evidence of
may exhibit vascular and perineural invasion, and portosystemic shunts such as esophageal varices and
these variants are thought to have moderate to high lack clinically overt jaundice, though up to 70% may
malignant potential. have a mild increase in unconjugated bilirubin. Just
160 C HA P T E R 7 Gastroenterology

as in patients with primary liver disease, patients with extrahepatic manifestations, may be present. Undiag-
CHF-associated liver disease who have ascites have a nosed Wilson disease sometimes presents with acute
SAAG >1.1 g/dL as a result of portal hypertension. liver failure, as in this case.
Hepatojugular relux is generally present and can be Nonalcoholic steatohepatitis (NASH) is now
useful in distinguishing hepatic congestion from pri- increasingly recognized as a major cause of crypto-
mary intrahepatic liver disease. genic cirrhosis. Associated medical conditions include
Weisberg IS, Jacobson IM. Cardiovascular diseases and the hypertension, dyslipidemia, obesity, insulin resistance,
liver. Clin Liver Dis. 2011;15(1):1–20. or frank diabetes. It is not associated with acute liver
failure.
31. ANSWER: C. Breastfeeding is not associated with Hemochromatosis is the most common inher-
increased risk of transmission. ited disorder among Caucasians of Northern Euro-
he mode of delivery (vaginal versus cesarean sec- pean descent, with a prevalence of 1 in 200 people.
tion), breastfeeding, and viral genotype are not asso- Untreated patients may develop cirrhosis, cardiac
ciated with increased risk of vertical transmission of dysfunction, or diabetes, but acute liver failure has
hepatitis C virus (HCV). Vertical transmission of not been reported. Early detection and treatment

t
ne
HCV is estimated to be 20% in women coinfected with phlebotomy may result in a normal lifespan for
with hepatitis C and HIV, compared with a 4%–7% afected patients.
risk in patients with HCV monoinfection. Vertical Alpha-1 antitrypsin deiciency occurs in approxi-

e.
transmission occurs almost exclusively among women mately 1 in 2000 people. Over 75 diferent protease
who have detectable HCV viremia. inhibitor (Pi) alleles have been described. he PiSZ

in
Checa Cabot CA, Stoszek SK, Quarleri J, et  al. Mother-to- and PiZZ phenotypes can be associated with develop-
child transmission of hepatitis C virus (HCV) among ment of cirrhosis, but acute liver failure has not been
HIV/HCV-coinfected women. J Pediatric Infect Dis Soc.

ic
described.
2013;2(2):126–135. Glycogenic hepatopathy occurs due to glycogen

32. ANSWER: D. Treatment with an H2 receptor


ed deposition in the liver in patients with poorly con-
trolled type 1 diabetes mellitus. It causes hepatomeg-
blocker may provide symptomatic relief. aly, abdominal pain, and elevated liver enzymes, but it
sm
In adults, Ménétrier disease is four times more does not typically cause acute liver failure.
common in men than in women. Symptoms of post- Rodriguez-Castro KI, Hevia-Urrutia FJ, Sturniolo GC. Wil-
prandial epigastric pain and weight loss are common. son’s disease: a review of what we have learned. World J
Patients may develop anemia due to gastrointesti- Hepatol. 2015;7(29):2859–2870.
ok

nal occult blood loss. he etiology of the disorder


is unknown. Patients often experience relief of epi- 34. ANSWER: C. Vitamin E
gastric pain with the use of H2 receptor blockers. In Vitamin E deiciency, if severe, may result in abnor-
bo

adults, Ménétrier disease typically is associated with malities afecting the posterior columns, leading to
chronic symptoms, whereas in children, it is usually ataxia, loss of proprioception, and arelexia. However,
a self-limited disease. Hypoalbuminemia, peripheral severe vitamin E deiciency occurs infrequently. Vita-
e

edema, and ascites may occur as a result of protein min A deiciency may lead to diminished nocturnal
://

loss from the hypersecretory gastric mucosa. Esopha- visual acuity. Vitamin D deiciency may result in met-
gogastroduodenoscopy (EGD) will show thickened abolic bone disease. Vitamin K deiciency may result
tp

gastric folds. A full-thickness biopsy of the stomach is in prolongation of prothrombin time and increased
usually required for diagnosis. susceptibility to bleeding. Although neurologic symp-
toms can be associated with vitamin B12 deiciency,
ht

Lambrecht NW. Ménétrier’s disease of the stomach: a clinical


challenge. Curr Gastroenterol Rep. 2011;13(6):513–517. vitamin B12 is not a fat-soluble vitamin.
Pfeifer RF. Neurologic manifestations of malabsorption
33. ANSWER: D. Wilson disease syndrome. Handb Clin Neurol. 2014;120:621–632.
Wilson disease presents on rare occasions with
acute liver failure, which is uniformly fatal without 35. ANSWER: A. Refractory ascites in a patient with
liver transplant. Wilson disease is a rare disorder of right heart failure
copper transport with a prevalence of 1 in 30,000 TIPS increases right-sided heart pressure due to
people. It typically presents in the second to fourth the creation of a portosystemic shunt and is therefore
decades of life. Early in the disease, hepatic steatosis contraindicated in patients with right heart failure.
may be present, and chronic liver disease is common if Acceptable indications for TIPS include management
it goes unrecognized and untreated. Neurologic symp- of acute refractory variceal bleeding, hemorrhage
toms such as tremors and choreic movements, as well from inaccessible gastric or intestinal varices, bleeding
as psychiatric problems, hemolytic anemia, and other from portal hypertensive gastropathy, recurrent severe
CHAPTER 7 Gastroenterology 161

variceal bleeding after endoscopic therapy, refractory although this patient does not have any clearly predis-
hepatic hydrothorax, Budd-Chiari syndrome, and posing conditions (such as diabetes) or nausea, and a
some other venoocclusive disorders. gastric emptying study can be performed for further
Ascha M, Abuqayyas S, Hanouneh I, et al. Predictors of mortal- evaluation. here is currently no evidence to support
ity after transjugular portosystemic shunt. World J Hepatol. a low-fat diet or probiotics in the treatment of func-
2016;8(11):520–529. tional dyspepsia.
Lu Y, Chen M, Huang Z, Tang C. Antidepressants in the
36. ANSWER: C. Positive anti–Scl-70 antibody treatment of functional dyspepsia: a systematic review and
Systemic sclerosis (scleroderma) is characterized meta-analysis. PLoS One. 2016;11(6):e0157798.
by thickened skin and internal organ involvement,
including myocardial ibrosis (resulting in cardiomy- 38. ANSWER: E. Testing for H. pylori infection
opathy) and esophageal involvement. Patients often he etiology of duodenal ulcers is most com-
have heartburn and later develop dysphagia due to monly related to aspirin, NSAIDs, and H. pylori.
esophageal hypomotility and GERD, and they fre- Even though he has a history of recent NSAID use,
quently develop relux esophagitis and peptic stric- he should still be evaluated for H. pylori infection

t
ne
tures. Esophageal manometry shows low-amplitude as a contributing factor. Patients with uncomplicated
contractions in the distal esophagus and incompe- duodenal ulcers generally do not need follow-up
tence of the lower esophageal sphincter (LES). he EGD, owing to the low likelihood of malignancy.

e.
anti–Scl-70 antibody is often positive in systemic If a patient requires aspirin for a cardiac indication,
sclerosis. interruption of aspirin therapy my increase the risk

in
Idiopathic achalasia, pseudoachalasia, and Chagas of adverse cardiac outcomes. However, if other non-
disease cause an achalasia-like picture, including distal critical medications, such as NSAIDs in this case, can

ic
“bird beak” narrowing seen on a barium esophagram. be avoided, this is advisable. Sucralfate can help ulcer
Achalasia does not typically have extraesophageal healing, but it is not generally recommended in addi-
manifestations, and characteristic manometric ind-
ings are poor to absent esophageal peristalsis, incom-
ed tion to PPIs. Because the recent use of NSAIDs was
the likely cause of his bleeding and can be stopped,
plete relaxation of the LES, and elevated LES pressure. he most likely does not need to be on lifelong proton
sm
Older age (especially over age 60), weight loss, and a pump inhibitor (PPI) therapy.
relatively short onset of symptoms (<6 months) raise Satoh K, Yoshino J, Akamatsu T, et al. Evidence-based clinical
concern for pseudoachalasia due to malignancy at the practice guidelines for peptic ulcer disease 2015. J Gastroen-
esophagogastric junction. Paraneoplastic manifesta- terol. 2016;51(3):177–194.
ok

tions of extraintestinal tumors such as pancreatic,


lung, and lymphoma may result in symptoms of dys- 39. ANSWER: A. No treatment at this time
phagia. Chagas disease is caused by infection with Try- his patient is a chronic carrier of hepatitis B who
bo

panosoma cruzi and occurs most commonly in people appears to have a precore mutant strain and low lev-
of South American or Central American origin. he els of viremia. Antiviral therapy is not indicated for
chronic form of Chagas disease can cause both car- HBeAg-negative patients with normal liver enzymes
e

diac and gastrointestinal disease. However, the cardiac and hepatitis B viral load <2000 IU/mL, as in this
://

disease is typically not ischemic in nature, and the case. his patient appears to have inactive disease and
esophageal picture is similar to achalasia, and testing no evidence of chronic liver disease.
tp

for Chagas disease is initially done with antibodies for All newborns born to mothers who are carriers
T. cruzi. should receive passive-active immunization consisting
of HBIG and hepatitis B vaccination, a strategy that
ht

Gyger G, Baron M. Systemic sclerosis: gastrointestinal disease


and its management. Rheum Dis Clin North Am. 2015; has high protective eicacy (95%). Tenofovir is the
41(3):459–473. drug of choice for pregnant patients requiring antivi-
ral therapy. Pegylated interferon is not recommended
37. ANSWER: D. Amitriptyline during pregnancy. Lamivudine is associated with rapid
his patient appears to have functional dyspep- virologic resistance and is considered a pregnancy class
sia. In a patient with no improvement after 8 weeks C drug (not recommended during pregnancy). Cesar-
of PPI therapy and no evidence of H. pylori disease, ean delivery does not reduce the risk of vertical trans-
treatment with a tricyclic antidepressant should be mission of hepatitis B. Mothers should be encouraged
considered. Low-dose amitriptyline or desipramine to follow up after childbirth to discuss treatment for
10–25 mg at night is most commonly used. An H2 hepatitis B.
blocker is less likely to be helpful if a PPI was inefec- Terrault NA, Bzo Wej NH, Chang KM, et al. AASLD guide-
tive. Promotility agents such as metoclopramide can lines for the treatment of chronic hepatitis B. Hepatology.
be considered if there is a concern for gastroparesis, 2016;63(1):261–283.
162 C HA P T E R 7 Gastroenterology

40. ANSWER: D. Increase prednisone to 20 mg/day a fever or evidence of pancreatic necrosis based on
and add azathioprine. CT imaging. Pancreatic debridement is indicated for
Mild to moderate ulcerative colitis is often initially management of infected necrosis. Interventional radi-
treated with 5-aminosalicylates (5-ASA) for induction ology drainage of luid collections might be beneicial
and then maintenance of remission. Short courses if there is a dominant collection that is causing gastric
of steroids are sometimes needed to induce remis- outlet or intestinal obstruction or if there is concern
sion or manage lares, but they are not recommended for a superinfected luid collection, but neither of
for maintenance therapy, owing to the side efects. these is present at this time.
Immunomodulator therapy such as 6-mercaptopru- Lodewijkx PJ, Besselink MG, Witteman BJ, et al. Nutrition in
ine or azathioprine, or an anti–tumor necrosis factor acute pancreatitis: a critical review. Expert Rev Gastroenterol
(TNF) biologic therapy such as inliximab, should be Hepatol. 2016;10(5):571–580.
considered for patients whose disease is not efectively
controlled with 5-ASA therapy. Switching to another 43. ANSWER: A. Prednisone
5-ASA agent at this time is unlikely to improve her Autoimmune pancreatitis is characterized by
symptoms. A slow-release form of budesonide has elevated serum IgG4 levels, characteristic imag-

t
ne
been approved for ulcerative colitis, but it is used simi- ing (most commonly a difusely enlarged pancreas,
larly to prednisone in this case to attempt to induce although focal lesions and pancreatic duct strictures
remission and not as a maintenance or steroid-sparing can also occur), and diagnostic histology (which

e.
agent. Antibiotics have not been shown to be efective often includes a lymphoplasmacytic iniltrate with
in the treatment of ulcerative colitis. IgG4-positive cells). IgG4-related disease can also

in
Chaparro M, Gisbert JP. Maintenance therapy option for ulcer- involve the biliary tract and salivary glands and may
ative colitis. Expert Opin Pharmacother. 2016;17(10): cause lung nodules, autoimmune thyroiditis, and
1339–1349.

ic
interstitial nephritis. Initial treatment of pancreatic
and extrapancreatic manifestations is typically gluco-
41. ANSWER: B. Supportive care
his patient has alcoholic hepatitis based on her
ed corticoids (such as prednisone). Alcohol cessation or
cholecystectomy would be helpful if she had a history
history and consistent laboratory indings and should of signiicant alcohol use or gallstones, respectively.
sm
be advised to abstain from alcohol use. Her paracente- Pancreaticoduodenectomy (Whipple procedure)
sis is not suggestive of spontaneous bacterial peritoni- may be performed if a patient has a pancreatic head
tis, so she should not receive cefotaxime at this time. lesion such as a mass or stricture that is causing acute
Her Maddrey discriminant function is less than 32 pancreatitis. Fibrate therapy can be used to treat
ok

based on her total bilirubin and INR, so she should patients with hypertriglyceridemia to reduce the risk
not receive prednisolone or pentoxifylline this time. of recurrent acute pancreatitis.
Alcoholic hepatitis can cause portal hypertension, but Madhani K, Farrell JJ. Autoimmune pancreatitis: an update on
bo

this will often reverse with cessation of alcohol. She diagnosis and management. Gastroenterol Clin North Am.
does not have clear evidence of cirrhosis at this time 2016;45(1):29–43.
and has not had an upper endoscopy to evaluate for
e

varices. Nadolol is not routinely started in the acute 44. ANSWER: D. Urinary 5-hydroxyindoleacetic acid
://

setting of alcoholic hepatitis. his patient most likely has carcinoid syndrome
Singal AK, Kodali S, Vucovich LA, et al. Diagnosis and treat- on the basis of secretory diarrhea, lushing, and evi-
tp

ment of alcoholic hepatitis: a systematic review. Alcohol Clin dence of heart failure with right-sided valvular lesions.
Exp Res. 2016;40(7):1390–1402. he most useful initial test for carcinoid syndrome
is a 24-hour urine for 5-hydroxyindoleacetic acid
ht

42. ANSWER: B. Enteral nutrition with a nasojejunal (5-HIAA), although it is more useful for midgut car-
tube cinoid tumors than for foregut and hindgut tumors.
Acute luid collections are an early complication of Vasoactive intestinal peptide is elevated in VIPomas,
acute pancreatitis. hey generally do not require any which can also cause secretory diarrhea and lushing
treatment. he patient has not had any nutrition in but are not associated with right-sided valvular lesions.
5 days and is not expected to be able to tolerate oral Calcitonin is elevated in medullary cancer of the thy-
intake in the near future; thus, enteral nutrition with a roid, and gastrin is elevated in gastrinomas, both of
nasojejunal feeding tube is recommended at this time. which can cause a secretory diarrhea. A fat pad biopsy
Enteral nutrition in acute pancreatitis has been shown with Congo red staining is performed to diagnose
to decrease infectious complications and disease sever- amyloid, which could cause diarrhea and heart fail-
ity compared with parenteral nutrition. Antibiotics ure with a low-amplitude QRS on an electrocardio-
may be indicated if there is concern for infected pan- gram but would not be expected to cause the tricuspid
creatic luid collections, but this patient does not have regurgitation and lushing.
CHAPTER 7 Gastroenterology 163

Sarshekeh AM, Halperin DM, Dasari A. Update on manage- in patients with diabetes, but the normal tissue trans-
ment of midgut neuroendocrine tumors. Int J Endocr Oncol. glutaminase IgA antibody (and serum IgA level), as
2016;3(2):175–189. well as the normal duodenal biopsies, makes this diag-
nosis highly unlikely. Irritable bowel syndrome would
45. ANSWER: C. Ceftriaxone not explain the vitamin B12 deiciency, anemia, and
his patient has upper gastrointestinal bleeding nocturnal stools.
concerning for variceal bleeding in light of the his- Quigley EM. Small intestinal bacterial overgrowth: what it is
tory of cirrhosis with evidence of portal hypertension. and what it is not. Curr Opin Gastroenterol. 2014;30(2):
Octreotide is the preferred pharmacologic treatment 141–146.
in the United States. Vasopressin can also be used in
this setting, but it has more side efects and would not 48. ANSWER: B. Low-oxalate diet and continue the
be administered concurrently with octreotide. Short- calcium supplementation
term (maximum 7 days) spontaneous bacterial perito- Patients with small intestinal Crohn disease, espe-
nitis (SBP) antibiotic prophylaxis should be given to cially those with small bowel resections, are predis-
any patient with cirrhosis and gastrointestinal hemor- posed to nephrolithiasis, particularly calcium oxalate

t
ne
rhage, especially those who have ascites. Intravenous stones. As a result of bile salt malabsorption, fat binds
ceftriaxone or ciproloxacin is the preferred initial to calcium in the intestines, leaving oxalate free to be
antibiotic prophylaxis. Penicillin is not recommended absorbed and deposited in the kidney, where it can

e.
for SBP prophylaxis. Intravenous albumin is not gen- form into stones. Spinach is a high–oxalate-contain-
erally given in the setting of gastrointestinal or variceal ing food, and eating large amounts of this may have

in
bleeding. N-acetylcysteine is used in acute liver failure contributed to stone formation. In addition to main-
due to acetaminophen toxicity but not for gastrointes- taining adequate hydration, the treatment for calcium

ic
tinal or variceal bleeding. oxalate stones is generally to eat a low-oxalate diet
Garcia-Tsao G. Current management of the complications of and ensure adequate calcium intake. Calcium intake
cirrhosis and portal hypertension: variceal bleeding, asci-
tes and spontaneous bacterial peritonitis. Dig Dis. 2016;
ed should not be reduced (unless it is excessive), because
a decrease in free calcium in the intestines can lead to
34(4)382–386. increased absorption of oxalate due to decreased bind-
sm
ing of oxalate by calcium. Potassium citrate is some-
46. ANSWER: C. Wireless capsule endoscopy times used to treat calcium stones that are thought to
his patient has obscure, occult gastrointestinal form due to hypocitraturia, which is not thought to be
bleeding, given the negative upper endoscopy, colonos- a common mechanism for nephrolithiasis formation
ok

copy, and small bowel imaging with CT enterography. in Crohn disease.


A small bowel source, such as angiodysplasia, is the most Worcester EM. Stones from bowel disease. Endocrinol Metab
likely cause. he next test would therefore be a wireless Clin North Am. 2002;31(4):979–999.
bo

capsule endoscopy. CT angiography and a tagged red


blood cell scan are not sensitive for detection of occult 49. ANSWER: E. Endoscopic retrograde cholangio-
gastrointestinal bleeding. Double-balloon and push pancreatography
e

enteroscopy are generally performed for further diag- his patient has acute pancreatitis most likely due
://

nosis and treatment of a small bowel bleeding source to gallstones on the basis of elevated liver enzymes as
identiied on capsule endoscopy or another study. well as the gallstones and dilated common bile duct
tp

Tanabe S. Diagnosis of obscure gastrointestinal bleeding. Clin seen on the ultrasound. he ultrasound did not iden-
Endosc. 2016;49(6):539–541. tify gallstones in the common bile duct, but it has
relatively poor sensitivity for detecting choledocho-
ht

47. ANSWER: D. Small intestinal bacterial overgrowth lithiasis. he persistent pain, elevated bilirubin, and
Small intestinal bacterial overgrowth (SIBO) typi- fever suggest that this patient has cholangitis and
cally causes loose stools and bloating and can also result persistent biliary obstruction. he most appropriate
in vitamin B12 deiciency (bacteria consume vitamin treatment is endoscopic retrograde cholangiopancrea-
B12) and elevated serum folate (bacteria synthesize tography (ERCP), which will provide both diagnostic
folate). Chronic pancreatitis and diabetes can predis- and therapeutic beneit. ERCP should be performed
pose to SIBO due to stasis. Steatorrhea due to exo- within 24 hours of admission for patients with gall-
crine pancreatic insuiciency is possible but typically stone pancreatitis and cholangitis.
improves with increased enzyme supplementation and Abdominal CT scan and magnetic resonance chol-
is not associated with increased folate levels. Crohn angiopancreatography (MRCP) would show acute
disease can result in similar symptoms, but colonos- pancreatitis (but may still be too early to show pan-
copy to the terminal ileum and abdominal imaging are creatic necrosis) and may detect choledocholithiasis
not suggestive of this. Celiac disease is more common but ofer no therapy. An elective cholecystectomy
164 C HA P T E R 7 Gastroenterology

should be considered in this patient who has a his- typical of IBS. his patient likely has constipation-
tory of gallstone-induced complications. Ursodiol predominant IBS. Colonic inertia (or slow-transit
can dissolve small gallbladder stones, but it is not constipation) and dyssynergic defecation (or pelvic
a suitable therapeutic option for treating choledo- loor dysfunction) are not typically associated with the
cholithiasis. abdominal pain symptoms and are typically associ-
da Costa DW, Schepers NJ, Römkens TE, et  al. Endoscopic ated with retention of radiopaque markers in the right
sphincterotomy and cholecystectomy in acute biliary side of the colon and rectum, respectively. Anorectal
pancreatitis. Surgeon. 2016;14(2):99–108. manometry can be performed to further diferenti-
ate between these two entities. Multiple sclerosis can
50. ANSWER: C. Constipation-predominant irritable cause chronic constipation, but this patient does not
bowel syndrome appear to have any other symptoms of this. Chronic
his patient meets the Rome IV criteria for irritable intestinal pseudoobstruction typically causes obstruc-
bowel syndrome (IBS), which are recurrent abdominal tive symptoms (nausea, vomiting) and is associated
pain, on average, at least 1 day per week in the last 3 with underlying neuropathic or myopathic diseases,
months and associated with two or more of the follow- which she does not appear to have.

t
ne
ing: (1) defecation, (2) change in stool frequency, and/ Siah KT, Wong RK, Whitehead WE. Chronic constipation and
or (3) change in stool form. Symptom onset should be constipation-predominant IBS: separate and distinct disor-
at least 6 months prior to the diagnosis of IBS. Alarm ders or a spectrum of disease? Gastroenterol Hepatol (N Y).

e.
features such as weight loss and rectal bleeding are not 2016;12(3):171–178.

in
ic
ed
sm
ok
e bo
://
tp
ht
8
Cardiovascular Disease
ANJU NOHRIA

1. A 32-year-old woman presents for evaluation of pal- 2. A 42-year-old corporate executive presents to the

t
ne
pitations. She notes the intermittent sensation of her emergency department with 2 hours of unrelenting
heart “lip-lopping” in her chest for the past month. chest pain. He has a salient medical history of tobacco
Review of systems is notable for fatigue and subjec- use and dyslipidemia. he pain is severe and exacer-

e.
tively decreased energy. On physical examination, she bated by movement. On physical examination, the
is well appearing. Her jugular venous pressure is 5 patient appears uncomfortable. he cardiopulmonary

in
cm H2O. She has a parasternal lift and a grade 2 of 6 examination is normal. A bedside echocardiogram
midpeaking systolic murmur at the left upper sternal reveals normal left ventricular function, a trace peri-

ic
border. he second heart sound is widely split without cardial efusion, and no wall motion abnormality. An
respiratory variation, and the pulmonic component of electrocardiogram is shown in Fig. 8.1.
the second heart sound is prominent. he extremities
are warm without edema. An electrocardiogram dem-
ed What is the most appropriate next step in manage-
ment?
onstrates a rightward axis, narrow QRS complex, and A. Aspirin and thrombolysis
sm
incomplete right bundle branch block. B. Aspirin, heparin, and clopidogrel
What is the most likely diagnosis? C. Prednisone
A. Wolf-Parkinson-White syndrome D. Ibuprofen
B. Arrhythmogenic right ventricular cardiomyopathy E. Ativan and nitroglycerin
ok

C. Atrial septal defect


D. Mitral stenosis 3. A 52-year-old man with hypertension, dyslipidemia,
E. Hyperthyroidism and a smoking history presents with 6 hours of severe,
e bo
://

I aVR V1 V4
tp
ht

II aVL V2 V5

III V3 V6
aVF

V1

• Fig. 8.1 Electrocardiogram for patient in Question 2.


165
166 C HA P T E R 8 Cardiovascular Disease

unrelenting chest burning. On examination, his blood D. hyroid storm


pressure is 88/66 mm Hg. His jugular venous pres- E. Mitral stenosis
sure is 14 cm H2O and increases with inspiration. His
lungs are clear. Cardiac auscultation reveals normal 5. A 64-year-old woman presents with increasing dys-
irst and second heart sounds without murmur. An pnea and fatigue. She has a medical history notable
electrocardiogram is shown in Fig. 8.2. for small cell lung cancer metastatic to the liver and
In addition to administering aspirin and activating bone and is currently undergoing chemotherapy. Her
the cardiac catheterization laboratory, the most appro- symptoms have been progressive over the past week;
priate management is: initially, she was dyspneic climbing stairs, and now
A. Infusion of normal saline she is dyspneic walking across the room. On exami-
B. Nitroglycerin infusion nation, her heart rate is 128 beats per minute with a
C. Furosemide bolus regular rhythm. he intensity and amplitude of the
D. Phenylephrine radial pulse are variable with respiration. he patient’s
E. Esmolol infusion blood pressure is 88/62 mm Hg. Her jugular venous
pressure is 15 cm H2O. Her heart sounds are quiet

t
ne
4. A 36-year-old woman who is 22 weeks pregnant pres- without murmur or gallop. Her lungs are clear. Her
ents with dyspnea and palpitations. Her dyspnea began strength is 5 of 5 in both upper and lower extremi-
at 16 weeks of pregnancy but has progressed to the ties, and an electrocardiogram reveals sinus tachycar-

e.
point where she is dyspneic at rest. She sleeps upright dia with low voltage in the limb leads and precordial
in a chair. On examination, her heart rate is 108 beats leads. A chest radiograph reveals a mass in the left

in
per minute with an irregularly irregular rhythm. Her upper lobe, which is increased in size compared with
blood pressure is 90/64 mm Hg. he irst heart sound 2 weeks prior.

ic
is louder than the second heart sound at the base. here What is the most appropriate next diagnostic test?
is a high-pitched, snapping, discrete, early diastolic A. Electromyography
sound that follows the second heart sound. here is a
diastolic rumbling murmur heard over the apex with
ed B. Echocardiography
C. CT pulmonary angiography
the patient positioned in the left lateral decubitus posi- D. Blood cultures
sm
tion. Pulmonary auscultation reveals bibasilar wet rales. E. Whole-body FDG-PET/CT scan
he patient’s jugular venous pressure is 16 cm H2O.
What is the most likely diagnosis? 6. A 67-year-old man presents for preoperative evalu-
A. Pregnancy-induced cardiomyopathy ation. He has a history of progressive osteoarthri-
ok

B. Pulmonary embolism tis of the right knee, which has been refractory to
C. Dyspnea secondary to normal progression of preg- medical treatment, and total knee replacement is
nancy planned. he remainder of the patient’s medical
e bo
://

I aVR V1 V4
tp
ht

II aVL V2 V5

III aVF V3 V6

V1

• Fig. 8.2 Electrocardiogram for patient in Question 3.


CHAPTER 8 Cardiovascular Disease 167

I aVR V1 V4

II aVL V2 V5

t
III aVF V3 V6

ne
e.
V1

in
• Fig. 8.3 Electrocardiogram for patient in Question 7.

ic
history includes type 2 diabetes, for which he is
ed terminating. A physical examination reveals a thin,
maintained on insulin therapy; diabetic nephropa- tall young man who is not distressed. he palate is
sm
thy with baseline creatinine of 2.5 mg/dL; hyper- slightly high arched. he arm span is normal. he
tension; dyslipidemia; and ST segment elevation irst and second heart sounds are normal; there is a
myocardial infarction 10 years prior, which had grade 1 of 6 systolic murmur at the left upper sternal
been treated successfully with thrombolytics. Knee border that diminishes with handgrip. he remainder
ok

pain limits his ability to ambulate, and he walks of the examination is normal. he electrocardiogram
with a limp. He has no angina or symptoms of is shown in Fig. 8.3.
heart failure. His medications include aspirin, meto- What is the best next step?
bo

prolol, simvastatin, NPH insulin, glyburide, and A. Electrophysiologic study and mapping
lisinopril. A physical examination includes a heart B. Signal-averaged electrocardiogram
rate of 88 beats per minute and blood pressure of C. Exercise stress test
e

126/86 mm Hg and is otherwise normal. D. Holter monitor


://

What is the most appropriate recommendation? E. CT coronary angiogram


A. Pharmacologic stress test prior to total knee
tp

replacement 8. A 52-year-old woman is referred to you by her den-


B. Increase dosage of metoprolol for a goal heart rate tist. She presented with multiple dental caries and is in
of 55–65 beats per minute, then proceed with need of several illings. Her medical history is notable
ht

total knee replacement. for hypertension and mitral valve prolapse. Other than
C. Echocardiogram prior to total knee replacement tooth pain, she feels well. On examination, there is a
D. Recommend against total knee replacement due to midsystolic click and late systolic murmur heard over
medical comorbidities. the apex.
E. Coronary angiography prior to total knee What is the most appropriate next step?
replacement A. Clindamycin
B. Amoxicillin
7. A 22-year-old collegiate cross-country runner pres- C. Repeat echocardiogram
ents with palpitations. Four times over the past D. Proceed with planned dental procedure.
month, he has noted the abrupt onset of a racing E. Blood cultures
heartbeat and a sensation of pounding in the neck.
hree of the episodes terminated after 1 minute, but 9. A 68-year-old man presents for evaluation of chest
the most recent episode lasted for 15 minutes before pain. He has a history of hypertension, diabetes,
168 C HA P T E R 8 Cardiovascular Disease

dyslipidemia, and cigarette smoking. Within the past An electrocardiogram demonstrates atrial lutter with
24 hours, he has had two severe episodes of subster- variable A-V conduction.
nal chest pain at rest. On examination, a fourth heart In addition to anticoagulation, what is the next step
sound is noted without murmurs. he extremities are in diagnosis and management?
warm without edema. he jugular venous pressure is 5 A. TEE and DC cardioversion
cm H2O. An electrocardiogram reveals sinus rhythm B. Beta blockade
with T-wave inversions and 1 mm of ST segment C. Transthoracic echocardiogram
depression in leads V5 and V6. he patient is pain- D. Exercise treadmill test
free and comfortable at the time of the evaluation. E. hyroid function testing and nocturnal polysom-
Troponin T measurement is 0.3 ng/mL (normal range, nogram
0–0.1 ng/mL).
In addition to aspirin and clopidogrel, what is the 12. A 32-year-old woman presents with 35 minutes of
most appropriate next step in management? palpitations. Her symptoms began abruptly. She has
A. Tenecteplase and unfractionated heparin slight lightheadedness but no syncope, dyspnea, or
B. Enoxaparin, nitrates, and coronary angiography in chest pain. A physical examination reveals a tachy-

t
ne
72 hours cardic, regular heart rhythm and blood pressure of
C. Unfractionated heparin, nitrates, and coronary 132/82 mm Hg and is otherwise normal. An electro-
angiography in 72 hours cardiogram is shown in Fig. 8.4.

e.
D. Fondaparinux and coronary angiography within Performing the Valsalva maneuver and carotid
24 hours sinus pressure fails to terminate the arrhythmia. he

in
E. Unfractionated heparin and coronary angiography next best step in management is:
within 24 hours A. DC cardioversion

ic
B. Adenosine
10. A 67-year-old man with a history of cigarette smok- C. Amiodarone
ing presents with accelerating anginal chest pain.
On initial evaluation, his troponin T is elevated to
ed D. Labetalol
E. Normal saline
2.0 (0–0.1), and an electrocardiogram demonstrates
sm
T-wave inversions in leads I and aVL. He is referred 13. A 45-year-old man with a heavy smoking history
for coronary angiography, whereupon a 60% stenosis presents by ambulance with 3 hours of crushing
in the middle right coronary artery and an 80% steno- substernal chest pain. On examination, he appears
sis in a small irst diagonal branch of the left anterior uncomfortable. His jugular venous pressure is 10 cm
ok

descending artery are demonstrated. Medical manage- H2O. he irst and second heart sounds are normal.
ment in lieu of revascularization is advised. In addition He has scant bibasilar rales. An electrocardiogram of
to aspirin, beta blocker, statin therapy, and smoking the patient is shown in Fig. 8.5. A cardiac catheter-
bo

cessation, you prescribe: ization laboratory is 2.5 hours away, accounting for
A. Fish oil transfer time.
B. Ranolazine In addition to aspirin, clopidogrel, morphine, oxy-
e

C. Isosorbide mononitrate gen, and nitrate therapy, what is the most appropriate
://

D. Clopidogrel next step in management?


E. Warfarin A. Transfer for primary percutaneous coronary angi-
tp

ography (PCI).
11. A 63-year-old long-term patient of yours presents with B. Administer tenecteplase and transfer to a PCI-
worsening breathlessness and palpitations. He has a capable facility.
ht

known history of mitral valve prolapse diagnosed years C. Prescribe colchicine and aspirin.
prior on the basis of auscultatory indings of a click– D. Administer half-dose thrombolytics and transfer
murmur complex and conirmed by echocardiogram. to a PCI-capable facility.
For the past 2 weeks, he describes progressive breath- E. Administer intravenous metoprolol boluses for
lessness, orthopnea, and palpitations. On examina- goal heart rate of 55 beats per minute.
tion, he appears uncomfortable and is dyspneic with
conversation. His heart rate is approximately 100 beats 14. A 48-year-old Caucasian woman presents for follow-
per minute and irregular. His jugular venous pres- up after a recent hospitalization for heart failure. She
sure is 16 cm H2O. He has a grade 4 of 6 pansystolic has a salient history of nonischemic cardiomyopa-
murmur with a thrill appreciated over the apex. he thy with an ejection fraction of 25%. She presented
murmur is blowing in quality and is heard through- with volume overload and dyspnea, which responded
out the precordium with particular radiation toward well to diuresis. Since discharge, however, she has
the base of the heart. he apex beat is hyperdynamic. remained dyspneic even climbing less than one light
CHAPTER 8 Cardiovascular Disease 169

I aVR V1 V4

II aVL V2 V5

t
III aVF V3 V6

ne
e.
V1

in
• Fig. 8.4 Electrocardiogram for patient in Question 12.

ic
ed
sm
I aVR V1 V4
ok

II aVL V2 V5
e bo
://

III aVF V3 V6
tp
ht

V1

• Fig. 8.5 Electrocardiogram for patient in Question 13.

of stairs. Her medications include lisinopril 40 mg heart sounds are normal, and a third heart sound is
daily, carvedilol 25 mg twice daily, spironolactone 25 appreciated. She has scant bilateral pitting edema of
mg daily, furosemide 40 mg twice daily, and potas- the lower extremities. Her lungs are clear.
sium supplementation. On examination, she appears An electrocardiogram demonstrates left bundle
fatigued. Her heart rate is 68 beats per minute, and branch block with QRS duration of 155 ms. Her
her blood pressure is 100/60 mm Hg. Her jugular serum creatinine level is 1.4 mg/dL, which is the base-
venous pressure is 6 cm H2O. he irst and second line level.
170 C HA P T E R 8 Cardiovascular Disease

What is the best next step in management? medical history includes hypertension. A review
A. Increase furosemide dose. of systems reveals he had an episode of syncope 3
B. Add metolazone. months prior for which he did not seek care, attribut-
C. Refer for cardiac resynchronization therapy (bivent- ing the event to dehydration secondary to the warm
ricular pacemaker placement). summer weather. On examination, he appears well.
D. Evaluate for placement of a left ventricular assist he carotid upstrokes are normal in contour and vol-
device. ume. he irst and second heart sounds are normal.
E. Add hydralazine. here is a grade 2 of 6 midpeaking systolic murmur
at the left upper sternal border and a grade 1 of 6
15. A 34-year-old woman presents for follow-up. She is in pansystolic murmur at the apex. A single pause is
week 32 of her irst pregnancy. She feels well overall heard during the period of auscultation. An electro-
and has been walking 20 minutes daily for exercise. cardiogram demonstrates sinus rhythm at a rate of
She has had several episodes of nocturnal heartburn. 75 beats per minute, left bundle branch block, and
On review of systems, she notes bilateral lower extrem- Mobitz type II AV block.
ity edema that is most pronounced at the end of her What is the next step in management?

t
ne
workday and improved by elevating the legs. On A. Initiate metoprolol and uptitrate for a goal heart
examination, her blood pressure is 96/52 mm Hg, rate of 55–65 beats per minute perioperatively.
and her heart rate is 88 beats per minute; her uterus B. Proceed with surgery without further diagnostic

e.
is gravid. Her jugular venous pressure is 5 cm H2O. testing.
A venous hum is appreciated over the right clavicu- C. Echocardiogram

in
lar fossa. S1 is normal, and S2 splits with inspiration. D. Prescribe 4 L of luid intake daily preoperatively.
here is a grade 2 of 6 midpeaking systolic murmur E. Refer for pacemaker placement prior to surgery.

ic
at the left upper sternal border. A third heart sound is
appreciated. here is 1+ bilateral pitting edema at the 18. A 76-year-old woman seeks care for palpitations. She
ankle.
What is the most appropriate next step?
ed notes sustained palpitations with exertion for the
past 2 weeks. Her medical history includes obstruc-
A. Urinalysis and liver function testing tive sleep apnea, hypertension, type 2 diabetes, and
sm
B. Echocardiogram chronic renal insuiciency with baseline creatinine of
C. Routine follow-up in 2 weeks 2.6 mg/dL. She underwent carotid endarterectomy
D. Lower extremity venous ultrasound 8 years ago for symptomatic left carotid stenosis. On
E. Holter monitor examination, her heart rate is 118 beats per minute,
ok

and the rhythm is irregularly irregular. here is a soft


16. A 48-year-old man is seen prior to discharge after left carotid bruit and a well-healed surgical scar over
being admitted with myocardial infarction. He pre- the left neck. he irst heart sound has variable inten-
bo

sented with a large anterolateral ST segment elevation sity, and the second heart sound splits with inspira-
myocardial infarction complicated by heart failure. tion. here are no murmurs or gallops. here is trace
An occlusion of the proximal left anterior descending pitting edema at both ankles. An electrocardiogram
e

artery was treated with PCI. At present, he feels well. conirms atrial ibrillation.
://

A physical examination reveals euvolemia, a third Which of the following strategies would best pre-
heart sound, and warm extremities. His renal func- vent future stroke?
tp

tion is normal. An electrocardiogram reveals anterior A. Aspirin 325 mg


Q waves. An echocardiogram demonstrates a left B. Warfarin dosed for INR 2–3 indeinitely
ventricular ejection fraction of 35%. he patient’s C. Dabigatran 150 mg twice daily
ht

medications include metoprolol, lisinopril, furose- D. Aspirin 325 mg and clopidogrel 75 mg


mide, atorvastatin, aspirin 81 mg, and clopidogrel E. DC cardioversion followed by warfarin for 1
75 mg. month
What is the most optimal next step in manage-
ment? 19. A 35-year-old woman presents with 2 days of severe
A. Increase aspirin from 81 to 325 mg daily. chest pain. She had an upper respiratory infection 1
B. Increase clopidogrel to 150 mg twice daily. week ago. Pain is worsened with deep inspiration and
C. Ranolazine relieved by leaning forward. Review of systems is nota-
D. Eplerenone ble for cough. On examination, she appears uncom-
E. Metolazone fortable. Her jugular venous pressure is less than 5
cm H2O. here is a tricomponent friction rub heard
17. An 82-year-old man is referred for preoperative risk at the left lower sternal border. An electrocardiogram
stratiication prior to planned total hip replacement reveals difuse ST segment elevation, which is concave
for severely painful, limiting osteoarthritis. His past upward in morphology. An echocardiogram reveals a
CHAPTER 8 Cardiovascular Disease 171

trace pericardial efusion and normal left ventricular prior. At the time, he was treated with placement of
function without wall motion abnormalities. a drug-eluting stent to a 90% stenosis of the right
What is the most optimal management? coronary artery. Since then, he has felt well without
A. Aspirin and prednisone angina, dyspnea, or syncope. He plays singles tennis
B. Prednisone twice per week in a competitive league. His medica-
C. Prednisone and ibuprofen tions include metoprolol, aspirin, clopidogrel, and
D. Aspirin atorvastatin. he result of his physical examination
E. Ibuprofen and colchicine is normal.
What is the next step in management?
20. A 16-year-old boy is evaluated for a preparticipation A. Proceed with surgery, maintaining the current
sports physical examination for swimming. He feels medication regimen.
well. His family history is negative for sudden cardiac B. Discontinue clopidogrel and proceed with sur-
death; an uncle has aortic stenosis. On examination, gery, continuing aspirin through the perioperative
he is tall and thin. he irst heart sound is normal. period.
here is an early systolic click and a grade 2 of 6 mid- C. Discontinue aspirin and clopidogrel prior to pro-

t
ne
peaking systolic murmur. here is a grade 1 of 6 dia- ceeding with surgery.
stolic murmur at the left upper sternal border. here D. Discontinue clopidogrel and atorvastatin prior to
is a harsh systolic bruit heard best over the posterior proceeding with surgery.

e.
chest. here is slight radiofemoral pulse delay. he E. Perform an exercise treadmill test prior to issuing a
boy’s blood pressure is 128/82 mm Hg in the right recommendation regarding the proposed surgery.

in
arm. An echocardiogram reveals a bicuspid aortic valve
with no stenosis and mild regurgitation, normal aortic 23. A 72-year-old man with a history of peripheral arterial

ic
root diameter, normal left ventricular chamber size, disease requiring femoral-popliteal bypass and with a
and normal left ventricular function. long-standing smoking history presents with subster-
What is the next best step?
A. Refer for aortic valve replacement.
ed nal chest pain. Pain is evoked by walking four blocks
briskly and abates readily with several minutes of rest.
B. Proceed with sports participation with serial echo- He has never had pain with activities of self-care or
sm
cardiographic monitoring. at rest. A coronary angiogram reveals a 60% stenosis
C. Aspirin 81 mg in the middle right coronary artery (fractional low
D. Magnetic resonance angiography of the thoracic reserve [FFR] = 0.92), a 70% stenosis in a small irst
and abdominal aorta diagonal branch of the left anterior descending artery,
ok

E. Losartan and a 40% stenosis in the middle left circumlex artery.


Left ventriculography reveals normal left ventricular
21. A 62-year-old man with diabetes and tobacco use function. he patient’s medications include meto-
bo

presents with 6 hours of stuttering substernal chest prolol, aspirin, and atorvastatin. On examination, his
pressure. On initial evaluation, there are ST seg- heart rate is 60 beats per minute at rest, increasing to
ment depressions in leads V5 and V6 of the elec- 80 with ambulation. His blood pressure is 132/82 mm
e

trocardiogram with a troponin T of 1.0 (0–0.1) ng/ Hg. he results of his cardiac and pulmonary exami-
://

mL. He is referred for coronary angiography, and a nations are normal.


95% stenosis of the irst obtuse marginal coronary What is the next step in management?
tp

artery is treated with percutaneous coronary inter- A. A PCI with stent placement of the irst diagonal
vention. After the procedure, he feels well, and his branch
chest pain has abated. He is prescribed metoprolol B. Increase dosage of metoprolol.
ht

25 mg, luvastatin 10 mg daily, aspirin 81 mg daily, C. Prescribe isosorbide mononitrate.


and prasugrel 10 mg. D. Prescribe ranolazine.
Which of the following would you recommend? E. Prescribe clopidogrel.
A. Change prasugrel to clopidogrel.
B. Change luvastatin to high-dose atorvastatin. 24. A 42-year-old man with known cardiomyopathy pres-
C. Increase aspirin 81 mg to 325 mg. ents for follow-up. His family history is notable for
D. Add ranolazine. dilated cardiomyopathy in his father and one brother.
E. Add isosorbide mononitrate. A paternal uncle died suddenly at a young age of
unknown cause. He presented 1 year prior with dys-
22. A 64-year-old man is seen in consultation for pre- pnea on exertion; a transthoracic echocardiogram
operative risk assessment prior to cholecystectomy revealed a dilated left ventricle with an ejection frac-
for biliary colic. He has a salient medical history tion of 25%. he result of coronary angiography was
of coronary artery disease presenting with non-ST- normal, and no other reversible cause of cardiomyopa-
elevation myocardial infarction (NSTEMI) 3 years thy was discovered.
172 C HA P T E R 8 Cardiovascular Disease

Since then, he has been maintained on furose- valve. She is hospitalized for evaluation and treatment
mide, lisinopril, carvedilol, spironolactone, and a and is initiated on nafcillin. On hospital day 3, her
multivitamin. He has one hospitalization for vol- fevers persist. An electrocardiogram demonstrates a
ume overload attributed to dietary indiscretion. He prolonged PR interval with periods of Mobitz II AV
feels winded shoveling snow and working out on his block.
exercise bicycle but otherwise has no current symp- What is the next step in diagnosis?
toms of heart failure. On examination, his blood A. Electrophysiologic testing
pressure is 110/60 mm Hg, and his heart rate is 70 B. Brain MRI
beats per minute. His jugular venous pressure is C. Transesophageal echocardiogram
5 cm H2O. he irst and second heart sounds are D. Request blood cultures to be incubated for 14
normal, and a third heart sound is noted. An elec- days.
trocardiogram reveals sinus rhythm with a single E. Serologies for Coxiella burnetii
premature ventricular beat. he QRS duration is
100 ms. 27. A 65-year-old woman presents with chest and arm
What is the best next step in management? pain. Her pain is substernal with radiation to the right

t
ne
A. Refer for cardiac resynchronization therapy arm and is evoked by exertion but not predictably
(CRT). relieved with rest. She has a history of severe lumbar
B. Add digoxin. disc disease and osteoarthritis of the knee, which limit

e.
C. Refer for implantable cardioverter-deibrillator. her functional capacity. In addition, she has hyperten-
D. Add metolazone. sion and depression. A physical examination reveals a

in
E. Refer for cardiac transplant evaluation. well-appearing, slightly overweight woman. he irst
and second heart sounds are normal, and the lungs are

ic
25. A 48-year-old man presents with dyspnea and dizzi- clear. An electrocardiogram demonstrates a left bundle
ness. He notes several months of progressive dyspnea, branch block, which was noted on prior electrocardio-
which has limited his ability to perform yard work. In
addition, he notes pronounced dizziness and presyn-
ed grams obtained before treatment with antidepressant
therapy.
cope when moving from squatting to a standing posi- What is the best next diagnostic step?
sm
tion as he weeds his garden. His past medical history A. Exercise treadmill test
includes hypertension. His medications include amlo- B. Exercise stress echocardiogram
dipine. On examination, he appears well. His jugular C. Dobutamine stress echocardiogram
venous pressure is 8 cm H2O. he irst heart sound D. Nuclear stress test with regadenoson
ok

is normal, and the second heart sound splits with E. Reassurance


inspiration. A fourth heart sound is present. here is a
grade 1 of 6 pansystolic murmur at the apex radiating 28. A 76-year-old man with a signiicant smoking his-
bo

to the back. here is a grade 2 of 6 harsh, late-peaking, tory presents with leg pain. He notes left-sided leg
diamond-shaped systolic murmur at the base with heaviness that occurs when he walks more than three
radiation to the clavicles and carotid arteries. blocks and resolves predictably with several minutes
e

he murmur increases in intensity with the Valsalva of rest, after which he can resume walking. He has
://

maneuver and with squat-to-stand maneuvers. he no chest pain or dyspnea. On physical examination,
patient’s lungs are clear. His extremities are warm and there is a left femoral bruit and diminished left dor-
tp

well perfused. salis pedis and posterior tibial pulses. he patient’s


What is the most likely diagnosis? ankle brachial index is 0.72 on the left and 0.9 on the
A. Dilated cardiomyopathy right.
ht

B. Mitral stenosis In addition to treating hypertension and dyslipid-


C. Aortic stenosis emia to target and prescribing aspirin, what is the best
D. Hypertrophic obstructive cardiomyopathy next step?
E. Atrial septal defect A. Supervised exercise program
B. Pentoxifylline
26. A 32-year-old woman presents with fever. She has a C. Lower extremity angiography and stenting
salient medical history of intravenous drug abuse. On D. Peripheral arterial bypass surgery
examination, her temperature is 102°F. She has a grade E. Amlodipine
2 of 6 diastolic blowing murmur at the base. here are
splinter hemorrhages in the nails. Blood cultures grow 29. A 60-year-old woman with a bicuspid aortic valve
methicillin-sensitive Staphylococcus aureus (MSSA). and known aortic regurgitation presents for fol-
An electrocardiogram reveals normal sinus rhythm. A low-up. She feels like she is slightly more limited
transthoracic echocardiogram reveals moderate aortic in her activities of daily living than she was 1 year
regurgitation and an 8-mm vegetation on the aortic ago; she feels dyspneic playing with her 2-year-old
CHAPTER 8 Cardiovascular Disease 173

grandchild. She has no chest pain, lower extremity Coronary angiography demonstrates a 90% ste-
edema, or paroxysmal nocturnal dyspnea. On exam- nosis of the proximal right coronary artery, a 40%
ination, her blood pressure is 100/40 mm Hg, and stenosis of a irst obtuse marginal branch of the left
her heart rate is 70 beats per minute. he carotid circumlex artery, and an 80% stenosis of the proximal
impulses are brisk with a rapid rise. he irst heart left anterior descending artery. A left ventriculogram
sound is normal, and the second heart sound splits demonstrates normal left ventricular function.
with inspiration. here is an early systolic click and What is the optimal approach to management?
a grade 2 of 6 midpeaking systolic murmur. here is A. Add metoprolol and isosorbide mononitrate.
a grade 2 of 4 early diastolic murmur with a blow- B. PCI with stent placement of the LAD artery
ing quality. he apex beat is laterally displaced. An C. PCI with stent placement of the RCA and LAD
echocardiogram demonstrates a left ventricular ejec- artery
tion fraction of 70%. he left ventricle is dilated D. Refer for coronary artery bypass grafting of the
with an end-diastolic diameter of 69 mm. he aortic LAD artery and RCA.
valve is bicuspid with a wide jet of aortic insui- E. Myocardial viability study
ciency, which is graded as severe. here is systolic

t
ne
low reversal noted in the descending thoracic aorta. 32. A 77-year-old man with hypertension and a
he aortic root diameter is 30 mm. 60-pack-year smoking history presents for evalua-
What is the next step in management? tion and routine follow-up. He feels well. His medi-

e.
A. Prescribe losartan. cations include lisinopril, aspirin, amlodipine, and
B. Aortic valve replacement atorvastatin. On examination, there is a soft bruit

in
C. Echocardiogram in 6 months over the epigastrium, and the aortic pulsation is
D. Exercise treadmill testing prominent. An abdominal ultrasound reveals an

ic
E. Carotid ultrasound abdominal aortic aneurysm measuring 4.1 cm in
transverse diameter.
30. A 58-year-old man is seen in follow-up for manage-
ment of hypertension. He has been maintained on
ed In addition to smoking cessation counseling, what
is the next step in management?
pharmacotherapy for 8 months with persistently poor A. Refer for open surgical repair.
sm
blood pressure control. He feels well and is asymptom- B. Refer for endovascular repair.
atic. His past medical history includes osteoarthritis, C. Urgent CT angiogram of the abdominal aorta
for which he takes over-the-counter analgesics. His D. Repeat abdominal ultrasound in 6 months.
medications otherwise include amlodipine, hydro- E. Exercise treadmill test
ok

chlorothiazide, and lisinopril. On examination, he is


an anxious-appearing, middle-aged man who is not 33. A 74-year-old man with hypertension, coronary
distressed. His body mass index is 28 kg/m2. His blood artery disease, GERD, and osteoarthritis presents for
bo

pressure in the right arm is 172/102 mm Hg. A fourth follow-up. He had an ST segment myocardial infarc-
heart sound is appreciated. here are no murmurs tion 2 years prior and underwent successful stenting
and no peripheral bruits. here is no lower extremity of a complete LAD arterial occlusion. For the past 3
e

edema. weeks, he has noted worsening dyspnea on light exer-


://

What is the best next step in evaluation? tion coupled with lower extremity swelling. He has
A. Twenty-four-hour urine free cortisol, serum renin, had no recurrent chest pain.
tp

and aldosterone levels; thyroid function testing; His medications include metoprolol, nifedipine,
and serum metanephrines aspirin, and rosuvastatin. On examination, his blood
B. CT angiography of the renal and mesenteric pressure is 126/80 mm Hg. His heart rate is 70 beats
ht

arteries per minute. His jugular venous pressure is 14 cm H2O.


C. Captopril renal scan he irst and second heart sounds are normal, and a
D. Obtain detailed over-the-counter medication use third heart sound is appreciated. here is lower extrem-
history. ity edema to the knee bilaterally. A stress echocardio-
E. Twenty-four-hour Holter monitor gram reveals mild anterior wall hypokinesis at rest, and
all walls augment appropriately with stress. he left ven-
31. A 66-year-old woman presents with arm pain and tricular ejection fraction at rest is estimated at 40%.
abdominal bloating that is evoked by climbing two In addition to diuresis and discontinuation of nife-
lights of stairs. Her past medical history includes dia- dipine, what is the most appropriate management?
betes and hypertension. Her medications include sim- A. Add hydralazine and isosorbide mononitrate.
vastatin, aspirin, and losartan. She is referred for an B. Add clopidogrel.
exercise treadmill test, which demonstrates difuse ST C. Add lisinopril.
segment depressions of 2–3 mm at 70% of maximum D. Add spironolactone.
predicted heart rate. E. Add digoxin.
174 C HA P T E R 8 Cardiovascular Disease

34. A 48-year-old man presents with chest pressure. He D. Referral for surgical removal
has a history of diabetes, dyslipidemia, hyperten- E. PET-CT of the chest, abdomen, and pelvis
sion, and active tobacco use. For the past 3 weeks, he
has noted substernal chest pressure that is evoked by 37. An 84-year-old woman presents with fatigue and
climbing two lights of stairs briskly and relieved over dyspnea of 3 months’ duration. Although previously
several minutes with rest. His family history includes active, she notes a decreased ability to garden, shop,
two brothers with myocardial infarction at ages 50 and and perform activities of daily living. She has no chest
52, respectively. he result of his physical examination pain, orthopnea, or lower extremity edema. Her past
is normal. An electrocardiogram reveals normal sinus medical history is unremarkable, and her only medica-
rhythm with no ischemic change and normal axis and tion is a multivitamin. On examination, her heart rate
intervals. is 42 beats per minute and regular. Her blood pressure
What is the best diagnostic step? is 128/70 mm Hg, and her oxygen saturation is 97%
A. Cardiac MRI breathing room air. Her skin is warm and dry, venous
B. Coronary artery calcium score pressure is low, and irst and second heart sounds are
C. Exercise treadmill test normal. Her lungs are clear. An electrocardiogram

t
ne
D. Coronary angiography reveals sinus bradycardia with a narrow QRS complex.
E. Pharmacologic stress myocardial perfusion With ambulation, her oxygen saturation is 96%, and
imaging her heart rate increases to 50 beats per minute. he

e.
result of thyroid function testing is normal.
35. A 78-year-old woman is seen in the oice for a pre- What is the next step in management?

in
operative evaluation. She has a notable history of A. Forty-eight-hour Holter monitor
low back pain secondary to spinal stenosis, and it B. hirty-day event monitor
has been recommended that she undergo lumbar

ic
C. Electrophysiologic study with measurement of
laminectomy. She is unable to ascend and descend sinus node recovery time
stairs secondary to low back pain. Her medical his-
tory includes hypertension and diabetes leading to
ed D. Serum Lyme titer
E. Referral for pacemaker placement
chronic kidney disease with baseline creatinine of
sm
2.1 mg/dL. She had a transient ischemic attack 2 38. A 32-year-old man is seen in follow-up. He feels well.
years prior. Her medications include aspirin, NPH At a recent “health day event” at his oice, his blood
insulin, rosuvastatin, and amlodipine. Her physical pressure was 120/60 mm Hg, and his total cholesterol
examination reveals a heart rate of 78 beats per min- was measured at 160 mg/dL. His past medical history
ok

ute, blood pressure of 138/68 mm Hg, and is oth- is notable for tension headache well controlled with
erwise normal. An electrocardiogram reveals sinus as-needed acetaminophen. On examination, his heart
rhythm, left anterior fascicular block, and Q waves rate is 103 beats per minute, and his blood pressure
bo

in leads V1 and V2. 156/98 mm Hg. A funduscopic examination reveals


What is the next step in diagnosis and manage- a normal optic disc and normal retinal vessels. His
ment? thyroid gland is slightly full to palpation. His carotid
e

A. Exercise treadmill test arteries are normal in contour and volume. he irst
://

B. Dobutamine stress echocardiogram and second heart sounds are normal. here are no
C. Coronary CT angiogram abdominal or lank bruits, and there is no radiofemo-
tp

D. Proceed with surgery. ral pulse delay.


E. Prescribe metoprolol. What is the best next diagnostic step?
A. Serum potassium and measurement of aldoste-
ht

36. A 26-year-old medical student is seen in the oice. rone-to-renin ratio


In the midst of a workshop with his classmates, while B. Renal artery Doppler ultrasound
learning to perform bedside cardiac ultrasound, an C. Give the patient a home blood pressure diary,
abnormality was appreciated. He feels entirely well, measure TSH, and repeat oice blood pressure in
and the result of his physical examination is normal. A 3 months.
formal echocardiogram reveals a left atrial mass mea- D. Prescribe chlorthalidone.
suring 2.1 × 2.3 cm. he mass is pedunculated, mobile, E. Measure serum free metanephrines.
and attached to the interatrial septum by a thin stalk.
he remainder of the echocardiogram is normal. 39. A 62-year-old man with diabetes and a tobacco use
What is the next step in management? history presents for follow-up of chest pain. He
A. Echocardiography in 6 months initially presented with chest pain 5 months prior;
B. Two sets of blood cultures 12 hours apart coronary angiography demonstrated a 60% stenosis
C. Anticoagulation with warfarin for goal INR of of the middle left anterior descending artery and an
2–3 80% stenosis of a moderate-sized obtuse marginal
CHAPTER 8 Cardiovascular Disease 175

branch of the left circumlex. He was initiated on high-pitched blowing murmur at the apex that radi-
aspirin, metoprolol, and isosorbide mononitrate. ates to the left sternal border. An EKG shows normal
For persistent chest pain with moderate exertion, sinus rhythm with occasional premature ventricular
ranolazine was added. He has undergone two contractions. An echocardiogram shows normal left
courses of supervised cardiac rehabilitation, but ventricular function (ejection fraction, 65%; left ven-
his symptoms persist. Currently, he gets chest pain tricular end-systolic dimension, 35 mm), thickened
with moderate exertion, which precludes him from and redundant mitral lealets with a lail posterior
playing doubles tennis and bowling. His medica- lealet, dilated left atrium, 3–4+ mitral regurgitation
tions include metformin, metoprolol, rosuvastatin, with systolic low reversal in the pulmonary veins,
isosorbide mononitrate, ranolazine, and lisinopril. and estimated pulmonary artery systolic pressure of
On examination, his heart rate is 56 beats per min- 36 mm Hg.
ute, and his blood pressure is 100/60 mm Hg. he What is an appropriate next step in her management?
results of cardiopulmonary examinations are nor- A. Referral for surgical mitral valve repair
mal. His total cholesterol is 100 mg/dL, with HDL B. Referral for surgical mitral valve replacement
cholesterol of 38 mg/dL. His hemoglobin A1c is C. Referral for MitraClip

t
ne
6.2%. D. Close follow-up with a transthoracic echocardio-
What is the best next step in management? gram every 6–12 months
A. Refer for coronary artery bypass surgery. E. Vasodilators to reduce afterload

e.
B. Refer for PCI of the obtuse marginal stenosis.
C. Increase metoprolol. 42. A 65-year-old woman with chronic systolic heart

in
D. Increase isosorbide mononitrate. failure (left ventricular ejection fraction, 30%)
E. Refer for cardiac rehabilitation. comes for a routine clinic visit. She reports that she

ic
is dyspneic climbing one light of stairs and uses two
40. A 65-year-old man presents for evaluation of worsen- pillows to sleep at night. She has intermittent lower
ing dyspnea on exertion. His past medical history is
notable for Hodgkin lymphoma treated with mantle
ed extremity edema, especially after eating a salty meal.
Her medications include lisinopril 20 mg daily,
radiation, coronary artery disease status post-CABG, carvedilol 25 mg twice daily, spironolactone 25 mg
sm
moderate restrictive lung disease, and stage 3 chronic daily, and torsemide 40 mg daily. On examination,
kidney disease. His physical examination is notable she has a heart rate of 70 beats per minute, blood
for jugular venous distention, decreased carotid pressure of 110/70 mm Hg, no jugular venous dis-
pulsation, normal S1, inaudible S2, late-peaking tention, normal heart sounds, a II/VI holosystolic
ok

crescendo–decrescendo systolic murmur at the base murmur at the apex, and trace-1+ peripheral edema.
radiating to both carotids, and a soft holosystolic Her laboratory values are notable for sodium 140
murmur at the apex. An echocardiogram shows a left mEq/L, potassium 4.8 mEq/L, blood urea nitrogen
bo

ventricular ejection fraction of 35%–40%, a severely 20 mg/dL, and creatinine 1.2 mg/dL.
calciied aortic valve with restricted opening (peak What is the next most appropriate step in her man-
and mean transaortic gradients of 64 and 40 mm Hg, agement?
e

respectively; calculated aortic valve area, 0.7 cm2), A. Continue her current medications.
://

and a calciied mitral valve with mild to moderate B. Increase lisinopril to 30 mg daily.
regurgitation. C. Stop lisinopril and start sacubitril/valsartan 49/51
tp

In addition to cautious diuresis, what is an appro- mg twice daily after 36-hour washout.
priate next step in his management? D. Increase torsemide to 60 mg daily.
A. Continued medical management for heart failure E. Add digoxin 0.125 mg daily.
ht

B. Aortic balloon valvuloplasty


C. Surgical aortic valve replacement 43. A 24-year-old woman with a history of nonisch-
D. Transcatheter aortic valve replacement emic cardiomyopathy (left ventricular ejection frac-
E. Surgical aortic and mitral valve replacement tion, 25%) comes for a postdischarge clinic visit
after a recent hospitalization with heart failure.
41. A 60-year-old woman presents to the emergency She reports postural lightheadedness and decreased
department with palpitations. Her past medical his- exercise tolerance due to fatigue and dyspnea. Her
tory is notable for well-controlled hypertension and medications include lisinopril 2.5 mg daily, meto-
mitral valve prolapse that were diagnosed when she prolol succinate 25 mg daily, spironolactone 12.5
was a teenager. On examination, her heart rate is 70 mg daily, furosemide 20 mg daily, and warfarin.
beats per minute with blood pressure 110/80 mm A physical examination reveals a heart rate of 100
Hg. She has no evidence of jugular venous disten- beats per minute and blood pressure of 90/70 mm
tion. Her cardiac examination shows regular rate and Hg. She has no jugular venous distention, her lungs
rhythm with normal S1 and S2. She has a holosystolic are clear, her heart sounds are irregularly irregular
176 C HA P T E R 8 Cardiovascular Disease

with an audible S3. She has no peripheral edema. room air oxygen saturation of 94%. She has evidence
Her laboratory values are notable for sodium 136 of jugular venous distention, bibasilar rales, and no
mEq/L, potassium 4.2 mEq/L, blood urea nitrogen peripheral edema. Her EKG shows sinus tachycardia
16 mg/dL, and creatinine 1.0 mg/dL. with 2-mm ST elevation in leads V1–V4.
What is the next most appropriate step in her man- What is the most likely diagnosis?
agement? A. Acute pulmonary embolism
A. Increase her furosemide dose. B. Amniotic luid embolism
B. Add digoxin. C. Peripartum cardiomyopathy
C. Increase her metoprolol succinate dose. D. Acute coronary dissection
D. Change lisinopril to sacubitril/valsartan. E. Acute pericarditis
E. Add ivabradine.
47. A 32-year-old recent immigrant from Haiti presents
44. A 40-year-old man presents to the emergency depart- for her irst clinic evaluation. When she was 12 years
ment with sudden onset of tachypnea and chest pain old, she had rheumatic fever that was treated with
after taking a trans-Atlantic light. On examination, antibiotics. She is asymptomatic. Her physical exami-

t
ne
he has a heart rate of 120 beats per minute, blood pres- nation shows a heart rate of 70 beats per minute with
sure of 100/80 mm Hg, and room air oxygen satu- a blood pressure of 122/77 mm Hg. She has no jugular
ration of 84%. Computed tomography of the chest venous distention. Her lungs are clear. She has a regu-

e.
shows an acute pulmonary embolus in the bilateral lar rate and rhythm with a loud S1 and physiologically
main pulmonary arteries. An echocardiogram shows split S2. A soft early diastolic rumble is heard at the

in
a moderately enlarged right ventricle with reduced apex with an opening snap after S2. here is wide sep-
systolic function and an estimated right ventricular aration between A2 and the opening snap. No periph-

ic
systolic pressure of 50 mm Hg. he result of a cardiac eral edema is present.
troponin test was negative. Which of the following is the most appropriate
What is the next best step in his management?
A. Systemic thrombolysis
ed next recommendation for this patient?
A. Take antibiotics prior to dental procedures.
B. Intravenous heparin B. Start furosemide.
sm
C. Emergent surgical thrombectomy C. Start anticoagulation.
D. Catheter-directed thrombolysis D. Start oral penicillin V 250 mg twice daily.
E. Placement of an inferior vena cava ilter E. Start metoprolol.
ok

45. A 70-year-old man with hypertension and diabetes 48. A 70-year-old woman presents to your oice for an
presents with new-onset atrial ibrillation. His physi- annual health examination. She has no known cardio-
cal examination is notable for an irregularly irregular vascular risk factors or history of cardiovascular disease.
bo

pulse of 120 beats per minute but is otherwise unre- She has never smoked and drinks alcohol socially. She
markable. His laboratory test results are notable for a leads an active lifestyle and is asymptomatic. Her phys-
creatinine level of 1.3 mg/dL. In addition to starting ical examination is notable for a heart rate of 70 beats
e

him on metoprolol for rate control, you recommend per minute and a blood pressure of 124/84 mm Hg.
://

anticoagulation with apixaban 5 mg twice daily. He he rest of her physical examination is unremark-
inquires if he needs to make any dietary or medication able. Her EKG is normal. She has a total cholesterol
tp

changes with apixaban, similar to what he has heard of 180 mg/dL and an HDL cholesterol of 50 mg/dL.
with warfarin. Her Framingham risk score for cardiovascular disease is
Which of the following drugs would be most likely 8.6%. Her pooled cardiovascular risk score is 8.8%.
ht

to increase his risk of bleeding if combined with apixa- Which of the following is the most appropriate test?
ban? A. Exercise stress test
A. Sotalol B. Abdominal ultrasound
B. Amiodarone C. Ankle brachial index
C. Verapamil D. Coronary artery calcium score
D. Phenytoin E. No further testing is needed.
E. Ketoconazole
49. A 28-year-old overweight woman presents for evalua-
46. A 32-year-old woman who is 4 weeks postpartum tion of recurrent headaches. She does not give a history
presents with acute-onset chest pain and dyspnea. She of snoring and denies any daytime hypersomnolence,
has no prior cardiac risk factors or known cardiac dis- lushing, palpitations, or diaphoresis. She has no sig-
ease. Her physical examination is notable for a heart niicant past medical history and is on no medications.
rate of 120 beats per minute, BP 100/60 mm Hg, and Her physical examination is notable for a heart rate of
CHAPTER 8 Cardiovascular Disease 177

75 beats per minute and blood pressure of 180/100 Which of the following tests is most likely to iden-
mm Hg on the right and 178/98 mm Hg on the left. tify the cause of her hypertension?
Her BMI is 28 kg/m2. Her lungs are clear. Her car- A. Renal artery Doppler study
diovascular examination is notable for an audible S4. B. Polysomnography
Her extremities reveal no edema and have equal pulses C. Twenty-four-hour urine for metanephrines and
without delay. Her electrolytes are normal, and her catecholamines
creatinine level is 1.0 mg/dL. D. Serum aldosterone and renin levels
E. Transthoracic echocardiogram

Chapter 8 Answers

1. ANSWER: C. Atrial septal defect myocardial infarction, acute pericarditis, early repolar-
his patient presents with symptoms of palpita- ization, and left ventricular aneurysm. In this case, the
tions, a common complaint. he physical examina- difuse nature of the ST segment elevation, the con-

t
ne
tion in such cases should be focused on identifying cave upward morphology of the ST segment, the lack
underlying structural heart disease. he parasternal of reciprocal ST segment depressions, and the PR seg-
lift, widened second heart sound with a “ixed split,” ment depressions on the electrocardiogram are all most

e.
prominent pulmonic closure sound, and a pulmo- consistent with acute pericarditis rather than ST-ele-
nary low murmur are all classic indings of a secun- vation myocardial infarction (STEMI). Patients with
dum atrial septal defect (ASD). Assuming normal acute pericarditis often assume a position of sitting

in
right and left ventricular compliance, ASDs cause up and leaning forward, which minimizes pericardial
left-to-right shunting of blood and chronic volume pain. Making the distinction between acute pericardi-

ic
loading of the right atrium and ventricle, leading to tis and acute STEMI is sometimes diicult, and bed-
progressive dilation and predisposing to arrhythmias, ed side echocardiography can be helpful. In this patient,
including atrial ibrillation, atrial tachycardia, and the pericardial efusion and lack of a wall motion
ventricular rhythms. he increased right-sided low abnormality also favor pericarditis rather than STEMI.
sm
gives rise to the pulmonary valve murmur and ixed Nonsteroidal antiinlammatory agents, often coupled
splitting of the second heart sound. he electrocar- with colchicine, are the treatment of choice. Option
diographic indings of rightward axis and incomplete A would be appropriate for a patient suspected of hav-
right bundle branch block are typical of a secundum ing STEMI, in which case urgent reperfusion therapy
ok

ASD. Wolf-Parkinson-White syndrome would pres- would be indicated. Option B would be appropriate
ent with a short PR interval and delta waves on the for patients with high-risk non-ST elevation acute
electrocardiogram, whereas the electrocardiogram coronary syndromes. Prednisone, Option C, has been
bo

of patients with arrhythmogenic right ventricular shown to increase the risk of recurrent disease when
cardiomyopathy classically demonstrates right ven- prescribed in patients with pericarditis and should be
tricular conduction delay; low voltage; and T-wave avoided. Option E is the management of choice for
e

inversions in the septal, anterior, and inferior leads. patients with cocaine-induced myocardial ischemia,
://

he diastolic rumbling murmur, loud S1, and open- which the history does not support in this case.
ing snap of mitral stenosis are not appreciated. Finally, Imazio M, Gaita F, LeWinter M. Evaluation and treatment
tp

hyperthyroidism can present with arrhythmia and of pericarditis: a systematic review. JAMA. 2015;314(14):
murmur related to the hyperdynamic state, but other 1498–1506.
indings of hyperthyroidism, including weight loss,
ht

stare, diaphoresis, and tremor, are not appreciated. 3. ANSWER: A. Infusion of normal saline
Geva T, Martins JD, Wald RM. Atrial septal defects. Lancet. his patient with multiple cardiac risk factors pres-
2014;383(9932):1921–1932. ents with an inferior ST segment myocardial infarc-
tion. Inferior STEMI can be due to occlusion of the
2. ANSWER: D. Ibuprofen right coronary or left circumlex coronary arteries; in
his patient presents with chest pain and electrocar- this case, the ST segment elevation in lead III that is
diographic indings of ST segment elevation; specii- of greater magnitude than in lead II, coupled with the
cally, the electrocardiogram reveals sinus tachycardia signiicant reciprocal depressions in lead aVL, suggests
and ST segment elevations of 1–3 mm in leads I, II, occlusion of the right coronary artery. he physical
III, aVF, and V2–V6. here are PR segment depres- examination reveals a triad of hypotension, jugular
sions most prominent in leads I, II, and V6. he dif- venous distention with the Kussmaul sign, and clear
ferential diagnosis for ST segment elevation on the lungs. his triad suggests a clinical diagnosis of right
electrocardiogram includes ST segment elevation, ventricular myocardial infarction. When the right
178 C HA P T E R 8 Cardiovascular Disease

coronary artery is occluded proximally, the acute RV patient’s physical examination is not consistent with
marginal branches of the RCA receive no low, leading normal physical indings of pregnancy.
to RV infarction. he hypocontractile, infarcted RV Carabello BA, Crawford FA Jr. Valvular heart disease. N Engl J
leads to elevated right atrial and jugular venous pres- Med. 1997;337(1):32–41.
sure, and the decreased right ventricular compliance
gives rise to the Kussmaul sign. he right ventricle is 5. ANSWER: B. Echocardiography
unable to maintain left ventricular preload, leading to his patient with known malignancy presents with
hypotension. he hypotension can be worsened mark- dyspnea. he diferential diagnosis of a patient with
edly with nitrate therapy. Appropriate hemodynamic cancer presenting with dyspnea includes hematologic
management includes judicious volume loading, often abnormalities, pulmonary embolism, paraneoplastic
guided by a pulmonary artery catheter. he low left syndromes, infection, and cardiac causes. Tachycar-
ventricular preload would be exacerbated by diuresis. dia, hypotension, and elevated jugular venous pres-
Intravenous beta blockers should not be administered sure are consistent with both pulmonary embolism
to patients in cardiogenic shock with acute myocardial and cardiac tamponade. he Kussmaul sign can be
infarction and would worsen hemodynamics in this seen in both conditions. he low electrocardiogram

t
ne
case; esmolol can be a useful agent to control blood voltage and quiet heart sounds, coupled with the
pressure in the management of acute aortic dissec- aforementioned abnormalities, suggest cardiac tam-
tion. Finally, phenylephrine, an alpha agonist, causes ponade as the cause of this patient’s symptoms, which

e.
increased afterload and can worsen myocardial perfor- is best evaluated with an echocardiogram. EMG may
mance in the setting of cardiogenic shock. be indicated to evaluate for neuromuscular parane-

in
O’Gara PT, Kushner FG, Ascheim DD, et al. 2013 ACCF/AHA oplastic syndromes in the setting of small cell lung
guideline for the management of ST-elevation myocardial cancer, but there is no evidence of weakness on exam-
infarction: a report of the American College of Cardiology

ic
ination. Cardiac tamponade secondary to cancer can
Foundation/American Heart Association Task Force on Prac- present insidiously with gradual onset of dyspnea, in
tice Guidelines. J Am Coll Cardiol. 2013;61(4):e78–e140. ed contrast to the acute tamponade observed after car-
diac surgery or intracardiac procedures. Cancers of
4. ANSWER: E. Mitral stenosis the lung, breast, and kidney as well as hematologic
sm
his patient who is 22 weeks pregnant presents with malignancies, are the malignancies most likely to
dyspnea; dyspnea is a common complaint in preg- metastasize to the pericardium.
nancy and has myriad causes. Salient symptoms here Spodick DH. Acute cardiac tamponade. N Engl J Med.
include orthopnea, which is a historical feature with 2003;349:684–690.
ok

high speciicity for elevated left heart illing pressures.


he examination suggests atrial ibrillation with an 6. ANSWER: B. Increase dosage of metoprolol for a
irregularly irregular heart rhythm. he loud S1, open- goal heart rate of 55–65 beats per minute, then pro-
bo

ing snap, and diastolic murmur are all cardinal physi- ceed with total knee replacement.
cal features of mitral stenosis. he state of pregnancy his patient is referred for preoperative cardiac risk
leads to signiicant hemodynamic changes even in nor- stratiication prior to noncardiac surgery. he irst step
e

mal cases, including decreased systemic vascular resis- in assessing for operative risk is to assess whether the
://

tance, increased plasma volume, and increased cardiac planned surgery is emergent and life-or-limb saving,
output. hese physiologic changes can lead to physical in which case the surgical procedure should proceed
tp

indings of systolic low murmurs, a third heart sound, without delay from further cardiovascular diagnostics
a venous hum, and a mammary soule, which are nor- or therapeutics. In this case, surgery is elective. Next,
mal indings in pregnancy. In patients with preexist- assess whether unstable cardiovascular conditions are
ht

ing heart disease, however, the hemodynamic stressors present that should be addressed prior to elective sur-
can lead to signiicant decompensation. Stenotic valve gery; these conditions would include unstable tachy-
lesions in particular tolerate pregnancy poorly, and it is or bradyarrhythmias, unstable ischemic syndromes,
often patients who had well-tolerated, asymptomatic decompensated heart failure, and critical symptomatic
occult mitral stenosis prior to pregnancy who present stenotic valvular disease. None of these syndromes
with symptoms as pregnancy progresses. Pregnancy- are present. Next, assess the risk of the planned sur-
induced cardiomyopathy and pulmonary embolism gery. For low-risk procedures (endoscopy, dermato-
are both important diferential diagnoses for the logic surgery, cataract surgery, and breast surgery),
pregnant patient with dyspnea, but the physical ind- no further cardiovascular diagnostics or therapeutics
ings in this case are inconsistent with these diagno- are necessary prior to the procedure. Orthopedic
ses. Similarly, thyroid storm can present with new procedures, intraabdominal surgery, and intratho-
atrial arrhythmia and high-output heart failure, but it racic surgery are considered intermediate-risk pro-
would not be associated with a systolic low murmur cedures. Next, assess functional capacity. If a patient
rather than a diastolic rumble and opening snap. his can achieve greater than 4 metabolic equivalents of
CHAPTER 8 Cardiovascular Disease 179

activity (corresponding roughly to walking briskly up but the 12-lead electrocardiogram is inconsistent with
two lights of stairs) without cardiovascular limitation, this diagnosis. An exercise stress test or CT coronary
the risk of perioperative cardiovascular events is low, angiogram could be helpful in diagnosing anomalous
and the planned operation can proceed. In this case, coronary or premature coronary obstruction. he Q
the functional capacity is limited by knee pain. Finally, wave on the electrocardiogram in lead aVL in this case
the Revised Cardiac Risk Index (RCRI) can be calcu- is more consistent with a pseudoinfarct pattern from
lated. his index assesses patient-speciic risk factors Wolf-Parkinson-White syndrome. A Holter monitor
for a cardiovascular event in the perioperative period. could document the presence of SVT; however, with
hese include (1) history of heart failure, (2) history of symptoms and a 12-lead electrocardiogram consistent
stroke or transient ischemic attack, (3) history of isch- with Wolf-Parkinson-White syndrome, there are suf-
emic heart disease (prior infarction, known coronary icient data to recommend an EP study as the initial
artery disease, or Q waves on the electrocardiogram, step.
(4) serum creatinine greater than 2.0 mg/dL, and (5) Cohen MI, Triedman JK, Cannon BC, et  al. PACES/HRS
diabetes requiring insulin. Depending on the RCRI expert consensus statement on the management of the
and the risk of the planned procedure, no therapy, beta asymptomatic young patient with a Wolf-Parkinson-White

t
(WPW, ventricular preexcitation) electrocardiographic pat-

ne
blockade, or rarely stress testing and possible revascu-
tern. Heart Rhythm. 2012;9(6):1006–1024.
larization could be considered. his patient has an
RCRI of 3 (including coronary disease, diabetes, and
8. ANSWER: D. Proceed with planned dental proce-

e.
renal insuiciency) and is already maintained on beta
blockade therapy. he most optimal course of therapy dure.
his patient has known valvular heart disease

in
is to uptitrate beta blockade over several weeks for a
goal heart rate of 55–65 beats per minute and thereaf- with mitral valve prolapse and is in need of den-
tal work. For some patients with heart disease,

ic
ter proceed with total knee replacement.
the sequelae of infectious endocarditis mandate
Fleisher LA, Fleischmann KE, Auerbach AD, et al. 2014 ACC/
AHA guideline on perioperative cardiovascular evaluation
ed antibiotic prophylaxis before routine dental work.
and management of patients undergoing noncardiac sur- Patients with prior infectious endocarditis, pros-
gery: a report of the American College of Cardiology/Amer- thetic heart valves, cyanotic congenital heart dis-
ease that is unrepaired, or congenital heart disease
sm
ican Heart Association Task Force on Practice Guidelines. J
Am Coll Cardiol. 2014;64:e77–e137. repaired with prosthetic material and patient sta-
tus post–cardiac transplant all mandate antibiotics
7. ANSWER: A. Electrophysiologic study and prior to dental procedures that manipulate gingival
ok

mapping tissue or involve the periapical area of the tooth.


his young athlete presents with palpitations. His- he patient in this case meets none of these crite-
torical features favoring a paroxysmal supraventricular ria, and the optimal management is to proceed with
bo

tachycardia include abrupt onset and ofset as well as planned dental work without antibiotic therapy.
the presence of neck pounding, which can be caused here is no indication for a repeat echocardiogram
by retrograde atrial activation in the setting of AV for mitral valve prolapse in the absence of a change
e

nodal reentrant tachycardia (AVNRT) or AV reen- in the physical examination or the development of
://

trant tachycardia (AVRT). he electrocardiogram in interval cardiac symptoms. here is no indication


this case is diagnostic, revealing sinus bradycardia with for blood cultures, because there is no current evi-
dence of infectious endocarditis.
tp

sinus arrhythmia that is a normal inding in a young,


Wilson W, Taubert KA, Gewitz M, et al. Prevention of infec-
highly conditioned athlete. he PR interval is short,
tive endocarditis: guidelines from the American Heart
less than 120 ms, and there are delta waves visible as a
ht

Association: a guideline from the American Heart Associa-


broad, slurred initial portion of the QRS best seen in tion Rheumatic Fever, Endocarditis, and Kawasaki Disease
leads II, III, and aVF as well as leads V2 and V3. Lead Committee, Council on Cardiovascular Disease in the
aVL also has a short PR interval with a Q wave that Young, and the Council on Clinical Cardiology, Council
represents a negative delta wave rather than evidence on Cardiovascular Surgery and Anesthesia, and the Quality
of prior infarction; this is a so-called pseudoinfarction of Care and Outcomes Research Interdisciplinary Working
pattern. A short PR interval and delta waves on the Group. Circulation. 2007;116(15):1736–1754.
electrocardiogram, coupled with symptoms suggestive
of arrhythmia, is diagnostic of the Wolf-Parkinson- 9. ANSWER: E. Unfractionated heparin and coronary
White syndrome. First-line therapy for this condition angiography within 24 hours
is referral to electrophysiology for an EP study with his patient presents with high-risk non-ST seg-
mapping and ablation of the accessory pathway. his ment elevation myocardial infarction (NSTEMI). he
procedure is curative in the vast majority of cases. Sig- non-ST segment acute coronary syndromes are com-
nal-averaged electrocardiogram can be used in diagnosis prised of unstable angina and non-ST segment myo-
of arrhythmogenic right ventricular cardiomyopathy, cardial infarction; serum biomarkers of myocardial
180 C HA P T E R 8 Cardiovascular Disease

necrosis (troponin in this case) are not present in therapy for 12 months, assuming there is not a high risk
unstable angina and are present in NSTEMI. It has of bleeding. Ranolazine and isosorbide mononitrate are
been established that there is no beneit to thromboly- antiischemic therapies that can improve anginal symp-
sis in NSTEMI; in contrast to ST-elevation myocar- toms but have no efect on cardiovascular mortality.
dial infarction (STEMI), the infarct-related artery is here is no role for ish oil or warfarin in managing
partially rather than completely occluded. A “selec- NSTEMI absent another indication for these therapies.
tively invasive” strategy consists of medical manage- Amsterdam EA, Wenger NK, Brindis RG, et al. 2014 AHA/
ment of the patient with antiplatelet, antithrombin, ACC Guideline for the Management of Patients with Non-
and antiischemic therapy followed by noninvasive ST-Elevation Acute Coronary Syndromes: a report of the
risk stratiication with stress testing. An “early inva- American College of Cardiology/American Heart Associa-
sive” strategy includes aggressive medical therapy and tion Task Force on Practice Guidelines. J Am Coll Cardiol.
coronary angiography with revascularization within 2014;64(24):e139–e228.
24–48 hours of hospitalization. It has been shown
that patients with higher-risk non-ST-elevation acute 11. ANSWER: C. Transthoracic echocardiogram
coronary syndrome (NSTE-ACS) have proportionally his patient has a history of mitral valve prolapse,

t
ne
more beneit from an “early invasive” strategy of man- which is a common cardiac valve lesion; myxomatous
agement with angiography within this time frame. valve degeneration leads to redundant chordal tissue
here are several validated scores available for risk and billowing of the lealets back into the left atrium

e.
stratiication of patients with NSTE-ACS, including during ventricular systole, causing the characteristic
the GRACE score and TIMI score. click–murmur complex. Consequences of mitral valve

in
his patient’s age, multiple risk factors, severe prolapse include progressive mitral regurgitation, an
angina at rest, ST depression on the electrocardio- increased risk of infective endocarditis, and chordal

ic
gram, and positive troponin all place him at high rupture leading to subacute decline. his patient pres-
risk, and an early invasive strategy of management ents with new atrial arrhythmia, heart failure, and aus-
would be favored. Fondaparinux is less favored as an
antithrombin agent in patients going to the catheter-
ed cultatory indings of severe mitral regurgitation rather
than the known click–murmur complex, all suggest-
ization laboratory, due to risk of catheter-associated ing that he has experienced a ruptured chord, lead-
sm
thrombus that is not present when unfractionated ing to subacute decompensation. An echocardiogram
heparin is used. should be ordered to conirm the diagnosis, and it is
Amsterdam EA, Wenger NK, Brindis RG, et al. 2014 AHA/ likely that surgical correction will be necessary. A sur-
ACC Guideline for the Management of Patients with geon with experience in mitral valve repair should be
ok

Non-ST-Elevation Acute Coronary Syndromes: a report consulted. In this circumstance, mitral valve repair is
of the American College of Cardiology/American Heart preferred over mitral valve replacement. Warfarin and
Association Task Force on Practice Guidelines. J Am Coll DC cardioversion could be considered for new atrial
bo

Cardiol. 2014;64(24):e139–e228. ibrillation of recent onset; however, that would not


address the underlying cause of dysrhythmia in this
10. ANSWER: D. Clopidogrel case. Similarly, thyroid testing and screening of sleep
e

his patient with cardiac risk factors presents with apnea are reasonable in an uncomplicated case of new
://

a non-ST segment myocardial infarction; the single atrial ibrillation or atrial lutter, but neither is the
severe stenosis demonstrated on angiography was not most likely diagnosis here. An exercise treadmill test
tp

amenable to percutaneous coronary intervention, and a is not recommended for a patient in decompensated
course of medical management was undertaken. Aspi- heart failure. Beta blockade would not be indicated.
rin, statin therapy, beta blockade, smoking cessation,
ht

Stout KK, Verrier ED. Acute valvular regurgitation. Circula-


and cardiac rehabilitation should be prescribed, and tion. 2009;119(25):3232–3241.
lipids and blood pressure should be treated to current
goals. In addition, a large randomized controlled trial 12. ANSWER: B. Adenosine
(he Clopidogrel in Unstable Angina to Prevent Recur- his young patient presents with supraventricu-
rent Events Trial Investigators. Efects of clopidogrel in lar tachycardia. he electrocardiogram demonstrates
addition to aspirin in patients with acute coronary syn- a narrow complex regular tachycardia at approxi-
dromes without ST-segment elevation. N Engl J Med. mately 150 beats per minute. No clear atrial activity
2001;345:494–502.) demonstrated the additive beneit is seen, and the diferential diagnosis includes AVRT,
of thienopyridine therapy added to aspirin therapy in AVNRT, atrial lutter, and ectopic atrial tachycardia.
patients with NSTEMI. Clopidogrel therapy in addi- Vagal maneuvers were an appropriate irst step but
tion to aspirin reduced a composite endpoint of car- were unsuccessful. Adenosine should be administered,
diovascular death, nonfatal myocardial infarction, and which will be both diagnostic and possibly therapeu-
stroke. hus, for this patient with NSTEMI man- tic by terminating AV node–dependent arrhythmias
aged medically, clopidogrel should be added to aspirin (AVNRT and AVRT) and unmasking occult atrial
CHAPTER 8 Cardiovascular Disease 181

activity in atrial lutter, ectopic atrial rhythm, or sinus scale, this patient would be categorized as NYHA
tachycardia. here is no need for DC cardioversion, class III. Patients with left bundle branch block have
because this patient is hemodynamically stable. Ami- delayed depolarization of the lateral left ventricle, and
odarone is not indicated in this case. Normal saline this dyssynchrony can worsen symptoms of heart fail-
would be prescribed if there were sinus tachycardia ure in patients with depressed ejection fraction. For
secondary to volume depletion. Labetalol is a drug patients with an ejection fraction <35%, a left bun-
better suited to hypertensive urgency than supraven- dle branch block with QRS duration >120 ms, and
tricular tachycardia. NYHA class III symptoms despite optimal medical
Page RL, Joglar JA, Caldwell MA, et al. 2015 ACC/AHA/HRS therapy, a biventricular pacemaker should be placed
Guideline for the Management of Adult Patients With for cardiac resynchronization. Of note, the trials of
Supraventricular Tachycardia: a report of the American Col- cardiac resynchronization therapy (CRT) enrolled
lege of Cardiology/American Heart Association Task Force patients with QRS duration >120 ms, but in sub-
on Clinical Practice Guidelines and the Heart Rhythm Soci- group analysis, the most profound beneit was seen in
ety. J Am Coll Cardiol. 2016;67(13):e27–e115. patients with QRS duration >150 ms. A recent trial
demonstrated the beneit of CRT in patients with left

t
ne
13. ANSWER: B. Administer tenecteplase and transfer bundle branch block and NHYA class II symptoms,
to a PCI-capable facility. but this indication has yet to be incorporated into
his patient is having an inferoposterolateral ST guidelines. Augmentation of diuretic therapy in this

e.
segment myocardial infarction. Timely reperfusion patient is not indicated, because there are no stigmata
therapy is paramount—“time is muscle.” he time of volume overload. Referral for LVAD placement

in
standards for reperfusion in the setting of STEMI would be premature.
include a “door-to-needle time” of 30 minutes or less Yancy CW, Jessup M, Bozkurt B, et  al. 2013 ACCF/AHA

ic
if thrombolytics are chosen as the reperfusion strat- guideline for the management of heart failure: a report of
egy and a “door-to-balloon time” of 90 minutes or the American College of Cardiology Foundation/American
less if primary PCI is chosen as the reperfusion strat-
egy. Timely PCI has been demonstrated to be supe-
ed Heart Association Task Force on Practice Guidelines. J Am
Coll Cardiol. 2013;62(16):e147–e239.
rior to timely lytic therapy with improved patency
sm
of the infarct-related artery, less reinfarction, and less 15. ANSWER: C. Routine follow-up in 2 weeks
hemorrhagic complications. If a patient with STEMI his pregnant patient presents with mild lower
presents to a non-PCI center, transfer for PCI should extremity edema, a venous hum and low murmur,
be considered if the irst medical contact to device and a third heart sound. hese are normal physical
ok

time is less than 120 minutes. First medical contact indings in pregnancy, and in the absence of symptoms
is deined as the time at which the EMTs arrive at the or other signs of cardiovascular disease, no further
bedside. In this case, there would be an unacceptable testing is indicated. In normal pregnancy, circulating
bo

delay (2.5 hours) associated with transfer, and on-site plasma volume nearly doubles, systemic vascular resis-
full-dose tenecteplase should be administered. Recent tance decreases, and cardiac output increases. hese
guidelines recommend transfer to a PCI-capable facil- changes lead to an exaggerated early diastolic illing
e

ity even if on-site thrombolysis is successful, and not phase in the cardiac cycle and increased outlow tract
://

just for failure of lytic therapy or for early reocclu- low with attendant S3 and pulmonary low murmur
sion. Intravenous beta blockers are no longer recom- in many patients. Venous hums and mammary soule
tp

mended acutely for myocardial infarction, owing to murmurs are also benign indings.
the potential to precipitate low cardiac output states. Lower extremity venous pressure increases due to
Half-dose thrombolytics prior to transfer for PCI compression of the pelvic veins lead to edema. Uri-
ht

(with or without a glycoprotein 2B/3A antagonist) is nalysis and liver function tests could be obtained if
not recommended. preeclampsia were suspected on the basis of signiicant
O’Gara PT, Kushner FG, Ascheim DD, et  al. 2013 ACCF/ edema, hypertension, and headache. An echocardio-
AHA guideline for the management of ST-elevation myo- gram could be obtained if peripartum cardiomyopa-
cardial infarction: a report of the American College of Car- thy were suspected, but the patient’s good functional
diology Foundation/American Heart Association Task Force capacity and low venous pressure make this less likely.
on Practice Guidelines. J Am Coll Cardiol. 2013;61(4):e78– Lower extremity ultrasound would be indicated to
e140. evaluate for deep vein thrombosis if leg pain or unilat-
eral edema, warmth, and redness were present, because
14. ANSWER: C. Refer for cardiac resynchronization pregnancy is a hypercoagulable state. Finally, there is
therapy (biventricular pacemaker placement). no indication of arrhythmia, and a Holter monitor
his patient with cardiomyopathy has persistent would not be indicated.
symptoms of dyspnea despite optimal medical ther- May L. Cardiac physiology of pregnancy. Compr Physiol.
apy. Using the New York Heart Association symptom 2015;5(3):1325–1344.
182 C HA P T E R 8 Cardiovascular Disease

16. ANSWER: D. Eplerenone severe stenotic heart disease were suspected, but this
his patient presented with a large myocardial patient’s murmurs are not consistent with severe aortic
infarction complicated by heart failure and depressed or mitral stenosis. he syncope is likely due to heart
ejection fraction. he EPHESUS trial demonstrated block rather than to dehydration, so luid loading
the beneit of the addition of an aldosterone antago- would not address the underlying cause.
nist to such patients. Trial inclusion criteria included Epstein AE, DiMarco JP, Ellenbogen KA, et al. 2012 ACCF/
an ejection fraction of 40% or less and a syndrome AHA/HRS focused update incorporated into the ACCF/
of heart failure after myocardial infarction. Random- AHA/HRS 2008 guidelines for device-based therapy of car-
ization occurred between 3 and 14 days after myo- diac rhythm abnormalities: a report of the American Col-
cardial infarction, and there were reductions in total lege of Cardiology Foundation/American Heart Association
mortality and cardiovascular mortality as well as in Task Force on Practice Guidelines and the Heart Rhythm
Society. J Am Coll Cardiol. 2013;61(3):e6–e75.
sudden cardiac death in the eplerenone arm. Impor-
tantly, aldosterone antagonists increase the risk of
hyperkalemia, so creatinine and potassium should be 18. ANSWER: B. Warfarin dosed for INR 2–3 indei-
monitored closely. here is no evidence that aspirin nitely

t
ne
325 mg per day has superior eicacy to aspirin 81 mg his patient presents with atrial ibrillation. Atrial
per day in this setting, and there is an increased risk of ibrillation is a common arrhythmia with aging; the
bleeding with the higher dose. A higher dose of clopi- main goals of treatment include prevention of stroke

e.
dogrel was tested in the OASIS-7 trial, which assessed and control of symptoms. With regard to the latter,
whether a higher loading dose of clopidogrel followed beta blockers and calcium channel blockers can be

in
by double-dose clopidogrel for 1 week was superior prescribed for rate control, and antiarrhythmic drugs
to standard dosing of 75 mg daily. he primary end- and atrial ibrillation ablation can be used to maintain

ic
point in this trial was negative; however, there may be sinus rhythm. With regard to prevention of stroke, the
a decreased risk of stent thrombosis with the increased irst step in choosing a therapy is assessing the base-
dosing regimen. here is no indication for ranolazine
in this patient, and given that he is euvolemic, there
ed line risk of stroke. he CHADS2 score is a scoring
system to determine stroke risk. One point is assigned
is no indication for augmented diuresis with metola- for each of a history of congestive heart failure, hyper-
sm
zone. tension, age >75 years, and diabetes, and 2 points are
Pitt B, Remme W, Zannad F, et  al. Eplerenone, a selective assigned for prior stroke or transient ischemic attack.
aldosterone blocker, in patients with left ventricular Patients with a score of 0 can be maintained on aspirin
dysfunction after myocardial infarction. N Engl J Med. or no anticoagulation. A score ≥2 would be an indi-
ok

2003;348(14):1309–1321. cation for warfarin therapy in the absence of a high


risk of bleeding. A score of 1 is indeterminate, and a
17. ANSWER: E. Refer for pacemaker placement prior discussion should be had with the patient about the
bo

to surgery. risks and beneits of anticoagulation. he CHADS2


his patient is planning elective noncardiac sur- score has been further reined with publication of the
gery. An important task in the perioperative medi- CHADS2-VASc score, which gives additional points
e

cine evaluation is an assessment for unstable cardiac for age between 65 and 75, female sex, and vascular
://

conditions that mandate treatment prior to surgery. disease. his patient’s scores based on both scoring sys-
his patient presents with syncope and evidence of tems suggest that anticoagulation should be pursued.
tp

signiicant conduction system disease on the 12-lead Warfarin is favored over dabigatran, given her age and
electrocardiogram with left bundle branch block and renal insuiciency. Even if cardioversion were pursued,
type II AV block. He has a high risk of progression anticoagulation would need to be maintained indei-
ht

to complete heart block, and a pacemaker should be nitely, given her high risk of stroke.
placed prior to elective surgery. Other unstable cardiac January CT, Wann LS, Alpert JS, et  al. 2014 AHA/ACC/
conditions that should be addressed prior to elective HRS guideline for the management of patients with atrial
noncardiac surgery include unstable cardiac isch- ibrillation: a report of the American College of Cardiol-
emic syndromes, including unstable and accelerating ogy/American Heart Association Task Force on Practice
angina, STEMI, and NSTEMI; symptomatic brady- Guidelines and the Heart Rhythm Society. Circulation.
cardia and supraventricular tachycardia; ventricular 2014;130(23):e199–e267.
tachycardia; decompensated heart failure; and critical
symptomatic stenotic valvular heart disease (aortic and 19. ANSWER: E. Ibuprofen and colchicine
mitral stenosis). Beta blockade is indicated to prevent his patient presents with acute pericarditis on the
perioperative events in patients at high risk but would basis of typical history, friction rub on physical exam-
be inappropriate in this patient with symptomatic ination, and characteristic electrocardiographic and
heart block. An echocardiogram could be ordered if echocardiographic indings. Coronary angiography
CHAPTER 8 Cardiovascular Disease 183

is not needed in such typical cases, although it is hours of randomization. Hence, this patient should be
reasonable in cases in which the distinction between transitioned from a low-potency statin to high-dose
pericarditis and ST segment myocardial infarction atorvastatin.
is unclear. he management of acute pericarditis
should include both a nonsteroidal antiinlamma- 22. ANSWER: B. Discontinue clopidogrel and proceed
tory agent and colchicine; the combination reduced with surgery, continuing aspirin through the peri-
recurrence of symptoms in the pivotal COPE trial operative period.
and was superior to NSAIDs alone. Prednisone is his patient with coronary artery disease and prior
efective for acute treatment but is associated with PCI presents prior to elective noncardiac surgery. He
an increased risk of relapses and should not be used has been maintained on dual antiplatelet therapy with
routinely. aspirin and clopidogrel since his PCI. Guidelines sug-
Lilly LS. Treatment of acute and recurrent idiopathic pericardi- gest that dual antiplatelet therapy be strictly main-
tis. Circulation. 2013;127(16):1723–1726. tained for 30 days after placement of a bare metal
stent and for 1 year after placement of a drug-eluting
20. ANSWER: D. Magnetic resonance angiography of stent to reduce the risk of acute stent thrombosis, a

t
ne
the thoracic and abdominal aorta catastrophic and life-threatening event. Given that
his pediatric patient presents with a bicuspid this patient’s stent placement was 3 years prior, it is
aortic valve, a common congenital cardiac condi- permissible to discontinue clopidogrel. Aspirin should

e.
tion. Regarding the valve itself, there is only mild be maintained perioperatively if possible, because the
regurgitation and no stenosis, so no speciic therapy inlammatory milieu of the postsurgical state is pro-

in
is indicated for aortic valve disease. Aortic root dila- thrombotic. here is emerging data that statin therapy
tion and aortic coarctation are both associated with in high-risk patients can reduce perioperative events,

ic
bicuspid aortic valve disease. his patient’s aortic root and statins should be continued in patients already
is normal in size; however, the presence of a poste- taking them. Given this patient’s excellent functional
rior bruit and a radiofemoral pulse delay suggest that
there may be concomitant coarctation. Magnetic res-
ed status (playing competitive singles tennis), there is no
role for exercise stress testing or other diagnostic test-
onance angiography would visualize the coarctation ing prior to surgery.
sm
and enable quantiication of severity. his should be Levine GN, Bates ER, Bittl JA, et al. 2016 ACC/AHA Guide-
performed prior to clearance for sports participa- line Focused Update on Duration of Dual Antiplatelet
tion. Losartan has been shown to decrease aortic root herapy in Patients With Coronary Artery Disease: a
diameter and aortic dilation associated with Marfan report of the American College of Cardiology/Ameri-
ok

and Loeys-Dietz syndromes but is not indicated in can Heart Association Task Force on Clinical Practice
this case. Guidelines: An Update of the 2011 ACCF/AHA/SCAI
Siu SC, Silversides CK. Bicuspid aortic valve disease. J Am Coll Guideline for Percutaneous Coronary Intervention, 2011
bo

Cardiol. 2010;55(25):2789–2800. ACCF/AHA Guideline for Coronary Artery Bypass Graft


Surgery, 2012 ACC/AHA/ACP/AATS/PCNA/SCAI/STS
Guideline for the Diagnosis and Management of Patients
21. ANSWER: B. Change luvastatin to high-dose ator-
e

With Stable Ischemic Heart Disease, 2013 ACCF/AHA


vastatin. Guideline for the Management of ST-Elevation Myo-
://

his patient with diabetes presents with NSTEMI cardial Infarction, 2014 AHA/ACC Guideline for the
and underwent successful PCI. Several medications Management of Patients With Non-ST-Elevation Acute
tp

have been shown to improve outcome in patients with Coronary Syndromes, and 2014 ACC/AHA Guideline on
diabetes and NSTEMI. Compared with clopidogrel, Perioperative Cardiovascular Evaluation and Management
prasugrel has been shown to improve a composite end- of Patients Undergoing Noncardiac Surgery. Circulation.
ht

point of cardiovascular death and repeat myocardial 2016;134(10):e123–e155.


infarction, with a greater beneit in patients with dia-
betes. here is no advantage to aspirin 325 mg per day 23. ANSWER: C. Prescribe isosorbide mononitrate.
over 81 mg per day at a higher cost of greater bleeding. his patient presents with typical angina that
Given that this patient has no residual angina, there has a stable pattern; that is, the angina is predict-
is no role for isosorbide mononitrate or ranolazine. ably evoked by a stable level of exertion and readily
High-dose atorvastatin therapy has been shown to be relieved with rest. His angiogram reveals nonob-
superior to moderate-dose statin therapy when admin- structive right coronary artery and left circumlex
istered early to patients with acute coronary syndrome artery disease and a 70% stenosis in a small branch
in the PROVE-IT trial. Patients receiving atorvastatin of the left anterior descending artery. In addition,
80 mg had improvement in a composite endpoint, FFR across the right coronary artery lesion indicates
including myocardial infarction and all-cause mortal- a non–ischemia-producing stenosis. Revasculariza-
ity, with the initial improvements beginning within 24 tion, either with PCI and stent placement or bypass
184 C HA P T E R 8 Cardiovascular Disease

surgery, in this circumstance does not provide any 25. ANSWER: D. Hypertrophic obstructive cardiomy-
survival advantage over medical therapy, and as opathy
such, a trial of medical therapy should be attempted. his patient presents with dizziness evoked by
Medical therapies for patients with stable angina squatting to standing and dyspnea. he examination
should include agents to decrease myocardial oxy- reveals a harsh systolic murmur that becomes louder
gen demand, including beta blockers, nitrates, and with maneuvers that decrease preload, a cardinal
calcium channel blockers. his patient’s heart rate physical examination inding present in hypertrophic
is well controlled, and increasing the dose of beta obstructive cardiomyopathy. In this condition, there
blockers is unlikely to be helpful. A long-acting is left ventricular outlow tract obstruction caused by
nitrate would be a good choice to decrease myo- a hypertrophied interventricular septum. Decreased
cardial oxygen consumption by decreasing pre- preload decreases left ventricular cavity size and wors-
load and wall stress. Ranolazine can be considered ens obstruction with an attendant increase in the
as an antianginal agent but typically is added after loudness of the murmur. he obstruction can cause
nitrates, beta blockers, and calcium channel block- a “Venturi efect,” leading to systolic anterior motion
ers are maximized. here is no indication for dual of the mitral valve and mitral regurgitation, as is also

t
ne
antiplatelet therapy with aspirin and clopidogrel in appreciated in this patient. An echocardiogram should
this patient. be ordered to conirm the diagnosis. Dilated cardio-
Fihn SD, Blankenship JC, Alexander KP, et  al. 2014 ACC/ myopathy can present with a third heart sound and

e.
AHA/AATS/PCNA/SCAI/STS focused update of the murmurs of mitral and tricuspid regurgitation. Mitral
guideline for the diagnosis and management of patients stenosis presents with a diastolic rumbling murmur

in
with stable ischemic heart disease: a report of the American at the apex. he harsh systolic murmur of aortic ste-
College of Cardiology/American Heart Association Task nosis would be expected to decrease with the Valsalva
Force on Practice Guidelines, and the American Associa-

ic
maneuver. An atrial septal defect presents with ixed
tion for horacic Surgery, Preventive Cardiovascular Nurses splitting of the second heart sound and murmurs of
Association, Society for Cardiovascular Angiography and ed tricuspid regurgitation as well as increased pulmonary
Interventions, and Society of horacic Surgeons. J Am Coll
Cardiol. 2014;64(18):1929–1949.
artery low.
Gersh BJ, Maron BJ, Bonow RO, et  al. 2011 ACCF/AHA
sm
Guideline for the Diagnosis and Treatment of Hypertro-
24. ANSWER: C. Refer for implantable cardioverter- phic Cardiomyopathy: a report of the American College
deibrillator. of Cardiology Foundation/American Heart Association
his patient has nonischemic dilated cardiomy- Task Force on Practice Guidelines. Developed in collabo-
ok

opathy, likely familial, and is doing well overall on ration with the American Association for horacic Surgery,
appropriate medical therapy with good functional sta- American Society of Echocardiography, American Society
of Nuclear Cardiology, Heart Failure Society of America,
tus. For patients with systolic dysfunction secondary
bo

Heart Rhythm Society, Society for Cardiovascular Angiog-


to nonischemic cardiomyopathy, an ejection fraction raphy and Interventions, and Society of horacic Surgeons.
less than 35% despite optimal medical therapy, and J Am Coll Cardiol. 2011;58(25):e212–e260.
functional status that is New York Heart Association
e

class II–III, placement of an implantable cardioverter-


://

deibrillator is indicated for primary prevention of 26. ANSWER: C. Transesophageal echocardiogram


sudden cardiac death. he pivotal SCD-HeFT trial his patient presents with Staphylococcus aureus bac-
tp

demonstrated beneit of an implantable cardioverter- teremia and a vegetation on the aortic valve consistent
deibrillator in such patients. his patient has a narrow with bacterial endocarditis. Appropriate antibiotic
therapy for MSSA was initiated with nafcillin. Van-
ht

QRS and as such has no indication for CRT, which


could be considered if the QRS were prolonged with comycin is appropriate therapy if MRSA is suspected;
a left bundle branch block pattern. he venous pres- however, nafcillin has bactericidal activity against
sure is low, and there would be no beneit to adding MSSA and is a preferred agent. Despite therapy,
an additional diuretic metolazone. Given his good however, the patient develops evidence of new heart
functional status, referral for transplant would be pre- block. he aortic valve and aortic root are in close ana-
mature. Digoxin improves symptoms of heart failure tomic proximity to the bundle of His, and an aortic
without an efect on mortality and can be considered root abscess complicating bacterial endocarditis can
for patients with persistent symptoms despite optimal progress to complete heart block. Serial monitoring
medical therapy. of AV conduction in patients with bacterial endocar-
Yancy CW, Jessup M, Bozkurt B, et  al. 2013 ACCF/AHA ditis is mandatory to identify this complication early.
guideline for the management of heart failure: a report of Transesophageal echo should be performed to conirm
the American College of Cardiology Foundation/American the suspected aortic root abscess and provide ana-
Heart Association Task Force on Practice Guidelines. J Am tomic information to guide surgical drainage. Surgi-
Coll Cardiol. 2013;62(16):e147–e239. cal intervention for bacterial endocarditis is indicated
CHAPTER 8 Cardiovascular Disease 185

for abscess formation, severe valvular destruction with therapy is mandatory. To treat PAD that is moder-
heart failure, failure to clear the infection, and per- ate and not limb threatening, an exercise program
sistently positive blood cultures, as well as for large has been demonstrated to increase pain-free walking
vegetation with signiicant risk for embolism. Blood distance and is an efective initial therapy. he data
cultures can be incubated for a prolonged period to for eicacy of pentoxifylline are marginal; cilostazol
increase yield if endocarditis associated with the fastid- can be efective in increasing walking distance, but
ious HACEK organisms is suspected; similarly, serolo- it should be avoided in patients with heart failure.
gies for C. burnetii can be requested if Q fever, a cause Cilostazol can be used in conjunction with an exer-
of culture-negative endocarditis, is suspected. Implan- cise program. Detailed anatomic imaging should be
tation of a pacemaker in this patient with active infec- pursued if limb-threatening ischemia is present or if
tion would not be the treatment of choice. medical therapy fails and revascularization is being
Baddour LM, Wilson WR, Bayer AS, et al. Infective endocardi- considered; referral for revascularization would be
tis in adults: diagnosis, antimicrobial therapy, and manage- premature at this time.
ment of complications: a scientiic statement for healthcare Gerhard-Herman MD, Gornik HL, Barrett C, et  al. 2016
professionals from the American Heart Association. Circu- AHA/ACC Guideline on the Management of Patients With

t
ne
lation. 2015;132(15):1435–1486. Lower Extremity Peripheral Artery Disease: a report of the
American College of Cardiology/American Heart Associa-
27. ANSWER: D. Nuclear stress test with regadenoson tion Task Force on Clinical Practice Guidelines. Circulation.
2017;135(12):e726–e779.

e.
his woman presents with a chest pain syndrome.
She has an intermediate pretest probability of coro-

in
nary artery disease based on her age and the charac- 29. ANSWER: B. Aortic valve replacement
teristics of her pain. Hence, a form of stress testing is his patient presents with bicuspid aortic valve

ic
appropriate rather than reassurance. Her severe knee and severe aortic regurgitation (AR). Patients with AR
osteoarthritis and lumbar disc disease make it unlikely should be referred for surgery if symptoms are present,
that exercise as a stressor will yield diagnostic informa-
tion; that is, it is unlikely she could exercise on a tread-
ed as in this case. If patients have asymptomatic severe
AR, they should undergo valve replacement when the
mill to reach her target heart rate. A dobutamine stress ejection fraction begins to fall or when the left ven-
sm
echocardiogram could be considered, but the presence tricle begins to dilate; repair at this stage forestalls
of left bundle branch block makes image interpreta- the eventual development of dilated cardiomyopathy
tion in this setting more diicult, although not impos- associated with chronic volume loading of the left ven-
sible. Nuclear stress testing with exercise in patients tricle. Bicuspid valve can be associated with aortic root
ok

with left bundle branch block can produce a perfusion dilation that in some instances mandates concomitant
abnormality in the septum, and therefore pharmaco- aortic root repair, but the aortic root diameter in this
logic stress testing is preferred in these patients. he case is normal. Trials of afterload reduction in aortic
bo

best diagnostic test for this patient who cannot exer- insuiciency have been negative, and medical therapy
cise with a left bundle branch block is a pharmacologic speciically for AR is not indicated. Serial echocar-
nuclear stress test. diography would not be appropriate in this patient
e

Wolk MJ, Bailey SR, Doherty JU, et  al. ACCF/AHA/ASE/ with symptomatic severe AR. Finally, the brisk carotid
://

ASNC/HFSA/HRS/SCAI/SCCT/SCMR/STS 2013 mul- impulses are an expected inding in severe AR and do


timodality appropriate use criteria for the detection and not mandate a carotid imaging study.
risk assessment of stable ischemic heart disease: a report of
tp

Nishinura RA, Otto CM, Bonow RO, et al. 2017 AHA/ACC


the American College of Cardiology Foundation Appropri-
Focused Update of the 2014 AHA/ACC Guideline for the
ate Use Criteria Task Force, American Heart Association,
Management of Patients With Valvular Heart Disease: a
ht

American Society of Echocardiography, American Society


report of the American College of Cardiology/American
of Nuclear Cardiology, Heart Failure Society of America,
Heart Association Task Force on Clinical Practice Guide-
Heart Rhythm Society, Society for Cardiovascular Angiog-
lines. Circulation. 2017;135(25):e1159–e1195.
raphy and Interventions, Society of Cardiovascular Com-
puted Tomography, Society for Cardiovascular Magnetic
Resonance, and Society of horacic Surgeons. J Am Coll 30. ANSWER: D. Obtain detailed over-the-counter
Cardiol. 2014;63(4):380–406. medication use history.
his patient presents with apparent resistant hyper-
28. ANSWER: A. Supervised exercise program tension, deined as blood pressure that is uncontrolled
his patient with tobacco use history presents despite use of three medications at good doses, one
with typical claudication and ankle-brachial index of which is a diuretic. he irst step in management
conirming moderate peripheral arterial disease of resistant hypertension is to conirm the diagnosis,
(PAD). Patients with PAD are also at risk for ath- and a home blood pressure diary or 24-hour ambula-
erosclerosis of other arterial beds, so an aggressive tory blood pressure monitoring can be used for con-
regimen of antihypertensive, antiplatelet, and lipid irmation. Next, ensure good medication compliance
186 C HA P T E R 8 Cardiovascular Disease

because noncompliance is a frequent cause of appar- versus open repair. Prior to repair, patients should be
ently resistant hypertension. Medications that can optimized medically, although stress testing should
contribute to hypertension, including possible NSAID be recommended only if it would afect manage-
use in this case as well as sympathomimetics, should be ment and lead to preoperative coronary angiogra-
minimized. Alcohol intake and obstructive sleep apnea phy and revascularization. AAA between 4 and 5.5
also can contribute to poorly controlled hypertension. cm should be followed with an imaging study every
Secondary causes of hypertension can be evalu- 6–12 months; AAA smaller than 4 cm can be fol-
ated, including Cushing syndrome, hyperaldosteron- lowed at a lesser interval, every 1–3 years unless new
ism, thyroid disease, and renal artery disease, although symptoms emerge. AAA is considered a coronary risk
these conditions are less common than the aforemen- equivalent, and all patients should undergo aggres-
tioned causes, which should be excluded irst. sive therapy of hypertension and dyslipidemia as well
Braam B, Taler SJ, Rahman M, et  al. Recognition and man- as smoking cessation.
agement of resistant hypertension. Clin J Am Soc Nephrol. Rooke TW, Hirsch AT, Misra S, et  al. 2011 ACCF/AHA
2017;12(3):524–535. Focused Update of the Guideline for the Management of
Patients With Peripheral Artery Disease (updating the 2005

t
ne
31. ANSWER: D. Refer for coronary artery bypass guideline): a report of the American College of Cardiol-
grafting of the LAD artery and RCA. ogy Foundation/American Heart Association Task Force
his patient with diabetes and normal left ven- on Practice Guidelines. J Am Coll Cardiol. 2011;58(19):
2020–2045.

e.
tricular function presents with obstructive coronary
stenoses in two vessels, one of which is the proximal
33. ANSWER: C. Add lisinopril.

in
left anterior descending artery. Coronary artery bypass
grafting has been demonstrated to be superior to his patient with prior myocardial infarction
now presents with systolic heart failure. In addition

ic
medical therapy in such patients and is a class I rec-
ommendation in current guidelines. PCI of one or to a diagnostic evaluation to determine underly-
both lesions could be considered if the patient were a
poor candidate for surgery, in which case physiologic
ed ing contributors to systolic dysfunction, appropriate
pharmacotherapy should be provided. he calcium
information, such as with stress myocardial perfu- channel blocker may impair LV function and should
sm
sion imaging (MPI) or FFR, about the extent and be stopped. First-line therapy for patients with systolic
anatomic distribution of ischemia may be helpful to dysfunction include blockade of the renin-angioten-
guide revascularization. Regardless of the method of sin-aldosterone system with an angiotensin-converting
revascularization, aggressive medical therapy should be enzyme (ACE) inhibitor or an angiotensin receptor
ok

instituted with good control of blood pressure and lip- blocker (ARB) coupled with a beta blocker (carvedilol,
ids as well as antiplatelet therapy with aspirin. Other extended-release metoprolol, bisoprolol, and nebivolol
class I indications for coronary artery bypass surgery are the agents that have been evaluated in randomized
bo

include patients with stenoses greater than 70% in controlled trials for use in heart failure). his patient
three coronary vessels and those patients with left has been maintained on beta blockers for coronary
main coronary artery stenosis greater than 50%. In a disease, and lisinopril should be added and uptitrated
e

subset of patients with anatomically amenable coro- as able while following blood pressure, serum potas-
://

nary artery disease who are at high risk for bypass sur- sium, and renal function. Hydralazine and nitrates can
gery, PCI could be considered. be used in patients intolerant of ACE and ARB and
tp

Hillis LD, Smith PK, Anderson JL, et  al. 2011 ACCF/AHA also have been shown to have incremental beneit in
guideline for coronary artery bypass graft surgery: execu- African American patients with heart failure, but they
tive summary: a report of the American College of Car- would not be a irst-line therapy in this case. here is
ht

diology Foundation/American Heart Association Task no indication for dual antiplatelet therapy with aspirin
Force on Practice Guidelines. J horac Cardiovasc Surg. and clopidogrel. Spironolactone should be prescribed
2012;143(1):4–34. for patients with heart failure with NYHA class II
or greater symptoms despite adequate doses of ACE
32. ANSWER: D. Repeat abdominal ultrasound in 6 inhibitors and beta blockers and normal potassium
months. handing; there is a survival beneit with spironolactone
his patient presents with an asymptomatic treatment in this population. Digoxin can improve
abdominal aortic aneurysm (AAA) measuring 4.1 symptoms but ofers no mortality beneit, and other
cm. AAA should be repaired electively when greater agents should be optimized irst.
than 5.5 cm or when expanding greater than 0.5 cm Yancy CW, Jessup M, Bozkurt B, et  al. 2013 ACCF/AHA
in 6 months or 1.0 cm in 1 year. If repair is planned, guideline for the management of heart failure: a report of
detailed anatomic information should be obtained the American College of Cardiology Foundation/American
with an imaging study such as a CT angiogram, Heart Association Task Force on Practice Guidelines. J Am
which can help guide the choice of endovascular Coll Cardiol. 2013;62(16):e147–e239.
CHAPTER 8 Cardiovascular Disease 187

34. ANSWER: C. Exercise treadmill test requiring insulin, and a history of cerebrovascular
his patient with multiple cardiac risk factors disease. Other risk factors include a history of heart
presents with substernal chest pain evoked by exer- failure and a history of coronary artery disease. Of
tion and relieved with rest. In evaluation of the note, when left anterior fascicular block is present,
patient with chest pain, the pretest probability of Q waves must be visualized from V1 through V3 to
coronary disease should be assessed. he patient’s diagnose septal myocardial infarction. For patients
age and sex as well as the characteristics of the pain with three of these risk factors, beta blockers should
inform the pretest probability. In this case, the pre- be prescribed and uptitrated over the weeks prior
test probability of coronary artery disease is consid- to the perioperative period for a goal heart rate of
ered intermediate to high, and further evaluation 55–65 beats per minute. Stress testing and coro-
with stress testing is appropriate. In patients where nary imaging should be pursued only as part of
the pretest probability of coronary disease is low the preoperative evaluation if it would afect man-
(<10%), a stress test that is positive likely represents agement—that is, if revascularization with coro-
a false-positive result. For patients with an interme- nary artery bypass grafting or PCI would follow if
diate pretest probability of coronary disease, stress needed, and no trial to date has demonstrated the

t
ne
testing is appropriate because the test characteristics beneit of routine revascularization prior to noncar-
meaningfully afect the posttest likelihood of disease. diac surgery. Given this patient’s high risk, however,
Research studies have yielded tools that can assist in proceeding to surgery without beta blockade would

e.
estimating the pretest probability of coronary disease not be appropriate.
(Sox HC Jr, Hickam DH, Marton KI, et  al. Am J

in
Med. 1990;89:7–14). he coronary artery calcium 36. ANSWER: D. Referral for surgical removal
score can be used to reine risk prediction in the his young patient presents with a cardiac mass

ic
primary prevention setting for patients at intermedi- that was incidentally discovered. Causes of intracar-
ate pretest risk of coronary disease, but at present it diac masses include thrombi, infectious and non-
should not be used for patients with active cardiac
symptoms. Proceeding directly to coronary angiog-
ed infectious vegetations, and cardiac tumors. Given
lack of fever or other symptoms and lack of comor-
raphy would be premature. Imaging with cardiac bidities, this is unlikely to represent a thrombus or
sm
MRI can be useful in evaluation of pericardial dis- vegetation. he description of a pedunculated mass
ease of myopathic diseases; however, a stress test is attached to the interatrial septum by a thin stalk is
more appropriate for this patient. Finally, an exercise consistent with atrial myxoma, which is the most
test yields more hemodynamic data than a pharma- common primary tumor of the heart. Myxomas
ok

cologic stress test and should be the irst-line assess- should be removed surgically due to a risk of embo-
ment in a patient who can exercise. lism; atrial myxomas can also secrete cytokines,
Wolk MJ, Bailey SR, Doherty JU, et al. ACCF/AHA/ASE/ leading to constitutional symptoms and paraim-
bo

ASNC/HFSA/HRS/SCAI/SCCT/SCMR/STS 2013 mune phenomena. Prognosis is good with surgical


multimodality appropriate use criteria for the detection removal.
and risk assessment of stable ischemic heart disease: a
e

report of the American College of Cardiology Foundation


37. ANSWER: E. Referral for pacemaker placement
Appropriate Use Criteria Task Force, American Heart
://

Association, American Society of Echocardiography,


his patient presents with symptomatic sinus bra-
American Society of Nuclear Cardiology, Heart Failure dycardia and failure to augment the heart rate with
tp

Society of America, Heart Rhythm Society, Society for exercise. Causes of sinus bradycardia include hypo-
Cardiovascular Angiography and Interventions, Society thyroidism, medications, and degenerative conduc-
tion system disease—the sick sinus syndrome. Given
ht

of Cardiovascular Computed Tomography, Society for


Cardiovascular Magnetic Resonance, and Society of ho- that thyroid function studies are normal and the
racic Surgeons. J Am Coll Cardiol. 2014;63(4):380–406. patient is on no agents that would depress sinus node
function, the most likely diagnosis is sick sinus syn-
35. ANSWER: E. Prescribe metoprolol. drome with chronotropic incompetence. Pacemaker
his patient presents for preoperative risk stratii- implantation is indicated for the symptomatic brady-
cation prior to lumbar laminectomy, an intermedi- cardia. here is no indication for further conirmation
ate-risk noncardiac surgical procedure. here is no and correlation of symptoms with arrhythmia using a
evidence of unstable cardiac conditions such as acute 48-hour Holter monitor. Event monitors are best for
coronary syndrome, tachy- or bradydysrhythmia, or diagnosis of paroxysmal symptoms hypothesized to
severe stenotic valvular disease that would preclude be related to dysrhythmia. here are no symptoms to
surgery, and the surgery is not emergent. he patient’s suggest Lyme disease. Sick sinus syndrome is a clini-
functional capacity is limited secondary to back pain. cal diagnosis, and invasive measurement of the sinus
She has multiple risk factors for a perioperative coro- node recovery time (a marker of sinus node function)
nary event, including creatinine >2.0 mg/dL, diabetes is rarely performed.
188 C HA P T E R 8 Cardiovascular Disease

38. ANSWER: C. Give the patient a home blood pres- of death. He is at very high risk for adverse outcomes
sure diary, measure TSH, and repeat oice blood after surgery due to his prior chest radiation and prior
pressure in 3 months. cardiac surgery. Such high-risk patients with a “hos-
his patient presents with a single elevated blood tile chest” were enrolled in the inoperable arm of the
pressure reading in the oice in the setting of a recent PARTNER trial, which randomized patients to trans-
normal blood pressure reading outside the oice. catheter aortic valve replacement (TAVR) or standard
he funduscopic examination reveals no evidence therapy. Compared with standard therapy, which often
of chronic hypertension; these facts should raise the included balloon valvuloplasty, TAVR reduced the
question whether white coat hypertension is present. rates of death and hospitalization as well as improved
A home blood pressure diary, or ambulatory 24-hour symptoms and valvular hemodynamics over a follow-
blood pressure measurement if available, may help up period of 2 years.
clarify whether sustained hypertension is present, and Makkar RR, Fontana GP, Jilaihawi H, et al. Transcatheter aor-
oice blood pressure should be repeated at a follow- tic-valve replacement for inoperable severe aortic stenosis. N
up visit. Given the abnormal thyroid on examination, Engl J Med. 2012;366(18):1696–1704.
hyperthyroidism as a cause of elevated blood pressure

t
ne
should be excluded. Patients with white coat hyper- 41. ANSWER: D. Close follow-up with a transthoracic
tension have a risk of developing sustained hyperten- echocardiogram every 6–12 months
sion and should be followed. here are no symptoms he patient has severe mitral regurgitation second-

e.
to suggest pheochromocytoma. White coat hyperten- ary to mitral valve prolapse and a lail mitral lealet.
sion should be excluded before further evaluation for She is otherwise asymptomatic and has no evidence

in
hyperaldosteronism and renal artery stenosis is under- of heart failure on examination. Her echocardiogram
taken. Empiric therapy is not indicated, given that shows preserved left ventricular function and dimen-

ic
only a single elevated blood pressure has been docu- sions. he class I indications for mitral valve sur-
mented. gery for primary severe mitral regurgitation include

39. ANSWER: B. Refer for PCI of the obtuse marginal


ed (1)  symptomatic severe mitral regurgitation in
patients with left ventricular ejection fraction >30%
stenosis. and (2) asymptomatic patients with severe mitral
sm
his patient presented originally with chest pain regurgitation and left ventricular dysfunction (ejec-
and was diagnosed with coronary disease with moder- tion fraction 30%–60%, left ventricular end-systolic
ate middle LAD artery stenosis and obstructive dis- dimension ≥40 mm). his patient does not meet
ease in the left circumlex system. Despite aggressive either of the criteria. Mitral valve repair is preferred
ok

medical therapy, lifestyle-limiting angina persists; to replacement for mitral valve prolapse. MitraClip
because of refractory symptoms, revascularization is reserved for patients who have prohibitive surgical
should be pursued. Given that only a single ves- risk. Vasodilators are not indicated in the absence of
bo

sel is afected with greater than 70% stenosis, PCI elevated blood pressure. he most appropriate next
would be favored over bypass surgery. Increasing the step is close follow-up with referral for surgery once
dose of metoprolol would be limited by heart rate, surgical criteria are met.
e

and increase of nitrates would be limited by blood Nishinura RA, Otto CM, Bonow RO, et al. 2017 AHA/ACC
://

pressure. Focused Update of the 2014 AHA/ACC Guideline for the


Cardiac rehabilitation could be considered, but this Management of Patients With Valvular Heart Disease: a
tp

patient is already it and active, and conditioning is report of the American College of Cardiology/American
unlikely to lead to abatement of symptoms. Heart Association Task Force on Clinical Practice Guide-
lines. Circulation. 2017;135(25):e1159–e1195.
ht

Fihn SD, Blankenship JC, Alexander KP, et  al. 2014 ACC/
AHA/AATS/PCNA/SCAI/STS focused update of the
guideline for the diagnosis and management of patients 42. ANSWER: C. Stop lisinopril and start sacubitril/
with stable ischemic heart disease: a report of the Ameri- valsartan 49/51 mg twice daily after 36-hour wash-
can College of Cardiology/American Heart Association out.
Task Force on Practice Guidelines, and the American Asso- he PARADIGM-HF trial randomized patients
ciation for horacic Surgery, Preventive Cardiovascular with symptomatic (NYHA class II-IV) heart failure
Nurses Association, Society for Cardiovascular Angiogra- and a left ventricular ejection fraction <40% to either
phy and Interventions, and Society of horacic Surgeons.
the angiotensin receptor-neprilysin inhibitor (ARNI),
J Am Coll Cardiol. 2014;64(18):1929–1949.
sacubitril/valsartan, or enalapril in addition to recom-
mended heart failure therapy. In this trial, sacubitril/
40. ANSWER: D. Transcatheter aortic valve replacement valsartan reduced the composite endpoint of cardio-
his patient presents with radiation-induced severe vascular death and heart failure hospitalizations by
symptomatic aortic stenosis. If left untreated, symp- 20% compared with enalapril alone over a median fol-
tomatic aortic stenosis is associated with a high rate low-up of 27 months. On the basis of these results, the
CHAPTER 8 Cardiovascular Disease 189

AHA/ACC/HFSA issued a guideline update recom- high risk and beneit from systemic thrombolytic ther-
mending that in patients with symptomatic systolic apy. Catheter-directed thrombolysis can be consid-
heart failure who are able to tolerate an angiotensin- ered in patients with hemodynamic decompensation
converting enzyme inhibitor or angiotensin receptor and high bleeding risk with systemic thrombolysis.
blocker, the angiotensin-converting enzyme inhibitor Patients with submassive pulmonary embolus pres-
or angiotensin receptor blocker should be replaced ent with hemodynamic stability but with right ven-
with an angiotensin receptor-neprilysin inhibitor to tricular dysfunction and positive biomarkers. Systemic
reduce morbidity and mortality. Because of the risk thrombolysis has an unfavorable risk-to-beneit ratio
of angioedema, patients should not start taking the in these patients. Patients with hemodynamic stability
angiotensin receptor-neprilysin inhibitor for at least and negative biomarkers are considered low risk and
36 hours after their last dose of angiotensin-convert- should be treated with intravenous heparin followed
ing enzyme inhibitor or angiotensin receptor blocker. by chronic anticoagulation with warfarin or direct
he patient is on a good dose of lisinopril and there- oral anticoagulants for at least 3 months. Routine use
fore would tolerate initiation of an ARNI, which is of inferior vena cava ilters is not recommended in
preferred to increasing lisinopril. he patient does patients who can receive anticoagulation.

t
ne
not have increased intravascular volume, and there- Konstantinides SV, Barco S, Lankeit M, Meyer G. Manage-
fore torsemide does not need to be increased. Digoxin ment of pulmonary embolism: an update. J Am Coll Car-
has been associated with no mortality beneit in heart diol. 2016;67(8):976–990.

e.
failure.
McMurray JJV, Packer M, Desai AS, et al. Angiotensin-neprily- 45. ANSWER: E. Ketoconazole
sin inhibition versus enalapril in heart failure. N Engl J Med.

in
Absorption of apixaban is mediated by P-glycopro-
2014;371(11):993–1004. tein (P-gp). P-gp inhibitors can increase absorption

ic
of apixaban and thus apixaban levels. Metabolism of
43. ANSWER: B. Add digoxin apixaban is mediated by CYP3A4. CYP3A4 inhibi-
he patient is complaining of postural light-
headedness and has borderline blood pressure and
ed tors can decrease the metabolism of apixaban and
increase levels. Drugs that are inhibitors of both
no evidence of jugular venous distention. herefore P-gp and CYP3A4 are therefore more likely to inter-
sm
increasing her furosemide dose does not seem appro- act with apixaban and increase the risk of bleeding
priate. She is in atrial ibrillation, and improved rate than drugs that inhibit either P-gp or CYP3A4 alone.
control may help her symptoms. She does not have Ketoconazole is an inhibitor of P-gp and a strong
adequate blood pressure for uptitration of metopro- inhibitor of CYP3A4. Sotalol is not metabolized
ok

lol, and therefore adding digoxin seems appropriate. and therefore does not interfere with either P-gp or
In the PARADIGM-HF trial, patients had to toler- CYP3A4. Amiodarone and verapamil are inhibitors
ate a minimum dose of enalapril 10 mg daily prior of p-GP and moderate inhibitors of CYP3A4; they
bo

to randomization, owing to the vasodilatory efects can be used with apixaban in patients with normal
of sacubitril/valsartan. his patient is on a very low renal function with a mild increase in apixaban levels
dose of lisinopril with marginal blood pressure and but should not be used in patients with creatinine
e

therefore would not be an ideal candidate to switch clearance <30 ml/min, age >80, or body weight <60
://

to sacubitril/valsartan. Ivabradine lowers the heart kg. Phenytoin is an inducer of CYP3A4 and reduces
rate by inhibiting the funny channel in the sinoatrial apixaban levels.
tp

node. In the SHIFT trial, patients with symptom- Amin A, Deitelzweig S. A case-based approach to imple-
atic heart failure and left ventricular ejection fraction menting guidelines for stroke prevention in patients
≤35% who were in sinus rhythm with a heart rate with atrial ibrillation: balancing the risks and beneits.
ht

≥70 beats per minute despite maximally tolerated hromb J. 2015;13:29.


beta blockers were randomized to receive ivabradine
or placebo on top of guideline-based medical ther- 46. ANSWER: D. Acute coronary dissection
apy. Ivabradine lowered heart rate and reduced heart Acute coronary dissection is a rare but well rec-
failure hospitalizations compared with placebo. ognized complication of pregnancy that occurs
his patient is not a good candidate for ivabradine, most commonly in the third trimester or in the
because she is in atrial ibrillation. irst 6 months after delivery. It occurs primarily in
Swedberg K, Komajda M, Böhm M, et al. Ivabradine and out- women >30 years of age and without traditional risk
comes in chronic heart failure (SHIFT): a randomised pla- factors for atherosclerosis. Chest pain and shortness
cebo-controlled study. Lancet. 2010;376(9744):875–885. of breath are common symptoms, and the major-
ity of patients present with an anterior ST elevation
44. ANSWER: B. Intravenous heparin myocardial infarction. Acute coronary dissection
Patients who present with shock or persistent hypoten- is associated with a high rate of maternal and fetal
sion from massive pulmonary embolus are considered mortality, and conservative management is favored
190 C HA P T E R 8 Cardiovascular Disease

in low-risk patients. Pregnancy is a hypercoagulable Interdisciplinary Council on Quality of Care and Outcomes
state, and the risk of acute pulmonary embolism Research: endorsed by the American Academy of Pediatrics.
is increased during pregnancy and in the postpar- Circulation. 2009;119(11):1541–1551.
tum period. his patient’s EKG is not consistent
with an acute pulmonary embolus. Amniotic luid 48. ANSWER: C. Ankle brachial index
embolism is a life-threatening complication of preg- he current guidelines recommend peripheral
nancy caused by the entry of amniotic luid into vascular disease screening for risk stratiication,
the maternal circulation during labor or in the early medical management, and reduction of cardiovas-
postpartum period. It is characterized by cardio- cular events in asymptomatic high-risk patients (age
pulmonary collapse and disseminated intravascular ≥70 years, age ≥50 years with a history of smoking
coagulation (DIC) and is associated with increased or diabetes, exertional leg symptoms, or nonheal-
maternal mortality. he patient’s presentation is not ing wounds). Screening for abdominal aortic aneu-
consistent with amniotic luid embolism. Peripar- rysms (AAAs) is recommended in men >60 years
tum cardiomyopathy is left ventricular dysfunction with a irst-degree relative with an AAA or aged
that occurs within the last month of pregnancy and 65–75 years with a history of smoking. his patient

t
ne
the irst 5 months after delivery, without any other is asymptomatic and at low risk for cardiovascular
identiiable cause. It is associated with shortness of events (Framingham risk score <10%). herefore
breath but does not have ST elevations on an EKG. routine exercise stress testing or coronary artery cal-

e.
he most common pericardial complication in cium scoring is not indicated. he patient’s pooled
pregnancy is hydropericardium, which is character- cardiovascular risk score is >7.5%, and she would

in
ized by a small, asymptomatic pericardial efusion beneit from statin therapy for primary prevention
that occurs in 40% of women in the third trimester of cardiovascular events.

ic
of pregnancy. his patient’s presentation and EKG Gof DC Jr, Lloyd-Jones DM, Bennett G, et  al. 2013 ACC/
are not consistent with acute pericarditis. AHA guideline on the assessment of cardiovascular risk: a
Havakuk O, Goland S, Mehra A, Elkayam U. Pregnancy
and the risk of spontaneous coronary artery dissection: an
ed report of the American College of Cardiology/American
Heart Association Task Force on Practice Guidelines. J Am
analysis of 120 contemporary cases. Circ Cardiovasc Interv. Coll Cardiol. 2014;63(25 Pt B):2935–2959.
2017;10(3):e004941.
sm

49. ANSWER: A. Renal artery Doppler study


47. ANSWER: D. Start oral penicillin V 250 mg twice In a young patient with severe hypertension, one
daily. must evaluate secondary causes of hypertension. he
ok

he patient’s examination is consistent with rheu- most likely etiology in a young woman is ibromuscu-
matic mitral stenosis. he severity of her mitral ste- lar dysplasia. Initial screening with renal Doppler can
nosis is mild based on her examination, and she is reveal increased velocities, and the next step would be
bo

asymptomatic. he current guidelines recommend evaluation with computed tomographic angiography


antibiotic prophylaxis for secondary prevention in or abdominal magnetic resonance angiography. Poly-
patients with rheumatic fever and carditis. For patients
e

somnography can lead to the detection of obstructive


with residual valvular disease, penicillin prophylaxis is sleep apnea, which can be a cause of hypertension, but
://

recommended for 10 years from the last episode of this patient does not give a history that is consistent
acute rheumatic fever or until the age of 40. Antibiotic with obstructive sleep apnea. Pheochromocytoma
prophylaxis is no longer recommended for patients
tp

is unlikely, given the lack of lushing, palpitations,


with mitral stenosis. Diuretic therapy is recommended or diaphoresis. Hyperaldosteronism is also unlikely,
for patients with symptoms of heart failure. Oral anti-
ht

given normal electrolytes. A transthoracic echocardio-


coagulation is indicated in the presence of atrial ibril- gram may show left ventricular hypertrophy, but she
lation or thromboembolic events. Beta blockers are is unlikely to have coarctation of the aorta, given the
recommended for rate control in patients with atrial lack of a murmur and no delayed pulse in the lower
ibrillation and can be considered in symptomatic extremities.
patients with sinus rhythm to increase diastolic illing Vongpatanasin W. Resistant hypertension: a review of diagnosis
time and reduce left atrial pressures. and management. JAMA. 2014:311(21):2216–2224.
Gerber MA, Baltimore RS, Eaton CB, et  al. Prevention of
rheumatic fever and diagnosis and treatment of acute Strep-
tococcal pharyngitis: a scientiic statement from the Ameri- Acknowledgment
can Heart Association Rheumatic Fever, Endocarditis, and
Kawasaki Disease Committee of the Council on Cardiovas- he author and editors gratefully acknowledge the contri-
cular Disease in the Young, the Interdisciplinary Council butions of the previous authors—homas S. Metkus Jr.,
on Functional Genomics and Translational Biology, and the Patrick O’Gara, and Donna M. Polk.
9
General Internal Medicine
LORI WIVIOTT TISHLER, SARAH P. HAMMOND, AND GALEN V. HENDERSON

1. A 55-year-old African American man comes for his irst cervical and vaginal examinations are entirely normal.

t
ne
primary care oice visit. He is without complaints but Her Pap test result is notable for “benign endometrial
presents because his father had a myocardial infarction cells.” She has not had any vaginal bleeding.
recently at age 75, and his mother is healthy at 74. He What is the best next step in management?

e.
does not smoke. His blood pressure is 120/71 mm Hg A. Nothing. he cells are benign; repeat at regular
on hydrochlorothiazide. His total cholesterol is 200 intervals until age 65.

in
mg/dL, and his high-density lipoprotein (HDL) is 25 B. Repeat in 6 months.
mg/dL. His 10-year risk for developing atherosclerotic C. Refer for endometrial biopsy.
cardiovascular disease, according to Pooled Cohort D. Repeat in 1 year.

ic
Equations, is 11.9%. E. Refer for colposcopy.
Which of the following is the most appropriate
management?
ed
4. A 35-year-old woman presents to the clinic with her
A. Recommend lifestyle change; follow up in 6 husband due to diiculty becoming pregnant. he
sm
months. couple has been trying to become pregnant for the last
B. Start a statin. 6 months without success, despite having intercourse
C. Obtain coronary calcium score. three times per week consistently. Her periods are usu-
D. Refer for cardiac stress test. ally regular on a 26-day cycle. Her partner had semen
ok

E. Refer for cardiac catheterization. analysis, the result of which was normal. Her physical
examination is unremarkable.
2. A 54-year-old woman with a remote history of What is the most appropriate next step in manage-
bo

migraines (but none in years) presents to her primary ment?


care physician with a new-onset headache. he pain is A. Reassurance and workup in 6 months if still not
retroorbital on the right side only and has been pro- pregnant
e

gressive for 3 weeks with intermittent responsiveness B. Day 20 serum progesterone


://

to acetaminophen. She denies visual changes, fevers, C. Hysterosalpingogram


chills, jaw claudication, or weakness. he result of D. Referral for in vitro fertilization
tp

her examination, including a thorough neurological E. Day 20 luteinizing hormone (LH) and follicle-
examination, is normal. stimulating hormone (FSH)
What is the most appropriate next step in manage-
ht

ment? 5. A 30-year-old woman presents to the clinic complain-


A. Prescribe oxycodone. ing of bilateral breast pain. She states that the pain is
B. Order a head MRI/MRA. worse toward the end of her menstrual cycle and also in
C. Prescribe amitriptyline. the evening. She has had the pain for approximately 3
D. Prescribe sumatriptan. months and describes it as an aching, difuse pain. Her
E. Refer for physical therapy. examination is notable for a BMI of 28 kg/m2 and pen-
dulous breasts without erythema, masses, or discharge.
3. A 56-year-old woman with obesity who has not had She has no lymphadenopathy or skin changes. She is
a period for 5 years comes in for her routine Pap test. concerned that she may have cancer and tells you that
All of her Pap smears in the past have been normal her maternal grandmother had cancer at age 76.
and HPV negative. She is at low risk for cervical can- What is the best next step in management?
cer and has been having screening Pap tests every 3–5 A. Bilateral mammography
years with consistently negative HPV test results. Her B. Bilateral breast MRI

191
192 C HA P T E R 9 General Internal Medicine

C. Bilateral breast ultrasound C. Lumbar puncture (LP)


D. BRCA testing D. Magnetic resonance venography
E. Reassurance and a more supportive bra
9. A 69-year-old man is found to have an alkaline phos-
6. A 31-year-old man with a family history of Hashimoto phatase level of 365 U/L. he results of radiographs
thyroiditis presents to the clinic for evaluation of hair and a radionuclide bone scan are consistent with
loss. He states that he noticed a small bald spot on his Paget disease of the bone involving the left humerus,
occiput several months ago. Hair did regrow in that right ischium, and several vertebrae. he patient is
area but was white instead of black. He now presents asymptomatic.
to the clinic because he has three well-circumscribed What is the next best step in management at this
round areas of complete hair loss on his scalp as well as time?
one in his beard. He otherwise denies any complaints. A. No further workup or treatment
he result of a thyroid-stimulating hormone (TSH) B. Start an oral bisphosphonate.
laboratory test was within normal range. C. Start an intravenous bisphosphonate.
What is the best next step in management? D. Start calcitonin.

t
ne
A. Intradermal injection of triamcinolone E. Refer for orthopedic surgery.
B. Minoxidil 2% solution applied twice a day
C. Biopsy of the bald area 10. A 28-year-old woman calls the oice with ongoing

e.
D. Ketoconazole shampoo symptoms of dysuria, urgency, and frequency. She
developed symptoms of a urinary tract infection 1

in
7. A 36-year-old Ashkenazi Jewish woman comes to see week ago that was treated with amoxicillin without
you. She is concerned because her 45-year-old sister resolution of her symptoms. Urine cultures were not

ic
was diagnosed with premenopausal breast cancer. As sent at that time.
you take her history, you learn that there are several What is the next most appropriate step in manage-
other irst-degree relatives who have had breast cancer
or ovarian cancer. You use the Gail model to determine
ed ment?
A. No further workup or treatment
the patient’s lifetime risk (based on your knowledge B. Prescribe a 3-day course of levoloxacin.
sm
of when she had menarche, her family history, and C. Prescribe another 7-day course of amoxicillin.
absence of previous breast biopsies). You determine D. Prescribe a 5-day course of nitrofurantoin.
that she has a 25% lifetime risk of developing breast E. Prescribe a 3-day course of ciproloxacin.
cancer. She and her family are considering genetic test-
ok

ing, and you refer her to a genetic counselor. 11. A 62-year-old man with a history of gout who is
In the meantime, what is the best screening proto- taking allopurinol 300 mg presents after a recently
col for our patient? inlamed irst metatarsal joint. A joint aspiration
bo

A. Annual mammograms starting at age 40 showed monosodium urate crystals within neutro-
B. Annual mammograms starting at age 45 phils. He is currently asymptomatic, but this is his
C. Annual MRI starting now second gout lare in the last 3 months. He has a his-
e

D. Mammograms and MRI annually, ideally 6 tory of hypertension and is being treated with lisino-
://

months apart pril. He states that he is taking all of his medications


E. MRI every 6 months regularly. His serum creatinine is 0.9 mg/dL, and his
tp

serum uric acid is 8.3 mg/dL.


8. A 30-year-old man is evaluated in the emergency What is the next best step in management?
department (ED) for a 3-day history of confusion A. No further workup or treatment
ht

and visual loss, and he has a 4-day history of gradu- B. Increase allopurinol with concurrent colchicine.
ally increasing headache. One week ago, he had a bout C. Colchicine alone
of severe gastroenteritis with diarrhea. A review of his D. Switch allopurinol to febuxostat.
medical records shows that he has a heterozygous fac- E. Switch allopurinol to probenecid.
tor V Leiden mutation. His examination is remarkable
for papilledema, a right pronator drift, right hom- 12. A 37-year-old man presented to his primary care phy-
onymous hemianopsia, and luent aphasia. he results sician with epigastric discomfort and was found to be
of a CT scan of the brain and laboratory studies are Helicobacter pylori positive. He did not have melena,
normal. weight loss, or any other concerning features. He was
Which of the following is the most appropriate treated with omeprazole 20 mg twice daily, amoxicillin
next diagnostic test for this patient? 1 g twice daily, and clarithromycin 500 mg twice daily
A. Carotid ultrasonography for 14 days. He now represents with the same epigas-
B. Electroencephalography (EEG) tric discomfort.
CHAPTER 9 General Internal Medicine 193

What is the next best step in management? D. Ceftriaxone 250 mg intramuscular injection × 1 +
A. Empirically treat with the same regimen for 14 days. azithromycin 1 g orally × 1
B. Empirically treat with omeprazole 20 mg twice E. Doxycycline 100 mg orally twice daily × 7 days
daily, bismuth subsalicylate two tabs four times
per day, tetracycline 500 mg four times daily, and 16. A 35-year-old man presents to an urgent care facility
metronidazole 250 mg four times per day. with a 2-week history of worsening purulent rhinorrhea
C. Order a repeat serology to conirm persistence of and right-sided maxillary sinus pain. His symptoms
H. pylori. have persisted despite treatment with acetaminophen,
D. Order a urea breath test to conirm persistence of luticasone propionate, and saline irrigation. Examina-
H. pylori. tion is notable for a temperature of 101.5°F, copious
rhinorrhea, and tenderness with percussion over the
13. A 31-year-old monogamous woman with no past med- maxillary sinuses. he results of cardiac and pulmo-
ical history presents for a routine Papanicolaou (Pap) nary examinations are unremarkable.
smear. he cytological result is negative for intraepi- He takes hydrochlorothiazide 25 mg daily and
thelial lesion or malignancy. Relex DNA testing for amlodipine 5 mg daily for hypertension. He has no
high-risk human papillomavirus (HPV) is performed, known drug allergies.
and the result is negative. What is the most appropriate next step in man-
What is the next step in management? agement?
A. Repeat cytology in 6 months. A. Amoxicillin 500 mg three times daily × 5–7 days
B. Repeat cytology in 1 year. B. Amoxicillin/clavulanate 500/125 mg three times
C. Repeat cytology in 2 years. daily × 5–7 days
D. Repeat cytology in 3 years. C. Azithromycin 500 mg po × 1 then 250 mg po
E. Repeat cytology in 5 years. daily × 4 days
D. Doxycycline 200 mg once daily × 5–7 days
14. A 22-year-old woman sustained a compound tibial E. Levoloxacin 750 mg daily × 5–7 days
fracture during a motor vehicle accident requiring
surgery. Her hospital course was complicated by lower 17. A 34-year-old woman is seen in the emergency depart-
extremity deep venous thrombosis (DVT). She has no ment with confusion, malaise, and lower extremity
family history of pulmonary embolism or DVT, and rash. Her past medical history is notable for aller-
the result of a hypercoagulability workup is negative. gic rhinitis. Her medications include loratadine and
She is not taking any medication. luticasone propionate nasal spray. he patient is
She is started on low-molecular-weight heparin alert and oriented to self only. Physical examination
followed by warfarin and achieves a target INR of is notable for jaundice and bilateral lower extremity
2.5. petechiae. Laboratory data are notable for a hemoglo-
How long should she be anticoagulated? bin of 7 g/dL, reticulocyte count of 15%, and plate-
A. 1 months lets of 45,000/µL. he patient’s lactate dehydrogenase
B. 3 months (LDH) level is 1500 mg/dL. he results of coagulation
C. 6–12 months studies are normal. Her serum creatinine is 3.6 mg/dL.
D. Lifelong anticoagulation A peripheral smear is shown in Fig. 9.1.

15. A 19-year-old woman presents with 3 days of


increasing vaginal discharge. She is sexually active
with one partner. Her last menstrual period was 4
days ago. Physical examination reveals temperature
99°F, blood pressure 100/60 mm Hg, and heart
rate of 90 beats per minute. Her pelvic examination
shows copious mucopurulent discharge from a red,
inlamed cervix. She has tenderness on palpation of
the cervix but no adnexal or uterine tenderness. he
result of a pregnancy test is negative, and you send a
gonorrhea/chlamydia probe that returns positive for
gonorrhea.
What is the next management step?
A. Ceftriaxone 125 mg intramuscular injection × 1
B. Ceftriaxone 250 mg intramuscular injection × 1
C. Ceftriaxone 125 mg intramuscular injection × 1 +
azithromycin 1 g orally × 1 • Fig. 9.1 Peripheral blood smear for patient in Question 17.
194 C HA P T E R 9 General Internal Medicine

Which of the following is the best next step? D. Eosinophilic granulomatosis with polyangiitis
A. Intravenous immunoglobulin E. Hodgkin disease
B. Antinuclear antibody test
C. Plasma exchange 21. An 18-year-old man presents for a physical exami-
D. Transthoracic echocardiogram nation prior to joining his college basketball team.
E. Direct antiglobulin (Coombs) test A II/VI crescendo–decrescendo murmur without
radiation is heard at the left lower sternal border on
18. An 80-year-old man is reevaluated after a 5-mm left cardiac examination. he murmur increases with Val-
middle cerebral artery aneurysm is discovered inci- salva maneuvers, and there is an extra heart sound
dentally on an MRI of the brain obtained because preceding S1.
of headaches. he patient has no other relevant per- What is the most likely underlying etiology?
sonal or family medical history and takes no medica- A. Congenital aortic stenosis
tions. On physical examination, the patient’s blood B. Marfan syndrome
pressure is 140/80 mm Hg, and his pulse is 80 beats C. Hypertrophic cardiomyopathy
per minute. Results of his physical and neurologi- D. Early-onset hypertension
cal examinations are normal. An MRA scan shows E. Rheumatic heart disease
an unruptured aneurysm but no additional intracra-
nial aneurysms. No hemorrhage, infarction, mass, or 22. A 22-year-old male college football player without a
mass efect is evident. signiicant past medical history presents to the emer-
Which of the following is the most appropriate next gency room with a small abscess on his neck. he
step in the management of this patient’s aneurysm? collection is drained, and a Gram stain demonstrates
A. Annual MRA gram-positive cocci in clusters.
B. Endovascular coiling of the aneurysm What treatment would you prescribe?
C. Nimodipine administration A. Oral vancomycin
D. Surgical clipping of the aneurysm B. Dicloxacillin
C. Oral trimethoprim-sulfamethoxazole
19. A 51-year-old man from western Massachusetts pres- D. Oral penicillin
ents to his primary care physician with a fever and E. Intravenous nafcillin
rash of 1 week’s duration. He has no other symptoms.
On examination, the rash is consistent with erythema 23. A 23-year-old man with a history of mild asthma pres-
migrans. ents for evaluation of intermittent dysphagia for solid
What is the next best step in management? foods. He denies heartburn. He has not lost weight.
A. Doxycycline 200 mg × 1 He reports that 1 year ago he underwent an upper
B. Azithromycin 500 mg × 1 day, then 250 mg × 4 endoscopy for removal of pieces of steak. He has taken
days 40 mg of omeprazole twice daily for the last month,
C. Amoxicillin-clavulanate 500/125 mg three times but his symptoms have persisted.
per day × 2 weeks His physical examination was unremarkable. An
D. Doxycycline 100 mg twice daily × 2 weeks upper endoscopy reveals circular rings in the mid-
E. Ceftriaxone 2 g IV daily × 3 weeks esophagus. A biopsy shows a dense eosinophilic
iniltrate.
20. A 68-year-old woman presents to her primary care Which of the following is the most appropriate
doctor with parasthesias in her right foot, progres- irst-line therapy?
sive right-sided foot drop, and erythematous rashes A. Increase the omeprazole to 80 mg twice daily.
over both lower extremities. Five months ago, she had B. Esophageal dilation
cough, sputum production, and a chest x-ray with C. Topical swallowed luticasone
patchy iniltrates, prompting treatment with anti- D. Oral nifedipine
biotics. She was diagnosed with asthma 2 years ear- E. Botulinum toxin injection into the lower esopha-
lier. Laboratory data show a white blood cell count geal sphincter
9600/µL, hematocrit 36.2%, platelets 271,000/µL,
with 58% neutrophils, 9% lymphocytes, and 31% 24. At a routine health review appointment, a 36-year-old
eosinophils. man with type 1 diabetes blurts out, “his sweating
Which of the following diseases is most likely to be thing is ruining my life.” He goes on to explain that
the cause of the patient’s presentation? he has to change his clothes at work because of under-
A. Granulomatosis with polyangiitis (formerly arm sweating, tries not to shake hands, and has been
Wegener granulomatosis) missing out on getting together with friends. He notes
B. Temporal arteritis that his wife is really upset by it. he sweating is not a
C. Schistosomiasis lifelong problem. In fact, it has been notable that he
CHAPTER 9 General Internal Medicine 195

has had it for only a couple of years. He is not particu- his lu shot annually, and he received his pneumococ-
larly anxious and denies any symptoms of hypo- or cal vaccine 3 years ago. He had a normal colonoscopy
hyperthyroidism. here does not seem to be a family 7 years ago. He is on amlodipine and allopurinol. His
history of this condition. It does not seem to be related vital signs are normal, and his physical examination is
to anxiety, to luctuations in his blood sugar, to his unremarkable.
hemoglobin A1c, or to any other conditions. He takes Which of the following screening tests is most
long- and short-acting human insulin, lisinopril, and appropriate for this patient based on most evidence
atorvastatin. of beneit?
What can you do for this distressed patient with A. Coronary calcium CT imaging
hyperhidrosis? B. Prostate-speciic antigen
A. Start an SSRI or SNRI; he is obviously too anx- C. hyroid-stimulating hormone
ious. D. Abdominal ultrasound
B. Stop the lisinopril, and see if he improves. E. Exercise treadmill test
C. Do a thorough evaluation for lymphoma, includ-
ing blood tests and scans. 28. An 80-year-old man is evaluated in the oice for an
D. Check his TSH and liver function tests (LFTs). If episode of hesitancy in speech and word-inding dif-
they are normal, treat symptomatically. iculty, right facial weakness, and weak right arm.
he episode occurred early yesterday, lasted 20 min-
25. A 35-year-old man presents to his primary care phy- utes, and was witnessed by his wife. he patient has a
sician for a routine physical examination. His only history of coronary artery disease, hypertension, and
medical history includes seasonal allergies, for which hyperlipidemia. His current medications are metopro-
he uses intranasal luticasone. He does not smoke ciga- lol, aspirin, hydrochlorothiazide, and lovastatin. On
rettes or drink alcohol. He has no family history of examination, his blood pressure is 150/80, and his
colorectal cancer, although his mother did have two heart rate is 70 beats per minute. he result of a neu-
adenomatous polyps at age 55. rological examination is normal.
When should he undergo his irst screening colo- Which of the following is the most appropriate
noscopy? next step in management?
A. Now A. Add clopidogrel.
B. Age 40 years B. Admit to the hospital.
C. Age 45 years C. Order outpatient diagnostic studies.
D. Age 50 years D. Schedule a follow-up visit in 1 week.

26. A 70-year-old woman presents with pain in her 29. A 32-year-old man is evaluated for what he calls a
hands and wrists of 9 months’ duration. Her hands “sinus headache.” he headache occurs two or three
are stif in the morning for 15 minutes. She has times per month and is accompanied by facial pres-
pain with sewing and typing. She has not noticed sure and occasional rhinorrhea; it worsens with
swelling or warmth. Her vital signs are normal. Her movement. Resting in a dark, quiet room results in
bilateral proximal interphalangeal joints are ten- subjective improvement. he symptoms resolve in
der to palpation and have bony enlargements. he 1 or 2 days, regardless of treatment. He has tried
irst carpometacarpal joints are also tender, and the multiple varieties of decongestants and antihista-
palms appear “square” due to misalignment of the mines without success. Acetaminophen-aspirin-cafeine
thumb base. he result of her metacarpal squeeze preparations offer minimal relief. He is currently
test is negative. he remainder of her examination symptomatic.
is normal. On examination, the patient is pale and moder-
Which of the following studies should be done to ately distressed. His temperature is 37.1°C (98.8°F),
establish the diagnosis? heart rate is 84 beats per minute, respiration rate is 16
A. ANA breaths per minute, and blood pressure is 132/75 mm
B. Erythrocyte sedimentation rate Hg. His face is tender on palpation.
C. Bilateral x-rays of the hands What is the most likely diagnosis?
D. Anticitrullinated protein antibody A. Cluster headache
E. No additional studies are needed. B. Migraine without aura
C. Sinus headache
27. A 68-year-old man with a history of hypertension and D. Tension headache
gout presents for his routine annual examination. He
has no complaints. He was a past smoker for 20 years 30. A 55-year-old obese (BMI, 35 kg/m2) woman with
but quit 30 years ago. He drinks one glass of red wine a history of hyperlipidemia and hypertension pres-
daily. He goes on long walks daily for exercise. He gets ents for follow-up after a recent liver biopsy for
196 C HA P T E R 9 General Internal Medicine

A B
P
• Fig. 9.2 (A) Noncontrast head computed tomography. (B) Axial T2-weighted FLAIR magnetic reso-
nance imaging. Both images are from the patient described in Question 31.

evaluation of persistently abnormal aminotransfer- C. Brain abscess: treatment with ceftazidime, vanco-
ases. She has no history of heavy alcohol use. Her mycin, micafungin, and acyclovir
medications include atorvastatin and hydrochloro- D. Ruptured brain abscess: treatment with ceftriax-
thiazide (HCTZ). Her alanine transaminase (ALT) one, vancomycin, and ampicillin, with neurosur-
was 85 U/L (reference range, 0–35 U/L), and her gery consultation if symptoms do not improve
aspartate transaminase (AST) was 66 U/L (refer- with 24 hours of antibiotics
ence range, 0–35 U/L). he results of viral serologic E. Ruptured brain abscess: treatment with ceftriax-
testing and autoimmune markers were negative. An one, vancomycin, and ampicillin, with emergency
abdominal ultrasound revealed hepatic steatosis. She neurosurgery consultation
underwent a liver biopsy, which showed steatosis
with inlammation consistent with steatohepatitis 32. A 92-year-old community-dwelling senior comes to
and stage 2 ibrosis. your oice for follow-up of a fall. She was fortunate,
In addition to weight loss, which of the following and all that was actually wounded was her pride. You
would you recommend? review her history. he fall occurred at home when
A. Pioglitazone she got up from a chair and headed for the bathroom.
B. Vitamin E She lives alone, does not drive, and ignores her walker.
C. Metformin She takes multiple medications, including atenolol,
D. No additional therapy HCTZ, lisinopril, insulin, and metformin. At night,
she takes amitriptyline for her diabetic neuropathy.
31. A 32-year-old man with no past medical history She also drinks 6–8 oz of hard liquor daily, though she
presents with low-grade fevers, anorexia, headache, repeatedly denies this, and you have learned it only
and neck stifness of 4 days’ duration, which started because her daughter told you.
shortly after a dental procedure. he night prior Of the one-third of elderly people who fall every
to presentation, he had one episode of emesis and year, what percentage of them have multiple falls?
a worsening posterior headache. his morning, his A. 20%
wife noticed that he seemed “not quite himself ” and B. 30%
was “walking into walls,” prompting her to bring C. 40%
him to the emergency department. In the emergency D. 50%
department, he undergoes head imaging, as shown in E. 60%
Fig. 9.2A, B.
What are the most likely diagnosis and best next 33. A 30-year-old woman with ulcerative colitis and
management choice? autoimmune hepatitis complicated by cirrhosis,
A. Meningitis: treatment with ceftazidime, vancomy- ascites, and esophageal varices presents with dys-
cin, and micafungin pnea and left-sided back pain. Abdominal ultra-
B. Meningitis: treatment with ceftriaxone, vancomy- sound shows minimal ascites, and a chest x-ray is
cin, and ampicillin shown in Fig. 9.3.
CHAPTER 9 General Internal Medicine 197

A B
• Fig. 9.3 (A–B) Chest x-ray for patient in Question 33.

Which of the following would be the next appro- neutropenia, normocytic anemia, high serum zinc,
priate step? and low ceruloplasmin; B12, folate, homocysteine, and
A. horacentesis methylmalonic acid levels are normal.
B. Chest tube Which of the following is the most likely cause of
C. Diuretics his condition?
D. TIPS (transjugular intrahepatic portosystemic A. Vitamin B12 deiciency
shunt) B. Paraneoplastic polyneuropathy
E. Evaluation for liver transplant C. Copper deiciency
D. Vitamin B6 toxicity
34. A 56-year-old woman with hypertension presents to E. Lead toxicity
the emergency room with abdominal pain in the left
lower quadrant and no bowel movements for several 36. A 73-year-old woman is evaluated in the ED for sudden
days. Her medications include lisinopril and omepra- onset of explosive headache starting 8 hours ago. She
zole. Her vital signs, including blood pressure, are initially rested in a dark room, but her condition was
normal. Her abdomen is soft, nontender, and nondis- unchanged, so then she went to the ED. he patient
tended. Abdominal CT suggests constipation but is has a history of hypertension controlled by lisinopril,
otherwise normal. he CT also shows a 3.5-cm right and her family history is noncontributory. In the ED,
adrenal lesion. she becomes nauseated, vomits, and then becomes rap-
In addition to checking a dexamethasone suppres- idly more progressively weak; she eventually becomes
sion test and plasma metanephrines, which of the fol- obtunded and requires intubation and mechanical
lowing tests is indicated to evaluate this adrenal lesion? ventilation. On physical examination, she is afebrile,
A. No additional tests necessary her blood pressure is 188/100 mm Hg, and her pulse
B. Plasma aldosterone and renin is 120 beats per minute. he patient exhibits laccid
C. Fine-needle aspiration for cytology and culture quadriplegia, and meningismus is present. Both pupils
D. Cosyntropin stimulation test are 4 mm and nonreactive; the oculocephalic relex is
absent, and the corneal relex is absent bilaterally. She
35. A 42-year-old man with a history of morbid obesity has a depressed level of consciousness and a GCS score
status post–bariatric surgery with 75-pound weight of 3. Subhyaloid hemorrhages are present bilaterally.
loss presents for a follow-up visit. He complains of 5 he results of CBC and the remainder of the blood
years of progressive gait instability and numbness and work are normal. CT of the head shows an extensive
weakness in his distal extremities. His family history is acute subarachnoid hemorrhage and mild prominence
unremarkable. He takes high doses of vitamin supple- of the temporal tips of the lateral ventricles.
ments, including B complex and zinc. Which of the following neurologic complications
On examination, he has an unsteady gait, Rom- is most likely to have caused this patient’s rapid dete-
berg sign, spasticity in the bilateral lower extremities, rioration?
bilateral hyperrelexia, and Babinski sign. here is A. Hydrocephalus
impaired vibration and position sense in the feet. Pain B. Rebleeding
and temperature sensation in the lower extremities are C. Syndrome of inappropriate antidiuretic hormone
normal. His laboratory test results reveal leukopenia, D. Vasospasm
198 C HA P T E R 9 General Internal Medicine

37. A 27-year-old woman presents for evaluation of sec- D. Clopidogrel


ondary amenorrhea. She had onset of menarche at age E. Pharmacologic stress test
15 and had regular periods until a few years ago. She
denies any other symptoms and states that she runs 39. A 28-year-old woman with no past medical history
approximately 10–15 miles per day. here is no evi- presents with nausea and vomiting after completing
dence of an eating disorder. Her pregnancy test result her irst marathon. She was able to complete the mara-
is negative, and her prolactin and TSH are normal. thon and thereafter immediately rehydrated. She took
Her FSH, LH, and estradiol levels are in the low- four 200-mg ibuprofen tablets and was at a postmara-
normal range. Her BMI is 22 kg/m2, and her exami- thon party when she started to feel ill, saying unusual
nation is unremarkable. he female athlete triad is things to her friends such as “I made a terrible mis-
suspected, and a bone density test shows signiicant take” and “I am drowning.” Her friends brought her
bone loss. to the emergency room. On physical examination, she
What is the best treatment for the patient? appears tired and is mildly confused. She has an oth-
A. Start oral contraceptive pills. erwise nonfocal neurological examination. Her jugular
B. Start a bisphosphonate. venous pressure is 6 cm H2O.
C. Encourage a decrease in exercise along with calcium What is the best next step in the workup and man-
and vitamin D. agement of this patient?
D. Start leptin. A. Administration of 1 L of normal saline
B. Encouraging oral rehydration with an electrolyte
38. A 42-year-old man presents to the emergency depart- replacement sports drink
ment 1 week after developing chest pain. One week C. Immediate electrolyte panel
ago, he developed severe left-sided chest pain in the D. Administration of hypertonic saline at a rate of
setting of cocaine use. he chest pain persisted for 1 ml/kg/h
1 day and then resolved. He has not had further E. Checking an ibuprofen level
chest pain. His EKG in the emergency department is
shown in Fig. 9.4. Cardiac biomarkers are notable for 40. A 36-year-old woman with depression, mild asthma,
a normal creatine kinase (CK) and CK-MB and an and obesity presents with 2 weeks of a nonproductive
elevated troponin at 13.2 µg/L. he best next step in cough. She also has paroxysms of coughing and post-
management is: tussive vomiting. She denies signiicant wheezing. She
A. Echocardiogram works at a day care. She received her vaccinations as a
B. Anticoagulation with heparin child. Vital signs, lung examination, complete meta-
C. Urgent cardiac catheterization bolic panel, and chest x-ray are unremarkable.

I aVR V1 V4

II aVL V2 V5

III aVF V3 V6

VI

• Fig. 9.4 Electrocardiogram for patient in Question 38.


CHAPTER 9 General Internal Medicine 199

he best treatment at this time would be: What is the most appropriate next step in manage-
A. Albuterol inhaler ment?
B. Azithromycin A. Increase the nifedipine dose.
C. Prednisone B. Start captopril.
D. Antitussive agents C. Start oral labetalol.
E. Admission to the hospital for IV antibiotics D. Begin IV methylprednisolone.
E. Start oral prednisone.
41. A 36-year-old man calls your oice with a complaint
that he woke up and cannot hear out of his left ear 43. A 36-year-old man presented to his primary care
very well. He has no history of ear problems and no physician with a weeklong history of severe pain
recent infections. He did travel recently on an airplane in his left Achilles tendon. Over the past few days,
and wonders if the plane trip is the cause. Your astute he has also developed pain and swelling in his in-
triage nurse had him hum, and the hum sound did not gers and toes (Fig. 9.5A, B). He has been having
lateralize. He has no vertigo, but he has tinnitus and a diiculty walking and bearing weight. Of note, 2
little bit of ear pain. weeks ago, he developed a weeklong course of diar-
What should be your next step? rhea accompanied by chills and sweats following a
A. Don’t worry. It’s the plane; take a decongestant, weekend camping trip.
and let me know if not better. he most appropriate treatment is:
B. Don’t worry. See if you can come in this week, and A. Ceftriaxone 1 g IV
we’ll take out the wax. B. Methylprednisolone 1000 mg IV
C. Worry a little. Maybe he has otitis. See him later or C. Prednisone 60 mg orally
tomorrow. D. Indomethacin 50 mg orally
D. Worry. See him today or refer him to ENT. E. Observation

42. A 38-year-old woman with a history of difuse cuta- 44. A 39-year-old woman of Greek descent presents to
neous systemic sclerosis presents with lower extremity the emergency room after experiencing a brief loss of
edema of 1 week’s duration. Her baseline blood pres- consciousness while at work. Workup reveals a white
sures are 120–140/70–80 mm Hg. Her medications blood cell count of 4000/µL, hematocrit 20%, and
include nifedipine and omeprazole. platelet count 207,000/µL.
On examination, she is afebrile. Her heart rate is She notes that she had a viral syndrome 1 week ago,
98 beats per minute. Her blood pressure is 170/100 which subsequently resolved. She has no history of
mm Hg. She has skin thickening over the face, hands, bleeding. She moved into a new house 3 months ago.
arms, chest, and abdomen. here are telangiectasias on She notes that she has had a propensity to chew ice for
her face and palms. Her cardiac examination is nota- the past 1 year. She has no family history of anemia.
ble for an S4. Her lungs are clear bilaterally. She has Additional workup reveals:
2+ lower extremity edema. Her laboratory test results • Mean corpuscular volume (MCV) 55 fL
reveal hemoglobin 9.8 g/dL, platelets 95,000/µL, • Iron less than assay, ferritin 1, total iron-binding
BUN 40 mg/dL, creatinine 2.4 mg/dL, and albumin capacity (TIBC) 400 µg/dL
3.4 g/dL, and urinalysis shows 2+ protein and 10 • Erythrocyte sedimentation rate (ESR) 8 mm/h
RBCs per high-power ield. A blood smear shows 2+ • Normal haptoglobin, LDH, vitamin B12, and folate
schistocytes. levels

A B

• Fig. 9.5 Photograph of lower extremity (A) and upper extremity (B) for patient in Question 43.
200 C HA P T E R 9 General Internal Medicine

A blood smear shows microcytic, hypochromic cells position for 1–2 weeks but now have been present
of varying shapes. Of note, a complete blood count 3 continuously for the last month. She previously had
years ago showed a hematocrit of 28% with an MCV occasional headaches with photo- and phonophobia
of 85 fL. beginning at age 14 years. herapy with triptans for
he most likely diagnosis is: her current headaches is only mildly efective; she
A. halassemia is on no other medications. She has had intermit-
B. Iron deiciency anemia tent blurred vision for the past month and notes a
C. Lead toxicity pulsatile sound in both ears when she lies in a quiet
D. Hemolysis room.
E. Anemia of chronic disease Physical examination is signiicant only for obesity
(BMI, 30 kg/m2). Funduscopic examination reveals
45. A 30-year-old woman comes to see you with wrist the presence of bilateral papilledema. No other abnor-
pain. She is having a really hard time, describing the mal indings are seen on neurologic examination.
pain as pain at the base of the wrist that is exacerbated Magnetic resonance imaging (MRI) of the brain with
by moving her thumb back and forth. She thinks it contrast and magnetic resonance venography of the
might be mildly swollen. Her past medical history is brain are normal.
notable for a recent uncomplicated pregnancy and Which of the following is the most appropriate next
healthy delivery. Her baby is 6 weeks old. She has no step in the management of this patient’s headache?
personal or family history of autoimmune disease. She A. Acetazolamide
had a recent normal TSH. here are no sick contacts. B. Amitriptyline
She has sustained no trauma. On examination, you do C. Lumbar puncture
not really appreciate the swelling she describes, but she D. Neurosurgical consultation
is tender to palpation. It is particularly painful when
you fold her ingers over her thumb and gently rotate 48. A 27-year-old woman originally from Brazil who is
her wrist in the ulnar direction. 25 weeks pregnant (G1P0) presents with dyspnea,
What is the most likely cause of her wrist pain? blood-tinged sputum, and pleuritic chest pain.
A. Carpal tunnel syndrome Upon physical examination, her heart rate is 137
B. De Quervain tenosynovitis beats per minute, blood pressure is 96/53 mm Hg,
C. Ulnar nerve entrapment and oxygen saturation is 82% on room air. Exami-
D. Stress fracture of her irst metacarpal nation of the pulmonary and cardiac systems reveals
E. Ganglion cyst diffuse rales in bilateral lung fields and a difficult-
to-auscultate, low-pitched diastolic rumble at the
46. A 37-year-old nurse presents to your oice because apex. An electrocardiogram shows sinus tachycar-
a tuberculin skin test (TST) that was done before he dia. A chest radiograph shows diffuse bilateral infil-
could start a job at a new hospital was reactive. He was trates. An echocardiogram reveals a normal ejection
born in Portugal and thinks he was given the BCG fraction, a diffusely thickened mitral valve, moder-
vaccine during childhood. He grew up in New York ate to severe mitral stenosis, and elevated pulmo-
City and most recently worked in a hospital with a nary artery systolic pressure. She is intubated for
large immigrant population. He does not recall taking respiratory support. Fetal ultrasound was reassur-
care of anyone with tuberculosis (TB). He currently ing. The most appropriate regimen for medical
takes no medications. He is anxious about starting a management is:
long course of treatment. You look at the red patch on A. Beta blockers and gentle diuresis
his arm but carefully measure the induration, inding B. Digoxin
that it is 22 mm across. A chest x-ray shows no abnor- C. Dopamine
malities. D. ACE inhibitors
What do you advise him to do? E. Hydralazine and nitrates
A. He does not need to be treated, because the TST
result is positive due to his BCG vaccine. 49. A 29-year-old woman presents to the emergency
B. Treat with weekly isoniazid and rifapentine for 12 department with sore throat, fever, and recurrent
weeks. hematuria. She was in her usual state of health until
C. Ofer to check a TB blood test because it is the 1 month ago, when she developed a sore throat and
“gold standard.” a fever of 101°F. he following day, she noticed frank
D. Treat with daily pyrazinamide and rifampin for 2 blood in her urine and went to the emergency depart-
months. ment. She was diagnosed with a presumed urinary
tract infection and given a 7-day course of cephalexin.
47. A 22-year-old woman is evaluated for daily head- After several days of antibiotics, her fevers resolved,
aches that seemed to be initially worse in the supine and her urine cleared. She remained in good health
CHAPTER 9 General Internal Medicine 201

until 1 week ago, when she again developed fever,


sore throat, and bloody urine, and she returned to the
emergency department. She notes a similar episode
about 3 years ago.
Urinalysis was notable for 3+ blood and 2+ protein.
Urine sediment showed 493 dysmorphic red blood
cells per high-power ield. No casts were seen. Labora-
tory values were notable for a serum creatinine level
of 3.4 mg/dL. Her complement component levels
were normal. Renal ultrasound showed no evidence of
obstruction, hydronephrosis, or perinephric luid col-
lections.
Which of the following is the most likely diagnosis?
A. Poststreptococcal glomerulonephritis
B. IgA nephropathy
C. Carcinoma of the bladder
D. Urinary tract infection
E. Nephrolithiasis

50. A 30-year-old woman with no past medical history


was incidentally found to have a hyperpigmented lin- • Fig. 9.6 Photograph of patient’s left arm in Question 50.
ear lesion on her left arm following travel to Hawaii
(Fig. 9.6).
Which activity mostly likely led to the develop- C. Applying sunscreen containing para-aminoben-
ment of this physical examination inding? zoic acid (PABA)
A. Handling ish in a saltwater tank D. Handling thorned roses
B. Squeezing limes while making mojitos E. Injection drug use

Chapter 9 Answers

1. ANSWER: B. Start a statin. of the American College of Cardiology/American Heart


he joint American College of Cardiology and Association Task Force on Practice Guidelines. Circulation.
American Heart Association guideline on the treat- 2014;129(25 Suppl 2):S1–S45.
ment of blood cholesterol identiied four major groups
of patients who would beneit from statin therapy: (1) 2. ANSWER: B. Order a head MRI/MRA.
those with clinical atherosclerotic cardiovascular dis- An MRI is indicated for this patient because she
ease (ASCVD), (2) those with LDL greater than or is presenting with a new headache after the age of 50
equal to 190 mg/dL, (3) diabetics aged 40–75 years years. Imaging should be performed in those patients
old with an LDL >70 mg/dL, and (4) those with an presenting with the following:
estimated 10-year ASCVD risk greater than or equal • “First or worst” headache
to 7.5%. In patients with LDL <190 mg/dL, ASCVD • Increased frequency and increased severity of head-
risk is calculated on the basis of age, sex, race, total ache
and HDL cholesterol, hypertension, and smoking sta- • New-onset headache after age 50
tus. (http://tools.acc.org/ASCVD-Risk-Estimator/). • New-onset headache with history of cancer or
his patient has a 10-year ASCVD risk of 11.9% immunodeiciency
that would qualify for statin therapy. Lifestyle change • Headache with mental status changes
should be encouraged but is insuicient to mitigate • Headache with fever, neck stifness, and meningeal
the patient’s risk of ASCVD. A coronary calcium score signs
is not necessary in this patient who already has an indi- • Headache with focal neurologic deicits if not pre-
cation for statin therapy. Stress testing and catheteriza- viously documented as a migraine with aura
tion are not indicated in this asymptomatic patient. Narcotics are generally not irst-line therapy for
Stone NJ, Robinson JG, Lichtenstein AH, et  al. 2013 ACC/ headaches; amitriptyline can be used for chronic
AHA guideline on the treatment of blood cholesterol to prophylaxis for migraines and tension headaches
reduce atherosclerotic cardiovascular risk in adults: a report but should not be used prior to a proper evaluation.
202 C HA P T E R 9 General Internal Medicine

Sumatriptan is a irst-line therapy for migraines, but Intradermal injection of triamcinolone is the best
this patient does not seem to be experiencing a classic studied and most effective treatment. Minoxi-
migraine. Physical therapy is not indicated. dil has been used but is of limited benefit when
used alone. Biopsy is unnecessary in this classic
3. ANSWER: C. Refer for endometrial biopsy. presentation.
In a postmenopausal woman, endometrial cells
(even benign ones) on a Pap smear can be indicative of 7. ANSWER: D. Mammograms and MRI annually,
endometrial cancer. In this question, the patient’s his- ideally 6 months apart
tory of obesity may put her at increased risk for endo- his patient is at high risk for developing breast
metrial cancer as well. In this setting, endometrial cells cancer. She has Ashkenazi Jewish heritage and sev-
are never normal, and the patient should be referred eral irst-degree relatives with cancer. his makes it
for endometrial biopsy. Some clinicians would also get more likely that she may carry BRCA1, BRCA2, or
an ultrasound to measure the patient’s endometrial other mutations that put her at increased risk. he
stripe. A decision not to biopsy based on the stripe is current recommendations for women at high risk
best made by the gynecologic consultant. is annual screening with both MRI and mammog-
Massad LS, Einstein MH, Huh WK, et al. 2012 updated con- raphy. he irst two choices would not be correct
sensus guidelines for the management of abnormal cervical even if she were not at markedly elevated risk. he
cancer screening tests and cancer precursors. J Low Genit age of her sister at diagnosis dictates that screening
Tract Dis. 2013;17(5 Suppl 1):S1–S27. starting at age 35 (10 years younger than the index
case) is important.
4. ANSWER: B. Day 20 serum progesterone Bevers TB, Anderson BO, Bonaccio E, et  al. NCCN clinical
Reassurance is inappropriate because the patient practice guidelines in oncology: breast cancer screening and
is 35 years old, so evaluation should begin after 6 diagnosis. J Natl Compr Canc Netw. 2009;7(10):1060–1096.
months of trying to become pregnant (or at 1 year if Bevers TB, Ward JH, Arun BK, et al. Breast cancer risk reduc-
the patient is under 35 years old). A Day 20 or 21 tion, version 2.2015. J Natl Compr Canc Netw. 2015;13(7):
serum progesterone can conirm the presence of ovula- 880–915.
tory cycles and should be the irst step in the workup
of female infertility. Up to 40% of women with infer- 8. ANSWER: D. Magnetic resonance venography
tility have ovulatory dysfunction. Although a hystero- he patient should undergo a MR venogram. he
salpingogram is useful to conirm tubal patency, it most likely diagnosis is venous sinus thrombosis,
should not precede assessing ovulation. IVF is inap- given his known hypercoagulability, dehydration,
propriate without completing a workup. LH and FSH and symptoms of mounting intracranial pressure
can assess ovulatory reserve but would be checked on and eventual focal deicits. Carotid ultrasound does
Day 3, not Day 20. not assist in the assessment of the intracranial vas-
Practice Committee of the America Society for Reproductive culature. Seizures are a recognized complication of
Medicine. Diagnostic evaluation of the infertile female: a venous sinus thrombosis; however, EEG does not
committee opinion. Fertil Steril. 2015;103(6):e44–e50. assist with the establishment of a preliminary diag-
nosis and should not be performed. An LP is con-
5. ANSWER: E. Reassurance and a more supportive traindicated in this patient with evidence of elevated
bra intracranial pressure.
he history is consistent with cyclic mastalgia, Stam J. hrombosis of the cerebral veins and sinuses. N Engl J
which may be due to Cooper ligament pain caused Med. 2005;352(17):1791–1798.
by inadequate support of her pendulous breasts.
Imaging would be overly aggressive, as would BRCA 9. ANSWER: A. No further workup or treatment
testing. here is no reason to treat asymptomatic Paget
Howard MB, Battaglia T, Prout M, Freund K. he efect of disease of the bone. When symptomatic, bisphospho-
imaging on the clinical management of breast pain. J Gen nates, orally or IV, can be efective in reducing pain
Intern Med. 2012;27(7):817–824. due to bone turnover. Calcitonin may be efective but
has less evidence than a bisphosphonate.
6. ANSWER: A. Intradermal injection of triamcino- Ralston SH. Paget’s disease of bone. N Engl J Med. 2013;368(7):
lone 644–650.
The patient has alopecia areata, which typically
presents before age 40, with 66% of patients pre- 10. ANSWER: D. Prescribe a 5-day course of nitrofu-
senting before 30. Alopecia areata causes rapid areas rantoin.
of well-circumscribed hair loss on the scalp but can his 28-year-old woman has uncomplicated cysti-
involve any and all hair growth. Growth of white tis and warrants treatment to relieve symptoms. he
hair in the alopecic regions is a common occurrence. large majority of cases of uncomplicated cystitis in a
CHAPTER 9 General Internal Medicine 203

premenopausal woman are caused by Escherichia coli. 13. ANSWER: E. Repeat cytology in 5 years.
he Infectious Diseases Society of America guidelines he U.S. Preventive Services Task Force (USP-
for the treatment of uncomplicated cystitis recom- STF) recommends cervical cancer screening every 3
mend trimethoprim-sulfamethoxazole, fosfomycin, years for women aged 21–65 with cytology (Papa-
and nitrofurantoin as irst-line treatment options. nicolaou). Women 30–65 years of age who desire
Neither quinolones (ciproloxacin, levoloxacin) nor longer screening intervals can be screened with a
amoxicillin is recommended as a irst-line agent. Qui- combination of cytology and HPV testing every 5
nolones and other broad-spectrum oral antibiotics are years. here is suicient evidence that screening with
associated with “collateral damage,” such as increased HPV testing (alone or in combination with cytology)
risk for colonization or infection with drug-resistant confers little to no beneit among women younger
organisms, whereas amoxicillin is associated with high than 30 years of age.
failure rates. Moyer VA. Screening for Cervical Cancer: U.S. Preventive Ser-
Gupta K, Hooton TM, Naber KG, et al. International Clini- vices Task Force Recommendation Statement. Ann Intern
cal Practice Guidelines for the Treatment of Acute Uncom- Med. 2012;156:880–891.
plicated Cystitis and Pyelonephritis in Women: A 2010
Update by the Infectious Diseases Society of America and 14. ANSWER: B. 3 months
the European Society for Microbiology and Infectious Dis- For patients with a irst episode of venous throm-
eases. Clin Infect Dis. 2011;52(5):e103–e120.
boembolism due to a reversible or time-limited risk
factor (i.e., oral contraceptive use, surgery/prolonged
11. ANSWER: B. Increase allopurinol with concurrent immobilization, trauma, pregnancy), the treatment
colchicine. course is 3 months. Anticoagulation beyond 3 months
his 62-year-old man is having continued attacks of is typically not required. Idiopathic or recurrent
gout while on allopurinol, but his serum uric acid level venous thromboembolism requires extended and/or
is higher than the goal level. His allopurinol should be lifelong therapy where the exact duration of therapy
increased to bring his serum uric acid down to <6.0 can vary on the basis of patient history and preference.
mg/dL. He should be on colchicine or an NSAID Kearon C, Akl EA, Ornelas J, et al. Antithrombotic herapy for
while the allopurinol is increased. here is no reason VTE Disease: CHEST Guideline and Expert Panel Report.
to switch medications. Chest. 2016;149(2):315–352.
Neogi T. Gout. N Engl J Med. 2011;364(5):443–452.
15. ANSWER: D. Ceftriaxone 250 mg intramuscular
12. ANSWER: D. Order a urea breath test to conirm injection × 1 + azithromycin 1 g orally × 1
persistence of H. pylori. he 2015 CDC guidelines for the management of
It is reasonable to use a test-and-treat strategy for sexually transmitted infections introduced the recom-
H. pylori in patients with epigastric discomfort without mendations to treat uncomplicated cervicitis and ure-
any red-lag symptoms. Persistence of symptoms after thritis due to Neisseria gonorrhea with dual-antibiotic
treatment can be related to lack of cure or may be unre- therapy due to emerging resistance among circulating
lated to H. pylori. Antibiotic resistance among patients N. gonorrhea internationally. he recommended irst-
with H. pylori has been increasing in the last decade. line treatment regimen in the United States includes
For this reason, 14-day treatment regimens are now intramuscular ceftriaxone and a single high dose of
recommended over shorter courses, and triple-therapy oral azithromycin, which also treats for the possibil-
regimens such as this patient received (typically two ity of Chlamydia, which is a common concurrent
antibiotics and a proton pump inhibitor) are recom- cause of infection. Note that because of the increased
mended only in areas where treatment success rates are prevalence of N. gonorrhea resistance to doxycycline
high and clarithromycin resistance is low; otherwise, compared with azithromycin, azithromycin is the
treatment regimens that include four agents (typically preferred agent for treatment of potential chlamydia
two antibiotics, a proton pump inhibitor, and bismuth) infection.
are recommended. Testing for cure of H. pylori after Workowski KA, Bolan GA; Centers for Disease Control and
therapy is now fairly standard practice, particularly if Prevention. Sexually transmitted diseases treatment guide-
the patient has persistent symptoms. H.  pylori serol- lines, 2015. MMWR Recomm Rep. 2015;64(RR-03):1–137.
ogy will remain positive even with adequate treatment,
thus a urea breath test or stool antigen must be used 16. ANSWER: B. Amoxicillin/clavulanate 500/125 mg
to conirm active infection. If found to be positive for three times daily × 5–7 days
H. pylori, quadruple therapy with alternative antibiot- Acute bacterial rhinosinusitis is characterized by
ics given for 14 days would be appropriate. at least one of the following three symptoms: persis-
Fallone CA. he Toronto Consensus for the Treatment of tent symptoms (≥7–10 days), severe symptoms (fever
Helicobacter pylori Infection in Adults. Gastroenterology. ≥102°F, purulent nasal discharge, or facial pain from
2016;151(1):51–69. onset lasting ≥3–4 days), or initial improvement
204 C HA P T E R 9 General Internal Medicine

followed by subsequent worsening of symptoms. he of neurologic disability associated with intervention


2012 Infectious Diseases Society of America guide- exceeds the potential beneit. After 3 successive years
lines recommend amoxicillin-clavulanate as irst-line of annual monitoring, an MRA or CTA obtained once
therapy, instead of amoxicillin alone. Doxycycline every 3 years will be suicient.
is the recommended alternative for patients with hompson BG, Brown RD Jr, Amin-Hanjani S, et al. Guide-
a penicillin allergy but should be dosed at 100 mg lines for the Management of Patients With Unruptured
by mouth twice daily. In 2016, the FDA issued a Intracranial Aneurysms: A Guideline for Healthcare Pro-
warning about the potential risk for serious and irre- fessionals From the American Heart Association/American
versible side efects with the use of luoroquinolones Stroke Association. Stroke. 2015;46(8):2368–2400.
such as levoloxacin and advised against using quino-
lones to treat uncomplicated acute bacterial sinusitis 19. ANSWER: D. Doxycycline 100 mg twice daily × 2
(and also uncomplicated urinary tract infections and weeks
acute bronchitis). Due to increasing rates of Strep- he treatment of choice for early Lyme disease is
tococcus pneumoniae resistance to azithromycin, this doxycycline × 10–21 days because it will also treat
drug is not recommended for the treatment of acute anaplasmosis. Amoxicillin is an alternative in pregnant
rhinosinusitis. women and in children <8 years of age. Intravenous
Chow AW, Benninger MS, Brook I, et al. IDSA clinical practice third-generation cephalosporins are required only in
guideline for acute bacterial rhinosinusitis in children and patients with advanced heart block, meningitis, neu-
adults. Clin Infect Dis. 2012;54(8):e72–e112. ritis, or other more advanced manifestations of this
U.S. Food and Drug Administration. FDA Drug Safety Com- infection. In Europe, studies have shown azithromy-
munication: FDA updates warnings for oral and injectable cin to be as efective as doxycycline. However, in the
luoroquinolone antibiotics due to disabling side efects. https: United States, studies have demonstrated azithromy-
//www.fda.gov/Drugs/DrugSafety/ucm511530.htm; accessed cin not to be as efective as amoxicillin (and presum-
3/17/17.
ably doxycycline) in the treatment of Lyme disease.
Wormser GP, Dattwyler RJ, Shapiro ED, et  al. he clinical
17. ANSWER: C. Plasma exchange assessment, treatment, and prevention of Lyme disease,
his is a case of thrombotic thrombocytopenic human granulocytic anaplasmosis, and babesiosis: clinical
purpura (TTP), also referred to as ADAMTS13 dei- practice guidelines by the Infectious Diseases Society of
ciency–mediated thrombotic microangiopathy. his syn- America. Clin Infect Dis. 2006;43(9):1089–1134.
drome is caused by a congenital or acquired deiciency
of ADAMTS13, which is a protease that cleaves large 20. ANSWER: D. Eosinophilic granulomatosis with
multimers of von Willebrand factor. When the prote- polyangiitis
ase is deicient, accumulation of von Willebrand factor his patient displays the classic triad of eosino-
multimers leads to microvascular damage. he smear philic granulomatosis with polyangiitis: asthma, sinus
is microangiopathic, demonstrating schistocytes (>5/ disease, and peripheral eosinophilia. he diagnosis is
high-power ield) and thrombocytopenia. he diagno- made primarily by clinical features, including asthma
sis of TTP is a clinical one; however, the presence of (particularly of late onset), mononeuropathy (includ-
microangiopathic hemolytic anemia and thrombocy- ing multiplex), or polyneuropathy; skin disease with
topenia without other clear cause is required for diag- variable rash patterns seen in two-thirds of patients;
nosis and is suicient, in this case, to make an initial paranasal sinus abnormalities (allergic rhinitis, recur-
diagnosis. he additional indings of renal failure, neu- rent sinusitis, nasal polyposis); eosinophilia with
rologic abnormalities, and fever constitute the remain- >10% or >1500 eosinophils/µL; and migratory or
ing classical clinical and laboratory features of TTP. A transient pulmonary opacities on x-ray.
low serum ADAMTS13 level supports the diagnosis. Mouthon L, Dunogue B, Guillevin L. Diagnosis and clas-
Plasma exchange is the treatment of choice for TTP, siication of eosinophilic granulomatosis with polyangiitis
and early initiation is key to decreasing patient mor- (formerly named Churg-Strauss syndrome). J Autoimmun.
bidity (see Fig. 9.1). 2014;48–49:99–103.
George JN, Nester CM. Syndromes of thrombotic microangi-
opathy. N Engl J Med. 2014;371(7):654–666. 21. ANSWER: C. Hypertrophic cardiomyopathy
his murmur is characteristic of hypertrophic car-
18. ANSWER: A. Annual MRA diomyopathy (HCM), speciically the increase with
his patient should have an annual MRA or CT Valsalva maneuvers and lack of radiation to the carotids.
angiogram to monitor aneurysmal growth. For patients HCM is an autosomal dominant disorder of the car-
without a prior SAH, the lowest-risk aneurysms are diac sarcomere and carries an increased risk of sudden
those in the anterior circulation and <7 mm in diam- cardiac death.
eter. he annual risk of rupture for an aneurysm of Aortic stenosis also has a crescendo–decrescendo
the size of this patient’s is 0.05% annually. he risk murmur but decreases or does not change signiicantly
CHAPTER 9 General Internal Medicine 205

with Valsalva maneuvers. he murmur associated with 24. ANSWER: B. Stop the lisinopril, and see if he
aortic stenosis also characteristically radiates to the improves.
carotids. Marfan syndrome can be associated with aor- his patient has hyperhidrosis. A relatively com-
tic insuiciency, which is a diastolic murmur. Rheu- mon condition, it can be quite debilitating for patients,
matic heart disease can cause systolic murmurs from causing them to have a lot of social isolation and dis-
aortic stenosis or mitral regurgitation. However, the comfort. Essential hyperhidrosis is a disorder of the
murmur described in this case is not consistent with eccrine glands. Sometimes it can be inherited. Second-
either of these valvular disorders. ary hyperhidrosis is suggested in this case because of its
later onset, and it can be due to a host of medications.
22. ANSWER: C. Oral trimethoprim-sulfamethoxazole In this case, lisinopril may be the most likely culprit. If
Community-acquired MRSA (CA MRSA) is a the patient improves after discontinuing the medicine,
common cause of skin and soft tissue infections. then it would be reasonable to consider other options.
Outbreaks of CA MRSA have been seen in particular Symptomatic treatment for patients can consist
groups, such as athletes and intravenous drug users, of using higher doses of aluminum chlorohydrate in
but it should be considered in anyone presenting with a preparation such as aluminum chloride (Drysol,
a soft tissue infection. his patient has a characteristic Hypercare), which can be used on the hands and feet
presentation and appropriately had a Gram stain and as well as under the arms and in other areas. Some
culture performed. patients choose to undergo more invasive procedures
CA MRSA is not susceptible to beta-lactam anti- for this condition.
biotics such as nafcillin, penicillin, and dicloxacillin.
Systemically ill-appearing patients with CA MRSA 25. ANSWER: B. Age 40 years
can be treated with intravenous vancomycin. here is For average-risk persons, the preferred method of
no role for oral vancomycin to treat infections other colon cancer screening is a colonoscopy beginning at
than Clostridium diicile infections, because it is not age 50. Patients with a family history of adenomatous
absorbed. When treating suspected CA MRSA in an polyps in a irst-degree relative should have a screening
outpatient setting, oral antibiotic options include colonoscopy at age 40 or 10 years before the diagnosis
clindamycin, trimethoprim-sulfamethoxazole, or a of adenomatous polyps in the family member, which-
long-acting tetracycline (e.g., minocycline or doxycy- ever comes irst.
cline). Rex DK, Johnson DA, Anderson JC, et al. American College of
Singer AJ, Talan DA. Management of skin abscesses in the era Gastroenterology Guidelines for Colorectal Cancer Screen-
of methicillin-resistant Staphylococcus aureus. N Engl J Med. ing 2008. Am J Gastroenterol. 2009;104(3):739–750.
2014;370(11):1039–1047.
Stevens DL, Bisno AL, Chambers HF, et al. Practice guidelines 26. ANSWER: E. No additional studies are needed.
for the diagnosis and management of skin and soft tissue his patient has irst carpometacarpal joint ten-
infections: 2014 update by the Infectious Diseases Society derness and squaring of the palm, consistent with
of America. Clin Infect Dis. 2014;59(2):e10–e52.
osteoarthritis (OA). Although symmetric polyarticu-
lar arthritis is typical of rheumatoid arthritis (RA),
23. ANSWER: C. Topical swallowed luticasone the joints involved are not consistent with RA, and
he patient has eosinophilic esophagitis. he diag- there are no signs of inlammation on exam. he
nostic criteria include clinical symptoms of esopha- distal interphalangeal joints are commonly involved
geal dysfunction, biopsy showing >15 eosinophils/ in osteoarthritis of the hand and rarely involved in
high-power ield, lack of responsiveness to high- RA. Involvement of the irst carpometacarpal joint is
dose PPI, and normal pH monitoring in the distal almost always a sign of osteoarthritis. Morning stif-
esophagus. Men are afected more commonly than ness lasting <30 minutes indicates degenerative rather
women. Adults with this disorder usually give a his- than inlammatory arthritis. Options A through D are
tory of intermittent dysphagia and food impaction. part of the workup for inlammatory arthritis involv-
First-line therapy includes topical corticosteroids. ing the hands. However, in this case, the diagnosis of
Fluticasone propionate at a dose of 440 µg twice OA in the hands can be made without additional test-
daily administered for 4–6 weeks leads to clinical ing (Option E).
and histologic improvement. Other therapies include Altman R, Alarcón G, Appelrouth D, et al. he American College
elimination diets, oral budesonide, and leukotriene of Rheumatology criteria for the classiication and report-
receptor antagonists. Dilation is reserved for those ing of osteoarthritis of the hand. Arthritis Rheum. 1990;33:
patients with ixed strictures who do not respond to 1601–1610.
medical therapy, because there is a risk of esophageal
tearing with dilation. 27. ANSWER: D. Abdominal ultrasound
Furuta GT, Katzka DA. Eosinophilic esophagitis. N Engl J Med. he U.S. Preventive Services Task Force (USPSTF)
2015;373:1640–1648. recommends that men between the ages of 65 and 75
206 C HA P T E R 9 General Internal Medicine

with any current or past history of smoking undergo type, because it lasts longer than 180 minutes (1–2
a one-time screening for abdominal aortic aneurysm days in this patient’s case). In addition, although
(AAA) with abdominal ultrasound (Option D). Sev- not part of the absolute criteria for cluster headache,
eral studies have demonstrated a survival beneit to patients with cluster headache prefer to be mobile
screening, including a population-based study of over because resting causes worsening of the pain. On the
67,800 men aged between 65 and 74 who were ran- basis of the lack of fever or discolored nasal discharge,
domized to AAA screening (with surgery for those the patient does not have the secondary headache of
found to have AAA >5.4 cm) or no screening. his sinus infection. Although sinus symptoms are not part
study showed AAA-related mortality was reduced of the formal criteria for migraine, they are quite com-
by an average of 42% (95% CI, 22%–58%) in the mon and can complicate the diagnosis. Tension-type
screened population compared with the unscreened headache can be ruled out because of the disabling
population. characteristic of the headache and the presence of both
Several studies have shown no beneit in male non- photo- and phonophobia.
smokers and in women. here are insuicient data Headache Classiication Subcommittee of the International Head-
to determine whether the association of coronary ache Society. he International Classiication of Headache Dis-
calcium with coronary artery disease risk warrants orders: 2nd edition. Cephalalgia. 2004;24 Suppl 1:9–160.
coronary calcium CT screening (Option A) in asymp-
tomatic men. Similarly, there are insuicient data to 30. ANSWER: B. Vitamin E
support the use of exercise treadmill testing (Option his patient has nonalcoholic fatty liver disease
E) for screening asymptomatic patients. he utility of (NAFLD). A biopsy is recommended in patients with
prostate cancer screening tests (Option B) to decrease NAFLD and increased risk of steatohepatitis and
mortality is uncertain. here is not enough evidence advanced ibrosis. Patients with metabolic syndrome
to recommend routine screening for thyroid disease fall into this category. Weight loss is the mainstay of
(Option C). therapy for NAFLD. Vitamin E has been shown to
LeFevre ML. U.S. Preventive Services Task Force. Screening improve liver histology in nondiabetic patients with
for abdominal aortic aneurysm: U.S. Preventive Services biopsy-proven nonalcoholic steatohepatitis (NASH).
Task Force recommendation statement. Ann Intern Med. Pioglitazone has been associated with improvement
2014;161(4):281–290. in some histologic changes of NASH. However, in a
large placebo-controlled trial, it was not superior to
28. ANSWER: B. Admit to the hospital. placebo. Metformin has not been shown to afect liver
his patient should be admitted to the hospital. histology or disease progression
His history is consistent with transient ischemic Bariatric surgery is not currently used speciically
attack (TIA). Risk for the development of stroke to treat NASH. However, bariatric surgery is recom-
after a TIA can be estimated by the “ABCD2” score, mended for patients with obesity and a BMI >40 kg/
which is a risk assessment tool that includes age, m2 or >35 kg/m2 and serious medical comorbidities.
blood pressure, clinical features of the TIA, dura- Chalasani N, Younossi Z, Lavine JE, et al. he diagnosis and
tion of the TIA, and diabetes. his patient has an management of non-alcoholic fatty liver disease: practice
elevated ABCD2 score of 5 based on his age over guideline by the American Association for the Study of
60, systolic blood pressure above 140 mm Hg, and Liver Diseases, American College of Gastroenterology, and
the American Gastroenterological Association. Hepatology.
symptoms including focal weakness, which suggests
2012;55(6):2005–2023.
his 2-day risk of stroke is 4.1% and that he would
likely beneit from hospitalization for immediate
further workup. 31. E. Ruptured brain abscess: treatment with ceftriax-
Johnston CS, Rothwell PM, Nguyen-Huynh MN, et al. Valida- one, vancomycin, and ampicillin, with emergency
tion and reinement of scores to predict very early stroke risk neurosurgical consultation
after transient ischaemic attack. Lancet. 2007;369(9558): he patient’s initial clinical syndrome of fevers,
283–292. anorexia, headache, and neck stifness is consistent
with a diagnosis of meningitis. he abrupt change in
29. ANSWER: B. Migraine without aura symptomatology and focal neurological symptoms
his patient presents with typical symptoms of reported the night prior to presentation are an indi-
migraine headache. he symptoms meeting criteria cation for emergent head imaging (noncontrast head
for migraine in this patient include worsening of the CT). Upon imaging, head CT demonstrates a brain
headache with movement, limitation of activities, abscess with likely surrounding vasogenic edema, and
and requiring absence of light and sound (dark, quiet magnetic resonance imaging (MRI) demonstrates pus
room). within the ventricle, indicating rupture of the brain
Although some autonomic features are present abscess. A ruptured brain abscess is considered a neu-
(congestion/rhinorrhea), the headache is not cluster rological emergency with a very high mortality rate.
CHAPTER 9 General Internal Medicine 207

Given the high mortality rate of ruptured brain metanephrines to evaluate for pheochromocytoma in
abscesses with pus draining into the ventricles, broad- all patients, and plasma aldosterone and renin if the
spectrum antibiotics as well as emergent neurosurgical patient has hypertension (as in this case). A cosyntro-
consultation are indicated in this case (Option E). pin stimulation test evaluates for adrenal insuiciency
and is not a standard part of the workup for an adre-
32. ANSWER: D. 50% nal incidentaloma. Fine-needle aspiration is not part
Falls in the elderly are responsible for 70% of acci- of the irst set of tests in the workup for an adrenal
dental deaths in people 75 and over. hey increase incidentaloma.
with age, transcend ethnic groups, and cause signii- Young WF Jr. he incidentally discovered adrenal mass. N Engl
cant morbidity, including decline of functional status J Med. 2007;356(6):601–610.
and risk for hospitalization. Hip fracture occurs in
1%–2% of falls. 35. ANSWER: C. Copper deiciency
he following are risk factors for falls: he patient has progressive upper motor neuron
Intrinsic deicits. In a patient with a history of bariatric surgery,
• Muscle weakness copper and vitamin B12 deiciency should be suspected.
• Gait and balance dysfunction However, the vitamin B12 level (Option A) in this case is
• Visual impairment normal. Zinc competes with copper for absorption, and
• Cognitive impairment the patient is taking supplemental zinc, so this may be
• Orthostatic hypotension contributing. Copper deiciency (Option C) afects the
• Medications (do not forget about alcohol) corticospinal tract (hyperrelexia and Babinski sign) and
Extrinsic posterior column (impaired vibration sensation).
• Poor lighting Vitamin B6 (Option D) toxicity causes peripheral
• Clutter neuropathy, not upper motor neuron signs. Lead tox-
• Environmental obstacles icity (Option E) is more common in children. Adults
• Bad shoes with lead poisoning frequently have sleep disorders
and may be hypersomnolent. A paraneoplastic poly-
33. ANSWER: A. horacentesis neuropathy (Option B) would not explain all of his
his patient most likely has pleural efusion due laboratory abnormalities.
to cirrhosis and ascites (also called hepatic hydrotho- Bal BS, Finelli FC, Shope TR, Koch TR. Nutritional deiciencies
rax). It is due to a diaphragmatic defect, which can be after bariatric surgery. Nat Rev Endocrinol. 2012;8(9):544–
microscopic. his occurs on the right side 85% of the 556.
time, on the left side 13% of the time, and bilaterally
2% of the time. horacentesis (Option A) should be 36. ANSWER: B. Rebleeding
the irst intervention performed to assess for other he most likely complication to have caused this
possible etiologies of pleural efusion and to rule out patient’s rapid deterioration is rebleeding. In the irst
infection. Diuretics (Option C) (and salt and luid few hours after an initial hemorrhage, up to 15% of
restriction) can be used to manage pleural efusion afected patients have a sudden deterioration of con-
once the diagnosis is conirmed. A transjugular intra- sciousness, which strongly suggests rebleeding. In
hepatic portosystemic shunt (TIPS) (Option D) is patients who survive the irst day, the rebleeding risk
used to manage refractory efusion occurring in this is evenly distributed during the next 4 weeks, with a
setting. Patients with hepatic hydrothorax should be cumulative risk of 40% without surgical or endovascu-
evaluated for liver transplant (Option E). A chest lar interventions. Occlusion of the responsible aneu-
tube (Option B) should never be placed in patients rysm is thus the irst aim in the management of the
with hepatic hydrothorax, because it can cause mas- subarachnoid hemorrhage and is usually performed by
sive protein and electrolyte depletion, infection, coiling or clipping.
renal failure, and bleeding. Connolly ES Jr, Rabinstein AA, Carhuapoma JR, et al. Guide-
Kumar S, Sarin SK. Paradigms in the management of hepatic lines for the management of aneurysmal subarachnoid hem-
hydrothorax: past, present, and future. Hepatol Int. orrhage: a guideline for healthcare professionals from the
2013;7:80–87. American Heart Association/American Stroke Association.
Stroke. 2012;43(6):1711–1737.
34. ANSWER: B. Plasma aldosterone and renin
his is an adrenal incidentaloma in a patient with a 37. ANSWER: C. Encourage a decrease in exercise
history of hypertension. Endocrinologic work to assess along with calcium and vitamin D.
for autonomous secretion of cortisol, catecholamines, his patient has functional hypothalamic amenor-
or aldosterone is indicated. he appropriate laboratory rhea resulting from suppression of the hypothalamic-
evaluation includes a dexamethasone suppression test pituitary-ovarian axis due to energy imbalance. he
to evaluate for Cushing disease in all patients, plasma best treatment for bone loss under these conditions is
208 C HA P T E R 9 General Internal Medicine

to treat the underlying energy imbalance by nutritional by the normal to slightly high jugular venous pressure
rehabilitation and decrease exercise along with calcium of 6 cm H2O, rehydration with normal saline (Option
and vitamin D supplementation. Oral contraceptive A) and oral rehydration (Option B) is inadequate to
pills will not correct the bone loss, though they would correct hyponatremia and can result in worsening
cause resumption of menses. Bisphosphonates would hyponatremia.
not be a good option in a young premenopausal woman he irst step in this emergency should be to
who may desire pregnancy. Leptin has been shown to obtain an immediate electrolyte panel (Option C)
restore menses in functional hypothalamic amenor- to conirm hyponatremia and to help guide further
rhea, but its efects on bone health are unknown. treatment. Empiric administration of hypertonic
Weiss Kelly AK, Hecht S. Council on Sports Medi- saline (Option D) should be avoided in a clini-
cine and Fitness. he female athlete triad. Pediatrics. cal setting where laboratory tests can be obtained
2016;138(2):e20160922. rapidly, because overly hasty correction of hypona-
tremia can lead to an osmotic demyelination syn-
38. ANSWER: A. Echocardiogram drome (also known as central pontine myelinolysis).
he clinical history of chest pain in the setting of he patient’s signs and symptoms are inconsistent
cocaine use 1 week ago suggests a cocaine-induced with ibuprofen overdose (Option E). Although her
myocardial infarction (MI). Following an MI, cardiac symptoms may be consistent with very severe acute
biomarkers peak at 18–24 hours. CK and CK-MB kidney injury in the setting of potential hypovole-
remain elevated for 48 hours, whereas troponins may mia and ingestion of ibuprofen as a second insult,
remain elevated for 10 days. he pattern of biomark- this is much less likely.
ers in this patient is consistent with an MI 1 week ago. Almond CSD, Shin AY, Fortescue EB, et  al. Hyponatremia
he EKG demonstrates ST elevations in leads V1–V4, among runners in the Boston Marathon. N Engl J Med.
suggestive of an anterior ST-elevation MI. here are 2005;352(15):1550–1556.
also Q waves in leads V1–V4. he presence of anterior
Q waves plus persistent ST elevations with a clinical 40. ANSWER: B. Azithromycin
story of a cocaine-induced MI 1 week ago is most sug- he clinical presentation suggests pertussis, which
gestive of a ventricular aneurysm. he diferential diag- can be a cause of persistent cough in adults, even those
nosis of chronic or persistent ST elevations includes who received vaccinations as child. he paroxysms of
early repolarization, ventricular aneurysm (when ST coughing and posttussive vomiting are also features
elevations occur with Q waves), a scar following a large suggestive of pertussis. he clinical course is usually
anterior MI, and chronic pericarditis. less severe than in children, but treatment is advised
An echocardiogram (Option A) should be within 3 weeks of symptoms in nonpregnant patients
obtained to look for the presence of a ventricular and within 6 weeks of symptoms in pregnant patients
aneurysm and ventricular thrombus. Ventricular in order to contain the spread of infection, which is
aneurysms are a common complication of anterior especially important, given that she works at a daycare.
MIs. hey are treated with afterload reduction and Treatment options for pertussis include azithromycin
anticoagulation. (Option B), clarithromycin, erythromycin, and trime-
Treatment with heparin (Option B) may be ini- thoprim-sulfamethoxazole.
tiated if the patient has evidence of a ventricular Albuterol inhaler (Option A) and prednisone
aneurysm, but it would not be the next best step in (Option C) might be used for an asthma attack. Anti-
management. Late catheterization of STEMI (after tussive agents (Option D) may be helpful, but they
24–48 hours) (Option C) should be done only for will not treat the infection. here is no clinical indi-
severe heart failure, electrical or hemodynamic insta- cation for hospitalization or IV antibiotics for this
bility, or persistent ischemia. A pharmacologic stress patient (Option E).
test (Option E) may be performed prior to discharge Tiwari T, Murphy TV, Moran J. National Immunization Pro-
for risk stratiication. Clopidogrel (Option D) and gram, Centers for Disease Control and Prevention. Rec-
aspirin are indicated for secondary prevention after a ommended antimicrobial agents for the treatment and
STEMI. postexposure prophylaxis of pertussis: 2005 CDC Guide-
Lange RA, Hillis LD. Cardiovascular complications of cocaine lines. MMWR Recomm Rep. 2005;54(RR-14):1–16.
use. N Engl J Med. 2001;345(5):351–358.
41. ANSWER: D. Worry. See him today or refer him
39. ANSWER: C. Immediate electrolyte panel to ENT.
his patient has symptoms associated with acute his patient has sudden sensorineural hearing loss
hyponatremia (confusion and altered mental status) (SSNHL). It is characterized by rapid loss of hear-
caused by extreme hypotonic losses (through sweat) ing, often in one ear and often noticed in the morn-
from exercise and replacement of the hypotonic luid ing. More than 90% of patients have tinnitus, and
with free water. In the setting of euvolemia, as indicated most have ear fullness as well. he etiologies include
CHAPTER 9 General Internal Medicine 209

autoimmune, microvascular, or viral cochleitis. Most 44. ANSWER: B. Iron deiciency anemia
would evaluate for retrocochlear tumor (acoustic his is a case of profound iron deiciency ane-
neuroma), which occurs in between 5% and 30% of mia (Option B), as evidenced by microcytic ane-
studies. Most ENT doctors will treat patients with mia with very low iron levels. Causes of microcytic
high-dose steroids, and a small subset of patients may anemia include iron deiciency, thalassemia, and
do well with antivirals. anemia of chronic disease. Rarer causes include
Rauch SD. Idiopathic sudden sensorineural hearing loss. N copper deiciency, lead poisoning, and sideroblastic
Engl J Med. 2008;359:833–840. anemia. Iron deiciency may be subtle and insidi-
ous, with typical presenting symptoms including
42. ANSWER: B. Start captopril. fatigue, weakness, exercise intolerance, headache,
he patient has scleroderma and newly elevated and irritability. Additional signs and symptoms
blood pressure, lower extremity edema, renal failure include tongue pain, dry mouth, pica/pagophagia,
with proteinuria, and microangiopathy, consistent and restless leg syndrome.
with scleroderma renal crisis. he drug of choice for halassemia (Option A) typically presents with
scleroderma renal crisis is an angiotensin-converting very low MCV, as in this case, but iron stores should
enzyme inhibitor (Option B), which should be rap- be normal to increased. he prior MCV of 85 fL also
idly titrated to reduce blood pressure. ACE inhibitors makes thalassemia highly unlikely, because it is an
help preserve or improve renal function in scleroderma inherited disorder. Family history is often positive.
renal crisis and have been shown to signiicantly reduce Lead poisoning (Option C) may cause microcytic
mortality in this setting. anemia. Basophilic stippling is often (but not always)
here is no evidence that this is an inlammatory pro- evident on peripheral blood smear. Other manifesta-
cess; thus, glucocorticoids (Options D and E) are not tions include abdominal pain, joint and muscle aches,
indicated. In addition, high-dose glucocorticoids increase memory problems, and irritability
the risk of scleroderma renal crisis. Calcium channel he normal haptoglobin and LDH make the diag-
blockers (Option A) may be used as additional treatment nosis of hemolytic anemia (Option D) less likely.
of resistant hypertension. Beta blockers (Option C) are Hemolysis usually results in normocytic anemia.
usually avoided in patients with scleroderma because of Anemia of chronic disease (Option E) typically
the theoretical risk of worsening vasospasm. presents with low iron, low TIBC, and a normal to
Bose N, Chiesa-Vottero A, Chatterjee S. Scleroderma renal cri- increased ferritin. he ESR would be expected to be
sis. Semin Arthritis Rheum. 2015;44(6):687–694. elevated.
Camaschella C. Iron-deiciency anemia. N Engl J Med.
43. ANSWER: D. Indomethacin 50 mg orally 2015;372:1832–1843.
his is a presentation of reactive arthritis, which
presents as an asymmetric mono-/oligoarthritis, pre- 45. ANSWER: B. De Quervain tenosynovitis
dominantly of lower extremity joints. Classically, his patient has the classic presentation of De
it also presents with enthesitis (inlammation of the Quervain tenosynovitis, which presents most com-
insertion of ligaments, tendons, joint capsule, or fascia monly in women between 30 and 50 years old, with a
to bone—typically the Achilles tendon) and dactyli- signiicant subset of them being recently postpartum.
tis (“sausage digits”). Extraarticular involvement may It afects the abductor pollicis longus and extensor
include urethritis, conjunctivitis, uveitis, oral ulcers, pollicis brevis in the irst extensor compartment. Most
and rashes. Reactive arthritis may occur following GU people think that it is related to some sort of repetitive
or enteric infections caused by Chlamydia trachoma- strain, though the etiology is unknown. It seems com-
tis, Yersinia, Salmonella, Shigella, Campylobacter, and mon in women with newborns because of the way that
possibly Clostridium diicile. Typically, there are a they hold and feed the baby. Treatment is generally
few days to a few weeks between infection and onset. conservative, and people do well with a thumb spica
Treatment is with NSAIDs such as indomethacin 50 splint and nonsteroidals. Steroid injections can help if
mg three times daily for at least 2 weeks. the pain continues.
Gonococcal arthritis similarly may present with the
abrupt onset of a mono- or oligoarthritis, but it typi- 46. ANSWER: B. Treat with weekly isoniazid and rifa-
cally does not cause sausage digits and frequently pres- pentine for 12 weeks.
ents with a rash. Ceftriaxone (Option A) would be an As a healthcare worker, our patient is at fairly
appropriate initial treatment for this. IV (Option B) or high risk of converting to active TB. He was likely
oral (Option C) steroids may be used to treat numer- exposed to TB in the past few years on the basis of
ous rheumatologic conditions, but they are not the his history (conversion is highest in the irst 2 years
treatment of choice for reactive arthritis. Observation after exposure and drops of after more time). BCG
(Option E) is not the best answer, given the severity of vaccination should not play a role in the decision
symptoms in this case. whether to treat additionally, on the basis of the
210 C HA P T E R 9 General Internal Medicine

large area of induration and how long ago he was patient likely had rheumatic fever as a child, resulting
vaccinated, BCG is unlikely to be contributing to in rheumatic heart disease and mitral stenosis, which
the test result. Neither the skin test nor the blood was asymptomatic until her pregnancy.
test is a “gold standard”; both can have false-posi- Medical management of symptomatic mitral ste-
tive and false-negative results. his patient should nosis in pregnancy involves beta blockers (to slow the
be treated for latent TB with one of the regimens heart rate and improve diastolic illing) and gentle
recommended by the Centers for Disease Con- diuresis (Option A). If this is not adequate, percutane-
trol and Prevention. hese include isoniazid for 9 ous mitral valvuloplasty can be considered.
months, rifampin for 4 months, or isoniazid and Inotropes such as digoxin (Option B) and dopa-
rifapentine given together weekly for 12 weeks. For mine (Option C) should be avoided in mitral steno-
patients who do not take other medications that sis. Vasodilators such as ACE inhibitors (Option D)
interact with rifampin or rifapentine and who do and hydralazine/nitrates (Option E) are not irst-line
not wish to take a long course of treatment, the iso- agents in the setting of decompensated mitral stenosis.
niazid/rifapentine option is reasonable. Treatment he safety of medications during pregnancy should
of latent TB with rifampin and pyrazinamide for also be considered.
4 months is no longer recommended by the CDC,
owing to excess risk of hepatotoxicity. he CDC 49. ANSWER: B. IgA nephropathy
website provides updated information about diag- IgA nephropathy (Option B) is the most common
nosis and management of latent tuberculosis (http: cause of primary glomerulonephritis in the devel-
//www.cdc.gov/tb/default.htm). oped world. It usually presents with recurrent gross
hematuria, usually less than 5 days after a urinary
47. ANSWER: C. Lumbar puncture tract infection or bacterial tonsillitis, though it may
his patient’s symptoms and signs of headache, also present with microscopic hematuria and mild
intermittent blurred vision, pulsatile tinnitus, and proteinuria.
papilledema are most consistent with increased intra- Poststreptococcal glomerulonephritis (Option A)
cranial pressure. Her normal imaging studies exclude a is an immune complex disease resulting from speciic
mass lesion, hydrocephalus, or venous sinus thrombo- nephritogenic strains of group A streptococcus. It usu-
sis, leaving idiopathic intracranial hypertension (pseu- ally occurs 1–3 weeks after pharyngitis and 3–6 weeks
dotumor cerebri) as the most likely diagnosis. his after skin infection. Children ages 5–12 and adults
condition is most commonly seen in obese women of >60 are at highest risk. he most common presenting
childbearing age. symptoms are edema, gross hematuria, and hyperten-
Because a mass lesion has been excluded by imag- sion, though the presentation ranges from asymp-
ing, lumbar puncture is indicated to conirm increased tomatic to microscopic hematuria to full-blown acute
cerebrospinal luid pressure and to initiate treatment nephritic syndrome. Complement components are
by removal of cerebrospinal luid. Urgent ophthalmo- usually decreased, and this may persist for 4–8 weeks.
logic consultation also is important for formal assess- Renal function resolves within 3–4 weeks; hematuria
ment and subsequent monitoring of visual ields, may persist for 3–6 months. Unlike IgA nephropathy,
because visual loss is a potential complication of this it rarely recurs.
condition. Acetazolamide therapy to reduce cerebro- Proteinuria and dysmorphic red cells suggest glo-
spinal luid production should be started only after merular (rather than extraglomerular) bleeding, mak-
lumbar puncture conirms the diagnosis. Amitripty- ing the remaining choices unlikely. Carcinoma of the
line therapy is not appropriate, because her current bladder (Option C) typically afects older patients,
headaches are not consistent with migraine, and this with a mean age at diagnosis of around 70 years. Uri-
therapy may cause increased weight gain. Neuro- nary tract infections (Option D) and nephrolithiasis
surgical consultation is not appropriate at this time, (Option E) would be expected to present with addi-
although patients with idiopathic intracranial hyper- tional signs and symptoms and without dysmorphic
tension who develop visual ield loss despite medical red blood cells in the urine.
management may require surgical intervention with Wyatt RJ, Julian BA. IgA nephropathy. N Engl J Med.
optic nerve sheath fenestration or cerebrospinal luid 2013;368:2402–2414.
shunting procedures.
Friedman DI, Liu GT, Digre KB, et al. Revised diagnostic cri- 50. ANSWER: B. Squeezing limes while making
teria for the pseudotumor cerebri syndrome in adults and mojitos
children. Neurology. 2013;81(13):1159–1165. his strange, often bizarre, well-demarcated rash
(phytophotodermatitis) is the result of a cutaneous
48. ANSWER: A. Beta blockers and gentle diuresis phototoxic eruption that results from contact with
Given the epidemiologic features of being raised light-sensitizing substances and exposure to UVA
in Brazil, as well as the examination and echocardio- radiation (Option B). Symptoms typically begin
graphic features suggestive of mitral valve stenosis, this within 24 hours of exposure, peak at 48–72 hours,
CHAPTER 9 General Internal Medicine 211

and may take weeks to resolve. Manifestation can • Sporothrix spp.—classically contracted through a
range from hyperpigmentation (as seen in Fig. 9.6) to rosebush thorn puncture (Option D)
bullous eruptions. he most frequently reported sen- • Streptococcus pyogenes—contracted through injec-
sitizing agents are lime juice, mangoes, celery, occa- tion drug use (Option E)
sionally roses/grasses, and bergamot oils in perfumes A reaction to sunscreen would not be expected to
with essential oils. he treatment for this condition is have such a linear pattern (Option C).
watchful waiting.
Given the linear pattern on the arm, lymphangitis Acknowledgment
is also in the diferential diagnosis, but in the absence
of systemic symptoms, this is highly unlikely. Lym- he authors and editors gratefully acknowledge the
phangitis can be caused by various organisms, such as contributions of the previous authors—Nikhil Wagle,
the following: Christopher Gibson, Ami Bhatt, Molly L. Perencevich,
• Mycobacterium marinum—common in saltwater William Martinez, Jason Ojeda, Rose Kakoza, and Lind-
ish tanks (Option A) say King.
Index

A Alendronate, for mild primary Aortic valve replacement, 172–173, 185


Abdominal aortic aneurysm (AAA), abdominal hyperparathyroidism, 104, 114 Argatroban, for heparin-induced thrombocytopenia
ultrasound for, 173, 186, 195, 205–206 Allopurinol with thrombosis (HITT), 85, 98–99
Abdominal compartment syndrome, measurement for Burkitt lymphoma, 37, 47 Arrhythmia, acute metabolic alkalemia causing,
of bladder pressure in, 84, 97 for gout, 192, 203 83, 95–96
Acetaminophen poisoning, N-acetylcysteine for, Alopecia areata, 202 Arterial blood gas with cooximetry, for hypoxemia,
84, 97 Alpha-1 antitrypsin deiciency, 82, 82f, 94 86, 101
N-acetylcysteine, for acetaminophen poisoning, emphysema and, 88 Arthritis
84, 97 Alport syndrome, 135, 143 pseudogout, acute, 49, 49f
Achilles tendon, quinolone-associated rupture of, Alternative immunosuppressive therapies, for renal septic, 49, 63
72 graft, 61, 74–75 aspiration with luid analysis for, 61, 75
Acinetobacter baumannii, colistimethate for, 6, Amitriptyline, 151, 161 viral, 51–52, 66
18–19 Amlodipine, for hypertension, 134, 142 Aspiration, for septic arthritis, 61, 75
Activated partial thromboplastin time (aPTT), Amoxicillin, for bacterial rhinosinusitis, 193, Aspirin, for NSTEMI, 171, 183
evaluation of, 31, 40 203–204 Aspirin-sensitive asthma, 89
Active pulmonary disease, chest x-ray for, 2, 13 Amoxicillin-clavulanate, oral, for asplenic patients, Asthma
Acute calciic tendonitis, 55, 69 dog bite and, 3, 15 aspirin-sensitive, 89
Acute chest syndrome (ACS), management of, Amphotericin B, lipid formulation of, for diabetic leukotriene modiiers for, 78, 89
29–30, 30t, 39 ketoacidosis, 4, 16 Atelectasis, 94f
Acute coronary dissection, 176, 189–190 Ampicillin Atherosclerotic cardiovascular disease (ASCVD),
Acute interstitial nephritis, 131, 139 as empiric antibiotic regimen, 7, 20–21 120
Acute ischemic stroke, 132–133, 140 for ruptured brain abscess, 196, 206–207 statin therapy for, 191, 201
Acute nonthyroidal illness, thyroid function with, Amyloidosis, AL, 133, 140–141 Atorvastatin, for patients with diabetes and
111, 123–124 ANCA-associated vasculitis, steroids for, 133, 141 NSTEMI, 171, 183
Acute promyelocytic leukemia (APL), 35, 44–45 Androgen deprivation therapy, prolactinoma and, Atrial septal defect, 165, 177
Acute pseudogout arthritis, 49, 49f, 63 32, 41 Azithromycin
Acute respiratory distress syndrome Anemia, red blood cell transfusion for, mortality for chlamydia, 193, 203
mandatory (assist-control) ventilator modes for, and, 85, 99 empiric, for Bordetella pertussis, 2, 14
84, 96 Angiotensin-converting enzyme (ACE) inhibitor, for pertussis, 198–199, 208
paralysis and, 84, 96 for focal lupus nephritis, 130, 138 for uncomplicated gonococcal urethritis, 8,
Acute toxoplasma infection, prevention of, during Anion gap acidosis, lorazepam and, 134, 142 21–22
pregnancy, 9, 23 Ankle brachial index, 176, 190
Acute uncomplicated cystitis, nitrofurantoin Antibiotics B
macrocrystals for, 2, 13 for exacerbation of chronic obstructive Bacteroides fragilis, 6, 19–20
Adalimumab, for ankylosing spondylitis, 62, 75–76 pulmonary disease, 83, 95 Barium esophagram, 147, 156
Adenocarcinoma, management of, 36–37, 47 prolonged, for Whipple disease, 146, 155 Barrett segment, biopsy of, 145, 154
Adenosine, for supraventricular tachycardia, 168, prophylactic, for cirrhosis, 149, 158 Bartonella henselae, 3, 15
169f, 180–181 Anticoagulation, for venous thromboembolism, Beta-agonist bronchodilators, long-acting inhaled,
Adnexal mass, needle-guided biopsy of, 34, 44 193, 203 adverse outcomes from, 78, 87–88
Adrenal incidentaloma, 197, 207 Antidiuretic hormone (ADH), 118 Beta blockers, for mitral stenosis, 200, 210
Adrenal myelolipoma, 113, 126–127 Antiglutamic acid decarboxylase (GAD) antibodies, Biopsy
Adrenal vein sampling, for hyperaldosteronism, for type I diabetes, 108, 119–120 of Barrett segment, 145, 154
130, 137–138 Antineutrophilic cytoplasmic antibodies (ANCA), renal, for focal lupus nephritis, 130, 138
Adult-onset Still disease, 60, 73–74 serum test for, 8, 22 skeletal muscle, 53, 53f, 68
AIMS65 score, 157 Antiresorptive therapy, level of, for osteopenia, temporal artery, 49, 63–64
Albuterol, inhaled, for rhinosinusitis, 7, 21 106, 117 for pulmonary-renal syndrome, 60–61, 74
Alcoholic hepatitis, supportive care, Antiretroviral preexposure prophylaxis (PrEP), 10, Biventricular pacemaker placement,
151–152, 162 24–25 168–170, 181
Aldosterone, plasma, in adrenal incidentaloma, Antiretroviral therapy (ART), 6, 20 Blood serologic test, for Coccidioides, 10–11, 25–26
197, 207 for HIV-associated nephropathy (HIVAN), 130, Blood smear, for mono-like illness, 5, 17–18
Aldosterone-to-renin ratio (ARR), 115 137 Blood test, for emphysema, 78, 88

Page numbers followed by f indicate igures and t indicate tables.

212
Index 213

Bone mineral density (BMD), in patient with Chronic lymphocytic leukemia Decompressive laminectomy, for lumbar spinal
primary hyperparathyroidectomy, 114 diagnosis of, 33–34, 43 stenosis, 60, 74
Bordetella pertussis, empiric azithromycin for, 2, 14 ibrutinib for, 36, 47 Dengue, 1, 11–12
Bowel ischemia, digoxin and, 145, 154 Chronic metabolic alkalosis, arrhythmia and, 83, Dental procedure, cardiovascular disease and, 167,
Brain abscess, ruptured, neurosurgery consultation 95–96 179
for, 196, 196f, 206–207 Chronic myelogenous leukemia, management of, Dermatomyositis, 53f, 68
BRCA2 mutation, 28, 37 35–36, 45–46 Dexamethasone suppression test, for adrenal mass,
Breast pain, 191–192, 202 Chronic obstructive pulmonary disease, 107–108, 119
Breastfeeding, hepatitis C virus (HCV) exacerbation of, antibiotics for, 83, 95 Dextromethorphan, oral, for rhinosinusitis, 7, 21
transmission and, 150, 160 Cirrhosis Diabetes insipidus, central, 34–35, 44, 107, 118
Bromocriptine, 116 decompensated, mortality in patients with, 145, Diabetic ketoacidosis, amphotericin B for, 4, 16
Bronchiectasis, serum immunoglobulin G and, 81, 154 Dialysis
92–93 primary biliary, 146, 154–155 for lupus nephritis, 133, 141
Bronchiolitis obliterans, 95 Cisplatin-based chemotherapy, neoadjuvant, vascular access for, 129, 136–137
Bronchoscopy 31–32, 40 Dietary interventions, for dysglycemia, 111–112,
lexible, for difuse alveolar hemorrhage, 61–62, Clavulanate, for bacterial rhinosinusitis, 193, 124–125
62f, 75 203–204 Difuse alveolar hemorrhage, lexible bronchoscopy
with lavage, rheumatoid arthritis and, 52, 67 Clindamycin, for severe babesiosis, 10, 25 for, 61–62, 62f, 75
Clopidogrel, for NSTEMI, 168, 180 Difuse intrathoracic airlow obstruction, low-
C Clostridium diicile infection, IBD and, 147, 156 volume loop for, 81, 81f–82f, 92, 92f
Cabergoline, 128 Clotrimazole troches, for oral candidiasis, 5–6, 18 Digoxin, 175–176, 189
Calcium, hypothalamic amenorrhea and, 198, Coccidioides, blood serologic test for, 10–11, 25–26 bowel ischemia and, 145, 154
207–208 Colchicine, 170–171, 182–183 1,25-dihydroxy-vitamin D, for granulomatous
Calcium oxalate, kidney stones, 135, 143 gout and, 192, 203 diseases, 109, 122
Calcium pyrophosphate dihydrate, of hands, 49f Colistimethate, for Acinetobacter baumannii, 6, Diuresis, for mitral stenosis, 200, 210
Campylobacter infection, in Guillain-Barré 18–19 Dopamine, for hypotension, 83, 96
syndrome, 2, 13 Colonoscopy, 195, 205 Dopamine agonist
Captopril, for scleroderma, 199, 209 Complete blood count, for osteoporosis, 113, 126 for prolactin-secreting pituitary adenoma, 114,
renal crisis, 57–58, 72, 130, 137 Computed tomography (CT), for pulmonary 128
Cardiac arrest, therapeutic hypothermia and, nodules, 33, 42, 42t for prolactinomas, 34, 43–44
85–86, 100 Congestive heart failure (CHF)-associated liver Doxycycline, for Lyme disease, 2–3, 14, 194, 204
Cardiac mass, surgical removal of, 174, 187 disease, indings in, 150, 159–160 Drug-drug interaction
Cardiac resynchronization therapy, 168–170, 181 Consolidation with air bronchograms, 93, 93f in inhaled luticasone, 6, 19
Cardiogenic shock, 86, 101 Constipation in serotonin syndrome, 10, 25
Cardiopulmonary resuscitation, end-tidal CO2 cystic ibrosis and, 147, 156 of warfarin with rifampin, 11, 26
detection and, 87, 102 predominant irritable bowel syndrome,
Cardiovascular disease, 165–190 153, 164 E
Cardioverter-deibrillator, implantable, 171–172, Copper deiciency, 148, 158, 197, 207 Echocardiography, 166, 178
184 Coronary angiography, heparin and, for NSTEMI, for myocardial infarction, 198, 198f, 208
Carpal tunnel syndrome (CTS), 71 167–168, 179–180 Eculizumab, for atypical hemolytic uremic
Caspofungin Coronary artery bypass grafting, of LAD artery and syndrome, 133, 141
intravenous, for multifocal pneumonia, 3, 15 RCA, 173, 186 Edema, pregnancy and, 170, 181
for neutropenic patients with fever, 5, 17 Corticosteroids Elderly, falls in, 196, 207
Catecholamine excess (pheochromocytoma), 119 systemic, for paradoxical tuberculosis immune Electrolyte panel, for hyponatremia, 198, 208
Cauda equina syndrome, MRI of lumbar spine for, reconstitution inlammatory syndrome, Electrophysiologic study and mapping, 167, 167f,
59, 73 4, 16 179
Cefepime vasopressor and, response to, 87, 102–103 Endocrinology, 104–128
for bacterial endocarditis, 4, 16–17 Cortisol Endometrial biopsy, 191, 202
for pneumonia, 5, 18 late-night salivary, 108, 120–121 Endoscopic retrograde cholangiopancreatography,
Ceftriaxone, 152, 163 value, interpretation of, 111, 123 153, 163–164
as empiric antibiotic regimen, 7, 20–21 Crystalloid Endotracheal suctioning, 86, 101–102
for gonorrhea, 193, 203 intravenous, for septic shock, 85, 98 Enteral nutrition, with nasojejunal tube, 152, 162
for ruptured brain abscess, 196, 206–207 for sepsis, 5, 17 Entrapment neuropathy, of left lateral femoral
for uncomplicated gonococcal urethritis, 8, Cushing syndrome, 119 cutaneous nerve, 57, 71
21–22 Cyclic mastalgia, 191–192, 202 Environmental contamination, chlorine-containing
Cellulitis, recurrent, penicillin for, 8–9, 23 Cyclophosphamide, for ANCA-associated cleaning agent for, 3, 15
Central apneas, 80, 91–92 vasculitis, 133, 141 Eosinophilic esophagitis, 147, 156
CFTR gene mutation, 112, 125 Cystic ibrosis (CF), 156 Eosinophilic granulomatosis with polyangiitis,
Chest radiograph, for Lofgren syndrome, 59, 73 Cytology, cervical cancer and, 193, 203 194, 204
Chest x-ray, for active pulmonary disease, 2, 13 Cytomegalovirus (CMV) serologies, for mono-like Eplerenone, 170, 182
Chlorine-containing cleaning agent, for illness, 5, 17–18 Epstein-Barr virus (EBV) serologies, for mono-like
environmental contamination, 3, 15 illness, 5, 17–18
Chlorthalidone, for hypertensive management, D Erosive esophagitis, treatment for, 149, 159
131, 134, 139, 142 De Quervain tenosynovitis, 200, 209 Erythropoiesis-stimulating agent (ESA), 35, 45
Chondrocalcinosis, of wrist, 49f, 56–57, 71 Death, from long-acting inhaled beta-agonist Erythropoietin deiciency, anemia due to, 28–29,
Chronic interstitial nephritis, 132, 140 bronchodilators, 78, 87–88 29t, 38
Chronic kidney disease, for vasopressin receptor Decompensated cirrhosis, mortality in patients Esophagogastroduodenoscopy, upper, 148,
antagonism, 133, 141 with, 145, 154 157–158
214 Index

Estradiol (vaginal pill), for genitourinary Health care maintenance, for smoking, I
menopausal symptoms, 107, 119 6–7, 20 Ibrutinib, for chronic lymphocytic leukemia, 36, 47
Ethylene glycol intoxication, 135, 143–144 Hematocrit, for patient receiving hormone Ibuprofen
Exercise replacement therapy, 110, 122 for acute pericarditis, 170–171, 182–183
hypothalamic amenorrhea and, 198, 207–208 Hematology, 28–48 for STEMI, 165, 165f, 177
for peripheral arterial disease, 172, 185 Hemochromatosis, 50, 51f, 65 Idiopathic pulmonary ibrosis, 79, 79f, 90, 90f
Hemodialysis, for hyperkalemia, 129, 137 pirfenidone for, 80, 90–91
F Hemolytic uremic syndrome, atypical, eculizumab IgA nephropathy, 131, 138–139
Fabry disease, 135, 143 for, 133, 141 glomerulonephritis and, 200–201, 210
Falls, in elderly, 196, 207 Hemosiderosis, pulmonary, 92–93 Immotile cilia syndrome, 92–93
Familial juvenile hyperuricemic nephropathy, 133, Heparin, intravenous, 176, 189 Indomethacin
141–142 Heparin-induced thrombocytopenia, management for reactive arthritis, 199, 199f, 209
Fanconi syndrome, tenofovir disoproxil and, 129, of, 30, 39 rectal, 149, 158–159
136 Heparin-induced thrombocytopenia with Infectious diseases, 1–27
Femoral cutaneous nerve, left lateral, entrapment thrombosis (HITT), argatroban for, 85, Insulin, intravenous, initiation of, with glucose,
neuropathy of, 57, 71 98–99 105, 115–116
Fentanyl, discontinuation of, 85, 99–100 Hepatic adenoma, 146, 155 Insulin (U-500), for severe insulin resistance,
Fibroblast growth factor 23 (FGF23), 110, 123 Hepatitis A vaccine, for rheumatoid arthritis, 112–113, 126
Fibromuscular dysplasia (FMD), renal, 134, 142 58–59, 72–73 Intermittent hemodialysis, renal replacement
Fibromyalgia syndrome, 57, 71 Hepatitis B infection therapy and, 84–85, 98
Flexible bronchoscopy, for difuse alveolar reactivation of, 2, 13–14 Internal medicine, 191–211
hemorrhage, 61–62, 62f, 75 treatment for, 151, 161 Interstitial lung diseases, test results in, 80, 90
Flow-volume loop, for difuse intrathoracic airlow Hepatitis B screening, 4, 15–16 Intraductal papillary mucinous neoplasm (IPMN),
obstruction, 81, 81f–82f, 92, 92f Hepatitis C antibody, 56, 56f, 70 management of, 28, 37–38
Fluticasone Hepatorenal syndrome (HRS), 134, 143 Ipratropium, intranasal, for rhinosinusitis, 7, 21
for eosinophilic esophagitis, 194, 205 Herpes simplex virus (HSV)-associated ulcer, Iron deiciency anemia, 199–200, 209
hoarse voice and, 82, 94–95 147–148, 157 Irritable bowel syndrome (IBS), 154
inhaled, in harmful drug-drug interaction, 6, 19 High-dose insulin therapy, for calcium channel colonoscopy for, 148, 157
Fomepizole, for ethylene glycol intoxication, 86, blocker overdose, 87, 103 constipation-predominant, 153, 164
100–101 Hip, osteoarthritis of, 50, 64 Ischemic colitis, 154
Fosphenytoin, lorazepam infusion followed by, for HIV 1/2 antigen/antibody test, in mono-like Isoniazid, for tuberculosis, 200, 209–210
status epilepticus, 87, 102 illness, 5, 17–18 Isosorbide mononitrate, 171, 183–184
Francisella tularensis, 11, 26–27 HIV-associated nephropathy (HIVAN), ART for, Isotonic saline
Fundic gland polyps, PPI-associated, cancer and, 130, 137 with hydrocortisone, 131–132, 139
146, 155 HIV infection, 145, 154 for rhabdomyolysis, 131, 138
Furosemide, for hypertension, 129, 136 HIV RNA test, in mono-like illness, 5, 17–18
Hodgkin lymphoma, management of, 33, 43 K
G Home blood pressure diary, 174, 188 Karyotyping, for Klinefelter syndrome, 109–110,
Gallbladder cancer, 147, 156–157 Honeycombing, 93, 93f 122
Gastric cancer, 28, 37 Hydrocortisone Ketoconazole, 176, 189
Gastroenterology, 145–164 for adrenal insuiciency, 105, 116 Ketosis-prone diabetes, of patient, 106, 117–118
Gastroesophageal relux disease, cystic ibrosis and, for pituitary apoplexy, 105, 116 Kidney stones
147, 156 Hydroxychloroquine, 50, 64–65 calcium oxalate, 135, 143
Giant cell arteritis (GCA), 69–70 1,25-hydroxyvitamin D supplementation, for uric acid, 135, 144
Gitelman syndrome, 134–135, 143 secondary hyperparathyroidism, 133, 141 Klinefelter syndrome, 109–110, 122
Global Initiative for Chronic Obstructive Lung Hypercortisolism (Cushing syndrome), 119
Disease (GOLD) Hyperhidrosis, 194–195, 205 L
COPD classiication with, 79, 89 Hyperprolactinemia, 127 Laboratory results, interpretation of, in primary
guidelines for, 88 Hypertension, 129–144 hyperaldosteronism, 105, 115
Glomerular iltration rate (GFR), 114 pulmonary, 52, 67 Lactic acidosis
Gluten-free diet, 112, 125 Hypertensive emergency, 129, 136 type A, 135, 143
Goiter, management of, 109, 121 chlorthalidone and lisinopril for, 134, 142 type B, 135, 143
Gout, tophaceous, 61f Hypertrophic cardiomyopathy, 194, 204–205 Laminectomy, decompressive, for lumbar spinal
Gouty bursitis, of right prepatellar bursa, 58, 72 Hypertrophic obstructive cardiomyopathy, 172, stenosis, 60, 74
Granulomatous arteritis, pulmonary-renal 184 Lane-Hamilton disease, 92–93
syndrome and, 60–61, 74 Hyperuricemia, 59, 73 Lavage, bronchoscopy with, for microscopic
Graves disease Hypoglycemia, complications of, 111, 124 organisms, 52, 67
methimazole for, 107, 118 Hypomagnesemia, 132, 140 Leukapheresis, 36, 46
radioactive iodine treatment for, 105, 116 Hyponatremia, multiple myeloma and, Leukotriene modiier, for asthma, 78, 89
Greater trochanteric pain syndrome, reassurance 130, 137 Levoloxacin
in, 55, 70 Hypoparathyroidism, as etiology of hypocalcemia, for Legionella pneumonia, 4, 16
Ground-glass opacity, 93, 93f 106, 117 for pneumococcal pneumonia, 58, 72
Group A streptococci, 11, 27 Hyporeninemic hypoaldosteronism, sodium for pneumonia, 5, 18
bicarbonate therapy for, 135, 144 Levothyroxine
H Hypotension, treatment for, 83, 96 for coronary artery disease, 113, 127
H. pylori infection, testing for, 151, 161 Hypothalamic amenorrhea, 110, 123 for hypothyroidism, 110, 122
HbA1c value, complete blood count and, 106–107, Hypothalamic-pituitary-adrenal (HPA) axis, secondary, 111, 124
118 120–121 pregnancy and, 114, 127–128
Index 215

Lifestyle interventions, for dysglycemia, 111–112, Myasthenia gravis, 146, 155 Permissive hypercapnia, 95–96
124–125 Mycophenolate mofetil, for focal lupus nephritis, Petechiae, lower-extremity, 56f
Linagliptin, for hyperglycemia, 110, 122–123 130, 138 Phospholipase A2 receptor (PLA2R), for idiopathic
Lisinopril, 173, 186 Mycoplasma genitalium, 9, 24 membranous nephropathy, 132, 140
for hypertensive urgency, 134, 142 Physical therapy
Liver function test, abnormal, 149, 158 N for ibromyalgia syndrome, 57, 71
Lorazepam Nasojejunal tube, enteral nutrition with, 152, 162 for greater trochanteric pain syndrome, 55, 70
anion gap acidosis and, 134, 142 Nephrogenic systemic ibrosis (NSF), 62–63, 76 Phytophotodermatitis, limes and, 201, 201f,
infusion, followed by fosphenytoin, for status Nephrology, 129–144 210–211
epilepticus, 87, 102 Nephrotic syndrome, 134, 142 Pickwickian syndrome, 91–92
Low-oxalate diet, calcium supplementation and, Neuroleptic malignant syndrome (NMS), 99–100 Pioglitazone, for hypoglycemia, 105–106, 117
153, 163 Niacin deiciency, 148, 157 Piperacillin-tazobactam, for nonhealing ulcer, 9, 23
Low-salt diet, 130–131, 138 Nitrofurantoin Pirfenidone, for idiopathic pulmonary ibrosis, 80,
Lumbago, 65–66 for cystitis, 192, 202–203 90–91
Lumbar puncture, 200, 210 macrocrystals, for acute uncomplicated cystitis, Pituitary mass, clinical considerations to, 35, 44
Lung cancer 2, 13 Plantar fasciitis, 58, 72
low-dose computed tomography for, 30, 39 Non-ST segment elevation myocardial infarction Plasma exchange, for thrombotic thrombocytopenic
radiographic studies of, 33, 42 (NSTEMI), management of, 167–168, purpura, 193–194, 193f, 204
surgical resection for, 33, 42–43 179–180 Plasmapheresis, for ANCA-associated vasculitis,
Lung nodules, surgical resection of, 82, 94 Noninvasive ventilation, for acute-on-chronic 133, 141
Lupus nephritis, 133, 141 respiratory acidosis, 84, 97 Pneumococcal 13-valent conjugate (PCV13)
Lyme disease Norepinephrine, for hypotension, 83, 96 vaccine, 1, 12–13
doxycycline for, 2–3, 14, 194, 204 Novel oral anticoagulant (NOAC) therapy, 34, 44 Pneumococcal pneumonia, treatment of, 58, 72
serologic testing for, 51, 66 Nuclear stress test, with regadenoson, 172, 185 Pneumonia, acute, management of, 79, 89
Lymphadenopathy, asymptomatic, management Pneumonitis, interventions for, 32, 41
of, 34, 43 O Polyarteritis nodosa, 132, 140
Lymphangioleiomyomatosis, 79, 89–90 Obesity hypoventilation syndrome (OHS), 91–92 Polychondritis, relapsing, 80, 91
Lymphoma, parotid, 62, 76 blood gas on room air and, 80, 92 Polymyalgia rheumatica (PMR), 73
Obstructive sleep apnea Positive anti-Scl-70 antibody, 150–151, 161
M continuous positive airway pressure (CPAP) for, Postenteric reactive arthritis, 155
Magnetic resonance angiography 133–134, 142 Poststreptococcal glomerulonephritis, 210
for cerebral artery aneurysm, 194, 204 motor vehicle accidents and, 80, 91 Prediabetes, 124–125
for headaches, 191, 201–202 retrognathia and, 80, 91 Prednisone
of thoracic and abdominal aorta, 171, 183 Oncology, 28–48 for focal lupus nephritis, 130, 138
Magnetic resonance imaging Opiate medication contract, in lumbago- for giant cell arteritis, 55, 69–70
for breast cancer, 192, 202 musculoskeletal low back pain, 50–51, 65–66 with or without azathioprine
for headaches, 191, 201–202 Orthodeoxia, 91–92 for elevated hepatic enzymes, 146–147,
Magnetic resonance venography, for venous sinus Osmotic demyelination syndrome, 131, 139 155–156
thrombosis, 192, 202 Osteoarthritis, 49, 63, 195, 205 for ulcerative colitis, 151, 162
Mammograms, for breast cancer, 192, 202 of hip, 50, 64 for pancreatitis, 152, 162
Mandatory (assist-control) ventilator mode, for Osteoporosis, due to increased aromatase inhibitor, Pregnancy
acute respiratory distress syndrome, 84, 96 34, 43 acute toxoplasma infection during, management
Maturity-onset diabetes of youth (MODY), of, 9, 23
119–120 P syphilis infection during, management of, 7, 21
Medication history, hypertension and, 173, Pacemaker, 170, 174, 182, 187 Prepatellar bursa, right, gouty bursitis of, 58, 72
185–186 Paget disease Primary adrenal insuiciency, 107, 118–119
Medication-induced hyperprolactinemia, 127 of bone, treatment for, 192, 202 Primary biliary cirrhosis, 146, 154–155
MEFV genetic testing, 53–54, 68 zoledronic acid infusion for, 113, 126 Primary sclerosing cholangitis (PSC), colonoscopy
Melena, presence of, 148, 157 Paradoxical tuberculosis immune reconstitution for, 148, 157
Ménétrier disease, 150, 160 inlammatory syndrome, systemic Progesterone, serum, pregnancy and, 191, 202
Meralgia paresthetica, 71 corticosteroids for, 4, 16 Programmed death receptor 1 (PD-1) inhibition,
Metformin therapy, for renal dysfunction, 104, 114 Paralysis, severe acute respiratory distress syndrome immune therapy with, 32, 41
Methimazole, for Graves disease, 107, 118 and, 84, 96 Prolactinoma
Methotrexate, 50, 64 Parapneumonic efusion, management of, 79, 89 androgen deprivation therapy and, 32, 41
Methylprednisolone, intramuscular, 54–55, 69 Parotid lymphoma, 62, 76 dopamine agonist for, 34, 43–44
Metoprolol, 174, 187 Patellofemoral syndrome, 54, 68–69 Prophylactic antibiotics, for cirrhosis, 149, 158
for heart rate, 166–167, 178–179 Pazopanib, for renal cell carcinoma, 31, 40 Propofol
Microcytic anemia, management of, 28, 29t, 38 PCV13 vaccine. see Pneumococcal 13-valent duration of mechanical ventilation and,
Midazolam, duration of mechanical ventilation conjugate (PCV13) vaccine 86, 100
and, 86, 100 Penicillin hyperkalemia and, 84, 96–97
Migraine without aura, 195, 206 oral, for rheumatic mitral stenosis, 176, 190 Prostate cancer
Minimal change disease, 132, 140 for recurrent cellulitis, 8–9, 23 diagnostic workup for, 32, 40
Mitral stenosis, 166, 178 Percutaneous coronary intervention (PCI), of management of, 32, 41
Montelukast, for asthma, 78, 89 obtuse marginal stenosis, 174–175, 188 Proton pump inhibitor (PPI)
Morning stifness, 59–60, 73 Percutaneous renal artery angioplasty, with stent associated fundic gland polyps, cancer and, 146,
Motor vehicle accidents, obstructive sleep apnea placement, 131, 139 155
and, 80, 91 Peripheral arterial disease (PAD), supervised for erosive esophagitis, 149, 159
MUTYH-associated polyposis (MAP), 149, 159 exercise program for, 172, 185 Pseudogout arthritis, acute, 49, 49f, 63
216 Index

Psychiatric counseling, for ibromyalgia syndrome, Sensorineural hearing loss, sudden, 199, 208–209 hiazolidinediones (TZDs), 126
57, 71 Septic arthritis, 49, 63 horacentesis, 196–197, 197f, 207
Pulmonary and critical care medicine, 78–103 aspiration with luid analysis for, 61, 75 hrombophilia, evaluation of, 31, 40
Pulmonary ibrosis, idiopathic, 79, 79f, 90, 90f Septic shock, intravenous crystalloid for, 85, 98 hrombotic thrombocytopenic purpura, 204
pirfenidone for, 80, 90–91 Serologic testing, for Lyme disease, 51, 66 management of, 30, 39
Pulmonary function test, 78, 88 Serotonin syndrome, 99–100 hyroid scan, for thyrotoxicosis, 108, 120
Pulmonary hemosiderosis, 92–93 Serous cystadenoma, 149, 159 hyrotropin-releasing hormone (TRH), 127
Pulmonary hypertension, 52, 67 Sertraline, discontinuation of, 85, 99–100 Tilt-table test, 53, 67–68
Pulmonary-renal syndrome, temporal artery biopsy Serum immunoglobulin G, bronchiectasis and, 81, Tophaceous gout, 61f
for, 60–61, 74 92–93 Total knee arthroplasty, right, 56, 70–71
Pulmonary sarcoidosis, stage 1, 83, 83f, 95 Serum protein electrophoresis (SPEP), 52, 66–67 Total testosterone, measurement of, 106, 117
Pyroglutamate, 135–136, 144 Sex-hormone-binding globulin, measurement of, Toxoplasma infection, prevention of, during
106, 117 pregnancy, 9, 23
Q Sickle cell disease, management of, 30, 30t, 39 Transcatheter aortic valve replacement, 175, 188
Quinine, for severe babesiosis, 10, 25 Sjögren syndrome, 60, 74 Transesophageal echocardiogram, 172, 184–185
Skeletal muscle biopsy, 53, 53f, 68 Transient ischemic attack, hospital admission and,
R Small cell lung cancer, 33, 42 195, 206
Radioactive iodine treatment, for Graves disease, Small intestinal bacterial overgrowth (SIBO), Transplantation, for lupus nephritis, 141
105, 116 152–153, 163 Transsphenoidal surgery, 128
Rasburicase, for uric acid breakdown, 132, Smoking cessation, for HIV patient, 6–7, 20 Transthoracic echocardiogram, 168, 180
139–140 Somatostatin analogue, medical therapy with, 109, close follow-up with, 175, 188
Raynaud phenomenon, 57, 71–72 121 Transurethral resection of bladder tumor
Reactive arthritis, indomethacin for, 199, 199f, 209 Spinal cord compression, epidural, management of, (TURBT), 31, 40
Rebleeding, 197, 207 37, 47–48 Treadmill test, for coronary disease assessment,
Recurrent cellulitis, penicillin for, 8–9, 23 Spirometry, interpretation of results from, 88 174, 187
Red blood cell transfusion, for anemia, mortality Spironolactone, for polycystic ovarian syndrome Tree-in-bud nodules, 81–82, 82f, 93, 93f
and, 85, 99 (PCOS), 104, 114–115 Triamcinolone, intradermal injection of, for
Red cell exchange transfusion, for severe babesiosis, Splenectomy, 29, 29t, 38 alopecia areata, 192, 202
10, 25 Spontaneous breathing trial, 84, 97–98 Trimethoprim-sulfamethoxazole
Refractory ascites, in patient with right heart Staphylococcus aureus, in reactive arthritis, 58, 72 for CA MRSA, 194, 205
failure, 150, 160–161 Statin therapy, for atherosclerotic cardiovascular for Pneumocystis jirovecii pneumonia, 9–10, 24
Reiter syndrome. see Postenteric reactive arthritis disease, 191, 201 for skin and soft tissue infection, 3, 14
Relapsing polychondritis, 80, 91 Status epilepticus, lorazepam infusion followed by 24-hour urine free cortisol, for Cushing syndrome,
Renal artery Doppler study, 176–177, 190 fosphenytoin for, 87, 102 112, 125
Renal biopsy, for focal lupus nephritis, 130, 138 Steroids, for ANCA-associated vasculitis, 133, 141 Typhoid vaccine, injectable, for rheumatoid
Renal replacement therapy, intermittent Still disease, adult-onset, 60, 73–74 arthritis, 58–59, 72–73
hemodialysis and, 84–85, 98 Stress ulcer prophylaxis, ventilator-associated
Renal tubular acidosis, distal (type 1), 130, 138 pneumonia and, 85, 99 U
Renin, in adrenal incidentaloma, 197, 207 Subglottic secretions, suctioning of, for ventilator- Ulcer, herpes simplex virus (HSV)-associated,
Renin-angiotensin-aldosterone system, 115 associated pneumonia, 1, 12 147–148, 157
Respiratory acidosis, acute-on-chronic, noninvasive Succinylcholine, hyperkalemia and, 84, 96–97 Ulcerative colitis, 82, 95
ventilation for, 84, 97 Sudden sensorineural hearing loss (SSNHL), 199, Ultrasound, abdominal, for abdominal aortic
Respiratory failure, from long-acting inhaled beta- 208–209 aneurysm, 173, 186, 195, 205–206
agonist bronchodilators, 78, 87–88 Sunitinib, for renal cell carcinoma, 31, 40 Upper esophagogastroduodenoscopy, 148,
Restless legs syndrome, 91–92 Superior vena cava (SVC) syndrome, 37, 48 157–158
Retrognathia, obstructive sleep apnea and, 80, 91 Synovial luid, elevated leukocyte count in, 146, Urea breath test, for H. pylori persistence, 192–193,
Rhabdomyolysis, isotonic saline for, 131, 138 155 203
Rheumatoid arthritis, 49, 63, 76–77 Syphilis infection, management of, during Uric acid kidney stones, urinary pH for, 135, 144
Rheumatology, 49–77 pregnancy, 7, 21 Urinary 5-hydroxyindoleacetic acid,
Rhinosinusitis, acute bacterial, 203–204 Systemic lupus erythematosus (SLE), tests for, 54, 152, 162–163
Rifampin, warfarin with, drug-drug interaction of, 69 Urinary tract infections, strategies for, 8, 22
11, 26 Systemic sclerosis (scleroderma), 161 Urine pregnancy test, for hyperprolactinemia, 113,
Rifapentine, for tuberculosis, 200, 209–210 127
Rifaximin, 145, 154 T Urine protein electrophoresis (UPEP), 52, 66–67
Rivaroxaban, for deep vein thrombosis, 31, 40 t(9;22) chromosomal translocation, 35–36, 45–46 Uromodulin kidney disease, 133, 141–142
Rosuvastatin, for type 2 diabetes, 108, 120 t(15;17) chromosomal translocation, 35, 44–45 Usual interstitial pneumonitis (UIP), 90
Tamoxifen, for breast cancer prevention, 32–33,
S 41–42 V
Sacubitril/valsartan, 175, 188–189 Telmisartan, for diabetic nephropathy, 109, Valacyclovir, for Ramsay Hunt syndrome type II,
Saline, normal, volume loading with, 165–166, 121–122 9, 23–24
166f, 177–178 Temporal artery biopsy, 49, 63–64 Vancomycin
Salmeterol Multicenter Asthma Research Trial for pulmonary-renal syndrome, 60–61, 74 for bacterial endocarditis, 4, 16–17
(SMART), 87–88 Tendonitis, acute calciic, 55, 69 as empiric antibiotic regimen, 7, 20–21
Salmonellosis, 1, 12 Tenecteplase, for STEMI, 168, 169f, 181 for nonhealing ulcer, 9, 23
Scaly erythematous plaques, 52–53, 67 Tenofovir disoproxil, Fanconi syndrome and, 129, for pneumonia, 5, 18
Scleroderma renal crisis (SRC), captopril for, 130, 136 for ruptured brain abscess, 196, 206–207
137 herapeutic hypothermia, cardiac arrest and, Vasopressin receptor antagonism, for chronic
Secondary adrenal insuiciency, 118–119 85–86, 100 kidney disease (CKD), 133, 141
Index 217

Venlafaxine Vitamin E, for nonalcoholic fatty liver disease, Wilson disease, 150, 160
for breast cancer, 34, 44 195–196, 206 Wireless capsule endoscopy, 152, 163
for elevated normetanephrine levels, 112, Vitamin E deiciency, 150, 160 Wrist, chondrocalcinosis of, 56–57, 71
125–126 von Willebrand disease, 36, 46
Ventilator-associated pneumonia evaluation of, 31, 39–40 X
stress ulcer prophylaxis and, 85, 99 X-ray imaging
subglottic secretions in, suctioning W chest, for Hodgkin lymphoma, 33, 43
of, 1, 12 Warfarin, 170, 182 of lateral cervical spine, for cervical myelopathy,
Ventilator support, for obstructive lung disease, with rifampin, drug-drug interaction of, 55–56, 70
86, 100 11, 26
Viral arthritis, 51–52, 66 Weight loss, in ibromyalgia syndrome, 57, 71 Z
Viral culture, of ulcers, 8, 22 Whipple disease, prolonged antibiotic treatment Zairlukast, for asthma, 78, 89
Vitamin B12 deiciency, 29, 29t, 38 for, 146, 155 Zenker diverticulum, 156
Vitamin D, hypothalamic amenorrhea and, 198, Whole-brain radiotherapy (WBRT), Zileuton, for asthma, 78, 89
207–208 36, 46–47 Zoledronic acid infusion, for Paget disease, 113, 126
This page intentionally left blank
Fast answers and trusted evidence

Drive better outcomes with a clinical search engine


nd and apply relevant knowledge.

Fast Convenient
Anticipates your query, recognizing Accessible at the patient’s bedside or
ering shortcuts on the go, making it easy to discover,
share, and apply content anywhere
Complete
Draws relevant answers from a wide Trusted
range of current, comprehensive Content from Elsevier, the
content across 30+ medical and name healthcare professionals
surgical specialties worldwide rely on

ckey.co/books

You might also like